You are on page 1of 301

Bài tập chuyên đề: PT, HPT, BPT, Hệ BPT Bồi dưỡng học sinh giỏi Toán Quốc

Bồi dưỡng học sinh giỏi Toán Quốc gia

Bài tập chuyên đề:


PHƯƠNG TRÌNH, HỆ PHƯƠNG TRÌNH,
BẤT PHƯƠNG TRÌNH, HỆ BẤT PHƯƠNG TRÌNH
VĂN PHÚ QUỐC
3
1. Giải PT: 3x 2  x  2012  3 3 x 2  6 x  2013  5 5 x  2014  3 2013 .
x 2x 3x 2012 x
2. Giải BPT:    ...  1
x  1  x  1 2 x  1  x  1 2 x  1 3 x  1  x  1 2 x  1 ...  2012 x  1
2 2 x1  cos x2

2 2 x2  cos x3
3. Giải HPT:  ; x1 , x2 , x3 , x4   .
2 2 x3  cos x4

2 2 x4  cos x1

 y
30 x 2  4 y  2012

 z
4. Giải HPT: 30 2  4 z  2012 ; x, y , z  
 y
 x
30 2  4 x  2012
 z

 x12  x22  x2  x1
 2 2
 x2  x3  x3  x2
..........................

5. Cho 2013 số dương: x1 , x2 ,..., x2013  0 thỏa mãn:  .
 ..........................
x2  x2  x  x
 2011 2012 2012 2011
2 2
 x2012  x2013  x2013  x2012

1
Chứng minh rằng trong 2013 số đó có hai số a, b sao cho: a  b  .
2012
x  y  z  3

6. Giải HPT:  x 2  y 2  z 2  3 ; x, y , z   .
 x 2012  y 2012  z 2012  3

7. Giải BPT:  
x  2012  x  2014  2 x 2  4028  2014 2  4024 x  4024 ; x.

VĂN PHÚ QUỐC- Trường THPT chuyên Nguyễn Bỉnh Khiêm -Quảng Nam (0982 333 443) 1
Bài tập chuyên đề: PT, HPT, BPT, Hệ BPT Bồi dưỡng học sinh giỏi Toán Quốc gia
30 x  4 x  2013 x 2012
1 2 3

30 x  4 x  2013 x 2012
2 3 4

8. Giải HPT: .......................................
 2013 2012
30 x2012  4 x1  x2
 x , x ,..., x  0
 1 2 2012

 1 2012 
 x1   x2  
 2 x2 

 x2  1  x3  2012 
9. Giải HPT:  2 x3  ; x1 , x2 ,..., x2012   .
..............................

 1 2012 
 x2012   x1  
 2 x1 

x2  2x  y
 2
y  2y  z
10. Giải HPT:  2 ; x, y, z , t   .
z  2z  t
t 2  2t  x

2
 2  k 1 
 x1  kx1     x2
  2 
 2
 k 1 
 x2  kx2  
2
11. Giải HPT:    x3 ; x1 , x2 ,..., x2012   , k là một số cho trước.
 2 
....................................

  k 1 
2
2
 x2012  kx2012     x1
  2 
12. Cho số nguyên n  3 . Giải hệ phương trình:
2012 x1  4025 x2  2013 x3  0

2012 x2  4025 x3  2013 x4  0
 ; x1 , x2 ,..., xn   .
...............................................
2012 xn  4025 x1  2013 x2  0

 2013 2 xy
 x  2012 2  x 2  y 2013
2012
 x  2x  2 1
13. Giải HPT:  ; x, y   .
y 
2013 2 xy 2 2013
 y x
 2012 2
x  2 y  22012  1

VĂN PHÚ QUỐC- Trường THPT chuyên Nguyễn Bỉnh Khiêm -Quảng Nam (0982 333 443) 2
Bài tập chuyên đề: PT, HPT, BPT, Hệ BPT Bồi dưỡng học sinh giỏi Toán Quốc gia
3 x x x 2 x x  x  2 x2  x x  2 x2  3 10
14. Giải PT: 2
 2
 2
  
x  x x  x3 x  x x 3 x  x 4 x x x 4 x x  x x 3 3
 x  y  2
15. Giải HPT:  2012 ; x, y   .
 x  y 2012  x 2011  y 2011
 y 2   5 x  4  4  x 
16. Giải HPT:  ; x, y   .
2 2
 y  5 x  4 xy  16 x  8 y  16  0
 
6 x 4  x3  x y 2   y  12  x 2  6

17. Giải HPT:  2
; x, y  
4
 2

5 x  x  1 y  11x  5

2 2

 3  2 x 2 y  x 4 y 2  x 2 1  2 x 2   y 4

18. Giải HPT:  ; x, y   .
2
1  1   x  y   x 2  x 4  2 x 2  2 xy 2  1  0

 2 xy  y x 2  y 2 x y x y
  
 14 2 2
19. Giải HPT:  ; x, y   .
3 3
  x y  x  y 
  2    2  9
    
 xy  2  x 4  y 4   1

20. Giải HPT:  ; x, y   .
2
 x 2009 y 2013  x 2013 y 2009  2011
 3
2 2
 x  y  1
21. Giải HPT: 2011 ; x, y   .
 x  
2011 y  2013 y  2013 x

 x  y  xy  2014 
4
 cos 2 x  6
22. Giải PT: 3  1  2   tan x  7
 cos x 
1 1
x  y  9

23. Giải HPT:  ( x, y   ).
 1 1   1   1 
   1 1    18
 3 x 3 y   3 x   3 y 
   
24. Giải BPT: 6  x 2  3 x  1  x 4  x 2  1  0 ; x  

 x  y  1  1  4  x  y 2  3  x  y 

25. Giải HPT:  , x, y   .
12 x  2 x 2
 3 y  7 xy   1  12 y 2
 3  5 x 
1 1
 x  2y  2 y  x 
4 4


26.Giải HPT:  , x, y  
 1  1   3 y   x 2  3x 2  y 2 
 x 2 y

VĂN PHÚ QUỐC- Trường THPT chuyên Nguyễn Bỉnh Khiêm -Quảng Nam (0982 333 443) 3
Bài tập chuyên đề: PT, HPT, BPT, Hệ BPT Bồi dưỡng học sinh giỏi Toán Quốc gia
27. Giải BPT: 4 x  6  3 x 3  7 x 2  12 x  6  x 2  2 ; x   .
3
28. Giải BPT: x5  x3  x  x 2
 
1  x2 x2  x 1 ; x   . 
 x 2  y  z 2  3x 2  x  1 y 2 z 2
 

 2

29. Giải HPT:  y 2  z  x   4 y 2  y  1 z 2 x 2  ; x, y, z   .
 2 2

 z  x  y   5 z 2  z  1 x 2 y 2 
y 3  m3 y 3  n3 y3  p3 3 3 ym yn y  p
30. (VMO 1975). Giải PT: 3
 3
 3
  . . .  0.
 y  m  y  n  y  p 2 2 ym yn y p

31. (VMO 1975). Không giải PT bậc ba: x 3  x  1  0 . Hãy tính tổng các lũy thừa bậc tám của ba
nghiệm số của nó.
x y 12
 x  y
32. (VMO 1976). Tìm nghiệm nguyên của HPT:  x  y 3
.
 y  x
1 1 x 1
33. (VMO 1977). Giải BPT: x
 1  .
x x x
34. (VMO 1978). Tìm tất cả các giá trị của m sao cho HPT sau đây chỉ có một nghiệm
 x 2  2 x  x  y  m
 2 .
2
 x  1  y
 x 2  y 2  z 2  t 2  50
 2
 x  y 2  z 2  t 2  24
35. (VMO 1981). Giải HPT:  .
 xz  yt
x  y  z  t  0

2t1  t2  a1
t  2t  t  a
 1 2 3 2

t  2t3  t4  a3
36. (Kỳ thi đặc biệt tại Huế, 1981). Giải HPT:  2 .
 ...........................
tn  2  2tn 1  tn  an 1

tn 1  2tn  an
37. Tìm nghiệm nguyên dương của PT: 2 x  2 y  2 z  2336  x  y  z .
38. (VMO 1993, bảng B). Giải BPT: 3 12  x  3 14  x  2 .
 x 2  3x  ln  2 x  1  y
40. (VMO 1994, bảng B). Giải HPT:  2 .
 y  3 y  ln  2 y  1  x
41. (VMO 1995, bảng A). Giải PT: x 3  3 x 2  8 x  40  8 4 4 x  4  0 .
42. (VMO 1995, bảng B). Giải PT: 2 x 2  11x  21  3 3 4 x  4  0 .

VĂN PHÚ QUỐC- Trường THPT chuyên Nguyễn Bỉnh Khiêm -Quảng Nam (0982 333 443) 4
Bài tập chuyên đề: PT, HPT, BPT, Hệ BPT Bồi dưỡng học sinh giỏi Toán Quốc gia
  1 
 3 x 1  2
  x y 
43. (VMO 1996, bảng A). Giải HPT:  .
 7 y 1  1   4 2
  
  x y
1  42 x  y  .51 2 x  y  1  22 x y 1

44. (VMO 1999, bảng A). Giải PT:  .
 y  4 x  1  ln  y  2 x   0
3 2

 7 x  y  2 x  y  5
45. (VMO 2001, bảng B). Giải HPT:  .
 2 x  y  x  y  2

46. (VMO 2002, bảng A). Giải PT: 4  3 10  3x  x  2 .


 x 3  x  x  z 2  2

 2
47. (VMO 2004, bảng A). Giải HPT:  y 3  y  z  x   30 .
 3 2
 z  z  x  y   16
3 2
 x  3xy  49
48. (VMO 2004, bảng B). Giải HPT:  2 2
.
 x  8 xy  y  8 y  17 x
 x 2  2 x  6.log  6  y   x
3

 2
49. (VMO 2006, bảng A). Giải HPT:  y  2 y  6.log 3  6  z   y .
 2
 z  2 z  6.log 3  6  x   z

 x 3  3x 2  2 x  5  y

50. (VMO 2006, bảng B). Giải HPT :  y 3  3 y 2  2 y  5  z .
 3 2
 z  3z  2 z  5  x
 12 
 1   x 2
 y  3 x 
51. (VMO 2007). Giải HPT:  .
 1  1  y  6
 
y  3x 

 x 2  y 3  29
52. (VMO 2008). Hãy xác định số nghiệm của HPT ( ẩn x, y ) sau:  .
log 3 x.log 3 y  1
 1 1 2
  
 1  2 x2 1  2 y2 1  2 xy
53. (VMO 2009). Giải HPT:  .
 2
 x 1  2 x   y 1  2 y   9

VĂN PHÚ QUỐC- Trường THPT chuyên Nguyễn Bỉnh Khiêm -Quảng Nam (0982 333 443) 5
Bài tập chuyên đề: PT, HPT, BPT, Hệ BPT Bồi dưỡng học sinh giỏi Toán Quốc gia
 x 4  y 4  240
54. (VMO 2010). Giải HPT:  3 2 .
 x  2 y  3  x  4 y   4  x  8 y 
3 2

55. Giải PT: x 2  3x 2  9  x 2  4 x 2  16  5 .


3 m x
56. Tìm giá trị lớn nhất của tham số m để BPT sau có nghiệm: m3  x  1  2
 4 m3 sin .
 x  1 2

57. Giải BPT: 2 x 2  2 x 30  2013. 30  4 x 2013  30. 2013 .


2 2 2
 x  y  z  2 xy  zx  zy  3
58. Giải HPT:  2 2
.
 x  y  yz  zx  2 xy  1
a 2  b 2  c 2  169

59. Với giá trị nào của u , v   thì HPT:   2u  v  .134
a  ua  vb   b  ub  vc   c  uc  va  
2 2 2 2 2 2 2 2 2

 4
có nghiệm nguyên dương a, b, c ?
2 2
60. Giải PT:  x  x9 .
x 1
12 x  8
61. Giải BPT: 2x  4  2 2  x  .
9 x 2  16
62. Giải PT: 13 x 2  x 4  9 x 2  x 4  16 .
x  3x 2
63. Giải PT:  2 x4  x3  7 x2  3x  3  2 .
2
 1 13 1 13
 y  x  16  x  y  x  x  16

 97
64. Giải HPT:  x 2  y 2  .
 36
x  0

y  0
 x  y  z  xyz
65. Giải HPT:  .
 x  y  1 z  1  y  x  1 z  1  z  x  1 y  1  0
2 2 2 2 2 2

2
66. Giải BPT: x 2   2x .
2
 x  6x  5
x2  y2  1

67. Trong các nghiệm thực  x, y, z , t  của HPT:  z 2  t 2  2 .

 xt  yz  2
Hãy tìm nghiệm sao cho tổng y  t nhỏ nhất.

VĂN PHÚ QUỐC- Trường THPT chuyên Nguyễn Bỉnh Khiêm -Quảng Nam (0982 333 443) 6
Bài tập chuyên đề: PT, HPT, BPT, Hệ BPT Bồi dưỡng học sinh giỏi Toán Quốc gia
68. Biết rằng PT: x 4  2 x 2  3 x  1  0 (1) có nghiệm dương x0 . Chứng minh rằng: 9
162  x0  2 và
x0 không phải là nghiệm của PT: 3  x  x x 1  4 .

69. Giải BPT: 2 x  1  x x 2  2   x  1 x 2  2 x  3  0 .


x  4  y2   8 y


70. Giải HPT:  y  4  z 2   8 z .

 z  4  x   8 x
2

71. Giải PT:  3  x  x  1  5  2 x   x 3  10 x 2  34 x  40 .


 x 2  3  2 x  y  3
72. Giải HPT: 
 y 2  3  2 y  x  3
2
 1   1 
2  sin x     sin x  7
 sin x   sin x 
74. Tìm m để BPT: 2
 2 vô nghiệm.
 1   1 
3  sin x   sin x   m  12
 sin x   sin x 
75. Giải PT: 11x 2  14 x  9  11x 2  2 x  3  17 x 2  2 x  3  2 2  x  2  .
 x y 2 2
  
 1  y  1  x 2  1  x  1  y 2  1  x 2 1  y 2 
76. Giải HPT:  .
 x y 1
  
 1  x
2
1  y2 1  x 1  y 2 
2

 
77. Giải PT: 3log 3 1  x  3 x  2 log 2 x .
 x  x 2  2 x  2  2012 y 1  1
78. Giải HPT: 
 y  y 2  2 y  2  2012 x 1  1
75 x  x 1  75 x1  2012 x  2012
79. Tìm m để hệ bất phương trình sau có nghiệm:  2
 x   m  2  x  2m  3  0
 2 2 2 xy
x  y  x  y  1
80. Giải HPT:  .
 x  y  x2  y

81. Giải HPT: 


 
log 2 x  y  5 log 32 x  y  2
.

2 2 2
 x  y  1  x  y  32

3
82. Giải PT:  8sin 3 x  1  162 sin x  27  0

VĂN PHÚ QUỐC- Trường THPT chuyên Nguyễn Bỉnh Khiêm -Quảng Nam (0982 333 443) 7
Bài tập chuyên đề: PT, HPT, BPT, Hệ BPT Bồi dưỡng học sinh giỏi Toán Quốc gia
 x  2 y  1  3
83. Giải HPT:  .
3 2
 x  4 x y  1  9 x  8 y  52  4 xy
 x4 y4  x2 y 2  x y
         2
84. Giải HPT:  y 4 x 4  y 2 x 2  y x .
 2 6
x  y  8x  6  0
1  xy  xy  x

85. Giải HPT:  1 1 .
  y y   3 y
x x x
 x3 y 1  y   x 2 y 2  2  y   xy 3  30

86. Giải HPT:  2 .
 2

 x y  x 1  y  y  y  11
 x  y  x  y  4x  y

87. Giải HPT:  .
 x 2  16  2  y  3 x
2 x 2  2  x  y  1  34  2 xy  x
88. Giải HPT:  .
2
2 y  2  x  y  1  34   xy  2 y
 1  4x
  2 x 1  log 32 y  1
89. Giải HPT:  log y 3 .

1  log 3 y  1  2   2
x

 2 7
 2 x  1 2 y  1  xy
2

90. Giải HPT:  2 .


 x  y  xy  7 x  6 y  14  0
2 2

 1 y 2 x
   2
91. Giải HPT:  x x y .
 
 y x2  1 1  3 x2 1
  
 6x
  2  3x  y  3 y
92. Giải HPT:  y
2 3x  3x  y  6 x  3 y  4

 x  y  z  t  12
 2 2 2 2
 x  y  z  t  50
93. Tìm các số  x; y; z; t  thỏa mãn HPT:  3 3 3 3
.
 x  y  z  t  252
 x 2t 2  y 2 z 2  2 xyzt

4x  6 y  9x  6 y
94. Tìm tất cả các nghiệm nguyên của PT:  313 .
x2  y 2
VĂN PHÚ QUỐC- Trường THPT chuyên Nguyễn Bỉnh Khiêm -Quảng Nam (0982 333 443) 8
Bài tập chuyên đề: PT, HPT, BPT, Hệ BPT Bồi dưỡng học sinh giỏi Toán Quốc gia
x 35
95. Giải BPT: x   .
x 2  1 12
96. Tìm tất cả các cặp số thực  x; y  thỏa mãn BPT: x  y  x   y 2  1  1 .


97. Xác định tất cả các nghiệm số nguyên của PT vô định: x 3  x 2 y  xy 2  y 3  8 x 2  xy  y 2  1 . 
4
98. Tìm tất cả các số tự nhiên x, y thỏa mãn PT:  x y   3361  11296320 .

99. Tìm m để PT sau có nghiệm duy nhất: x  1  x  2m x 1  x   2 4 x 1  x   m3 .


100. Giải PT: 16 x 6  16 x 5  20 x 4  20 x3  5 x 2  2 x  7  0 .
45
101. Giải PT: x 2  2 x  5  x 2  4 x  40  x 2  5 x  .
4
102. Cho a, b, c, d   \ 30; 4;14;10 là bốn tham số đôi một phân biệt và x, y , z , t là các ẩn số.
 x y z t
 a  30  a  4  a  14  a  10  1

 x  y  z  t 1
 b  30 b  4 b  14 b  10
Hãy giải HPT:  .
 x y z t
   1
 c  30 c  4 c  14 c  10
 x y z t
    1
 d  30 d  4 d  14 d  10
 x  y  z  kx  y  z   k 3  2k 2

103. Cho k  1 . Giải HPT:  x  y  z  x  ky  z   4k 2  8k .

 x  y  z  x  y  kz   4k  8
3
104. Tìm tất cả các cặp  x; y  với x, y   thỏa x y  1  2 y x  1  xy .
2
x  y  z  3
105. Tìm tất cả các bộ  x; y; z  với x, y, z là những số nguyên thỏa mãn HPT:  3 3 3
.
x  y  z  3
x  y  xy  16 xy

y  z  yz  25 yz
106. Tìm m để HPT:  có nghiệm x, y, z  0 .
z  x  zx  36 zx

 xy  yz  zx  m xyz

VĂN PHÚ QUỐC- Trường THPT chuyên Nguyễn Bỉnh Khiêm -Quảng Nam (0982 333 443) 9
Bài tập chuyên đề: PT, HPT, BPT, Hệ BPT Bồi dưỡng học sinh giỏi Toán Quốc gia
3 2
 1  2 x  x
  y
 2012 
 2 y3  y2
 1 
107. Giải HPT:   z.
 2012 
 2 z3  z2
  1 
x
 2012 

2 x 2012  y 2010  z 2010

108. Tìm tất cả các bộ ba số dương  x; y; z  thỏa HPT: 2 y 2012  z 2010  x 2010 .
 2012
2 z  x 2010  y 2010

x2  y2   y  x  z 

109. Giải HPT:  x 2  x  y  2 yz .
3 x 2  8 y 2  8 xy  8 yz  2 x  4 z  2

 x1  x2  x32012
 2012
 x2  x3  x4

110. Giải HPT: .................. .
 2012
 x2011  x2012  x1
 x  x  x 2012
 2012 1 2

 y 3  6 x 2  12 x  8  0

111. Giải HPT:  z 3  6 y 2  12 y  8  0 .
 3 2
 x  6 z  12 z  8  0
x2  y  1

112. Giải HPT:  y 2  z  1 ..
 2
z  x 1
 x y  2 y x  3 x 2 x  1
113. Giải HPT:  .
 y x  2 x y  3 y 2 y  1
2 x 3  3x 2  18  y 2  y

114. Giải HPT: 2 y 3  3 y 2  18  z 2  z .
 3 2 2
2 z  3 z  18  x  x
 x  y  z  2012

115. Giải HPT:  1 1 .
 3 x  2 y  x  2 y  2 z

VĂN PHÚ QUỐC- Trường THPT chuyên Nguyễn Bỉnh Khiêm -Quảng Nam (0982 333 443) 10
Bài tập chuyên đề: PT, HPT, BPT, Hệ BPT Bồi dưỡng học sinh giỏi Toán Quốc gia
 
6 x 2 x 3  6 x  5  x 2  2 x  6 x 3  4


116. Giải HPT:  2 2 .
x   1 2
 x y
 a2 1
 xa  ya  za 3
 1
117. Giải HPT:   a  1 .
 a2  1
 a  x  a  y  a  z  3
 a
 2013
 1  x1  1  x2  ...  1  x2012  2012
 2012
118. Giải HPT:  .
 1  x  1  x  ...  1  x 2011
 1 2 2012  2012
2012
1 1 1
x  y  z  2

1 1 1
119. Giải HPT:    .
y zx 3
1 1 1
  
z x y 4
 1 2 x y
4 x 
4 2
  x  y 
120. Giải HPT:  .
4 1 2 x y
 y   1
4 x  y 
 
121. Giải PT: 32 x 4  80 x3  50 x 2  4 x  3  4 x  1  0 .
 2 5
x  y  z    3

122. Giải HPT:  y 2  z  x   3

z2  x  y   1
 3
3 3 2
 x  8 y  4 xy  1
123. Giải HPT:  4 4
.
2 x  8 y  2 x  y  0
 x 1  y 2 1  z 2  x  yz


124. Giải HPT:  y 1  z 2 1  x 2  y  zx .
 2 2
 z 1  x 1  y  z  xy

VĂN PHÚ QUỐC- Trường THPT chuyên Nguyễn Bỉnh Khiêm -Quảng Nam (0982 333 443) 11
Bài tập chuyên đề: PT, HPT, BPT, Hệ BPT Bồi dưỡng học sinh giỏi Toán Quốc gia
125. Tìm số a lớn nhất để PT: x  ax  bx  cx  d  0  a; b; c; d    có 4 nghiệm x1 ; x2 ; x3 ; x4 thỏa
4 3 2

2012 2012 2012 2012


x12  x22  x32  x42  4 . Trong trường hợp đó hãy tính b; c; d .

126. Giải PT: x 3  6 x 2  12 x  7  3  x3  9 x 2  19 x  11 .


 12  2 x 2  y  4
127. Giải HPT:  .
2
 1  2 y  y  5  2 x
 2 1 2
 5 x  5 2  y  3
 x
128. Giải HPT:  .
1  x  1   2 y  3
 2  x 
ax n ax n bx n bx
129. Giải PT: n    ( với a, b là các số thực dương )
ax ax b x b x
x2  y2  2x  4 y  4  m  0
130. Định m để hệ sau có nghiệm:  .
3 x  4 y  5  0
 u  1 v  2   6

131. Giải HPT:  v  2  t  3  12 ; u , v, t    .

 t  3 u  1  8
132. Giải PT: x 6 y  y 6 x  5 x  5 y  12 xy  0 ; x, y    .
 x1  x2  2 x2 x3

...

133. Giải HPT:  x2012  x2013  2 x2013 x1 .

 x2013  x1  2 x1 x2
 x  2013, x  0
 1 2

 x 2  y  y 2  x  2
134. Giải HPT:  .
2 2
 x  y  x  y  2
 2 x12
 2
 x2
1  x 1
 2x2
 2  x3
135. Giải HPT: 1  x22 .
...

 2 xn2
 2
 x1
1  x n

136. Giải PT: x 3  3x 2  3 3 3x  5  1  3x .

VĂN PHÚ QUỐC- Trường THPT chuyên Nguyễn Bỉnh Khiêm -Quảng Nam (0982 333 443) 12
Bài tập chuyên đề: PT, HPT, BPT, Hệ BPT Bồi dưỡng học sinh giỏi Toán Quốc gia
x2  y2  2

137. Giải HPT:  z 2  2 z  x  y   8 .

 z  y  x   4 3
2 37
138. Giải PT: 4 x  1  9 x 2  26 x   0.
3 3
2

139. Tìm nghiệm nguyên dương của PT: y 2  x 2 x 2  x  1   x  1 
140. Giải PT: 2  x 2  4 2x  2  1.
  
2 x y 2  1  y y 2  9



  
141. Giải HPT: 2 y z 2  1  z z 2  9 . 
 2
  
2
2 z x  1  x x  9 
16 x 3 y 3  9 y 3   2 xy  y   4 xy 2  3
142. Giải HPT:  .
2 2 2 2
 4 x y  2 xy  y  3
143. Giải PT: 4 x  1  4 8 x  3  4 x 4  3x 2  5 x .
144. Tìm các cặp số nguyên  x; y  thỏa: y 2010  x 2010  x1340  x 670  2 .
 3 1
 x  x2  y  y 2  z  z2 
 3 .
145. Giải HPT: 
 2 3
 x  y  y  z  z  x  x  y  z   9

 x  y 2011
 
146. Tìm các bộ ba  x; y; z  nguyên dương sao cho:  y  z 2011 .
 2 2 2
 x  y  z 
( Ký hiệu , lần lượt là tập hợp các số hữu tỉ , tập hợp các số nguyên tố).
147. Giải PT: 4  x 2  4 x  1  x 2  y 2  2 y  3  4 x 4  16  5  y .
x  y  z  3

148. Biết HPT:  x 3  y 3  z 3  15 có một bộ nghiệm  x; y; z  thỏa x 2  y 2  z 2  10 .
 x 4  y 4  z 4  35

Hãy tính x 5  y 5  z 5 .
 1 
149. Giải PT: 4 x 4 y 2  16 x 2 y  9  x 2 y 2  2 y 2  2  x 2  2  với x  0 .
 x 

VĂN PHÚ QUỐC- Trường THPT chuyên Nguyễn Bỉnh Khiêm -Quảng Nam (0982 333 443) 13
Bài tập chuyên đề: PT, HPT, BPT, Hệ BPT Bồi dưỡng học sinh giỏi Toán Quốc gia
 2  x  3x  2 z   3  z
 3
 y  3 y 2  x 2  3x  2
150. Giải HPT:  .
2 2
 y  z  6z
z  3

 x  1 4 .  y  2 2 .z 3 .t 6  1024

151. Giải HPT: 4 x 2  z 3  16 y  t 6  8 x  16 .
 x  1; y  2; z  0; t  0

152. Tìm x; y; z   thỏa mãn PT: 3 x 2  6 y 2  2 z 2  3 y 2 z 2  18 x  6  0 .
  1  1  1
3  x    4  y    5  z  
153.(USAOP 1990). Giải HPT:   x  y  z .
 xy  yz  zx  1

 y 6  y 3  2 x 2  xy  x 2 y 3

154. Tìm mọi cặp số thực  x; y  thỏa hệ:  1 2
.
3 3 2
4 xy  y   2 x  1   2 x  y 
 2
2 x  y  3z  2
155. Tìm nghiệm nguyên của HPT:  3 3 3 .
 2 x  1   y  1  6   2  3 z 
 y  x  2012

156. Cho HPT:  z  2 y  2012 . Tìm nghiệm  x; y; z; w  sao cho x; y; z; w  0 và x có giá trị bé nhất.
w  3z  2012

157. Tìm nghiệm nguyên dương của PT:
 
x 3  2  3 y  z  x 2  12 y 2  8 yz  z 2 x  8 y 3  8 y 2 z  2 z 2 y  2 z 3  40  0 .
2012 x y 2011 2
158. Tìm nghiệm nguyên dương của PT:     .
x  y y  2011 4023 2012  x z
x  y  z  0

159. Giải HPT:  x 2  y 2  z 2  50 .
 x 7  y 7  y 7  350

160. Gọi x1 ; x2 ; x3 là ba nghiệm phân biệt của PT: x 3  x  1  0 . Tính tổng x111  x11 11
2  x3 .

 xi  xi 1  xi  2  xi  3 2012  2012 xi  4


161. Giải HPT:  với x11  x1; x12  x2 ; x13  x3 ; x14  x4 .
i  1,10
 x y 
162. Giải PT: 4 3 4 x  x 2 sin 2  2cos  x  y    13  4 cos2  x  y  .
 2 
163. Giải PT: 1  x1 1  x2  ... 1  x120   1  x1 1  x2  ... 1  x120   2120 trên đoạn  1;1 .

VĂN PHÚ QUỐC- Trường THPT chuyên Nguyễn Bỉnh Khiêm -Quảng Nam (0982 333 443) 14
Bài tập chuyên đề: PT, HPT, BPT, Hệ BPT Bồi dưỡng học sinh giỏi Toán Quốc gia
 x1  a1 x2  a2 x  an
   ...  n
164. Giải HPT:  b1 b2 bn .
 x  x  ...  x  a
 1 2 n

 x1  x2  x3  0

 x2  x3  x4  0
.......................
165. Giải HPT:  .
 x98  x99  x100  0
 x99  x100  x1  0

 x100  x2  x1  0
 x 2  xy  y 2  37

166. Giải HPT:  x 2  xz  z 2  28 .
 2 2
 y  yz  z  19
 x  y  z  13

167. Giải HPT:  x 2  y 2  z 2  61 .
 xy  xz  2 yz

x2  y 2  z2

168. Giải HPT:  xy  yz  zx  47 .
 zx zy 2
  
 xy  x  y  z

169. Giải HPT:  xz  2  x  y  z  .

 yz  3   x  y  z 
 x 2  y 2  z 2  50

170. Giải HPT:  xyz  60 .
 xy  4 x  5 y  20  0

x  y  z  1
171. Giải HPT:  4 4 4
.
 x  y  z  xyz
 x 3  y 3  x 2  y  z   xyz  14

172. Giải HPT:  y 3  z 3  y 2  z  x   xyz  21 .
 3 3 2
 z  x  z  x  y   xyz  7
1  x1 x2  0

1  x1 x3  0
173. Giải HPT: ................. .
1  x x  0
n 1 n

1  xn x1  0

VĂN PHÚ QUỐC- Trường THPT chuyên Nguyễn Bỉnh Khiêm -Quảng Nam (0982 333 443) 15
Bài tập chuyên đề: PT, HPT, BPT, Hệ BPT Bồi dưỡng học sinh giỏi Toán Quốc gia

 x1 x2  x1 x3  x2 x3  x4  2

x x  x x  x x  x  2
174. Giải HPT:  1 2 1 4 2 4 3 .
x x  x x
 1 3 1 4 3 4 2  x x  x  2
 x2 x3  x2 x4  x3 x4  x1  2

 
175. Giải PT:  sin x  2  sin 2 x  sin x  1  3 3 3sin x  1  1 .
17  3 x  5  x   3 y  14  4  y  0
176. Giải HPT:  .
2
2 2 x  y  5  3 3 x  2 y  11  x  6 x  13
 x sin   ycos  z   2  x  y  z   3 
177. Gọi  x; y; z  là nghiệm của HPT:  với    ;  .
2 x  3 y  13 z  7  2 

Hãy tính  x  y  .z .
178.(AIME 1984). Xác định x 2  y 2  z 2  t 2 biết  x; y; z; t  là nghiệm của HPT sau:
 x2 y2 z2 t2
 22  12 22  32 22  52 22  7 2  1
  
 2
 x y2 z2 t2
 42  12 42  32 42  52 42  7 2  1
  
 2 2
.
 x  y z2 t2
  1
 62  12 62  32 62  52 62  7 2
 2 2
 x  y z2 t2
  1
 82  12 82  32 82  52 82  7 2
 x1  x2  x3  2  x2  x4  x5  x2 

 x2  x3  x4 2  x3  x5  x1  x3 

 2
179. Giải HPT:  x3  x4  x5   x4  x1  x2  x4  .
 2
 x4  x5  x1   x5  x2  x3  x5 
 2
 x5  x1  x2   x1  x3  x4  x1 

 
1  x  1  x 2 1  x 4  1  y 7
 
180. Giải HPT:  .
 2
 4
1  y  1  y 1  y  1  x
7

 2


181. Giải PT: 2  2 x  1 x 2  2 x  2 x  2 x 1  2 .
2
182. Tìm tất cả các cặp số nguyên  x; y  thỏa mãn: x 3  y 3   x  y  .
183. (Korean Mathematics Competition 2000). Giải PT: 2 x  3x  4 x  6 x  9 x  1 .
2
184. Giải PT:  z  1 z  2  z  3  z  4  z  5  360 ; z   .

VĂN PHÚ QUỐC- Trường THPT chuyên Nguyễn Bỉnh Khiêm -Quảng Nam (0982 333 443) 16
Bài tập chuyên đề: PT, HPT, BPT, Hệ BPT Bồi dưỡng học sinh giỏi Toán Quốc gia
185. Giải PT: x  y 2 z 2  u  v  x y z u v .

186. Giải PT: x  4 x  16 x  ...  4 n x  3  x  1 (*)



187. Giải PT: x1  1  2 x2  4  ...  n xn  n 2   x1  x2  ...  xn  .
2
 y 2  u 2  v 2  w2  4 x  1
 2 2 2 2
x  u  v  w  4 y 1

188. Giải HPT:  x 2  y 2  v 2  w2  4u  1 .
 x 2  y 2  u 2  w2  4v  1

 x 2  y 2  u 2  v 2  4w  1


 x 4  8 y  4 x3  1  16 3
 
189. Giải HPT:  .
 y 4
 8 x  4 y 3
 
1  16 3 
 2 7
190. Giải HPT: 
 2

 2 x  1 2 y  1  xy
2
 .
 x  y  xy  7 x  6 y  14  0
2 2

1 2
191. Giải PT: x 4  x  1  4  5   x  2 .
x  x 1
192. Giải PT: 3 12 x 2  46 x  15  3 x3  5 x  1  2  x  1 .
 3 x  y  x  3 y  xy  14
193. Giải HPT:  .
2
 2
 x  y  x  14 xy  y  36 
 4 4 121x  122 y
x  y  4 xy

194. Giải HPT:  .
 x  14 x y  y 
4 2 2 4 122 x  121 y
 x2  y 2
 x 4  2 x 3 y  2 x 2 y 2  12 xy 3  8 y 4  1  0
195. Giải HPT:  3 4
.
2 x y  y  1
3
196. Giải PT: x2  1  x3  2  x  0 .
 y 2  x2 x 2  1
e  2
197. Giải HPT:  y 1 .
3log  x  2 y  6   2 log  x  y  2   1
 3 2

VĂN PHÚ QUỐC- Trường THPT chuyên Nguyễn Bỉnh Khiêm -Quảng Nam (0982 333 443) 17
Bài tập chuyên đề: PT, HPT, BPT, Hệ BPT Bồi dưỡng học sinh giỏi Toán Quốc gia
 x  y sin x
e  sin y


198. Giải HPT: 10 x 6  1  3 y 4  1 .  

 x, y    ; 5 
  
 4 
199. Giải PT: 13
3125.x14  1339 25. x 6  4 x 2 13 3125  4 6 5.13 3125  0 .
2 2 2 2 2 2
200. Giải PT: 252sin x  252cos x  212sin x  212cos x  42sin x  42cos x .
 y  2   3  x 3

 2 z  y  y  2   9  4 y
201. Giải HPT:  .
x2  z2  4x

z  0
 x  33  3  2 y

z2  4 y2  8 y
202. Giải HPT:  .
 2 z  x  x  3   5 x  16

z  0
2  yz  zx  xy   xyz

203. Giải HPT: 3  zx  xy  yz   xyz .

4  xy  yz  zx   xyz
 x  y  y  z   4 xy 2 z

204. Giải HPT:  y  z  z  x   4 yz 2 x .
 2
 z  x  x  y   4 zx y
9 x  4 y  17
205. (HSG tỉnh Thừa Thiến Huế năm 2009-2010). Giải HPT:  .
log
 17 3 x
 2y
log 5 
3 x
 2 y
 1
206. (HSG tỉnh Gia Lai năm 2009-2010).
1  cos2 x 2 cos 2 x 2009.cos x
Giải PT:   .
1  cos x  sin x  cos2 x 3cos x  sin x  2 cos 2 x  cos3 x  sin 3 x 2010
 y  x3

 
207. Giải HPT:  2 z  2  y y  1  4 y .
 2
 x  z  4 x  0


2 x 2  y x 2  1




208. Giải HPT: 3 y 3  z y 4  y 2  1 . 
 4 6
4
4 z  x z  z  z  1
2

VĂN PHÚ QUỐC- Trường THPT chuyên Nguyễn Bỉnh Khiêm -Quảng Nam (0982 333 443) 18
Bài tập chuyên đề: PT, HPT, BPT, Hệ BPT Bồi dưỡng học sinh giỏi Toán Quốc gia
1 1
1 1 2
 1 x  1  x
209. Giải PT: x ln  1    x  x3 ln  1  2   1 ( với x  0 ).
 x  x 
4x2  2
210. Giải PT: log 2012 6 2
 x6  3x 2  1 .
x  x 1
 1 1 1
 x  y  z 3 3

211. Giải HPT:  x  y  z  1 .
 7
 xy  yz  zx   2 xyz
 27
2013 2012
 x  y  xy  z 2  2 z 2
 2014
212. Giải HPT:  x 4  y 4  2 z 2 .
 z 1 x y
 x  y    z  2014 
log 2  x  1  x  y
213. Giải HPT:  1 x  y 1 x  y 1 x  y .
9 6 3  61 x  y  31 x  y  91 x  y
 2013
 3  x   y  2
 1
214. Giải HPT: log 3  log 1  y  2   log 1 9  4 y để tìm nghiệm  x0 ; y0 ; z0  thỏa z0  0 .
 2z  y 3 3
 2

2

log 2 x  z  2  log 2 x
3 2 4
 x y  m  y
215. Chứng minh rằng m   , HPT sau có nghiệm duy nhất  2 2 2 3
.
 x y  2 xy    y
 2x2
 2 y
 x 1
 3 y3
216. Giải HPT:  4 2
z .
 y  y  1
 4z4
 6 x
 z  z4  z2 1

 x3 y 2  3 y  3  3 y 2




217. Giải HPT:  y 3 z 2  3 z  3  3 z 2 . 
 3 2

 z x  3x  3  3x
2

VĂN PHÚ QUỐC- Trường THPT chuyên Nguyễn Bỉnh Khiêm -Quảng Nam (0982 333 443) 19
Bài tập chuyên đề: PT, HPT, BPT, Hệ BPT Bồi dưỡng học sinh giỏi Toán Quốc gia
 2013
 x1  x2   x3
x1 x2

 2013
 x2  x3  x x  x4
 2 3

218. Tìm nghiệm nguyên dương của HPT: ... .


 2013
 x2012  x2013   x1
 x2012 x2013
 2013
 x2013  x1   x2
 x2013 x1
219. Cho a  1 , hãy tìm tất cả các bộ ba số thực  x; y; z  sao cho y  1 thỏa PT:

2 8  4z  y2
log 2
a  xy   log a  x 3 3
y  xyz   0.
2
  y  y2  9 
220. Giải HPT: 

 x  y  x 2  xy  y 2  2  6 ln  
 x  x2  9  .

 
 3 2
 x  2 x  1  y
 y 1
x  1
 x
 z 1
221. Giải HPT:  y  1  .
 y

z  1 x 1
 z
2 x 3  3x 2  18  y 3  y

222. Giải HPT: 2 y 3  3 y 2  18  z 3  z .
  2 3
2 z  3 z  18  x  x
223. Giải PT: sin x.2012 sin 2 x  2012   cos x  1 2012 cos 2 x  2 cos x  2013  cos x  sin x  1 .
 x 2  2 x  22  y  y 2  2 y  1
224. Giải HPT:  .
2 2
 y  2 y  22  x  x  2 x  1
225. Tìm nghiệm nguyên dương của PT sau:
 x3  3x  x 2  1
  x2  4 
x3  3x  x 2  1  x 2  4 
8 3  3
 y 2  z 2  16 với điều kiện: 2  y  x  10
 2 2 
 
 
 x2 y 2 y2z2 z2 x2
 4 2   1
226. Giải HPT:  x y  x 2 y 2  1 y 4 z 2  y 2 z 2  1 z 4 x 2  z 2 x 2  1 .
 xyz  1

VĂN PHÚ QUỐC- Trường THPT chuyên Nguyễn Bỉnh Khiêm -Quảng Nam (0982 333 443) 20
Bài tập chuyên đề: PT, HPT, BPT, Hệ BPT Bồi dưỡng học sinh giỏi Toán Quốc gia
 x  1  x  3  x  5  y  1  y  3  y  5
227. Giải HPT:  .
2 2
 x  y  x  y  80
 4 2 697
x  y 
228. Giải HPT:  81 .
 x 2  y 2  xy  3 x  4 y  4  0

 x5  x4  2 x 2 y  2

229. Giải HPT:  y 5  y 4  2 y 2 z  2 .
 5 4 2
z  z  2z x  2
ln x  y  1

 y 1 x  1
230. Giải HPT: e  1 .
 x
 x  1  log y 2  1  1
 
 2

231. Giải HPT:


2


2
3x  2  92 y 1  2 2 y  x
  .
 2
3  x y  2
 2 x  y  29
4 x2 16  3 x  x 2  1  4 y 2 8 y  3 y  4  y 2  8 y  17

232. Giải HPT:  .
 
2 2
 2

 y x  1  4 x  3x  8  ln x  3 x  3  0
 6 1 4
8 x  xy  y  3x
233. Giải HPT:  2 .
x  4x y  y
3 2

log 2 x  1  log x  y  4 x 2  4 x  2  x  y 2  1  3 x 2  y 2  4 x  2 xy  1
 3  3   
234. Giải HPT:  .
log 3  2 x   4 x 2  4 x 2  1  1  2

235. Giải HPT: 


 
 x  2012  x 2 y  2012  y 2  2012
  .
 x 6 x  2 xy  1  4 xy  6 x  1
sin x  2 sin  x  y  z   0

236. (Russia). Giải HPT: sin y  2 sin  x  y  z   0 .

sin z  4sin  x  y  z   0
237. (Moscow). Giải HPT với n = 100
 x1  2 x2  2 x3  2 x4  2 x5  ...  2 xn  1

 x1  3 x2  4 x3  4 x4  4 x5  ...  4 xn  2

 x1  3 x2  5 x3  6 x4  6 x5  ...  6 xn  3 .
...

 x1  3 x2  5 x3  7 x4  9 x5  ...   2n  1 xn  n

VĂN PHÚ QUỐC- Trường THPT chuyên Nguyễn Bỉnh Khiêm -Quảng Nam (0982 333 443) 21
Bài tập chuyên đề: PT, HPT, BPT, Hệ BPT Bồi dưỡng học sinh giỏi Toán Quốc gia
 x1 x2 x3 ...x1962  1

 x1  x2 x3... x1962  1

238. (Moscow 1962). Cho HPT:  x1 x2  x3 x4 ...x1962  1 . Tìm giá trị của x25 .
...

 x1 x2 x3 ...x1961  x1962  1
 x1  x2  x32
 2
 x2  x3  x4

239. (Moscow). Tìm tất cả các nghiệm dương của hệ:  x3  x4  x52 .
 2
 x4  x5  x1
x  x  x2
 5 1 2

 x1  x2  x3  x4  x5  x6  x7  150
 x  x  43
 2 5
 x3  x6  52

240. (Moscow). Giải HPT:  x4  x7  53 .
 x x  52
 1 5
 x2 x3  480

 x3 x4  135
 
2

cos x  y  cos
2
 2 2
 a 2  2a  3
  x  1  y
241.(Moscow). Giải HPT:  .



cos x 2  y 2  cos  2 2
 2  a  1
  x  1  y
24 x 2  25 xy  73 x  25 y  35  0
242. (Hungari). Giải HPT:  2 2
.
 x  y  2 x  2 y  7  0
x2  y 2  u2  v2  4

 xu  yv   xv  yu
243. (Austria - Poland).Tìm bộ bốn số  x; y; u; v  thỏa mãn HPT:  .
 xyu  yuv  uvx  vxy  2
 xyuv  1
244. (England 1975). Chứng minh rằng với n   tùy ý, tồn tại đúng một bộ số  x1 ; x2 ;...; xn  thỏa PT:
2 2 1
2
1  x1    x1  x2   ...   xn1  xn   xn2 
.
n 1
245. (IMO 1966). Chứng minh rằng với mọi số tự nhiên n và mọi số thực x sao cho
1 1 1
 
sin 2n x  0 n  1;2; ... ta có: 
sin 2x sin 4x
 ...  n
sin 2 x
 cot x  cot 2n x .

246. (IMO 1966). Cho a1, a 2 , a 3 , a 4 là bốn số thực khác nhau cho trước.

VĂN PHÚ QUỐC- Trường THPT chuyên Nguyễn Bỉnh Khiêm -Quảng Nam (0982 333 443) 22
Bài tập chuyên đề: PT, HPT, BPT, Hệ BPT Bồi dưỡng học sinh giỏi Toán Quốc gia
a  a x  a  a x  a  a x  1
 1 2 2 1 3 3 1 4 4

 a  a1 x 1  a 2  a 3 x 3  a 2  a 4 x 4  1
Giải hệ phương trình sau:  2 .
a
 3  a 1
x 1
 a 3
 a 2
x 2
 a 3
 a 4
x 4
 1
a  a x  a  a x  a  a x  1
 4 1 1 4 2 2 4 3 3

247. (IMO 1968). Cho hệ phương trình với các ẩn số x 1 ; x 2 ;...; x n :


ax 2  bx  c  x
 12 1 2

ax 2  bx 2  c  x 3

.........................
ax 2  bx  c  x
 n 1 n 1 n

ax n2  bx n  c  x 1

trong đó a,b, c là những số thực và a  0 . Chứng minh rằng:
2

a) Hệ không có nghiệm thực nếu b  1  4ac  0 . 
2

b) Hệ có nghiệm duy nhất nếu b  1  4ac  0 . 
2

c) Hệ có hơn một nghiệm thực nếu b  1  4ac  0 . 
248. (IMO 1972). Tìm tất cả các nghiệm thực dương của hệ:

 1

 x2  x x x2  x x  0
3 5 2 3 5  
2
 2
 x 2  x 4x 1 x 3  x 4x 1  0
 2
 
 2
 x 3  x 5x 2 x 4  x 5x 2  0  
 2
 2
 x 4  x 1x 3 x 5  x 1x 3  0  
 x2  x x x2  x x  0
 5  2 4 1 2 4  
249. (IMO 1976). Cho số nguyên dương n và n  2m . Với mọi i, j thỏa mãn điều kiện:
1  i  n,1  j  m . Gọi aij là các số nhận giá trị 0;1; 1 . Xét hệ phương trình:
a11x 1  a12x 2  ...  a1m x m  0

a21x 1  x 22x 2  ....  a 2m x m  0

...........................................
an 1x 1  an 2x 2  ...  a nm x m  0

 
Chứng minh rằng hệ này có một nghiệm x 1; x 2 ;...; x m sao cho các thành phần x i , i  1, m là các số

nguyên không đồng thời bằng 0 và x i  m, i  1, m .


a x  a x  a x  0
 11 1 12 2 13 3
250. (IMO 1965). Cho hệ phương trình: a 21x 1  a22x 2  a 23x 3  0 với các hệ số thỏa mãn các điều
a x  a x  a x  0
 31 1 32 2 33 3

kiện sau đây:


(i). a11, a 22 , a 33 dương

VĂN PHÚ QUỐC- Trường THPT chuyên Nguyễn Bỉnh Khiêm -Quảng Nam (0982 333 443) 23
Bài tập chuyên đề: PT, HPT, BPT, Hệ BPT Bồi dưỡng học sinh giỏi Toán Quốc gia
(ii). Tất cả các hệ số còn lại đều âm
(iii). Trong mỗi phương trình, tổng các hệ số là dương.
Chứng minh rằng: nghiệm x 1  x 2  x 3  0 là nghiệm duy nhất của hệ phương trình ấy.
251. (IMO 1965). Tìm bốn số thực x 1 ; x 2 ; x 3 ; x 4 sao cho mỗi số cộng với tích các số còn lại đều bằng 2.

252. (IMO 1965). Tìm tất cả các giá trị x   0;2  sao cho: 2 cos x  1  sin 2x  1  sin 2x  2 .

 
253. (IMO 1963). Tìm tất cả các nghiệm x 1; x 2 ; x 3 ; x 4 ; x 5 của hệ phương trình:
x  x 2  yx 1
 5
x 1  x 3  yx 2

x 2  x 4  yx 3 .
x  x 5  yx 4
 3
x 4  x 1  yx 5
trong đó y là tham số.
254. (USA 1995). Giả sử a,b, c là các số phức và các nghiệm z của phương trình
x 3  ax 2  bx  c  0 thỏa z  1 . Chứng minh rằng phương trình x 3  a x 2  b x  c  0 có ba
nghiệm w thỏa w  1 .

255. (IMO 1963). Với những giá trị nào của p thì phương trình: x 2  p  2 x 2  1  x có nghiệm
thực? Hãy tìm các nghiệm đó.
x  y  z  a

256. (IMO 1961). Giải hệ phương trình: x 2  y 2  z 2  b trong đó a,b là những số cho trước.
xy  z 2

 
Các số a,b phải thỏa mãn điều kiện gì để các nghiệm x ; y; z của hệ là dương và khác nhau?
257. (IMO 1961). Giải phương trình: cosn x  sinn x  1 với n   .
2 2 2 2 2 2
258. Giải phương trình: 196sin x  16sin x  100sin x  100cos x  16cos x  196cos x .
259. (IMO 1973). Cho phương trình: x 4  ax 3  bx 2  ax  1  0 có ít nhất một nghiệm thực, với a,b
là các số thực. Tìm giá trị nhỏ nhất của a 2  b 2 .
 4x 2
 2
y
1  42x
 4y
260. (Canada 1996). Giải hệ phương trình:  2
z.
1  42y
 4z
1  4z 2  x


x 3y 2  1  y y 2  3

  

261. (IMO Shortlist 2007). Giải hệ phương trình: y 3z 2  1  z z 2  3 .   
 2
 2
z 3x  1  x x  3   
262. (Romania 2008). Xác định số nguyên x sao cho: log3 1  2   log 1  x  .
x
2

VĂN PHÚ QUỐC- Trường THPT chuyên Nguyễn Bỉnh Khiêm -Quảng Nam (0982 333 443) 24
Bài tập chuyên đề: PT, HPT, BPT, Hệ BPT Bồi dưỡng học sinh giỏi Toán Quốc gia
263. (THPT chuyên Năng khiếu - ĐHQG TP.HCM 2004). Tìm nghiệm x ; y; u; v của hệ PT sau: 
u  v  2

ux  vy  3
 2 2 .
ux  vy  5
ux 3  vy 3  9


3 x 2  y 2  z 2  1

264. (Poland 1997). Giải hệ phương trình sau: 

3
2 2 2 2 2 2
x y  y z  z x  xyz x  y  z  
265. (MO Treasures- Titu Andreescu, Bogdan Enescu). Tìm tất cả các số thực dương x , x 1, x 2 ,..., x n
x  x  x 
     
sao cho: log xx 1  log xx 2  ...  log xx n  log    log    ...  log  
x  x  x 
 1  2  n
 log x 1  log x 2  ...  log x n .
266. (MO Treasures- Titu Andreescu, Bogdan Enescu). Tìm tất cả các số thực a,b, c sao cho:
ax  by  cz  bx  cy  az  cx  ay  bz  x  y  z , x , y, z   .
267. (MO Treasures- Titu Andreescu, Bogdan Enescu). Tìm các số thực của hệ phương trình:
x  y  z  0
 3 3 3
x  y  z  18 .
x 7  y 7  z 7  2058

268. (MO Treasures- Titu Andreescu, Bogdan Enescu). Tìm tất cả các số thực của hệ phương trình:
a  b  8

ab  c  d  23

ad  bc  28
cd  12

3x 4  9x 3  17x 2  11x  8
269. (diendantoanhoc.net) . Giải phương trình: 2
3x  4x  5

 x  1 x2  3 
 1  1  1 1
270. (diendantoanhoc.net). Giải hệ phương trình: 6  x    3  y    2  z    xyz  .
 y  z  x xyz


 x 2 y  z  6



271. (diendantoanhoc.net). Giải hệ phương trình:  xy 1  2xy  9 
x 2  y 2  z 2  3

272. (diendantoanhoc.net). Giải phương trình sau:
x x x 1   
x x 1 x 2     
n x x  1 ... x  n  1 
x 
1 1.2

1.2.3
 ...  1 .   n!
 0.

VĂN PHÚ QUỐC- Trường THPT chuyên Nguyễn Bỉnh Khiêm -Quảng Nam (0982 333 443) 25
Bài tập chuyên đề: PT, HPT, BPT, Hệ BPT Bồi dưỡng học sinh giỏi Toán Quốc gia
 x  3  y3  6

273. (diendantoanhoc.net). Giải hệ phương trình sau:  y  2  z 3  25 .
 3
 z  1  x  1
 x  y  4 2012  3 4 xy

274. (diendantoanhoc.net). Giải hệ phương trình:  6 6 4
x  y  2012 x  y  4

275. (VMO 2013). Giải hệ phương trình với x , y   :
 1 1 20y
 sin2 x  2
 cos2 y  2

 sin x cos y x y
 .
 sin2 y  1 1 20x
 cos2 x  
 2
sin y cos2 x x y

x 1  x 2  ...  x 2012  1

 
x 1 x 2  x 3  ...  x 2012  1

  
276. (MSS 2013). Giải hệ phương trình:  x 1  x 2 x 3  x 4  ...  x 2012  1 .
..........................

 
 x 1  x 2  ...  x 2011 x 2012  1

277. (Gzeta, Romania). Giải hệ phương trình:
x 1  y 2  y 1  x 2  1

 2 2
y 1  x  z 1  y  2 ; x , y, z   .
 2 2
z 1  x  x 1  z  3
4
278. Giải phương trình: 1  2x  x 2  1  2x  x 2  2 x  1  2x 2

 4x  1 .

28 27
279. Giải phương trình: 2 4 27x 2  24x 
1 x 6 .
3 2
280. (Romania 2002, Titu Andreescu). Tìm tất cả các số thực a,b, c, d,e   2;2  thỏa hệ phương
a  b  c  d  e  0

trình: a 3  b 3  c 3  d 3  e 3  0 .
a 5  b 5  c 5  d 5  e 5  10

281. (Crux-Canada 1999). Giải phương trình nghiệm nguyên sau:
1 3
 2
    
x  y y  z z  x  x  y  z  1  xyz .
282. (Crux-Canada 1999). Cho n  1 là một số nguyên dương lẻ. Giả sử rằng các số nguyên dương
x 1, x 2 ,..., x n thỏa mãn hệ phương trình:

VĂN PHÚ QUỐC- Trường THPT chuyên Nguyễn Bỉnh Khiêm -Quảng Nam (0982 333 443) 26
Bài tập chuyên đề: PT, HPT, BPT, Hệ BPT Bồi dưỡng học sinh giỏi Toán Quốc gia
 x  x 2  2 x  x  1  n2
 2  
1  2 
1
 x  x 2  2 x  x  1  n 2
 3
 
2  3 2

 ............................................
 2
   
 x 1  x n  2 x 1  x n  1  n
2

Chứng minh rằng: hoặc x 1  x n hoặc tồn tại j với 1  j  n  1 sao cho x j  x j 1 .
283. (Iranian MO 1995, Crux - Canada 2002). Cho a,b, c là các số thực dương. Tìm tất cả các số
x  y  z  a  b  c
thực x , y, z thỏa hệ phương trình sau:  .
2 2 2
4xyz  a x  b y  c z  abc 
284. (Austrian-Polish MO 1993, Crux-Canada 1997). Xác định tất cả các nghiệm thực x , y, z của hệ
x 3  y  3x  4
 3
phương trình: 2y  z  6y  6 .
3z 3  x  9z  8

285. (Crux-Canada 1996). Xác định tất cả các số thực x , y, z  1 thỏa mãn phương trình:
3 3 3
x y z  
x 1 y 1 z 1
 
2 x 2  y 2  z 2 . 
286. (Crux-Canada 1996). Tìm tất cả các số thực x và y thỏa hệ phương trình:
2x 2 y  2x y 2  128

 x  y  2 2
x y 1  (y  1)x

287.(Saigon 2012) Giải hệ phương trình :  2x 2  9x  6 ; x, y  
2
 4 x  18x  20   y 1
 2x 2  9x  8
x 3  3xy 2  6xy  3x  49

288. (Haiphong 2012). Giải hệ phương trình :  2 2
x  8xy  y  10y  25x  9
289.(Nghean 2012)
 2 2 8 xy
x  y   16
 x y
a) Giải hệ phương trình 
2
x 2x x3 x2 y
8y 3   
 3y 4 2
3
b) Giải phương trình : 2 4 x2  x  1  2 x  3 2 x2  x3  9 x2  4 x  4
 x( y3  x3 )  7
290.(Ninhbinh 2012). Giải hệ phương trình 
 x4  x3 y  9 y  y3 x  x2 y2  9 x
291. (Hatinh 2012). Cho các số thực a, b, c thỏa a  b  c  0 . Chứng minh rằng phương trình sau có
ab
nghiệm duy nhất : x  a  x  b   0.
xc

VĂN PHÚ QUỐC- Trường THPT chuyên Nguyễn Bỉnh Khiêm -Quảng Nam (0982 333 443) 27
Bài tập chuyên đề: PT, HPT, BPT, Hệ BPT Bồi dưỡng học sinh giỏi Toán Quốc gia
292. (Binhdinh 2012)
a) Giải phương trình x  2  4  x  2 x2  5 x  1

 x3  y3  3 y2  3 x  2  0
b) Giải hệ phương trình : 
 x2  1  x2  3 2 y  y2  2  0
293. (Hanoi 2012)
a) Giải phương trình : x4  1  x2  1
 x2  y2  2 xy  1
b) Giải hệ phương trình : 
 x5  y3  1  0
294. (Namdinh 2012).
 x  y  2  m2  1
a) Tìm m để hệ sau có nghiệm duy nhất: 
 y  x  2  m2  1
 4 x  y  2 x  y  2
b) Giải hệ phương trình : 
 2 x  y  x  y  1
295. Giải các hệ phương trình sau:
 2 2 2 2 2 2
 
 x y  3z  y z  3x  2 6  
 2 2
 2 2 2 2
a)  
 y z  3x  z x  3y 2 6  
 2 2
 z 2  x 2  3  y  x 2  y2  3  z  2 6
   

 2 2 2 2 2 2
 
 x  y 6 z  y  z 6 x  4 6  
 2 2
 2 2 2 2
b)  
 y  z 6 x  z  x 6 y  4 6  
 2 2
 z 2  x  6  y2  x 2  y  6  z 2  4 6
   

x 6  y 8  z 10  1

296. Giải hệ bất phương trình sau:  2009
x  y 2011  z 2013  1

x  2  2  2  y
297. Giải hệ phương trình:  ; x, y   .
y  2  2  2  x

 2x  2 4 6  x  y 2  2 2

298. Giải hệ phương trình:  4 ; x, y   .
 2x  2 6  x  8  2  2 2y
2

299. Giải phương trình: 3 1  2 sin 4 2x  2 40  4 sin6 x  cos6x  1  5 11 .
  
300. (Mathematical and youth 8/2012). Giải hệ phương trình sau:

VĂN PHÚ QUỐC- Trường THPT chuyên Nguyễn Bỉnh Khiêm -Quảng Nam (0982 333 443) 28
Bài tập chuyên đề: PT, HPT, BPT, Hệ BPT Bồi dưỡng học sinh giỏi Toán Quốc gia


 xy  x  y

 
xy  2  x  y  y
; x, y  
  
 x  1 y  xy  x 1  x  4


301. (Mathematical and Youth 1/2013). Giải hệ phương trình:
 5x 2  2xy  2y 2  2x 2  2xy  5y 2  3 x  y

 .
 
 2x  y  1  2 7x  12y  8  2xy  y  5
3

302. (BMO 2013, Round 1). Giải hệ phương trình sau:


x 2  4y  7  0
 2
y  6z  14  0 .
z 2  2x  7  0

303. (Vinhphuc 2012-2013). Giải hệ phương trình sau:
 2 8
x  3x  2   5y  1
 y
 2 8
y  3y  2   5z  1 ; x , y, z   .
 z
8
z 2  3z  2   5x  1

 x

 
304. (British MO 1996). Tìm tất cả các nghiệm thực dương a;b; c;d của hệ phương trình:
a  b  c  d  12
 .
abcd  27  ab  ac  ad  bc  bd  cd
x2 x 2  3x  18
305. (Ireland 1999). Tìm tất cả các số thực 1  x  0 thỏa mãn: 2
 2
.
x  1  x 1   x 1 
306. (Hanoi). Giải hệ phương trình:
 x 2 2  y 3 2   y 3 x z 2

      
2
x  5x  9z  7y  15  3yz
8x 2  18y 2  18xy  18yz  84x  72y  24z  176

 
307. (ĐH KHTN Hanoi). Tìm nghiệm x ; y; z của hệ phương trình:
2z  x  y   1  x 2
 y2
 2 
2
y  z  1  2xy  2zx  2yz
 2
 2

y 3x  1  2x x  1  
308. (THPT chuyên Quang Trung, Binhphuoc 2010-2011). Giải hệ phương trình sau:
hệ phương trình:

VĂN PHÚ QUỐC- Trường THPT chuyên Nguyễn Bỉnh Khiêm -Quảng Nam (0982 333 443) 29
Bài tập chuyên đề: PT, HPT, BPT, Hệ BPT Bồi dưỡng học sinh giỏi Toán Quốc gia
2009x  2010y  x  y 2
  
 2
 
2010y  2011z  y  z ; x , y, z  
 2
2011z  2009x  z  x

 
309. (Ukraina 1997). Tìm tất cả các nghiệm thực của hệ phương trình sau:
x 1  x 2  ...  x 1997  1997
 4 4 4 3 3 3 .
x
 1  x 2
 ...  x 1997
 x 1
 x 2
 ...  x 1997

310. (Moscow). Giải hệ phương trình:


x 1  x 2  x 3  6

x 2  x 3  x 4  9
x  x  x  3
 3 4 5
x
 4  x 5
 x 6
 3

x 5  x 6  x 7  9
x 6  x 7  x 8  6

x 7  x 8  x 1  2
x  x  x  2
 8 1 2

311. (Moscow). Cho 100 số a1; a 2 ;...; a100 thỏa mãn các điều kiện:
a  3a  2a  0
 1 2 3
a
 2  3 a 3
 2a 4
0

......................... .
a  3a  2a  0
 99 100 1

a100  3a1  2a 2  0
Chứng minh rằng: tất cả các số đã cho đều bằng nhau.
x 12  x 22  ...  x 100
2
 10000
312. (Moscow). Cho 100 số dương x 1 ; x 2 ;...; x 100 thỏa điều kiện:  .
x 1  x 2  ...  x 100  300
Chứng minh rằng trong chúng ta tìm được 3 số mà tổng lớn hơn 100.

313. (Moscow). Giải hệ phương trình:


10x  3x  4x  x  x  0
 1 2 3 4 5

 211x  2 x 3
 2 x 4
 3 x 5
 x 6
0
15x  4x  5x  4x  x  0
 3 4 5 6 7
2
 1x  x 2
 3x 3
 12 x 4
 3 x 5
 x6  x 7  0 .
6x  5x  3x  x  17x  x  0
 1 2 3 4 5 6

3x 1  2x 2  3x 3  4x 4  x 5  16x 6  2x 7  0
4x  8x  x  x  3x  19x  0
 1 2 3 4 5 7

VĂN PHÚ QUỐC- Trường THPT chuyên Nguyễn Bỉnh Khiêm -Quảng Nam (0982 333 443) 30
Bài tập chuyên đề: PT, HPT, BPT, Hệ BPT Bồi dưỡng học sinh giỏi Toán Quốc gia
 3x 4y 2z
   1
314. Giải hệ phương trình:  x  1 y  1 z  1 ; x , y, z  0 .
89.x 3 .y 4 .z 2  1

315. Tìm nghiệm nguyên dương của hệ phương trình:
 x 2  xy  y 2  y 2  yz  z 2  x 2  xz  z 2
 .
x  y  z  11
316. (Indian MO 2011). Tìm tất cả các số thực x , y, z  0 thỏa mãn hệ phương trình:

  
 x 2  xy  y 2 y 2  yz  z 2 z 2  zx  x 2  xyz

 4 .

 2 2 4 4 2 2
 4 4 2 2
 x  x y  y y  y z  z z  z x  x  x y z
4

3 3 3

317. (IMO 1979). Tìm tất cả các số thực a sao cho tồn tại 5 số thực không âm x 1 ; x 2 ; x 3 ; x 4 ; x 5 thỏa
5 5 5
3
mãn các hệ thức:  kx k
a ; k x k
 a2 ; k x 5
k
 a3 .
k 1 k 1 k 1

318. (Vietnam TST 1993). Với a là số thực và a  1 , giải hệ phương trình:


x 2  ax  1
 12 2

x
 2  ax 3
1

................. .
x 2  ax 1
 999 1000
2
x 1000  ax1  1

319. (Turkey 1995). Tìm nghiệm thực của hệ phương trình:
x 2  2ax  b 2  x
 12 1
2
2

x 2  2ax 2  b  x 3

........................... ; ở đây b  a  0 là các số thực.
x 2  2ax  b 2  x
 n 1 n 1 n
x n2  2ax n  b 2  x 1

320. (Mathematical and Youth 8/2010). Cho các số thực x ; y; z thỏa mãn hệ phương trình:
 2 y2
x  xy   25
 2 3
y 2
 z 9 .
3
 2 2
z  xz  x  16


Tính giá trị của biểu thức P  xy  2yz  3xz .
321. (Mathematical and Youth 10/2010). Cho các số thực dương x ; y; z thỏa mãn hệ phương trình:
x 2  y 2  9
 2 2
y  z  16 .
y 2  xz

VĂN PHÚ QUỐC- Trường THPT chuyên Nguyễn Bỉnh Khiêm -Quảng Nam (0982 333 443) 31
Bài tập chuyên đề: PT, HPT, BPT, Hệ BPT Bồi dưỡng học sinh giỏi Toán Quốc gia
Tính giá trị của biểu thức G  xy  yz .
322. (Mathematical and Youth 10/2010).
 2 29
x  y 
 4
Cho các số thực x ; y; z với y  0 thỏa mãn hệ phương trình: y 2  z  2
 2
y  x  1. 2  z

Tính giá trị của biểu thức: H  y  x 1  2z . 
323. (Mathematical and Youth 10/2010). Cho các số thực dương x ; y; z thỏa mãn:
 2 2
y  z  50
 y2
2
x  xy   169
 2
2
x 2  xz  z  144
 2
Tính giá trị của biểu thức: K  xy  yz  zx .
324. (Mathematical and Youth 8/2010). Giải hệ phương trình:
3 x  3 y  3 3 x  y

 3
  .
2 4
4x  6x  4x  1  15y
325. (Mathematical and Youth 12/2010). Giải hệ phương trình:
 3x  y
  x2
 x  3y
 3y  z
  y2 .
 y  3z
 3z  x  z 2
 z  3x

326. (Mathematical and Youth 3/2011). Giải phương trình:
x  y  z  xyz  2  xy  yz  zx  2 . 
 x 2  y2 x 2  xy  y 2
  x y
327. (Mathematical and Youth 9/2011). Giải hệ phương trình:  2 3
x 2xy  5x  3  4xy  5x  3

x  y  z  1

328. Giải hệ phương trình:  x y z x  y y  z .
   
y z x y  z x  y  1

x 19  y 5  1890z  z 2013

329. Giải hệ phương trình: y 19  z 5  1890x  x 2013 .
z 19  x 5  1890y  y 2013

VĂN PHÚ QUỐC- Trường THPT chuyên Nguyễn Bỉnh Khiêm -Quảng Nam (0982 333 443) 32
Bài tập chuyên đề: PT, HPT, BPT, Hệ BPT Bồi dưỡng học sinh giỏi Toán Quốc gia
 1 1 1 8
 x  y  z    
 x y z 3
 1 1 1 118
330. Giải hệ phương trình: x  y  z     .
 x y z 9
x x  y y  z z  1  1  1  728
 27
 x x y y z z

  
x 3  2y 3  2 x 2  3y 2  3 x  2y  1  0


  
331. Giải hệ phương trình: y 3  2z 3  2 y 2  3z 2  3 y  2z  1  0 . 
 3 3 2 2

z  2x  2 z  3x  3 z  2x  1  0   
332. (Trại hè Hùng Vương 2010). Giải phương trình:
4
 x  1 4 1
2 
 x2  3   2
 3x 2  2x  5 .
x 2
3   x  1
x2  x  2 x2  x
333. (Trại hè Hùng Vương 2010). Giải phương trình:   x2  1
2 2
1  x  x  2 1  x  x  4

334. (Mathematical and Youth 4/332). Giải phương trình: x  x  x  x 5 5 .


x  y 4  y

 
 
335. (Trại hè Hùng Vương 2010). Cho x ; y; z là nghiệm của hệ phương trình: y  z 4  z .  
z  x 4  x
  
x 2005  y 2005  z 2005  a 2005

336. (OLP Hùng Vương, lần thứ 3). Cho a  0 . Giải hệ phương trình: x 2006  y 2006  z 2006  a 2006 .
x 2007  y 2007  z 2007  a 2007

337. (Balkan MO 1984). Cho a,b, c là các số thực dương. Tìm tất cả các nghiệm thực x ; y; z của hệ  
ax  by  x  y 2
  
 2

phương trình: by  cz  y  z . 
 2

cz  ax  z  x


338. (Junior Balkan MO 2008). Tìm tất cả các số thực a,b, c thỏa:
a  b  c  d  20
 .
ab  ac  ad  bc  bd  cd  150

339. (IberoAmerican 1989). Tìm tất cả các bộ ba số thực x ; y; z thỏa hệ phương trình: 
x  y  z  1
 2 2 2
x  y  z  1
x 3  y 3  z 3  1

VĂN PHÚ QUỐC- Trường THPT chuyên Nguyễn Bỉnh Khiêm -Quảng Nam (0982 333 443) 33
Bài tập chuyên đề: PT, HPT, BPT, Hệ BPT Bồi dưỡng học sinh giỏi Toán Quốc gia
y z x
340. (CentroAmerican 2011). Tìm tất cả các số thực dương x , y, z thỏa : x   y   z   2 .
z x y
Tìm giá trị dương của x  y  z
x 5  y  y 5
 5 5
y  z  z
341. (Baltic Way 1993). Giải hệ phương trình:  5 5
.
z  t  t
t 5  x  x 5

 
342. (Baltic Way 1995). Tìm tất cả các bộ ba x ; y; z nguyên dương thỏa hệ phương trình:


x 2  2 y  z  .
 6 6 6
x  y  z  31 y  z
2 2
 
343. (Baltic Way 1999). Tìm tất cả các số thực a,b, c, d thỏa hệ phương trình:
abc  ab  bc  ca  a  b  c  1

bcd  bc  cd  db  b  c  d  9

cda  cd  da  ac  c  d  a  9
dab  da  ab  bd  d  a  b  9

344. (Baltic Way 2000). Tìm tất cả các nghiệm thực của hệ phương trình:
x  y  z  t  5

xy  yz  zt  tx  4
 .
xyz  yzt  ztx  txy  3
xyzt  1

345. (Baltic Way 2000). Xác định tất cả các số thực dương x , y thỏa phương trình:
1 1
x y 
x y

  4  2 2x  1  2y  1 . 
346. (Baltic Way 2002). Giải hệ phương trình:
a 3  3ab 2  3ac 2  6abc  1
 3 2 2
b  3ba  3bc  6abc  1 ; a,b, c   .
c 3  3ca 2  3cb 2  6abc  1

 
347. (Baltic Way 2010). Tìm tất cả các bộ số a;b; c;d thỏa hệ phương trình:
 b  c  d 2010  3a
  
 a  c  d 2010  3b


 
2010

 a b d  3c 
 2010

 a  b  c  3d 
348. (Mediterranean MO 2010). Cho các số thực a,b, c, d .

VĂN PHÚ QUỐC- Trường THPT chuyên Nguyễn Bỉnh Khiêm -Quảng Nam (0982 333 443) 34
Bài tập chuyên đề: PT, HPT, BPT, Hệ BPT Bồi dưỡng học sinh giỏi Toán Quốc gia
x 2  yz  zu  yu  a
 2
y  zu  ux  xz  b
 
Tìm nghiệm x , y, z , u của hệ phương trình:  2
z  ux  xy  yu  c
.

u 2  xy  yz  zx  d

 
349. (Middle European MO 2012). Tìm tất cả các bộ ba số thực x ; y; z thỏa hệ phương trình:
2x 3  1  3zx
 3
2y  1  3xy ; x , y, z   .
2z 3  1  3yz

350. (Canada National Olympiad 2003). Tìm tất cả các nghiệm thực dương thỏa
x 3  y 3  z 3  x  y  z
 2 2 2
.
x  y  z  xyz
 
351. (Canada National Olympiad 2004). Tìm tất cả các bộ ba số thực x , y, z thỏa hệ phương trình:
xy  z  x  y

xz  y  x  z .
yz  x  y  z

 
352. (Germany Bundeswettbewerb Mathematik 2003). Tìm tất cả các bộ ba x , y, z nguyên thỏa hệ
x 3  4x 2  16x  60  y

phương trình: y 3  4y 2  16y  60  z .
z 3  4z 2  16z  60  x

353. (Polish MO 1999). Tìm tất cả các số nguyên n  2 của hệ phương trình:
x 12  x 22  50  16x 1  12x 2
 2 2
x 2  x 3  50  16x 2  12x 3
x 2  x 2  50  16x  12x
 3 4 3 4

......................................
x 2  x 2  50  16x  12x
 n2 1 2 n n 1 n

x
 n  x 1
 50  16 x n
 12x 1

36x 2y  60x 2  25y  0



354. Giải hệ phương trình: 36y 2z  60y 2  25z  0 ; x , y, z   .
36z 2x  60z 2  25x  0

355. (Hungary 1988). Tìm tất cả các nghiệm thực x , y, z của hệ phương trình:



 
x  y  z 2  8z  14 x  y  2  1
.
2x  5y  xy  z  3

VĂN PHÚ QUỐC- Trường THPT chuyên Nguyễn Bỉnh Khiêm -Quảng Nam (0982 333 443) 35
Bài tập số học Bồi dưỡng HSG Toán Quốc gia

1. LÝ THUYẾT CHIA HẾT


a 2  b2
1. (Hanoi 2002). Cho a,b    sao cho a 2  b 2 ab . Tính A  .
ab
 
2. (Kvant, Russia). Cho a1, a2,..., an  1;1 , n  * và thỏa mãn
a1a 2  a 2a 3  ...  ana1  0 .
Chứng minh rằng: n  4 .
3. (IMO 2001). Cho a  b  c  d là những số nguyên dương và giả sử
 
ac  bd  b  d  a  c b  d  a  c . 
Chứng minh rằng: ab  cd không phải là số nguyên tố.
a 1 b 1
4. (Spanish MO 1996). Cho a,b   sao cho:  .
b a
Chứng minh rằng ước chung lớn nhất của a và b không vượt quá a  b .
 
5. (Russia 2001). Cho a,b   và a  b thỏa: ab a  b chia hết cho a 2  ab  b 2 .

Chứng minh rằng: a  b  3 ab .


6. (HMMT 2002). Hãy tính 2002  2;20022  2;20023  2;... . 
..
7. (K u rschák 1953). Cho n, d    sao cho d 2n 2 . Chứng minh rằng n 2  d không thể
là số chính phương.
8. (IMO 1960). Tìm tất cả các số có ba chữ số chia hết cho 11 sao cho thương số trong
phép chia số đó cho 11 bằng tổng bình phương các chữ số.
x 4  1 y4  1
 
9. (VMO 2007). Cho x , y  \ 1 sao cho
y 1

x 1
  . Chứng minh rằng:

x 4y 44  1 x  1 .
10. (IMO 1967). Cho k , m, n  * sao cho m  k  1 là số nguyên tố lớn hơn n  1 . Đặt
n

 
cs  s s  1 . Chứng minh rằng: c1c2 ...cn  c
i 1
m i
 ck .
11. (Romania 1999). Cho a,b, c là những số nguyên khác không, a  c sao cho
a a 2  b2
 2 2
. Chứng minh rằng: a 2  b 2  c 2 không phải là số nguyên tố.
c c b
12. (Romania 1999). Cho p, q, r là các số nguyên tố và n là một số nguyên dương sao
cho p n  q n  r 2 . Chứng minh rằng: n  1 .
13. (IMO 1969). Chứng minh rằng tồn tại vô số số nguyên dương a thỏa z  n 4  a
không phải là số nguyên tố với mọi số nguyên dương n .
14. (IMO 1984). Tìm hai số nguyên dương a,b thỏa mãn hai điều kiện:
7
 
(i). ab a  b không chia hết cho 7 ; 
(ii). a  b   a 7  b 7 chia hết cho 7 7 .

Văn Phú Quốc , GV. Trường THPT chuyên Nguyễn Bỉnh Khiêm DĐ: 0982 333 443
Bài tập số học Bồi dưỡng HSG Toán Quốc gia

15. (Vietnam 1983).Cho n  , n  3 . Chứng minh rằng nếu 2n  10a  b 0  b  10  


thì tích ab chia hết cho 6.
16.(Romania 2003). Cho n là một số nguyên dương chẵn và cho a,b là hai số nguyên
dương nguyên tố cùng nhau. Tìm a và b nếu a  b a n  b n .
5 6
17. Chứng minh rằng 34  45 là một tích của hai số nguyên mà mỗi số này lớn hơn
102002 .
18. Cho p  2 là một số lẻ và n là một số nguyên dương. Chứng minh rằng
n n pn

p 1p  2p  ...  p  1 
 
19. Tìm tất cả các cặp số nguyên m, n sao cho những số
A  n  2mn  3m  2, B  2n 2  3mn  m 2  2,C  3n 2  mn  2m 2  1
2 2

có một ước chung lớn hơn 1


20. Cho M là một tập hợp tất cả các giá trị ước chung lớn nhất của d của các số
A  2n  3m  13, B  3n  5m  1,C  6n  8m  1 với m, n là những số nguyên
dương. Chứng minh rằng M là một tập hợp của tất cả các ước của một số nguyên k .
21. (St. Petersburg City MO 1998). Chứng minh rằng với mỗi số tự nhiên n , giữa n 2
2

và n  1  có thể tìm được ba số tự nhiên a,b, c sao cho a 2  b 2 chia hết cho c .

  
22. (India 1998). Tìm tất cả các bộ ba x , y, n nguyên dương sao cho x , n  1  1 và 
x n  1  y n 1 .
23. (APMO 1999). Tìm số nguyên lớn nhất chia hết cho tất cả các số nguyên dương bé
hơn căn bậc ba của nó.
24. (Russia 2001). Tìm số nguyên dương lẻ n  1 sao cho a và b là hai ước nguyên tố
cùng nhau bất kì của n thì a  b  1 cũng là ước của n .
25. (Vietnam 1979). Cho m, n là hai số nguyên tố cùng nhau. Hãy tìm m  n, m 2  n 2  
26. Cho a,b, c, d là các số nguyên dương thỏa mãn ab  cd . Chứng minh rằng
A  a n  b n  c n  d n là hợp số với mọi n nguyên dương.
27. Có tồn tại hay không 2013 số nguyên dương a1, a 2,..., a 2013 sao cho các số
a12  a22 , a12  a22  a32,..., a12  a22  ...  a2013
2
đều là số chính phương?
5125  1
30. (Vietnam TST 1992). Chứng minh rằng: N  không phải là số nguyên tố.
525  1
31. Cho x , y, p là các số nguyên và p  1 sao cho mỗi số x 2012 và y 2013 đều chia hết cho
p . Chứng minh rằng: A  1  x  y không chia hết cho p .
32. Tìm tất cả các số nguyên tố p sao cho tổng tất cả các ước số tự nhiên của p 4 là một
số chính phương.
n
33. Chứng minh rằng một số nguyên tố tùy ý có dạng 22  1, n   không thể biểu diễn
được dưới dạng hiệu các lũy thừa bậc 5 của hai số tự nhiên.

Văn Phú Quốc , GV. Trường THPT chuyên Nguyễn Bỉnh Khiêm DĐ: 0982 333 443
Bài tập số học Bồi dưỡng HSG Toán Quốc gia

34. Chứng minh rằng với m  2 , giữa m và m ! có ít nhất một số nguyên tố. Từ đó suy
ra rằng có vô số số nguyên tố.
35. Có tồn tại một số tự nhiên n có thể viết dưới dạng: n  x ! y ! với x , y    và
x  y bằng hai cách khác nhau hay không?
2
36. Chứng tỏ rằng số 444444  303030 3 không thể biểu diễn dưới dạng x  y 3   với
x, y   .
n 1

C  
2 n k
37. Chứng minh rằng với mọi số tự nhiên n  2 ta có: 2n 1
.C n1k chia hết cho 4n 1 .
k 0

1 1
38. Tìm tất cả các số hữu tỉ dương x , y sao cho x  y và  là các số nguyên.
x y
39. Cho p là số tự nhiên lẻ và các số nguyên a,b, c, d,e thỏa mãn các điều kiện:
a  b  c  d  e ; a 2  b 2  c 2  d 2 đều chia hết cho p . Chứng minh rằng số
a 5  b 5  c 5  d 5  e 5  5abcde cũng chia hết cho p .
40. Cho 5 số nguyên phân biệt tùy ý a1, a 2, a 3 , a 4, a 5 . Xét tích sau đây:

       
P  a1  a 2 a1  a 3 a1  a 4 a1  a 5 a 2  a 3 a2  a 4 a 2  a 5 a 3  a 4 a 3  a 5 a 4  a 5   
Chứng minh rằng: P chia hết cho 288.
41. Giả sử phương trình x 2003  ax 2  bx  c  0 với các hệ số nguyên a,b, c có ba
nghiệm nguyên x 1, x 2 , x 3 . Chứng minh rằng: a,b, c có ba nghiệm nguyên

a  b  c  1x 1   
 x 2 x 2  x 3 x 3  x 1 chia hết cho 2003 .
42. Cho ba số nguyên dương khác nhau x , y, z . Chứng minh rằng:
5 5 5
x  y   y  z   z  x   
chia hết cho 5 x  y y  z z  x  
43. Giả sử rằng số nguyên tố p có thể được viết thành hiệu hai lập phương của hai số
nguyên dương khác nhau. Chứng minh rằng khi đem 4p chia cho 3, nếu loại bỏ phần dư
đi thì sẽ nhận được số là bình phương của một số nguyên lẻ.
44. (Komal - Hungary C.640, 2001). Tìm tất cả các số tự nhiên thỏa mãn tính chất sau:
Nếu thay đổi hai chữ số cuối cùng của bình phương số tự nhiên đó, ta nhận được bình
phương của số tự nhiên liền sau nó.
4
45. (Komal - Hungary C.676, 2002). Tìm số nguyên a,b sao cho a 4  a  b    b 4 là
số chính phương.
46. (Komal - Hungary B.3525, 2002). Chứng minh rằng trong dãy 1; 31; 331; 3331;... có
vô hạn các hợp số.
47. (Komal - Hungary B.3474, 2001).
2

Xác định chữ số thứ 73 tính từ bên phải của số : 111...1 .
 

112
48. (Komal - Hungary A.243, 2000). Xác định tất cả các số nguyên tố p, q sao cho:

Văn Phú Quốc , GV. Trường THPT chuyên Nguyễn Bỉnh Khiêm DĐ: 0982 333 443
Bài tập số học Bồi dưỡng HSG Toán Quốc gia

p 2n 1  1 q 3  1
 với n  1, n   .
p 1 q 1
49.(Komal - Hungary A.244, 2000). Tìm tất cả các số nguyên dương n sao cho
n  a 2  b 2 với a,b là các số nguyên dương nguyên tố cùng nhau và ab chia hết cho
mọi số nguyên tố bé hơn hoặc bằng n .
50. (The Winter Mathematical Competitions in Bulgaria 1995). Chứng minh rằng với
mọi số nguyên dương n , mệnh đề sau đây đúng: “ Số 7 là một ước số của 3n  n 3 nếu
chỉ nếu 7 là một ước số của 3n n 3  1 ”.
51. (The Winter Mathematical Competitions in Bulgaria 1995).
1 2x  2
Cho A  và B  . Tìm tất cả các giá trị nguyên của x sao
4x 2  4x  1 x 2  2x  1
2A  B
cho C  là một số nguyên.
3
Giải
52. (The Winter Mathematical Competitions in Bulgaria 1996). Cho số nguyên dương
1
n và số thực  sao cho cos  . Tìm tất cả các số nguyên dương k sao cho cos k
n
là một số nguyên.
53. (The Winter Mathematical Competitions in Bulgaria 1997). Tìm tổng tất cả các số
tự nhiên dạng: a1a2 ...a2n sao cho:
(i). Không có một chữ số ai nào bằng 0.
(ii). Tổng a1a2  a 3a 4  ...  a 2n 1a 2n là một số chẵn.
54.(The Winter Mathematical Competitions in Bulgaria 1999). Tìm số tự nhiên n
2
nhỏ nhất sao cho tổng bình phương các ước số của nó ( kể cả 1 và n ) bằng n  3 .  
55.(The Winter Mathematical Competitions in Bulgaria 2000).
Chứng minh rằng chữ số hàng trăm của số 21999  22000  22001 là một số chẵn.
2 22 2 3 2n p
56. Viết tổng    ...  thành phân số tối giản . Chứng minh rằng: p 8
1 2 3 n q
với mọi n  4 .
 
57. (APMO 1999). Xác định tất cả các cặp số nguyên a;b sao cho hai số a 2  4b và
b 2  4a đều là những số chính phương.
58. (Singapore 1995-1996). Với mỗi số nguyên dương k . Hãy chứng minh rằng tồn tại
một số chính phương có dạng: n 2k  7 , trong đó n là số nguyên dương.
59. (Singapore 1996-1997). Ta viết bốn số nguyên a 0 ,b0 , c0, d0 trên một đường tròn theo
chiều kim đồng hồ. Bước đầu tiên ta thay a 0 ,b0 , c0, d0 bằng các số a1,b1, c1, d1 với
a1  a 0  b0 , b1  b0  c0 , c1  c0  d 0 , d1  d0  a 0 . Ở bước tiếp theo ta thay a1,b1, c1, d1
bằng các số a 2 , b2 , c2 , d2 sao cho a 2  a1  b1, b2  b1  c1, c2  c1  d1, d2  d1  a1 .

Văn Phú Quốc , GV. Trường THPT chuyên Nguyễn Bỉnh Khiêm DĐ: 0982 333 443
Bài tập số học Bồi dưỡng HSG Toán Quốc gia

Tổng quát ở bước thứ k , ta nhận được các số ak , bk , ck , dk trên đường tròn sao cho:
ak  ak 1  bk 1,bk  bk 1  ck 1, ck  ck 1  dk 1, dk  dk 1  a k 1 .
Sau 1997 lần thay thế như trên ta đặt a  a1997 , b  b1997 , c  c1997 , d  d1997 .
Hỏi tất cả các số bc  ad , ac  bd , ab  cd có đồng thời là các số nguyên tố hay
không? Chứng minh cho câu trả lời.
60. (Hungary 2000). Tìm tất cả các số nguyên tố p sao cho với số p đó tồn tại các số
nguyên dương n, x , y thỏa p n  x 3  y 3 .
61. (China 2001). Cho 7 số nguyên tố khác nhau có thể được viết thành:
a;b; c; a  b  c;a  b  c; a  b  c; a  b  c
trong đó, hai trong ba số a;b; c có tổng bằng 800. Gọi d là khoảng cách giữa số lớn nhất
và số nhỏ nhất trong 7 số nguyên tố. Hỏi giá trị lớn nhất có thể có của d ?

62. Cho đa thức P x có các hệ số nguyên, biết rằng tồn tại số nguyên dương c sao cho

  
không có số nào trong các số: P 1 , P 2 ,..., P c chia hết cho c . Chứng minh rằng với


mọi số nguyên b , ta có: P b  0 .
63.Chứng minh rằng nếu x 2  2 y là một số chính phương với x, y    thì x 2  y là
tổng của hai số chính phương.
64. Tìm số nguyên tố p sao cho 2p  1 là lập phương của một số tự nhiên.
65. Chứng minh rằng đa thức:

P x  x 9999  x 8888  x 7777  x 6666  x 4444  x 3333  x 2222  x 1111  1

 
chia hết cho đa thức: Q x  x 9  x 8  x 7  x 6  x 5  x 4  x 3  x 2  x  1 .
66. (Bulgaria MO 1995, Round 3 ).Tìm tất cả các số nguyên dương x , y sao cho
x 2  y2
là số nguyên và nó là ước của 1995.
x y
67. (Bulgaria MO 1995, Round 4 ). Giả sử x ; y là các số thực khác nhau sao cho có bốn
x n  yn
số nguyên dương n liên tiếp nhau để là một số nguyên. Chứng minh rằng:
x y
x n  yn
là một số nguyên với mọi số nguyên dương n .
x y
68. (Bulgaria MO 1996, Round 3). Chứng minh rằng với mọi số nguyên n  3 , tồn tại
các số nguyên dương lẻ x n và yn sao cho: 7x n2  yn2  2n .
69.(Bulgaria MO 1998,Round 4). Gọi m, n là các số tự nhiên sao cho
n

A
m  3  1
là số nguyên. Chứng minh rằng A là một số nguyên lẻ.
3m
70. (diendantoanhoc.net). Cho a;b; c; d    thỏa mãn ac  bd a 2  b 2 .

Văn Phú Quốc , GV. Trường THPT chuyên Nguyễn Bỉnh Khiêm DĐ: 0982 333 443
Bài tập số học Bồi dưỡng HSG Toán Quốc gia


Chứng minh rằng: a 2  b 2 ; c 2  d 2  1 . 
71. Tìm n nguyên dương sao cho:  3 1    3 2   ...   3 n3  2n  4   7225 .
 
1999
72. Tính:  45  1999
  , trong đó: a là ký hiệu phần nguyên của số a.
  
 
n
73. Chứng minh rằng:  2  3  là số lẻ với mọi số tự nhiên n .
 
 
2. QUAN HỆ ĐỒNG DƯ
1. (Komal-Hungary C.691, 2002). Cho hình lập phương có ba cạnh là các số nguyên.
Tổng thể tích của chúng bằng 2002 đơn vị được không?
2. (Komal - Hungary A.271, 2001).
C pk
Chứng minh rằng với mọi số nguyên tố p  5 , số  chia hết cho p .
2p p
0 k 
3
3. (Komal - Hungary A.271, 2001).
 
Tìm các cặp số a;b sao cho a,b    và a 2  ab  b 2 là bội số của 7 5 .
p
4. Cho p là số nguyên tố lẻ. Chứng minh: C C k
p
k
pk   2 p  1 chia hết cho p 2 .
k 0

5. Chứng minh rằng số: 222555  555222 chia hết cho 7.


6. Tìm bộ số nguyên dương  m; n  sao cho p  m2  n 2 là số nguyên tố và m3  n3  4
chia hết cho p
n
2 k 1
7. Chứng minh rằng: C 2 n 1 23 k không chia hết cho 5 với mọi n là số tự nhiên.
k 0

8. Cho p là số nguyên tố khác 2 và a, b là hai số tự nhiên lẻ sao cho a  b chia hết cho
p và a  b chia hết cho p  1 . Chứng minh rằng: ab  b a chia hết cho 2 p .
9. Cho số nguyên tố p  3 và m, n là hai số nguyên tố cùng nhau sao cho
m 1 1 1
 2  2  ...  2
. Chứng minh rằng m chia hết cho p .
n 1 2  p  1
10. (Baltic 2001). Cho a là số nguyên dương lẻ, m và n là hai số nguyên dương phân
 n
biệt. Chứng minh rằng: a 2  22 ; a 2  22
n m m

  1.
 n  1 !
11. Cho n  5 là số tự nhiên. Chứng minh rằng: 
 
 chia hết cho n  1 . Biết a 
 n   
 
là ký hiệu phần nguyên của a .
12. Tồn tại hay không một số nguyên x sao cho x 2  x  1 2003 ?

Văn Phú Quốc , GV. Trường THPT chuyên Nguyễn Bỉnh Khiêm DĐ: 0982 333 443
Bài tập số học Bồi dưỡng HSG Toán Quốc gia

13. Tìm tất cả các số nguyên dương n sao cho n là ước số của 312  1 nhưng n không
là ước số của 3i  1 với mọi i  1,2, 3,...,11 . Có bao nhiêu số n chẵn và bao nhiêu số n
lẻ?
14. Chứng minh rằng với mọi số nguyên tố p , tồn tại vô số số nguyên dương n thỏa
mãn: 2n  n  p .
2
15. Tìm tất cả các số nguyên tố p sao cho 5p  1  0 mod p 2 .  
16. Cho a;b là các số nguyên dương nguyên tố cùng nhau. Chứng minh rằng tồn tại các
số nguyên dương m, n sao cho: a m  b n  1ab .
17. Cho n  5 là số nguyên dương lẻ và có các số nguyên tố là p1, p2 ,..., pk . Chứng minh
 n 
rằng 2 
 1 có ước số nguyên tố không thuộc tập p1; p2 ;...; pk . 
18. (IMO 1978). Cho m và n là những số tự nhiên với n  m  1 . Trong cách viết thập
phân ba chữ số cuối cùng của 1978m theo thứ tự bằng ba chữ số cuối cùng của 1978n .
Tìm các số m và n sao cho tổng m  n nhỏ nhất.
19. (VMO 2001 A). Cho số nguyên dương n và hai số nguyên tố cùng nhau a,b lớn
n n
hơn 1 . Giả sử p, q là hai ước lẻ lớn hơn 1 của a 6  b 6 . Hãy tìm số dư trong phép chia
n n
p 6  q 6 cho 6.12n .
20. (VMO 2008). Đặt m  20072008 . Hỏi có tất cả bao nhiêu số tự nhiên n mà n  m và
  
n 2n  1 5n  2 chia hết cho m .
21. Chứng minh rằng không tồn tại một dãy vô hạn tăng các số nguyên tố pn thỏa mãn  
pn 1  2pn  1 với mọi n  1 .


22. Cho p là số nguyên tố, r1; r2 ;...; rp và s ;s ;...; s  là các hệ thặng dư đầy đủ
1 2 p

modulo p . Hỏi tập hợp r s ; r s ;...; r s  có phải là một hệ thặng dư đầy đủ modulo p
1 1 2 2 p p

không?
 
23. Chứng minh rằng nếu p là một số nguyên tố thì p  2 ! 1 p nhưng nếu p  5 thì

 p  2  ! 1 không phải là một lũy thừa của p .


n
24. Chứng minh rằng: 52013  7 chia hết cho 12 với mọi số tự nhiên n .
25. (Nordic 1998). Cho n là một số nguyên dương. Chứng minh rằng các số
k  0;1;2;...;n thỏa mãn C nk lẻ là một lũy thừa của 2.
 
2n  1
26. (Korea 1999). Tìm tất cả các số nguyên dương n sao cho 2n  1 3 và là ước
3
số của một số nguyên có dạng 4m 2  1 .

Văn Phú Quốc , GV. Trường THPT chuyên Nguyễn Bỉnh Khiêm DĐ: 0982 333 443
Bài tập số học Bồi dưỡng HSG Toán Quốc gia

 
27. (Bulgaria). Cho an  7  4 3 , n  1 . Chứng minh rằng trong biểu diễn thập phân

của an có ít nhất n chữ số 9 nằm sau dấu phẩy.


28. (Ukraine 1976). Tìm bốn chữ số cuối cùng của số:
 
a  19761976  19741974 19761975  19741973 . 
10000
1995
29. Tìm ba chữ số tận cùng của số M  19931994 .

3. DÃY SỐ VÀ SỐ HỌC

u1  1

1. Cho dãy số un  xác định bởi :   3 3 .
u n 1   1   u n  2  , n  1
  n n
Chứng minh rằng tất cả các số hạng của dãy số đều là số nguyên.
u0  u1  u2  1

2. Cho dãy số un  xác định như sau :  un 3 un  2 .
u  n! , n  
 n 1 un
Chứng minh rằng tất cả các số hạng của dãy đều nguyên.
4n  4n 2  1
3. Cho dãy số un  xác định bởi : un  , n  1.
2n  1  2n  1
Chứng minh rằng : u1  u2  ...  u40 là một số nguyên.

4. Cho dãy số vô hạn un  xác định như sau : un  3 n 2  n  7 , n  * . 
Chứng minh rằng không có phần tử nào của dãy là lập phương của một số nguyên.
5. Cho dãy số un  được xác định như sau:
n n
 3 5   3 5 
un        2, n  
 2   2 
Chứng minh rằng u2k 1 , k   là một số chính phương.
6. Chứng minh rằng mọi số hạng của dãy un  xác định bởi:
u0  1
 2
đều nguyên.
u
 n1  2u n  3u n  2
7. Cho m    . Dãy un  được xác định theo công thức:
u1  1
 .
un1  5un  m.un  8 n  
2 *

Tìm m để dãy un  là một dãy số nguyên.

Văn Phú Quốc , GV. Trường THPT chuyên Nguyễn Bỉnh Khiêm DĐ: 0982 333 443
Bài tập số học Bồi dưỡng HSG Toán Quốc gia

u1  2
8. Dãy số un  được xác định theo công thức:  3 2
.
un  3un1  2n  9n  9n  3 n  2
Chứng minh rằng với mọi số nguyên tố p thì dãy các tổng tương ứng:
u1  u2  ...  u p 1 đều chia hết cho p .
9. (Putnam 1999). Dãy số nguyên un  được xác định như sau:
u1  1, u2  2, u3  24

 6un21un 3  8un 1un22 .
u
 n  n  4
 un  2 u n  3
Chứng minh rằng un luôn là bội của n .
u  7
10. Dãy số un  được xác định như sau:  1 .
un 1  4un  5un 1  1975 n  2
Chứng minh rằng u1996 chia hết cho 1997.
u1  2012, u2  2013
11. Cho dãy số un  xác định như sau:  .
un 1  un  un  1  2 n  2
Chứng minh rằng số A   u12  1 u22  1 ...  u2013
2
 1  1 là số chính phương.
1 1 1
12. Cho dãy số un  xác định bởi: un    .
 n 3 5
cos cos n cos n

7 7 7
Chứng minh rằng un luôn là số nguyên và chia hết cho 8.
13. (VMO 1997). Cho dãy số nguyên un  được xác định như sau:
u0  1, u1  45, un 2  45un1  7un n   .
a) Tính số các ước dương của un21  unun  2 theo n .
b) Chứng minh rằng 1997un2  7 n 1.4 là số chính phương với mỗi n .
u1  u2  0

14. Cho dãy số un  xác định bởi:  un2  2un  un 1  2 .
u
 n1  n  2
 un1  1
Chứng minh rằng un nguyên với mọi n   .
15. Cho k  * và dãy số un  thỏa mãn điều kiện: u0  1; u1  1; un 2  4un1  un .
Chứng minh rằng: un  3k khi và chỉ khi n 3k .
u1  5, u2  7
16. Cho dãy số un  được xác định bởi công thức:  3 2008
.
un 1  un  6un 1  3.2
Chứng minh rằng un  không thể biểu diễn được dưới dạng tổng lũy thừa bậc 6 của ba số
nguyên dương.

Văn Phú Quốc , GV. Trường THPT chuyên Nguyễn Bỉnh Khiêm DĐ: 0982 333 443
Bài tập số học Bồi dưỡng HSG Toán Quốc gia

17 . Gọi  là nghiệm dương của phương trình: t 2  2014t  1  0 và dãy un  được xác định
như sau: u0  1, un 1    un  n   , ở đây  x  là ký hiệu phần nguyên của số thực x . Tìm số
dư u2014 khi chia cho 2014.
2 2 2
18. Cho dãy số un  xác định như sau: un  n 2   n  1   n  2    n  3 n  * .
Tìm tất cả các số hạng của dãy chia hết cho 10.
19. Dãy số un  xác định như sau: un   n 2  n   . Chứng minh rằng có vô số số
hạng của dãy là số chính phương.
20. Cho dãy số un  , vn  xác định bởi: u0  3, u1  2, un  3un1  4vn1 , vn  2un 1  3vn1
với mọi n  * . Chứng minh rằng dãy wn  xác định bởi wn  1  4un2 vn2 không chứa các
số nguyên tố.
21. Cho dãy số un  xác định bởi: u0  u1  3 , un1  7un  un1 n  1 . Chứng minh
rằng un  2 là một số chính phương n  1 .
22. (VMO 2011).
Cho dãy số nguyên un  xác định bởi: u0  1, u1  1, un  6un 1  5un 2 n  2 .
Chứng minh rằng: u2012  2010 chia hết cho 2011 .
u0  1

23. Cho dãy số un  xác định bởi:  7un  45un2  36 .
un1  , n  
 2
Chứng minh rằng:
a) un là số nguyên dương n   .
b) unun 1  1 là số chính phương n   .
n n

24. Cho dãy số un  xác định như sau: un 


2  3  2  3 , n   .
2 3
a) Chứng minh rằng: un là số nguyên n   .
b) Tìm tất cả các số hạng của dãy chia hết cho 3
25. Cho f  x     x  thỏa f  0   f 1  1, x0  , xn 1  f  xn  n   . Chứng minh
rằng với mọi m, n  , m  n ta luôn có:  xm , xn   1 .
2n
26. Cho k  * , xét f n   2k   1 n  * . Chứng minh rằng f n đôi một nguyên tố
cùng nhau.
27. (Journal of Mathematical youth ).
Cho dãy số an  với a1  a2  1 và an  2  an 1  an n  1 . Tìm tất cả các cặp số nguyên
dương  a; b  , a  b thỏa an  2na n chia hết cho b n  1 .
28. Cho dãy số xn  4n 2  38n n   . Tìm lim
n 
 x  ,  x là phần lẻ của x .
n

29. Xét dãy số un  , vn  xác định bởi:

Văn Phú Quốc , GV. Trường THPT chuyên Nguyễn Bỉnh Khiêm DĐ: 0982 333 443
Bài tập số học Bồi dưỡng HSG Toán Quốc gia

u1  3, v1  2 và un 1  3un  4vn , vn 1  2un  3vn n  1 .


Chứng minh rằng: un  2vn   un  vn  2  n  1 .
30. Cho dãy Fibonacci: u1  u2  1, un  2  un 1  un . Chứng minh rằng với mọi n  6 giữa
un và un 1 có một số chính phương.

an 2
31.(IMO 1994). Cho dãy số an  : a0  1994; an 1  ( n  N ) . Chứng minh rằng khi
1  an
0  n  998 thì  an   1994  n

32. (Bulgaria 1999). Cho dãy các số nguyên an  thoả mãn

(n  1)an1  (n  1)an  2(n  1), n  * .

Biết a1999 chia hết cho 2000. Tìm số n nhỏ nhất sao cho an chia hết cho 2000  n  2
33. (Bulgaria 1978). Cho dãy số an  xác định như sau:

a12  a22  a an21  a


a1 , a2 ,   ; an  0 n   và an  2 
*
n  *
a1a2 an
( a là số cho trước ). Chứng minh rằng dãy số đã cho gồm toàn số nguyên.
34. ( Vietnam TST 1982). Cho a  * và an  là dãy số xác định bởi:

a0  0, a1  1
 .
an1  2an   a  1 an 1
Với p0 là số nguyên cố định lớn hơn 2. Hãy tìm giá trị bé nhất của a sao cho hai khẳng
định sau đúng:
a) Nếu p là số nguyên tố, p  p0 thì a p  p .

b) Nếu p là số nguyên tố, p  p0 thì a p không chia hết cho p .

an1  3an  an 1 n
35. Cho dãy số an  xác định bởi a0  0, a1  1 và   1 với n  * .
2
Chứng minh an là số chính phương với mọi n  0 .

36. Cho dãy số an  xác định bởi: a0  0, a1  1, a2  2, a3  6 và


an  4  2an 3  an  2  2an1  an với mọi n  * . Chứng minh rằng: an chia hết cho n ,
n  1 .
37. Cho k  , k  1 . Xét dãy số an  xác định bởi:

Văn Phú Quốc , GV. Trường THPT chuyên Nguyễn Bỉnh Khiêm DĐ: 0982 333 443
Bài tập số học Bồi dưỡng HSG Toán Quốc gia

a  4, a  a  k 2  2 2
 0

1 2   .
an1  an an 1  2  an  an 1   an  2  8 n  2

Chứng minh rằng: 2  an là số chính phương với mọi n  0 .

38. Chứng minh rằng với mọi số nguyên dương n  4 thì Fn  1 không là số nguyên tố.

39. Cho dãy số nguyên  xn  xác định bởi: x0  3; x1  11 và xn 2  2 xn 1  7 xn n   .

Tìm tất cả các số nguyên dương lẻ a sao cho với mọi m, n nguyên dương tồn tại k
nguyên dương mà xnk  a chia hết cho 2m .

1 a2 1
40. Cho dãy số nguyên an  xác định bởi: a1  2; a2  7 và   an 1  n  , n  2 .
2 an1 2
Chứng minh rằng: an là số lẻ với mọi n  2 .

41. Cho a, b là hai số thực khác 0. Xét dãy số un  xác định
u  0; u1  1
bởi:  0 .
un 2  aun 1  bun n  0
Chứng minh rằng nếu có bốn số hạng liên tiếp của dãy un  là số nguyên thì mọi số hạng
của dãy đều là số nguyên.
42. Cho dãy số nguyên an  thỏa mãn điều kiện: 0  an  7 an1  10an 2  9, n  0 .

Chứng minh rằng tồn tại số tự nhiên n0 sao cho với mọi n  n0 thì an  0 .

a  29, a1  105, a2  381


43. Cho dãy số an  xác định bởi  0 .
an3  3an  2  2an 1  an n  0
Chứng minh rằng với mỗi số nguyên dương m luôn tồn tại số tự nhiên n sao cho các số
an , an1  1, an  2  2 đều chia hết cho m .

44. (BMO 2000, Round 3). Cho dãy số an  xác định bởi : a1  43 , a2  142 và
an 1  3an  an 1 với mọi n  2 .

a) Chứng minh rằng :  an , an 1   1 với mọi n  1 .

b) Với mọi số tự nhiên m , tồn tại số tự nhiên n sao cho an  1 và an 1  1 đều chia hết
cho m

Văn Phú Quốc , GV. Trường THPT chuyên Nguyễn Bỉnh Khiêm DĐ: 0982 333 443
Bài tập số học Bồi dưỡng HSG Toán Quốc gia

45. (Slovenia 1999). Cho dãy các số thực a1 , a2 ,..., thỏa mãn điều kiện:
a  an1 an 1
a1  2, a2  500, a3  2000 và n 2  n  2 . Chứng minh rằng tất cả các số
an1  an 1 an1
hạng của dãy đều là số dương và a2000 chia hết cho 22000 .
1 1 1
46. (WMSETS 1999 - 2000). Giả sử   ...   1 với các số a1 , a2 ,..., am nguyên
a1 a2 am
dương và đôi một khác nhau. Nếu các số lớn nhất trong các số ai là 2 p với p là số
nguyên tố nào đó, hãy tìm tập a1 , a2 ,..., am  .

47. (WMSETS 2000 - 2001). Cho a1  14, a2  144 và an  1444...4 với n số 4. Tìm tất
cả các số nguyên dương n sao cho an là số chính phương.
48. (USAMTS 2000 - 2001). Xét dãy số thực s0 , s1 , s2 ,... thỏa mãn tính chất:

(i). si s j  si  j  si  j với mọi số nguyên không i, j sao cho i  j

(ii). si  si 12 với mọi số nguyên không âm i ; s0  s1  s2  0 .

Tìm ba số s0 , s1 , s2 .

49. (Putnam 1990). Xét dãy số 2,3, 6,14, 40,152, 784... với số hạng đầu là a0  2 và số
hạng tổng quát an   n  4  an 1  4nan  2   4n  8 an 3 với mọi n  3 .

Chứng minh rằng mỗi số hạng của dãy trên có thể được viết thành tổng của các số hạng
tương ứng của hai dãy quen thuộc.
50. (Putnam 1991). Với mọi số nguyên dương n ta định nghĩa d  n   n  m 2 trong đó
m là số nguyên lớn nhất sao cho m2  n . Cho số nguyên dương b0 , ta thiết lập dãy
b0 , b1 ,..., bk ,... với bk 1  bk  d  bk  . Những giá trị b0 như thế nào thì dãy trên có bi là
hằng số với i đủ lớn?
51. (Putnam 1997). Cho dãy số an  xác định bởi a1  1, an1  2an . Chứng minh rằng
an  an 1  mod n  với n  2 .
52. (Czech and Slovak Republic 2000). Chứng minh rằng tồn tại một dãy các số tự
   
nhiên tăng dần an với mọi k  0 là dãy k  an chỉ chứa một số hữu hạn số nguyên
tố.
53. (The Winter mathematical competitions in Bulgaria 1997).
Cho    là hai nghiệm của phương trình x 2  px  q  0 . Với mọi số tự nhiên n ta
n  n
ký hiệu: an  .
 
a) Tìm p và q sao cho với mọi số tự nhiên n đẳng thức sau đây đúng:

Văn Phú Quốc , GV. Trường THPT chuyên Nguyễn Bỉnh Khiêm DĐ: 0982 333 443
Bài tập số học Bồi dưỡng HSG Toán Quốc gia

n
 
an 1an 2  anan  3  1 .
b) Chứng minh rằng với p, q nói trên ta có: an  an 1  an 2 với mọi số tự nhiên.
c) Chứng minh rằng với mọi số tự nhiên n , an là số nguyên và nếu n  3 thì an là số
chẵn.
54. (The Winter mathematical competitions in Bulgaria 1998).
 
Cho dãy số nguyên am mà khi viết trong hệ thập phân, các số hạng của dãy bao gồm

 
các chữ số chẵn a1  2, a 2  4, a 3  6,... . Tìm tất cả các số nguyên m sao cho
am  12m .
 
55. Lập dãy ak như sau: a 0  1;a1  1 với k  1 thì ak 1  2ak  qak 1 trong đó q là số
nguyên tố cố định nào đó. Tìm q sao trong dãy có số hạng a 3n nhận giá trị 1 .
56. Dãy an thỏa mãn tính chất sau: a1  5; 7 , an 1  5an ; 7an
      với n  1;2;...
Tìm mọi giá trị có thể của hai chữ số cuối của a 2011 .
 
57. (BMO 2001, Round 4). Cho dãy an thỏa mãn a 0  4;a1  22 và

  
an  6an 1  an 2  0 với n  2 . Chứng minh rằng tồn tại các dãy x n , yn gồm các số
yn2  7
nguyên dương sao cho an  với mọi n  0 .
x n  yn
58. (Romania 1996). Tìm số nguyên lớn nhất n để tồn tại n số nguyên không âm
x 1 ; x 2 ;...; x n không đồng thời bằng 0, sao cho với mọi dãy số  1,  2 ,...,  n gồm các phần

 
tử không đồng thời bằng 0 lấy từ tập 1; 0;1 ta có: n 3 không chia hết
 1x 1   2x 2  ...   n x n .
an n
 
59. (BMO 1996, Round 4). Cho dãy số an xác định bởi: a1  1; an 1 
n

an
.

Chứng minh rằng: an2   n với n  4 ( ký hiệu x  là phần nguyên của x ).
u  0; u2  1
 
60. Cho dãy số un xác định bởi:  1
u  u  u  1
.
 n  2 n n 1

 
Chứng minh rằng: Nếu p  5 là số nguyên tố thì u p u p  1 chia hết cho p .

61. (St. Petersburg City MO 2002).Cho dãy số a  xác định bởi:


n

 an  1
 , if an  1
 2

an 1
2an
 , if an  1
 1  an

Văn Phú Quốc , GV. Trường THPT chuyên Nguyễn Bỉnh Khiêm DĐ: 0982 333 443
Bài tập số học Bồi dưỡng HSG Toán Quốc gia

Cho a 0 là một số nguyên dương, an  2 với mỗi n  1;2;...;2011 và a 2002  2 . Tính


a0 ?

 
62. Cho dãy số x n xác định bởi: x 0  0; x 1  1; x 2  0 và

n 2

n 1 n 1 x n 1
x n 3
n
 n 2  
 n 2  n  1 x n 1 
n
xn .

Chứng minh rằng x n là số chính phương với mọi n  0 .


  
63. (VMO 1987). Cho dãy số x n , yn xác định bởi:

 
x 0  365 ; x n 1  x n x n1986  1  1622 , n  0
và y 0  16 ; yn 1  yn y  1  1952 ,
3
n
n  0 .
Chứng minh rằng: x n  yk  0 , n, k  1 .

 
64. (VMO 1995). Một dãy số an được xác định bởi:

a  9an , if n is even


a 0  1, a1  3, an 2   n 1
9an 1  5an , if n is odd
Chứng minh rằng:
2000
2
a) a k
chia hết cho 20 ; b) a 2n 1 không phải là một số chính phương n   *
k 1995

65. (VMO 1998 A). Cho dãy số nguyên dương an   n 0


xác định bởi:
a 0  20, a1  100, an 2  4an 1  5an  20 với n  0 .
Tìm số dương h nhỏ nhất có tính chất: an h  a n chia hết cho 1998 với mọi n   .
66. (VMO 1989). Xét dãy số Fibonacci 1,1,2, 3,5, 8,13,... Đặt
 
f n  1985n 2  1956n  1960 .
a) Chứng rằng tồn tại vô hạn số F của dãy trên sao cho f F chia hết cho 1989.  
b) Tồn tại hay không một số G của dãy sao cho f G  2 chia hết cho 1989?  

Văn Phú Quốc , GV. Trường THPT chuyên Nguyễn Bỉnh Khiêm DĐ: 0982 333 443
Inequalities proposed in
“Crux Mathematicorum”
(from vol. 1, no. 1 to vol. 4, no. 2 known as “Eureka”)
Complete and up-to-date: November 24, 2004

The best problem solving journal all over the world; visit http://journals.cms.math.ca/CRUX/

(An asterisk (F) after a number indicates that a problem was proposed without a solution.)

2. Proposed by Léo Sauvé, Algonquin College.


A rectangular array of m rows and n columns contains mn distinct real numbers. For i =
1, 2, . . . , m, let si denote the smallest number of the ith row; and for j = 1, 2, . . . , n, let lj denote
the largest number of the j th column. Let A = max{si } and B = min{lj }. Compare A and B.

14. Proposed by Viktors Linis, University of Ottawa.


1 1
If a, b, c are lengths of three segments which can form a triangle, show the same for a+c , b+c ,
1
a+b .

17. Proposed by Viktors Linis, University of Ottawa.


Prove the inequality
1 3 5 999999 1
· · ··· < .
2 4 6 1000000 1000
23. Proposed by Léo Sauvé, Collège Algonquin.
Déterminer s’il existe une suite {un } d’entiers naturels telle que, pour n = 1, 2, 3, . . ., on ait

2un < 2n + 1 < 21+un

25. Proposed by Viktors Linis, University of Ottawa.


Find the smallest positive value of 36k − 5l where k and l are positive integers.

29. Proposed by Viktors Linis, University of Ottawa.


Cut a square into a minimal number of triangles with all angles acute.

36. Proposed by Léo Sauvé, Collège Algonquin.


Si m et n sont des entiers positifs, montrer que
m m nn
sin2m θ cos2n θ ≤ ,
(m + n)m+n
et dèterminer les valeurs de θ pour lesquelles il y a égalité.

54. Proposed by Léo Sauvé, Collège Algonquin.


Si a, b, c > 0 et a < b + c, montrer que
a b c
< + .
1+a 1+b 1+c
66. Proposed by John Thomas, University of Ottawa.
What is the largest non-trivial subgroup of the group of permutations on n elements?

74. Proposed by Viktors Linis, University of Ottawa.


Prove that if the sides a, b, c of a triangle satisfy a2 + b2 = kc2 , then k > 21 .

1
75. Proposed by R. Duff Butterill, Ottawa Board of Education.
M is the midpoint of chord AB of the circle with centre C shown
in the figure. Prove that RS > M N . C P

M
A B
79. Proposed by John Thomas, University of Ottawa. R
Show that, for x > 0, N
¯Z x+1 ¯ S
¯< 2 .
¯ 2
¯
¯
¯ sin(t ) dt ¯ x2
x

84. Proposed by Viktors Linis, University of Ottawa.


Prove that for any positive integer n
r

n
2
n<1+ .
n
98. Proposed by Viktors Linis, University of Ottawa.
Prove that, if 0 < a < b, then

b2 b a
ln < − .
a2 a b
100. Proposed by Léo Sauvé, Collège Algonquin.
Soit f une fonction numérique continue et non négative pour tout x ≥ 0. On suppose qu’il existe
un nombre réel a > 0 tel que, pout tout x > 0,
Z x
f (x) ≤ a f (t) dt.
0

Montrer que la fonction f est nulle.

106. Proposed by Viktors Linis, University of Ottawa.


Prove that, for any quadrilateral with sides a, b, c, d,
1
a 2 + b 2 + c 2 > d2 .
3
108. Proposed by Viktors Linis, University of Ottawa.
Prove that, for all integers n ≥ 2,
n
X 1 3n
2
> .
k 2n + 1
k=1

110. Proposed by H. G. Dworschak, Algonquin College. P


(a) Let AB and P R be two chords of a circle intersecting at Q. If A
Q
B
A, B, and P are kept fixed, characterize geometrically the position
of R for which the length of QR is maximal. (See figure).
(b) Give a Euclidean construction for the point R which maximizes
the length of QR, or show that no such construction is possible.
R
115. Proposed by Viktors Linis, University of Ottawa.
Prove the following inequality of Huygens:
π
2 sin α + tan α ≥ 3α, 0≤α< .
2

2
119. Proposed by John A. Tierney, United States Naval Academy.
A line through the first quadrant point (a, b) forms a right triangle with the positive coordinate
axes. Find analytically the minimum perimeter of the triangle.

120. Proposed by John A. Tierney, United States Naval Academy.


Given a point P inside an arbitrary angle, give a Euclidean construction of the line through P
that determines with the sides of the angle a triangle
(a) of minimum area;
(b) of minimum perimeter.

135. Proposed by Steven R. Conrad, Benjamin N. Cardozo H. S., Bayside, N. Y.


How many 3×5 rectangular pieces of cardboard can be cut from a 17×22 rectangular piece of
cardboard so that the amount of waste is a minimum?

145. Proposed by Walter Bluger, Department of National Health and Welfare.


A pentagram is a set of 10 points consisting of the vertices and the intersections of the diagonals
of a regular pentagon with an integer assigned to each point. The pentagram is said to be magic
if the sums of all sets of 4 collinear points are equal.
Construct a magic pentagram with the smallest possible positive primes.

150. Proposed by Kenneth S. Williams, Carleton University, Ottawa, Ontario.


If bxc denotes the greatest integer ≤ x, it is trivially true that
$µ ¶ %
3 k 3k − 2 k
> for k ≥ 1,
2 2k

and it seems to be a hard conjecture (see G. H. Hardy & E. M. Wright, An Introduction to the
Theory of Numbers, 4th edition, Oxford University Press 1960, p. 337, condition (f)) that
$µ ¶ %
3 k 3k − 2 k + 2
≥ for k ≥ 4.
2 2k − 1

Can one find a function f (k) such that


$µ ¶ %
3 k
≥ f (k)
2

3k −2k 3k −2k +2
with f (k) between 2k
and 2k −1
?

160. Proposed by Viktors Linis, University of Ottawa.


P9 − 2
10
Find the integral part of n 3.
n=1
This problem is taken from the list submitted for the 1975 Canadian Mathematics Olympiad
(but not used on the actual exam).

162. Proposed by Viktors Linis, University of Ottawa.


If x0 = 5 and xn+1 = xn + x1n , show that

45 < x1000 < 45.1.

This problem is taken from the list submitted for the 1975 Canadian Mathematics Olympiad
(but not used on the actual exam).

3
165. Proposed by Dan Eustice, The Ohio State University.
Prove that, for each choice of n points in the plane (at least two distinct), there exists a point
on the unit circle such that the product of the distances from the point to the chosen points is
greater than one.

167. Proposed by Léo Sauvé, Algonquin College.


The first half of the Snellius-Huygens double inequality
1 3 sin α π
(2 sin α + tan α) > α > , 0<α< ,
3 2 + cos α 2
was proved in Problem 115. Prove the second half in a way that could have been understood
before the invention of calculus.

173. Proposed by Dan Eustice, The Ohio State University.


For each choice of n points on the unit circle (n ≥ 2), there exists a point on the unit circle such
that the product of the distances to the chosen points is greater than or equal to 2. Moreover,
the product is 2 if and only if the n points are the vertices of a regular polygon.

179. Proposed by Steven R. Conrad, Benjamin N. Cardozo H. S., Bayside, N. Y.


The equation 5x + 7y = c has exactly three solutions (x, y) in positive integers. Find the largest
possible value of c.

207. Proposed by Ross Honsberger, University of Waterloo.



Prove that 2r+5
r+2 is always a better approximation of 5 than r.

219. Proposed by R. Robinson Rowe, Sacramento, California.


Find the least integer N which satisfies

N = aa+2b = bb+2a , a 6= b.

223. Proposed by Steven R. Conrad, Benjamin N. Cardozo H. S., Bayside, N. Y.


Without using any table which lists Pythagorean triples, find the smallest integer which can
represent the area of two noncongruent primitive Pythagorean triangles.

229. Proposed by Kenneth M. Wilke, Topeka, Kansas.


On an examination, one question asked for the largest angle of the triangle with sides 21, 41,
and 50. A student obtained the correct answer as follows: Let C denote the desired angle; then
50 9 9
sin C = 41 = 1 + 41 . But sin 90 ◦ = 1 and 41 = sin 12 ◦ 400 4900 . Thus

C = 90 ◦ + 12 ◦ 400 4900 = 102 ◦ 400 4900 ,

which is correct. Find the triangle of least area having integral sides and possessing this property.

230. Proposed by R. Robinson Rowe, Sacramento, California.


Find the least integer N which satisfies

N = ama+nb = bmb+na

with m and n positive and 1 < a < b. (This generalizes Problem 219.)

4
247F . Proposed by Kenneth S. Williams, Carleton University, Ottawa, Ontario.
On page 215 of Analytic Inequalities by D. S. Mitrinović, the following inequality is given: if
0 < b ≤ a then
1 (a − b)2 a+b √ 1 (a − b)2
≤ − ab ≤ .
8 a 2 8 b
Can this be generalized to the following form: if 0 < a1 ≤ a2 ≤ · · · ≤ an then
P P
(ai − aj )2 (ai − aj )2
1≤i<j≤n a1 + · · · + a n √ 1≤i<j≤n
k ≤ − n a1 · · · a n ≤ k ,
an n a1
where k is a constant?

280. Proposed by L. F. Meyers, The Ohio State University.


A jukebox has N buttons.
(a) If the set of N buttons is subdivided into disjoint subsets, and a customer is required to
press exactly one button from each subset in order to make a selection, what is the distribution
of buttons which gives the maximum possible number of different selections?
(b) What choice of n will allow the greatest number of selections if a customer, in making a
selection, may press any n distinct buttons out of the N ? How many selections are possible
then?
(Many jukeboxes have 30 buttons, subdivided into 20 and 10. The answer to part (a) would
then be 200 selections.)

282. Proposed by Erwin Just and Sidney Penner, Bronx Community College.
On a 6×6 board we place 3×1 trominoes (each tromino covering exactly three unit squares of
the board) until no more trominoes can be accommodated. What is the maximum number of
squares that can be left vecant?

289. Proposed by L. F. Meyers, The Ohio State University.


Derive the laws of reflection and refraction from the principle of least time without use of calculus
or its equivalent. Specifically, let L be a straight line, and let A and B be points not on L. Let
the speed of light on the side of L on which A lies be c1 , and let the speed of light on the other
side of L be c2 . Characterize the points C on L for which the time taken for the route ACB is
smallest, if
(a) A and B are on the same side of L (reflection);
(b) A and B are on opposite sides of L (refraction).

295. Proposed by Basil C. Rennie, James Cook University of North Queensland, Australia.
If 0 < b ≤ a, prove that
√ 1 (a − b)2
a + b − 2 ab ≥ .
2 a+b
303. Proposed by Viktors Linis, University of Ottawa.
Huygens’ inequality 2 sin α + tan α ≥ 3α was proved in Problem 115. Prove the following hyper-
bolic analogue:
2 sinh x + tanh x ≥ 3x, x ≥ 0.
304. Proposed by Viktors Linis, University of Ottawa.
Prove the following inequality:

ln x 1+ 3x
≤ √ , x > 0, x 6= 1.
x−1 x+ 3x

5
306. Proposed by Irwin Kaufman, South Shore H. S., Brooklyn, N. Y.
Solve the following inequality, which was given to me by a student:
1
sin x sin 3x > .
4
307. Proposed by Steven R. Conrad, Benjamin N. Cardozo H. S., Bayside, N. Y.
It was shown in Problem 153 that the equation ab = a + b has only one solution in positive
integers, namely (a, b) = (2, 2). Find the least and greatest values of x (or y) such that

xy = nx + ny,

if n, x, y are all positive integers.

310. Proposed by Jack Garfunkel, Forest Hills H. S., Flushing, N. Y.


Prove that
a b 2ab 3
√ +√ +√ √ ≤ .
a2 + b2 9a2 + b2 a2 + b2 · 9a2 + b2 2
When is equality attained?

318. Proposed by C. A. Davis in James Cook Mathematical Notes No. 12 (Oct. 1977), p. 6.
Given any triangle ABC, thinking of it as in the complex plane, two points L and N may be
defined as the stationary values of a cubic that vanishes at the vertices A, B, and C. Prove that
L and N are the foci of the ellipse that touches the sides of the triangle at their midpoints,
which is the inscribed ellipse of maximal area.

323. Proposed by Jack Garfunkel, Forest Hills H. S., Flushing, N. Y., and Murray S. Klamkin,
University of Alberta.
If xyz = (1 − x)(1 − y)(1 − z) where 0 ≤ x, y, z ≤ 1, show that
3
x(1 − z) + y(1 − x) + z(1 − y) ≥ .
4
344. Proposed by Viktors Linis, University of Ottawa.
Given is a set S of n positive numbers. With each nonempty subset P of S, we associate the
number

σ(P ) = sum of all its elements.

Show that the set {σ(P ) | P ⊆ S} can be partitioned into n subsets such that in each subset the
ratio of the largest element to the smallest is at most 2.

347. Proposed by Murray S. Klamkin, University of Alberta.


Determine the maximum value of
q p q p
3 3
4 − 3x + 16 − 24x + 9x2 − x3 + 4 − 3x − 16 − 24x + 9x2 − x3

in the interval −1 ≤ x ≤ 1.

358. Proposed by Murray S. Klamkin, University of Alberta.


Determine the maximum of x2 y, subject to the constraints
p
x + y + 2x2 + 2xy + 3y 2 = k (constant), x, y ≥ 0.

6
362. Proposed by Kenneth S. Williams, Carleton University, Ottawa, Ontario.
In Crux 247 [1977: 131; 1978: 23, 37] the following inequality is proved:
P P
(ai − aj )2 (ai − aj )2
1 1≤i<j≤n a1 + · · · + a n √ 1 1≤i<j≤n
≤ − n a1 · · · a n ≤ 2 .
2n2 an n 2n a1
1
Prove that the constant 2n2
is best possible.

367F . Proposed by Viktors Linis, University of Ottawa.


(a) A closed polygonal curve lies on the surface of a cube with edge of length√1. If the curve
intersects every face of the cube, show that the length of the curve is at least 3 2.
(b) Formulate and prove similar theorems about (i) a rectangular parallelepiped, (ii) a regular
tetrahedron.

375. Proposed by Murray S. Klamkin, University of Alberta.


A convex n-gon P of cardboard is such that if lines are drawn parallel to all the sides at
distances x from them so as to form within P another polygon P 0 , then P 0 is similar to P . Now
let the corresponding consecutive vertices of P and P 0 be A1 , A2 , . . . , An and A01 , A02 , . . . , A0n ,
respectively. From A02 , perpendiculars A02 B1 , A02 B2 are drawn to A1 A2 , A2 A3 , respectively, and
the quadrilateral A02 B1 A2 B2 is cut away. Then quadrilaterals formed in a similar way are cut
away from all the other corners. The remainder is folded along A01 A02 , A02 A03 , . . . , A0n A01 so as
to form an open polygonal box of base A01 A02 . . . A0n and of height x. Determine the maximum
volume of the box and the corresponding value of x.

394. Proposed by Harry D. Ruderman, Hunter College Campus School, New York.
A wine glass has the shape of an isosceles trapezoid rotated about its axis of symmetry. If R, r,
and h are the measures of the larger radius, smaller radius, and altitude of the trapezoid, find
r : R : h for the most economical dimensions.

395F . Proposed by Kenneth S. Williams, Carleton University, Ottawa, Ontario.


In Crux 247 [1977: 131; 1978: 23, 37] the following inequality is proved:
P P
(ai − aj )2 (ai − aj )2
1 1≤i<j≤n 1 1≤i<j≤n
≤A−G≤ 2 ,
2n2 an 2n a1
where A (resp. G) is the arithmetic (resp. geometric) mean of a1 , . . . , an . This is a refinement of
the familiar inequality A ≥ G. If H denotes the harmonic mean of a1 , . . . , an , that is,
µ ¶
1 1 1 1
= + ··· + ,
H n a1 an

find the corresponding refinement of the familiar inequality G ≥ H.

397. Proposed by Jack Garfunkel, Forest Hills H. S., Flushing, N. Y.


Given is 4ABC with incenter I. Lines AI, BI, CI are drawn to meet the incircle (I) for the
first time in D, E, F , respectively. Prove that

(AD + BE + CF ) 3

is not less than the perimeter of the triangle of maximum perimeter that can be inscribed in
circle (I).

7
402. Proposed by the late R. Robinson Rowe, Sacramento, California.
An army with an initial strength of A men is exactly decimeted each day of a 5-day battle and
reinforced each night wirh R men from the reserve pool of P men, winding up on the morning
of the 6th day with 60 % of its initial strength. At least how large must the initial strength have
been if
(a) R was a constant number each day;
(b) R was exactly half the men available in the dwindling pool?

404. Proposed by Andy Liu, University of Alberta.


Let A be a set of n distinct positive numbers. Prove that
(a) the number of distinct sums of subsets of A is at least 21 n(n + 1) + 1;
2n
(b) the number of distinct subsets of A with equal sum to half the sum of A is at most n+1 .

405. Proposed by Viktors Linis, University of Ottawa.


A circle of radius 16 contains 650 points. Prove that there exists an annulus of inner radius 2
and outer radius 3 which contains at least 10 of the given points.

413. Proposed by G. C. Giri, Research Scholar, Indian Institute of Technology, Kharagpur,


India.
If a, b, c > 0, prove that

1 1 1 a8 + b8 + c 8
+ + ≤ .
a b c a3 b3 c3
417. Proposed by John A. Tierney, U. S. Naval Academy, Annapolis, Maryland.
It is easy to guess from the graph of the folium os Descartes,

x3 + y 3 − 3axy = 0, a>0
¡ 3a ¢
that the point of maximum curvature is 2 , 3a
2 . Prove it.

423. Proposed by Jack Garfunkel, Forest Hills H. S., Flushing, N. Y.


In a triangle ABC whose circumcircle has unit diameter, let ma and ta denote the lengths of
the median and the internal angle bisector to side a, respectively. Prove that
A B−C
ta ≤ cos2 cos ≤ ma .
2 2
427. Proposed by G. P. Henderson, Campbellcroft, Ontario.
A corridor of width a intersects a corridor of width b to form an “L”. A rectangular plate is
to be taken along one corridor, around the corner and along the other corridor with the plate
being kept in a horizontal plane. Among all the plates for which this is possible, find those of
maximum area.

429. Proposed by M. S. Klamkin and A. Liu, both from the University of Alberta.
On a 2n × 2n board we place n × 1 polyominoes (each covering exactly n unit squares of the
board) until no more n×1 polyominoes can be accomodated. What is the number of squares
that can be left vacant?
This problem generalizes Crux 282 [1978: 114].

8
440F . Proposed by Kenneth S. Williams, Carleton University, Ottawa, Ontario.
My favourite proof of the well-known result

1 1 1 π2
ζ(2) = + + + · · · =
12 22 32 6
uses the identity
n
X kπ n(2n − 1)
cot2 =
2n + 1 3
k=1

and the inequality


1 π
cot2 x < < 1 + cot2 x, 0<x<
x2 2
to obtain
n · ¸
π2 n(2n − 1) X 1 π2 n(2n − 1)
· < < n+ ,
(2n + 1)2 3 k2 (2n + 1)2 3
k=1

from which the desired result follows upon letting n → ∞.


Can any reader find a new elementary prrof simpler than the above? (Many references to this
problem are given by E. L. Stark in Mathematics Magazine, 47 (1974) 197–202.)

450F . Proposed by Andy Liu, University of Alberta.


Triangle ABC has a fixed base BC and a fixed inradius. Describe the locus of A as the incircle
rools along BC. When is AB of minimal length (geometric characterization desired)?

458. Proposed by Harold N. Shapiro, Courant Institute of Mathematical Sciences, New York
University.
Let φ(n) denote the Euler function. It is well known that, for each fixed integer c > 1, the
equation φ(n) = n − c has at most a finite number of solutions for the integer n. Improve this
by showing that any such solution, n, must satisfy the inequalities c < n ≤ c2 .

459. Proposed by Vedula N. Murty, Pennsylvania State University, Capitol Campus, Middle-
town, Pennsylvania.
If n is a positive integer, prove that

X 1 π2 1
≤ · .
k 2n 8 1 − 2−2n
k=1

468. Proposed by Viktors Linis, University of Ottawa.


(a) Prove that the equation

a1 xk1 + a2 xk2 + · · · + an xkn − 1 = 0,

where a1 , . . . , an are real and k1 , . . . , kn are natural numbers, has at most n positive roots.
(b) Prove that the equation

axk (x + 1)p + bxl (x + 1)q + cxm (x + 1)r − 1 = 0,

where a, b, c are real and k, l, m, p, q, r are natural numbers, has at most 14 positive roots.

9
484. Proposed by Gali Salvatore, Perkins, Québec.
Let A and B be two independent events in a sample space, and let χA , χB be their characteristic
functions (so that, for example, χA (x) = 1 or 0 according as x ∈ A or x ∈
/ A). If F = χA + χB ,
show that at least one of the three numbers a = P (F = 2), b = P (F = 1), c = P (F = 0) is not
less than 49 .

487. Proposed by Dan Sokolowsky, Antioch College, Yellow Springs, Ohio.


If a, b, c and d are positive real numbers such that c2 + d2 = (a2 + b2 )3 , prove that

a3 b3
+ ≥ 1,
c d
with equality if and only if ad = bc.

488F . Proposed by Kesiraju Satyanarayana, Gagan Mahal Colony, Hyderabad, India.


Given a point P within a given angle, construct a line through P such that the segment inter-
cepted by the sides of the angle has minimum length.

492. Proposed by Dan Pedoe, University of Minnesota.


(a) A segment AB and a rusty compass of span r > 12 AB are given. Show how to find the
vertex C of an equilateral triangle ABC using, as few times as possible, the rusty compass only.
(b)F Is the construction possible when r < 21 AB?

493. Proposed by Robert C. Lyness, Southwold, Suffolk, England.


1
(a) A, B, C are the angles of a triangle. Prove that there are positive x, y, z, each less than 2,
simultaneously satisfying
B C
y 2 cot + 2yz + z 2 cot = sin A,
2 2
C A
z 2 cot + 2zx + x2 cot = sin B,
2 2
A B
x2 cot + 2xy + y 2 cot = sin C.
2 2
1
(b)F In fact, 2 may be replaced by a smaller k > 0.4. What is the least value of k?

495. Proposed by J. L. Brenner, Palo Alto, California; and Carl Hurd, Pennsylvania State
University, Altoona Campus.
Let S be the set of lattice points (points having integral coordinates) contained ina bounded
convex set in the plane. Denote by N the minimum of two measurements of S: the greatest
number of points of S on any line of slope 1, −1. Two lattice points are adjoining if they are
exactly one unit apart. Let the n points of S be numbered by the integers from 1 to n in such
a way that the largest difference of the assigned integers of adjoining points is minimal. This
minimal largest difference we call the discrepancy of S.
(a) Show that the discrepancy of S is no greater than N + 1.
(b) Give such a set S whose discrepancy is N + 1.
(c)F Show that the discrepancy of S is no less than N .

505. Proposed by Bruce King, Western Connecticut State College and Sidney Penner, Bronx
Community College.
Let F1 = F2 = 1, Fn = Fn = Fn−1 + Fn−2 for n > 2 and G1 = 1, Gn = 2n−1 − Gn−1 for n > 1.
Fn
Show that (a) Fn ≤ Gn for each n and (b) lim G n
= 0.
n→∞

10
506. Proposed by Murray S. Klamkin, University of Alberta.
It is known from an earlier problem in this journal [1975: 28] that if a, b, c are the sides of a
triangle, then so are 1/(b + c), 1/(c + a), 1/(a + b). Show more generally that if a 1 , a2 , . . . , an
are the sides of a polygon then, for k = 1, 2, . . . , n,
n+1 X 1 (n − 1)2
≥ ≥ ,
S − ak S − ai (2n − 3)(S − ak )
i=1
i6=k

where S = a1 + a2 + · · · + an .

517F . Proposed by Jack Garfunkel, Flushing, N. Y.


Given is a triangle ABC with altitudes ha , hb , hc and medians ma , mb , mc to sides a, b, c, respec-
tively. Prove that
hb hc ha
+ + ≤ 3,
mc ma mb
with equality if and only if the triangle is equilateral.

529. Proposed by J. T. Groenman, Groningen, The Netherlands.


The sides of a triangle ABC satisfy a ≤ b ≤ c. With the usual notation r, R, and rc for the in-,
circum-, and ex-radii, prove that

sgn(2r + 2R − a − b) = sgn(2rc − 2R − a − b) = sgn(C − 90 ◦).

535. Proposed by Jack Garfunkel, Flushing, N. Y.


Given a triangle ABC with sides a, b, c, let Ta , Tb , Tc denote the angle bisectors extended to the
circumcircle of the triangle. Prove that
8√
Ta Tb Tc ≥ 3abc,
9
with equality attained in the equilateral triangle.

544. Proposed by Vedula N. Murty, Pennsylvania State University, Capitol Campus, Middle-
town, Pennsylvania.
Prove that, in any triangle ABC,
µ ¶
B C C A A B A B C
2 sin sin + sin sin + sin sin ≤ sin + sin + sin ,
2 2 2 2 2 2 2 2 2
with equality if and only if the triangle is equilateral.

552. Proposed by Vedula N. Murty, Pennsylvania State University, Capitol Campus, Middle-
town, Pennsylvania.
Given positive constants a, b, c and nonnegative real variables x, y, z subject to the constraint
x + y + z = π, find the maximum value of

f (x, y, z) ≡ a cos x + b cos y + c cos z.

563. Proposed by Michael W. Ecker, Pennsylvania State University, Worthington Scranton


Campus.
For n a positive integer, let (a1 , a2 , . . . , an ) and (b1 , b2 , . . . , bn ) be two permutations (not neces-
sarily distinct) of (1, 2, . . . , n). Find sharp upper and lower bounds for

a1 b1 + a2 b2 + · · · + an bn .

11
570. Proposed by Vedula N. Murty, Pennsylvania State University, Capitol Campus, Middle-
town, Pennsylvania.
If x, y, z > 0, show that
X 2x2 (y + z)
≤ x + y + z,
(x + y)(x + z)
cyclic

with equality if and only if x = y = z.

572F . Proposed by Paul Erdös, Technion – I.I.T., Haifa, Israel.


It was proved in Crux 458 [1980: 157] that, if φ is the Euler function and the integer c > 1, then
each solution n of the equation

φ(n) = n − c (1)

satisfies c + 1 ≤ n ≤ c2 . Let F (c) be the number of solutions of (1). Estimate F (c) as well as you
can from above and below.

583. Proposed by Charles W. Trigg, San Diego, California.


A man, being asked the ages of his two sons, replied: “Each of their ages is one more than three
times the sum of its digits.” How old is each son?

585. Proposed by Jack Garfunkel, Flushing, N. Y.


Consider the following three inequalities for the angles A, B, C of a triangle:
B−C C −A A−B A B C
cos cos cos ≥ 8 sin sin sin , (1)
2 2 2 2 2 2
A B−C B C −A C A−B
csc cos + csc cos + csc cos ≥ 6, (2)
2 2 2 2 2 2
A B C
csc + csc + csc ≥ 6.
2 2 2
Inequality (3) is well-known (American Mathematical Monthly 66 (1959) 916) and it is trivially
implied by (2). Prove (1) and show that (1) implies (2).

589. Proposed by Ngo Tan, student, J. F. Kennedy H. S., Bronx, N. Y..


In a triangle ABC with semiperimeter s, sides of lengths a, b, c, and medians of lengths m a , mb ,
mc , prove that:

(a) There exists a triangle with sides of lengths a(s − a), b(s − b), c(s − c).
³ m ´2 ³ m ´2 ³ m ´2 9
a b c
(b) + + ≥ , with equality if and only if the triangle is equilateral.
a b c 4
602. Proposed by George Tsintsifas, Thessaloniki, Greece.
Given are twenty natural numbers ai such that

0 < a1 < a2 < · · · < a20 < 70.

Show that at least one of the differences ai − aj , i > j, occurs at least four times. (A student
proposed this problem to me. I don’t know the source.)

12
606F . Proposed by George Tsintsifas, Thessaloniki, Greece.
Let σn = A0 A1 . . . An be an n-simplex in Euclidean space Rn and let σn0 = A00 A01 . . . A0n be an
n-simplex similar to and inscribed in σn , and labeled in such a way that

A0i ∈ σn−1 = A0 A1 . . . Ai−1 Ai+1 . . . An , i = 0, 1, . . . , n.

Prove that the ratio of similarity

A0i A0j 1
λ≡ ≥ .
Ai Aj n

[If no proof of the general case is forthcoming, the editor hopes to receive a proof at least for
the special case n = 2.]

608. Proposed by Ngo Tan, student, J. F. Kennedy H. S., Bronx, N. Y..


ABC is a triangle with sides of lengths a, b, c and semiperimeter s. Prove that

A B C s3
cos4 + cos4 + cos4 ≤ ,
2 2 2 2abc
with equality if and only if the triangle is equilateral.

613. Proposed by Jack Garfunkel, Flushing, N. Y.


If A + B + C = 180 ◦, prove that
B−C C −A A−B 2
cos + cos + cos ≥ √ (sin A + sin B + sin C).
2 2 2 3
(Here A, B, C are not necessarily the angles of a triangle, but you may assume that they are if
it is helpful to achieve a proof without calculus.)

615. Proposed by G. P. Henderson, Campbellcroft, Ontario.


Let P be a convex n-gon with vertices E1 , E2 , . . . , En , P
perimeter L and area A. Let 2θi be the
measure of the interior angle at vertex Ei and set C = cot θi . Prove that

L2 − 4AC ≥ 0

and characterize the convex n-gons for which equality holds.

623F . Proposed by Jack Garfunkel, Flushing, N. Y.


If P QR is the equilateral triangle of smallest area inscribed in a given triangle ABC, with P on
BC, Q on CA, and R on AB, prove or disprove that AP , BQ, and CR are concurrent.

624. Proposed by Dmitry P. Mavlo, Moscow, U. S. S. R.


ABC is a given triangle of area K, and P QR is the equilateral triangle of smallest area K 0
inscribed in triangle ABC, with P on BC, Q on CA, and R on AB.
(a) Find ratio

K
λ= ≡ f (A, B, C)
K0
as a function of the angles of the given triangle.
(b) Prove that λ attains its minimum value when the given triangle ABC is equilateral.
(c) Give a Euclidean construction of triangle P QR for an arbitrary given triangle ABC.

13
626. Proposed by Andy Liu, University of Alberta.
A (ν, b, r, k, λ)-configuration on a set with ν elements is a collection of b k-subsets such that
(i) each element appears in exactly r of the k-subsets;
(ii) each pair of elements appears in exactly λ of the k-subsets.
Prove that k r ≥ ν λ and determine the value of b when equality holds.

627. Proposed by F. David Hammer, Santa Cruz, California.


Consider the double inequality

3
6<3 < 7.

Using only the elementary properties


√ of exponents and inequalities (no calculator, computer,
table of logarithms, or estimate of 3 may be used), prove that the first inequality implies the
second.

628. Proposed by Roland H. Eddy, Memorial University of Newfoundland.


Given a triangle ABC with sides a, b, c, let Ta , Tb , Tc denote the angle bisectors extended to the
circumcircle of the triangle. If R and r are the circum- and in-radii of the triangle, prove that

Ta + Tb + Tc ≤ 5R + 2r,

with equality just when the triangle is equilateral.

644. Proposed by Jack Garfunkel, Flushing, N. Y.


If I is the incenter of triangle ABC and lines AI, BI, CI meet the circumcircle of the triangle
again in D, E, F , respectively, prove that
AI BI CI
+ + ≥ 3.
ID IE IF
648. Proposed by Jack Garfunkel, Flushing, N. Y.
Given a triangle ABC, its centroid G, and the pedal triangle P QR of its incenter I. The segments
AI, BI, CI meet the incircle in U , V , W ; and the segments AG, BG, CG meet the incircle in
D, E, F . Let ∂ denote the perimeter of a triangle and consider the statement

∂P RQ ≤ ∂U V W ≤ ∂DEF.

(a) Prove the first inequality.


(b)F Prove the second inequality.

650. Proposed by Paul R. Beesack, Carleton University, Ottawa.


(a) Two circular cylinders of radii r and R, where 0 < r ≤ R, intersect at right angles (i. e.,
their central axes intersect at an angle of π2 ). Find the arc length l of one of the two curves of
intersection, as a definite integral.
(b) Do the same problem if the cylinders intersect at an angle γ, where 0 < γ < π2 .
(c) Show the the arc length l in (a) satisfies
Z π/2 p
5πr
l ≤ 4r 1 + cos2 θ dθ < .
0 2

14
653. Proposed by George Tsintsifas, Thessaloniki, Greece.
For every triangle ABC, show that
X B−C Y A
cos2 ≥ 24 sin ,
2 2
where the sum and product are cyclic over A, B, C, with equality if and only if the triangle is
equilateral.

655. Proposed by Kaidy Tan, Fukien Teachers’ University, Foochow, Fukien, China.
If 0 < a, b, c, d < 1, prove that
³X ´3 X X µ1¶
2
a > 4bcd a + 8a bcd ,
a
where the sums are cyclic over a, b, c, d.

656. Proposed by J. T. Groenman, Arnhem, The Netherlands.


P is an interior point of a convex region R bounded by the arcs of two intersecting circles C 1 and
C2 . Construct through P a “chord” U V of R, with U on C1 and V on C2 , such that |P U | · |P V |
is a minimum.

664. Proposed by George Tsintsifas, Thessaloniki, Greece.


An isosceles trapezoid ABCD, with parallel bases AB and DC, is inscribed in a circle of diameter
AB. Prove that
AB + DC
AC > .
2
665. Proposed by Jack Garfunkel, Queens College, Flushing, N. Y.
If A, B, C, D are the interior angles of a convex quadrilateral ABCD, prove that
√ X A+B X A
2 cos ≤ cot
4 2
(where the four-term sum on each side is cyclic over A, B, C, D), with equality if and only if
ABCD is a rectangle.

673F . Proposed by Vedula N. Murty, Pennsylvania State University, Capitol Campus, Midd-
letown, Pennsylvania.
Determine for which positive integers n the following property holds: if m is any integer satisfying
n(n + 1)(n + 2) n(n + 1)(2n + 1)
≤m≤ ,
6 6
then there exist permutations (a1 , a2 , . . . , an ) and (b1 , b2 , . . . , bn ) of (1, 2, . . . , n) such that

a1 b1 + a2 b2 + · · · + an bn = m.

(See Crux 563 [1981: 208].)

682. Proposed by Robert C. Lyness, Southwold, Suffolk, England.


Triangle ABC is acute-angled and ∆1 is its orthic triangle (its vertices are the feet of the
altitudes of triangle ABC). ∆2 is the triangular hull of the three excircles of triangle ABC (that
is, its sides are external common tangents of the three pairs of excircles that are not sides of
triangle ABC). Prove that the area of triangle ∆2 is at least 100 times the area of triangle ∆1 .

15
683. Proposed by Kaidy Tan, Fukien Teachers’ University, Foochow, Fukien, China.
Triangle ABC has AB > AC, and the internal bisector of angle A meets BC at T . Let P be
any point other than T on line AT , and suppose lines BP , CP intersect lines AC, AB in D, E,
respectively. Prove that BD > CE or BD < CE according as P lies on the same side or on the
opposite side of BC as A.

684. Proposed by George Tsintsifas, Thessaloniki, Greece.


Let O be the origin of the lattice plane, and let M (p, q) be a lattice point with relatively prime
positive coordinates (with q > 1). For i = 1, 2, . . . , q − 1, let Pi and Qi be the lattice points, both
with ordinate i, that are respectively the left and right endpoints of the horizontal unit segment
intersecting OM . Finally, let Pi Qi ∩ OM = Mi .
q−1
X
(a) Calculate S1 = P i Mi .
i=1

(b) Find the minimum value of Pi Mi for 1 ≤ i ≤ q − 1.


(c) Show that Ps Ms + Pq−s Mq−s = 1, 1 ≤ s ≤ q − 1.
q−1
X P i Mi
(d) Calculate S2 = .
i?1
Mi Q i

685. Proposed by J. T. Groenman, Arnhem, The Netherlands.


Given is a triangle ABC with internal angle bisectors ta , tb , tc meeting a, b, c in U, V, W , respec-
tively; and medians ma , mb , mc meeting a, b, c in L, M, N , respectively. Let

ma ∩ tb = P, mb ∩ tc = Q, mc ∩ ta = R.

Crux 588 [1980: 317] asks for a proof of the equality


AP BQ CR
· · = 8.
P L QM RN
Establish here the inequality
AR BP CQ
· · ≥ 8,
RU P V QW
with equality if and only if the triangle is equilateral.

689. Proposed by Jack Garfunkel, Flushing, N. Y.


Let ma , mb , mc denote the lengths of the medians to sides a, b, c, respectively, of triangle ABC,
and let Ma , Mb , Mc denote the lengths of these medians extended to the circumcircle of the
triangle. Prove that
Ma Mb Mc
+ + ≥ 4.
ma mb mc
696. Proposed by George Tsintsifas, Thessaloniki, Greece.
Let ABC be a triangle; a, b, c its sides; and s, r, R its semiperimeter, inradius and circumradius.
Prove that, with sums cyclic over A, B, C,
3 1X B−C X
(a) + cos ≥ cos A;
4 4 2
µ ¶
X B−C 2r
(b) a cos ≥s 1+ .
2 R

16
697. Proposed by G. C. Giri, Midnapore College, West Bengal, India.
Let
a = tan θ + tan φ, b = sec θ + sec φ, c = csc θ + csc φ.

If the angles θ and φ such that the requisite functions are defined and bc 6= 0, show that
2a/bc < 1.

700. Proposed by Jordi Dou, Barcelona, Spain.


Construct the centre of the ellipse of minimum excentricity circumscribed to a given convex
quadrilateral.

706. Proposed by J. T. Groenman, Arnhem, The Netherlands.


Let F (x) = 7x11 + 11x7 + 10ax, where x ranges over the set of all integers. Find the smallest
positive integer a such that 77|F (x) for every x.

708. Proposed by Vedula N. Murty, Pennsylvania State University, Capitol Campus.


A triangle has sides a, b, c, semiperimeter s, inradius r, and circumradius R.
(a) Prove that
(2a − s)(b − c)2 + (2b − s)(c − a)2 + (2c − s)(a − b)2 ≥ 0,
with equality just when the triangle is equilateral.
(b) Prove that the inequality in (a) is equivalent to each of the following:

3(a3 + b3 + c3 + 3abc) ≤ 4s(a2 + b2 + c2 ),


s2 ≥ 16Rr − 5r 2 .

715. Proposed by Vedula N. Murty, Pennsylvania State University, Capitol Campus.


Let k be a real number, n an integer, and A, B, C the angles of a triangle.
(a) Prove that
8k(sin nA + sin nB + sin nC) ≤ 12k 2 + 9.
(b) Determine for which k equality is possible in (a), and deduce that

3 3
| sin nA + sin nB + sin nC| ≤ .
2
718. Proposed by George Tsintsifas, Thessaloniki, Greece.
ABC is an acute-angled triangle with circumcenter O. The lines AO, BO, CO intersect BC,
CA, AB in A1 , B1 , C1 , respectively. Show that
3
OA1 + OB1 + OC1 ≥ R,
2
where R is the circumradius.

723. Proposed by George Tsintsifas, Thessaloniki, Greece.


Let G be the centroid of a triangle ABC, and suppose that AG, BG, CG meet the circumcircle
of the triangle again in A0 , B 0 , C 0 , respectively. Prove that

(a) GA0 + GB 0 + GC 0 ≥ AG + BG + CG;


AG BG CG
(b) 0
+ 0
+ = 3;
GA GB GC 0
(c) GA0 · GB 0 · GC 0 ≥ AG · BG · CG.

17
729. Proposed jointly by Dick Katz and Dan Sokolowsky, California State University at Los
Angeles.
Given a unit square, let K be the area of a triangle which covers the square. Prove that K ≥ 2.

732. Proposed by J. T. Groenman, Arnhem, The Netherlands.


C′
Given is a fixed triangle ABC with angles α, β, γ and a variable C
circumscribed triangle A0 B 0 C 0 determined by an angle φ ∈ [0, π), γ φ
as shown in the figure. It is easy to show that triangles ABC and
A0 B 0 C 0 are directly similar. φ
B′
(a) Find a formula for the ratio of similitude α β
A φ B
B0C 0
λ = λ(φ) = .
BC A′

(b) Find the maximal value λm of λ as φ varies in [0, π), and show how to construct triangle
A0 B 0 C 0 when λ = λm .
(c) Prove that λm ≥ 2, with equality just when triangle ABC is equilateral.

733F . Proposed by Jack Garfunkel, Flushing, N. Y.


A triangle has sides a, b, c, and the medians of this triangle are used as sides of a new triangle.
If rm is the inradius of this new triangle, prove or disprove that

3abc
rm ≤ ,
4(a2 + b2 + c2 )

with equality just when the original triangle is equilateral.

736. Proposed by George Tsintsifas, Thessaloniki, Greece.


Given is a regular n-gon V1 V2 . . . Vn inscribed in a unit circle. Show how to select, among the n
vertices Vi , three vertices A, B, C such that
(a) The area of triangle ABC is a maximum;
(b) The perimeter of triangle ABC is a maximum.

743. Proposed by George Tsintsifas, Thessaloniki, Greece.


Let ABC be a triangle with centroid G inscribed in a circle with center O. A point M lies on
the disk ω with diameter OG. The lines AM , BM , CM meet the circle again in A0 , B 0 , C 0 ,
respectively, and G0 is the centroid of triangle A0 B 0 C 0 . Prove that
(a) M does not lie in the interior of the disk ω 0 with diameter OG0 ;
(b) [ABC] ≤ [A0 B 0 C 0 ], where the brackets denote area.

762. Proposed by J. T. Groenman, Arnhem, The Netherlands.


ABC is a triangle with area K and sides a, b, c in the usual order. The internal bisectors of
angles A, B, C meet the opposite sides in D, E, F , respectively, and the area of triangle DEF
is K 0 .
(a) Prove that

3abc K0 1
3 3 3
≤ ≤ .
4(a + b + c ) K 4

(b) If a = 5 and K 0 /K = 5/24, determine b and c, given that they are integers.

18
768. Proposed by Jack Garfunkel, Flushing, N. Y.; and George Tsintsifas, Thessaloniki, Gree-
ce.
If A, B, C are the angles of a triangle, show that
4X Y B−C 2X
sin B sin C ≤ cos ≤ cos A,
9 2 3
where the sums and product are cyclic over A, B, C.

770. Proposed by Kesiraju Satyanarayana, Gagan Mahal Colony, Hyderabad, India.


Let P be an interior point of triangle ABC. Prove that

P A · BC + P B · CA > P C · AB.

787. Proposed by J. Walter Lynch, Georgia Southern College.


(a) Given two sides, a and b, of a triangle, what should be the length of the third side, x, in
order that the area enclosed be a maximum?
(b) Given three sides, a, b and c, of a quadrilateral, what should be the length of the fourth
side, x, in order that the area enclosed be a maximum?

788. Proposed by Meir Feder, Haifa, Israel.


A pandigital integer is a (decimal) integer containing each of the ten digits exactly once.
(a) If m and n are distinct pandigital perfect squares, what is the smallest possible value of
√ √
| m − n|?
√ √
(b) Find two pandigital perfect squares m and n for which this minimum value of | m − n|
is attained.

790. Proposed by Roland H. Eddy, Memorial University of Newfoundland.


Let ABC be a triangle with sides a, b, c in the usual order, and let la , lb , lc and la0 , lb0 , lc0 be two
sets of concurrent cevians, with la , lb , lc intersecting a, b, c in L, M , N , respectively. If

la ∩ lb0 = P, lb ∩ lc0 = Q, lc ∩ la0 = R,

prove that, independently of the choice of concurrent cevians la0 , lb0 , lc0 , we have
AP BQ CR abc
· · = ≥ 8,
P L QM RN BL · CM · AN
with equality occuring just when la , lb , lc are the medians of the triangle.
(This problem extends Crux 588 [1981: 306].)

793. Proposed by Vedula N. Murty, Pennsylvania State University, Capitol Campus.


Consider the following double inequality for the Riemann Zeta function: for n = 1, 2, 3, . . .,
1 1
+ ζn (s) < ζ(s) < ζn (s) + ,(1)
(s − 1)(n + 1)(n + 2) · · · (n + s − 1) (s − 1)n(n + 1) · · · (n + s − 2)
where ∞ n
X 1 X 1
ζ(s) = and ζn (s) = .
ks ks
k=1 k=1

Go as far as you can in determining for which of the integers s = 2, 3, 4, . . . the inequalities (1)
hold. (N. D. Kazarinoff asks for a proof that (1) holds for s = 2 in his Analytic Inequalities, Holt,
Rinehart & Winston, 1964, page 79; and Norman Schaumberger asks for a proof of disproof that
(1) holds for s = 3 in The Two-Year College Mathematics Journal, 12 (1981) 336.)

19
795. Proposed by Jack Garfunkel, Flushing, N. Y.
Given a triangle ABC, let ta , tb , tc be the lengths of its internal angle bisectors, and let Ta , Tb ,
Tc be the lengths of these bisectors extended to the circumcircle of the triangle. Prove that
4
Ta + Tb + Tc ≥ (ta + tb + tc ).
3
805. Proposed by Murray S. Klamkin, University of Alberta.
If x, y, z > 0, prove that
x+y+z yz + zx + xy
√ ≥p √ p ,
3 3 y + yz + z + z 2 + zx + x2 + x2 + xy + y 2
2 2

with equality if and only if x = y = z.

808F . Proposed by Stanley Rabinowitz, Digital Equipment Corp., Merrimack, New Hampshi-
re.
Find the length of the largest circular arc contained within the right triangle with sides a ≤ b < c.

815. Proposed by J. T. Groenman, Arnhem, The Netherlands.


Let ABC be a triangle with sides a, b, c, internal angle bisectors ta , tb , tc , and semiperimeter s.
Prove that the following inequalities hold, with equality if and only if the triangle is equilateral:
µ ¶
√ 1 1 1 4s
(a) 3 + + ≥ ;
ata btb ctc abc
1 1 1 s
√ ata + btb + ctc 2s
(b) 3 3 · ≥ 4 .
ata + btb + ctc (abc)3

816. Proposed by George Tsintsifas, Thessaloniki, Greece.


Let a, b, c be the sides of a triangle with semiperimeter s, inradius r, and circumradius R. Prove
that, with sums and product cyclic over a, b, c,
Y
(a) (b + c) ≤ 8sR(R + 2r),
X
(b) bc(b + c) ≤ 8sR(R + r),
X
(c) a3 ≤ 8s(R2 − r2 ).

823. Proposé par Olivier Lafitte, élève de Mathématiques Supérieures au Lycée Montaigne à
Bordeaux, France.
(a) Soit {a1 , a2 , a3 , . . .} une suite de nombres réels strictement positifs. Si
µ ¶
a1 + an+1 n
vn = , n = 1, 2, 3, . . . ,
an
montrer que lim sup vn ≥ e.
n→∞
(b) Trouver une suite {an } pour laquelle intervient l’égalité dans (a).

20
825F . Proposed by Jack Garfunkel, Flushing, N. Y.
Of the two triangle inequalities (with sum and product cyclic over A, B, C)
X A Y A
tan2 ≥1 and 2−8 sin ≥ 1,
2 2
Q
the first is well known and the second is equivalent to the well-known inequality sin(A/2) ≤
1/8. Prove or disprove the sharper inequality
X A Y A
tan2 ≥2−8 sin .
2 2

826F . Proposed by Kent D. Boklan, student, Massachusetts Institute of Technology.


It is a well-known consequence of the pingeonhole principle that, if six circles in the plane have
a point in common, the one of the circles must entirely contain a radius of another.
Suppose n spherical balls have a point in common. What is the smallest value of n for which it
can be said that one ball must entirely contain a radius of another?

832. Proposed by Richard A. Gibbs, Fort Lewis College, Durango, Colorado.


Let S be a subset of an m × n rectangular array of points, with m, n ≥ 2. A circuit in S is a
simple (i.e., nonself-intersecting) polygonal closed path whose vertices form a subset of S and
whose edges are parallel to the sides of the array.
Prove that a circuit in S always exists for any subset S with S ≥ m + n, and show that this
bound is best possible.

835. Proposed by Jack Garfunkel, Flushing, N. Y.; and George Tsintsifas, Thessaloniki, Gree-
ce.
Let ABC be a triangle with sides a, b, c, and let Rm be the circumradius of the triangle formed
by using as sides the medians of triangle ABC. Prove that

a2 + b2 + c 2
Rm ≥ .
2 (a + b + c)

836. Proposed by Vedula N. Murty, Pennsylvania State University, Capitol Campus.


(a) If A, B, C are the angles of a triangle, prove that

(1 − cos A)(1 − cos B)(1 − cos C) ≥ cos A cos B cos C,

with equality if and only if the triangle is equilateral.


(b) Deduce from (a) Bottema’s triangle inequality [1982: 296]:

(1 + cos 2A)(1 + cos 2B)(1 + cos 2C) + cos 2A cos 2B cos 2C ≥ 0.

843. Proposed by J. L. Brenner, Palo Alto, California.


For integers m > 1 and n > 2, and real numbers p, q > 0 such that p + q = 1, prove that

(1 − pm )n + npm (1 − pm )n−1 + (1 − q n − npq n−1 )m > 1.

21
846. Proposed by Jack Garfunkel, Flushing, N. Y.; and George Tsintsifas, Thessaloniki, Gree-
ce.
Given is a triangle ABC with sides a, b, c and medians ma , mb , mc in the usual order, circumra-
dius R, and inradius r. Prove that
ma mb mc
(a) ≥ r;
+ m2b + m2c
m2a

(b) 12Rma mb mc ≥ a(b + c)m2a + b(c + a)m2b + c(a + b)m2c ;


(c) 4R(ama + bmb + cmc ) ≥ bc(b + c) + ca(c + a) + ab(a + b);
µ ¶
1 1 1 ma mb mc
(d) 2R + + ≥ + + .
bc ca ab mb mc mc ma ma mb

850. Proposed by Vedula N. Murty, Pennsylvania State University, Capitol Campus.


Let x = r/R and y = s/R, where r, R, s are the inradius, circumradius, and semiperimeter,
respectively, of a triangle with side lengths a, b, c. Prove that
√ √ √
y ≥ x ( 6 + 2 − x),

with equality if and only if a = b = c.

854. Proposed by George Tsintsifas, Thessaloniki, Greece.


For x, y, z > 0, let
yz zx xy
A= + +
(y + z)2 (z + x)2 (x + y)2
and
yz zx xy
B= + + .
(y + x)(z + x) (z + y)(x + y) (x + z)(y + z)
It is easy to show that a ≤ 34 ≤ B, with equality if and only if x = y = z.
(a) Show that the inequality a ≤ 34 is “weaker”than 3B ≥ 49 in the sense that
3 9
A + 3B ≥ + = 3.
4 4
When does equality occur?
(b) Show that the inequality 4A ≤ 3 is “stronger” than 8B ≥ 6 in the sense that

4A + 8B ≥ 3 + 6 = 9.

When does equality occur?

856. Proposed by Jack Garfunkel, Flushing, N. Y.


For a triangle ABC with circumradius R and inradius r, let M = (R − 2r)/2R. An inequality
P ≥ Q involving elements of triangle ABC will be called strong or weak, respectively, according
as P − Q ≤ M or P − Q ≥ M .
(a) Prove that the following inequality is strong:
A B C 3
sin2 + sin2 + sin2 ≥ .
2 2 2 4
(b) Prove that the following inequality is weak:
A B C
cos2 + cos2 + cos2 ≥ sin B sin C + sin C sin A + sin A sin B.
2 2 2

22
859. Proposed by Vedula N. Murty, Pennsylvania State University, Capitol Campus.
Let ABC be an acute-angled triangle of type II, that is (see [1982: 64]), such that A ≤ B ≤ π3 ≤
C, with circumradius R and inradius r. It is known [1982: 66] that for such a triangle x ≥ 41 ,
where x = r/R. Prove the stronger inequality

3−1
x≥ .
2
862. Proposed by George Tsintsifas, Thessaloniki, Greece.
A
P is an interior point of a triangle ABC. Lines through P par-
B1
allel to the sides of the triangle meet those sides in the points C2
A1 , A2 , B1 , B2 , C1 , C2 , as shown in the figure. Prove that A2 A1
1
(a) [A1 B1 C1 ] ≤ [ABC],
3
B C1 B2 C
2
(b) [A1 C2 B1 A2 C1 B2 ] ≤ [ABC],
3
where the brackets denote area.

864. Proposed by J. T. Groenman, Arnhem, The Netherlands.


Find all x between 0 and 2π such that

2 cos2 3x − 14 cos2 2x − 2 cos 5x + 24 cos 3x − 89 cos 2x + 50 cos x > 43.

866. Proposed by Jordi Dou, Barcelona, Spain.


Given a triangle ABC with sides a, b, c, find the minimum value of

a · XA + b · XB + c · XC,

where X ranges over all the points of the plane of the triangle.

870F . Proposed by Sidney Kravitz, Dover, New Jersey.


Of all the simple closed curves which are inscribed in a unit square (touching all four sides), find
the one which has the minimum ratio of perimeter to enclosed area.

882. Proposed by George Tsintsifas, Thessaloniki, Greece.


The interior surface of a wine glass is a right circular cone. The glass, containing some wine,
is first held upright, then tilted slightly but not enough to spill any wine. Let D and E denote
the area of the upper surface of the wine and the area of the curved surface in contact with the
wine, respectively, when the glass is upright; and let D1 and E1 denote the corresponding areas
when the glass is tilted. Prove that
D1 D
(a) E1 ≥ E, (b) D1 + E1 ≥ D + E, (c) ≥ .
E1 E
882. Proposed by George Tsintsifas, Thessaloniki, Greece.
The interior surface of a wine glass is a right circular cone. The glass, containing some wine,
is first held upright, then tilted slightly but not enough to spill any wine. Let D and E denote
the area of the upper surface of the wine and the area of the curved surface in contact with the
wine, respectively, when the glass is upright; and let D1 and E1 denote the corresponding areas
when the glass is tilted. Prove that
D1 D
(a) E1 ≥ E, (b) D1 + E1 ≥ D + E, (c) ≥ .
E1 E

23
883. Proposed by J. Tabov and S. Troyanski, Sofia, Bulgaria.
Let ABC be a triangle with area S, sides a, b, c, medians ma , mb , mc , and interior angle bisectors
ta , tb , tc . If

ta ∩ mb = F, tb ∩ mc = G, tc ∩ ma = H,

prove that
σ 1
< ,
S 6
where σ denotes the area of triangle F GH.

895. Proposed by J. T. Groenman, Arnhem, The Netherlands.


Let ABC be a triangle with sides a, b, c in the usual order and circumcircle Γ . A line l through C
meets the segment AB in D, Γ again in E, and the perpendicular bisector of AB in F . Assume
that c = 3b.
(a) Construct the line l for which the length of DE is maximal.
(b) If DE has maximal length, prove that DF = F E.
(c) If DE has maximal length and also CD = DF , find a in terms of b and the measure of
angle A.

896. Proposed by Jack Garfunkel, Flushing, N. Y.


Consider the inequalities
X A 1Y B−C 3
sin2 ≥1− cos ≥ ,
2 4 2 4
where the sum and product are cyclic over the angles A, B, C of a triangle. The inequality
between the second and third members is obvious, and that between the first and third members
is well known. Prove the sharper inequality between the first two members.

897. Proposed by Vedula N. Murty, Pennsylvania State University, Capitol Campus.


If λ > µ and a ≥ b ≥ c > 0, prove that

b2λ c2µ + c2λ a2µ + a2λ b2µ ≥ (bc)λ+µ + (ca)λ+µ + (ab)λ+µ ,

with equality just when a = b = c.

899. Proposed by Loren C. Larson, St. Olaf College, Northfield, Minnesota.


Let {ai } and {bi }, i = 1, 2, . . . , n, be two sequences of real numbers with the ai all positive.
Prove that
X X
ai bj = 0 =⇒ bi bj ≤ 0.
i6=j i6=j

908. Proposed by Murray S. Klamkin, University of Alberta.


Determine the maximum value of

P ≡ sinα A · sinβ B · sinγ C,

where A, B, C are the angles of a triangle and α, β, γ are given positive numbers.

24
914. Proposed by Vedula N. Murty, Pennsylvania State University, Capitol Campus.
If a, b, c > 0, then the equation x3 − (a2 + b2 + c2 )x − 2abc = 0 has a unique positive root x0 .
Prove that
2
(a + b + c) ≤ x0 < a + b + c.
3

915F . Proposed by Jack Garfunkel, Flushing, N. Y.


If x + y + z + w = 180 ◦, prove or disprove that

sin(x + y) + sin(y + z) + sin(z + w) + sin(w + x) ≥ sin 2x + sin 2y + sin 2z + sin 2w,

with equality just when x = y = z = w.

922F . Proposed by A. W. Goodman, University of South Florida.


Let
n−1
n(n − 1) X
Sn (z) = + (n − k)2 z k ,
2
k=1

where z = e i θ . Prove that, for all real θ,

sin θ
< (Sn (z)) = (n sin θ − sin nθ) ≥ 0.
2(1 − cos θ)2

939. Proposed by George Tsintsifas, Thessaloniki, Greece.


Triangle ABC is acute-angled at B, and AB < AC. M being a point on the altitude AD, the
lines BM and CM intersect AC and AB, respectively, in B 0 and C 0 . Prove that BB 0 < CC 0 .

940. Proposed by Jack Garfunkel, Flushing, N. Y.


Show that, for any triangle ABC,

7 A B C 9
sin B sin C + sin C sin A + sin A sin B ≤ + 4 sin sin sin ≤ .
4 2 2 2 4

948. Proposed by Vedula N. Murty, Pennsylvania State University, Capitol Campus.


If a, b, c are the side lengths of a triangle of area K, prove that

27K 4 ≤ a3 b3 c2 ,

and determine when equality occurs.

952. Proposed by Jack Garfunkel, Flushing, N. Y.


Consider the following double inequality, where the sum and product are cyclic over the angles
A, B, C of a triangle:
µ ¶
X Y A 9
sin2 A ≤ 2 + 16 sin2 ≤ .
2 4

The inequality between the first and third membersQis well


¡ Aknown,
¢ 1 and that between the second
and third members is equivalent to the well-known sin 2 ≤ 8 . Prove the inequality between
the first and second members.

25
954. Proposed by W. J. Blundon, Memorial University of Newfoundland.
The notation being the usual one, prove that each of the following is a necessary and sufficient
condition for a triangle to be acute-angled:

(a) IH < r 2,
(b) OH < R,
(c) cos2 A + cos2 B + cos2 C < 1,
(d) r2 + ra2 + rb2 + rc2 < 8R2 ,
(e) m2a + m2b + m2c > 6R2 .

955. Proposed by Geng-zhe Chang, University of Science and Technology of China, Hefei,
Anhui, People’s Republic of China.
If the real numbers A, B, C, a, b, c satisfy

A + a ≥ b + c, B + b ≥ c + a, C + c ≥ a + b,

show that

Q ≡ Ax2 + By 2 + Cz 2 + 2ayz + 2bzx + 2cxy ≥ 0

holds for all real x, y, z such that x + y + z = 0.

957. Proposed by George Tsintsifas, Thessaloniki, Greece.


Let a, b, c be the sides of a triangle with circumradius R and area K. Prove that

bc ca ab 2K
+ + ≥ ,
b+c c+a a+b R
with equality if and only if the triangle is equilateral.

958. Proposed by Murray S. Klamkin, University of Alberta.


If A1 , A2 , A3 are the angles of a triangle, prove that

tan A1 + tan A2 + tan A3 ≥ or ≤ 2(sin 2A1 + sin 2A2 + sin 2A3 )

according as the triangle is acute-angled or obtuse-angled, respectively. When is there equality?

959. Proposed by Sidney Kravitz, Dover, New Jersey.


Two houses are located to the north of a straight east-west highway. House A is at a perpendicular
distance a from the road, house B is at a perpendicular distance b ≥ a from the road, and the
feet of the perpendiculars are one unit apart. Design a road system of minimum total length (as
a function of a and b) to connect both houses to the highway.

965. Proposed by George Tsintsifas, Thessaloniki, Greece.


Let A1 A2 A3 be a nondegenerate triangle with sides A2 A3 = a1 , A3 A1 = a2 , A1 A2 = a3 , and let
P Ai = xi (i = 1, 2, 3), where P is any point in space. Prove that
x1 x2 x3 √
+ + ≥ 3,
a1 a2 a3

and determine when equality occurs.

26
968. Proposed by J. T. Groenman, Arnhem, The Netherlands.
For real numbers a, b, c, let Sn = an + bn + cn . If S1 ≥ 0, prove that

12S5 + 33S1 S22 + 3S15 + 6S12 S3 ≥ 12S1 S4 + 10S2 S3 + 20S13 S2 .

When does equality occur?

970F . Proposed by Walther Janous, Ursulinengymnasium, Innsbruck, Austria.


Let a, b, c and ma , mb , mc denote the side lengths and median lengths of a triangle. Find the set
of all real t and, for each such t, the largest positive constant λt , such that
ma mb mc mt + mtb + mtc
≥ λt · a
abc a+b+c
holds for all triangles.

972F . Proposed by Stanley Rabinowitz, Digital Equipment Corp., Nashua, New Hampshire.
(a) Prove that two equilateral triangles of unit side cannot be placed inside a unit square
without overlapping.
(b) What is the maximum number of regular tetrahedra of unit side that can be packed without
overlapping inside a unit cube?
(c) Generalize to higher dimensions.

974. Proposed by Jack Garfunkel, Flushing, N. Y.


Consider the following double inequality, where A, B, C are the angles of any triangle:
A B C 1
cos A cos B cos C ≤ 8 sin2 sin2 sin2 ≤ .
2 2 2 8
The inequality involving the first and third members and that involving the second and third
members are both well known. Prove the inequality involving the first and second members.

978. Proposed by Andy Liu, University of Alberta.


Determine the smallest positive integer m such that

529n + m · 132n

is divisible by 262417 for all odd positive integers n.

982. Proposed by George Tsintsifas, Thessaloniki, Greece.


Let P and Q be interior points of triangle A1 A2 A3 . For i = 1, 2, 3, let P Ai = xi , QAi = yi , and
let the distances from P and Q to the side opposite Ai be pi and qi , respectively. Prove that
√ √ √ √ √ √
x1 y1 + x2 y2 + x3 y3 ≥ 2( p1 q1 + p2 q2 + p3 q3 ).

When P = Q, this reduces to the well-known Erdös-Mordell inequality.


(See the article by Clayton W. Dodge in this journal [1984: 274–281].)

987F . Proposed by Jack Garfunkel, Flushing, N. Y.


If triangle ABC is acute-angled, prove or disprove that
µ ¶
A B C 4 A B C
(a) sin + sin + sin ≥ 1 + sin sin sin ,
2 2 2 3 2 2 2
µ ¶
A B C 4 A B C
(b) cos + cos + cos ≥ √ 1 + sin sin sin .
2 2 2 3 2 2 2

27
992. Proposed by Harry D. Ruderman, Bronx, N. Y.
Let α = (a1 , a2 , . . . , amn ) be a sequence of positive real numbers such that ai ≤ aj whenever
i < j, and let β = (b1 , b2 , . . . , bmn ) be a permutation of α. Prove that
n Y
X m n Y
X m
(a) am(j−1)+i ≥ bm(j−1)+i ;
j=1 i=1 j=1 i=1

Y m
n X n X
Y m
(b) am(j−1)+i ≤ bm(j−1)+i .
j=1 i=1 j=1 i=1

993. Proposed by Walther Janous, Ursulinengymnasium, Innsbruck, Austria.


Let P be the product of the n + 1 positive real numbers x1 , x2 , . . . , xn+1 . Find a lower bound
(as good as possible) for P if the xi satisfy
n+1
X 1
(a) = 1;
1 + xi
i=1
n+1
X ai
(b)F = 1, where the ai and bi are given positive real numbers.
bi + xi
i=1

999F . Proposed by Jack Garfunkel, Flushing, N. Y.


Let R, r, s be the circumradius, inradius, and semiperimeter, respectively, of an acute-angled
triangle. Prove or disprove that

s2 ≥ 2R2 + 8Rr + 3r 2 .

When does equality occur?

1003F . Proposed by Murray S. Klamkin, University of Alberta.


Without using tables or a calculator, show that
µ ¶2
2 5
ln 2 > .
5

1006. Proposed by Hans Havermann, Weston, Ontario.


Given a base-ten positive integer of two or more digits, it is possible to spawn two smaller base-
ten integers by inserting a space somewhere within the number. We call the left offspring thus
created the farmer (F) and the value of the right one (ignoring leading zeros, if any) the ladder
(L). A number is called modest if it has an F and an L such that the number divided by L
leaves remainder F. (For example, 39 is modest.)
Consider, for n > 1, a block of n consecutive positive integers all of which are modest. If the
smallest and largest of these are a and b, respectively, and if a − 1 and b + 1 are not modest,
then we say that the block forms a multiple berth of size n. A multiple berth of size 2 is called
a set of twins, and the smallest twins are {411, 412}. A multiple berth of size 3 is called a set of
triplets, and the smallest triplets are {4000026, 4000027, 4000028}.
(a) Find the smallest quadruplets.
(b)F Find the smallest quintuplets. (There are none less than 25 million.)

28
1012. Proposed by G. P. Henderson, Campbellcroft, Ontario.
An amateur winemaker is siphoning wine from a carboy. To speed up the process, he tilts the
carboy to raise the level of the wine. Naturally, he wants to maximize the height, H, of the
surface of the liquid above the table on which the carboy rests. The carboy is actually a circular
cylinder, but we will only assume that its base is the interior of a smooth closed convex curve,
C, and that the generators are perpendicular to the base. P is a point on C, T is the line tangent
to C at P , and the cylinder is rotated about T .
(a) Prove that H is a maximum when the centroid of the surface of the liquid is vertically
above T .
(b) Let the volume of the wine be V and let the area inside C be A. Assume that V ≥ AW/2,
where W is the maximum width of C (i. e., the maximum distance between parallel tangents).
Obtain an explicit formula for HM , the maximum value of H. How should P be chosen to
maximize HM ?

1019. Proposed by Weixuan Li and Edward T. H. Wang, Wilfrid Laurier University, Water-
loo, Ontario.
Determine the largest constant k such that the inequality
x ≤ α sin x + (1 − α) tan x
£ ¢
holds for all α ≤ k and for all x ∈ 0, π2 .
(The inequality obtained when α is replaced by 23 is the Snell-Huygens inequality, which is fully
discussed in Problem 115 [1976: 98–99, 111–113, 137–138].)

1025. Proposed by Peter Messer, M. D., Mequon, Wisconsin. A C B


A paper square ABCD is folded so that vertex C falls on
AB and side CD is divided into two segments of lengths l
l
and m, as shown in the figure. Find the minimum value of
the ratio l/m.
D m
1030. Proposed by J. T. Groenman, Arnhem, The Netherlands.
Given are two obtuse triangles with sides a, b, c and p, q, r, the longest sides of each being c and
r, respectively. Prove that
ap + bq < cr.
1036. Proposed by Gali Salvatore, Perkins, Québec.
Find sets of positive numbers {a, b, c, d, e, f } such that, simultaneously,
abc a+b+c a b c d e f
< 1, < 1, + + > 3, + + > 3,
def d+e+f d e f a b c
or prove that there are none.

1045. Proposed by George Tsintsifas, Thessaloniki, Greece.


Let P be an interior point of triangle ABC; let x, y, z be the distances of P from vertices A, B,
C, respectively; and let u, v, w be the distances of P from sides BC, CA, AB, respectively. The
well-known Erdös-Mordell inequality states that
x + y + z ≥ 2(u + v + w).
Prove the following related inequalities:
x2 y2 z2 x y z
(a) + + ≥ 12, (b) + + ≥ 3,
vw wu uv v+w w+u u+v
x y z
(c) √ +√ + √ ≥ 6.
vw wu uv

29
1046. Proposed by Jordan B. Tabov, Sofia, Bulgaria.
The Wallace point W of any four points A1 , A2 , A3 , A4 on a circle with center O may be defined
by the vector equation
³ ´
−−→ 1 −−→ −−→ −−→ −−→
OW = OA1 + OA2 + OA3 + OA4
2
(see the article by Bottema and Groenman in this journal [1982: 126]).
Let γ be a cyclic quadrilateral the Wallace point of whose vertices lies inside γ. Let a i (i =
1, 2, 3, 4) be the sides of γ, and let Gi be the midpoint of the side opposite to ai . Find the
minimum value of

f (X) ≡ a1 · G1 X + a2 · G2 X + a3 · G3 X + a4 · G4 X,

where X ranges over all the points of the plane of γ.

1049F . Proposed by Jack Garfunkel, Flushing, N. Y.


Let ABC and A0 B 0 C 0 be two nonequilateral triangles such that A ≥ B ≥ C and A0 ≥ B 0 ≥ C 0 .
Prove that
s s0
A − C > A0 − C 0 ⇐⇒ > 0,
r r
where s, r and s0 , r0 are the semiperimeter and inradius of triangles ABC and A0 B 0 C 0 , respec-
tively.

1051. Proposed by George Tsintsifas, Thessaloniki, Greece.


Let a, b, c be the side lengths of a triangle of area K, and let u, v, w be positive real numbers.
Prove that
ua4 vb4 wc4
+ + ≥ 8K 2 .
v+w w+u u+v
When does equality occur? Some interesting triangle inequalities may result if we assign specific
values to u, v, w. Find a few.

1057. Proposed by Jordi Dou, Barcelona, Spain.


Let Ω be a semicircle of unit radius, with diameter AA0 . Consider a sequence of circles γi , all
interior to Ω, such that γ1 is tangent to Ω and to AA0 , γ2 is tangent to Ω and to the chord
AA1 tangent to γ1 , γ3 is tangent to Ω and to the chord AA2 tangent to γ2 , etc. Prove that

r1 + r2 + r3 + · · · < 1,

where ri is the radius of γi .

1058. Proposed by Jordan B. Tabov, Sofia, Bulgaria.


Two points X and Y are choosen at random, independently and uniformly with respect to
length, on the edges of a unit cube. Determine the probability that

1 < XY < 2.

1060. Proposed by Murray S. Klamkin, University of Alberta.


If ABC is an obtuse triangle, prove that

sin2 A tan A + sin2 B tan B + sin2 C tan C < 6 sin A sin B sin C.

30
1064. Proposed by George Tsintsifas, Thessaloniki, Greece.
Triangles ABC and DEF are similar, with angles A = D, B = E, C = F and ratio of similitude
λ = EF/BC. Triangle DEF is inscribed in triangle ABC, with D, E, F on the lines BC, CA,
AB, not necessarily respectively. Three cases can be considered:

Case 1: D ∈ BC, E ∈ CA, F ∈ AB;


Case 2: D ∈ CA, E ∈ AB, F ∈ BC;
Case 3: D ∈ AB, E ∈ BC, F ∈ CA.
1
For Case 1, it is known that λ ≥ 2 (see Crux 606 [1982: 24, 108]). Prove that, for each of Cases
2 and 3,
λ ≥ sin ω,
where ω is the Brocard angle of triangle ABC. (This inequality also holds a fortiori for Case 1,
since ω ≤ 30 ◦.)

1065. Proposed by Jordan B. Tabov, Sofia, Bulgaria.


The orthocenter H of an orthocentric tetrahedron ABCD lies inside the tetrahedron. If X ranges
over all the points of space, find the minimum value of
f (X) = {BCD} · AX + {CDA} · BX + {DAB} · CX + {ABC} · DX,
where the braces denote the (unsigned) area of a triangle.
(This is an extension to 3 dimensions of Crux 866 [1984: 327].)

1066F . Proposed by D. S. Mitrinović, University of Belgrade, Belgrade, Yugoslavia.


Consider the inequality

(y p + z p − xp )(z p + xp − y p )(xp + y p − z p )
≤ (y q + z q − xq )r (z q + xq − y q )r (xq + y q − z q )r .

(a) Prove that the inequality holds for all real x, y, z if (p, q, r) = (2, 1, 2).
(b) Determine all triples (p, q, r) of natural numbers for each of which the inequality holds for
all real x, y, z.

1067. Proposed by Jack Garfunkel, Flushing, N. Y.


(a)F If x, y, z > 0, prove that
p √
xyz(x + y + z + x2 + y 2 + z 2 ) 3+ 3
≤ .
(x2 + y 2 + z 2 )(yz + zx + xy) 9
(b) Let r be the inradius of a triangle and r1 , r2 , r3 the radii of its three Malfatti circles (see
Crux 618 [1982: 82]). Deduce from (a) that

3+ 3
r ≤ (r1 + r2 + r3 ) .
9
1075. Proposed by George Tsintsifas, Thessaloniki, Greece.
Let ABC be a triangle with circumcenter O and incenter I, and let DEF be the pedal triangle
of an interior point M of triangle ABC (with D on BC, etc.). Prove that
r
OM ≥ OI ⇐⇒ r 0 ≤ ,
2
where r and r 0 are the inradii of triangles ABC and DEF , respectively.

31
1077F . Proposed by Jack Garfunkel, Flushing, N. Y.
For i = 1, 2, 3, let Ci be the center and ri the radius of the Malfatti circle nearest Ai in triangle
A1 A2 A3 . Prove that
(r1 + r2 + r3 )3 − 3r1 r2 r3
A 1 C1 · A 2 C2 · A 3 C3 ≥ .
3
When does equality occur?

1079. Proposed by Walther Janous, Ursulinengymnasium, Innsbruck, Austria.


Let
X a b − 4c
g(a, b, c) = · ,
a + 2b b + 2c
where the sum is cyclic over the sides a, b, c of a triangle.
(a) Prove that − 35 < g(a, b, c) ≤ −1.
(b)F Find the greatest lower bound of g(a, b, c).

1080F . Proposed by D. S. Mitrinović, University of Belgrade, Belgrade, Yugoslavia.


Determine the maximum value of
¯ ¯
¯b − c c − a a − b¯
f (a, b, c) = ¯¯ + + ¯,
b + c c + a a + b¯
where a, b, c are the side lengths of a nondegenerate triangle.

1083F . Proposed by Jack Garfunkel, Flushing, N. Y.


Consider the double inequality
µ ¶
2 X X B−C 2 X A
√ sin A ≤ cos ≤√ cos ,
3 2 3 2
where the sums are cyclic over the angles A, B, C of a triangle. The left inequality has already
been established in this journal (Problem 613 [1982: 55, 67, 138]). Prove or disprove the right
inequality.

1085. Proposed by George Tsintsifas, Thessaloniki, Greece.


Let σn = A0 A1 . . . An be a regular n-simplex in Rn , and let πi be the hyperplane containing the
face σn−1 = A0 A1 . . . Ai−1 Ai+1 . . . An . If Bi ∈ πi for i = 0, 1, . . . , n, show that
X −−−→ n+1
|B i B j | ≥ e,
2
0≤i<j≤n

where e is the edge length of σn .

1086. Proposed by Murray S. Klamkin, University of Alberta.


The medians of an n-dimensional simplex A0 A1 . . . An in Rn intersect at the centroid G and are
extended to meet the circumsphere again in the points B0 , B1 , . . . , Bn , respectively.
(a) Prove that

A0 G + A1 G + · · · + An G ≤ B0 G + B1 G + · · · + Bn G.

(b)F Determine all other points P such that

A0 P + A1 P + · · · + An P ≤ B0 P + B1 P + · · · + Bn P.

32
1087. Proposed by Robert Downes, student, Moravian College, Bethlehem, Pennsylvania.
Let a, b, c, d be four positive numbers.
(a) There exists a regular tetrahedron ABCD and a point P in space such that P A = a,
P B = b, P C = c, and P D = d if and only if a, b, c, d satisfy what condition?
(b) This condition being satisfied, calculate the edge length of the regular tetrahedron ABCD.
(For the corresponding problem in a plane, see Problem 39 [1975: 64; 1976: 7].)

1088F . Proposed by Basil C. Rennie, James Cook University of North Queensland, Australia.
If R, r, s are the circumradius, inradius, and semiperimeter, respectively, of a triangle with largest
angle A, prove or disprove that

s T 2R + r according as A S 90 ◦.

1089. Proposed by J. T. Groenman, Arnhem, The Netherlands.


Find the range of the function f : R → R defined by

X
f (θ) = 3−k cos kθ, θ ∈ R.
k=1

1093F . Proposed by Jack Garfunkel, Flushing, N. Y.


Prove that
à P !3
sin A Y A
P ¡A¢ ≥ 8 sin ,
cos 2 2

where the sums and product are cyclic over the angles A, B, C of a triangle. When does equality
occur?

1095. Proposed by Edward T. H. Wang, Wilfrid Laurier University, Waterloo, Ontario.


Let Nn = {1, 2, . . . , n}, where n ≥ 4. A subset A of Nn with |A| ≥ 2 is called an RC-set
(relatively composite) if (a, b) > 1 for all a, b ∈ A. Let f (n) be the maximum cardinality of all
RC-sets A in Nn . Determine f (n) and find all RC-sets in Nn of cardinality f (n).

1096. Proposed by Murray S. Klamkin, University of Alberta.


Determine the maximum and minimum values of
A B C A B C
S ≡ cos cos cos + sin sin sin ,
4 4 4 4 4 4
where A, B, C are the angles of a triangle. (No calculus, please!)

1098. Proposed by Jordi Dou, Barcelona, Spain.


Characterize all trapezoids for which the circumscribed ellipse of minimal area is a circle.

1102. Proposed by George Tsintsifas, Thessaloniki, Greece.


Let σn = A0 A1 . . . An be an n-simplex in n-dimensional Euclidean space. Let M be an interior
point of σn whose barycentric coordinates are (λ0 , λ1 , . . . , λn ) and, for i = 0, 1, . . . , n, let pi be
its distances from the (n − 1)-face

σn−1 = A0 A1 . . . Ai−1 Ai+1 . . . An .

Prove that λ0 p0 + λ1 p1 + · · · + λn pn ≥ r, where r is the inradius of σn .

33
1111. Proposed by J. T. Groenman, Arnhem, The Netherlands.
Let α, β, γ be the angles of an acute triangle and let
α β β γ γ α
f (α, β, γ) = cos cos + cos cos + cos cos .
2 2 2 2 2 2

(a) Prove that f (α, β, γ) > 23 3 2.

(b)F Prove or disprove that f (α, β, γ) > 21 + 2.

1114. Proposed by George Tsintsifas, Thessaloniki, Greece.


Let ABC, A0 B 0 C 0 be two triangles with sides a, b, c, a0 , b0 , c0 and areas F , F 0 respectively. Show
that
√ √
aa0 + bb0 + cc0 ≥ 4 3 F F 0 .

1116. Proposed by David Grabiner, Claremont High School, Claremont, California.


(a) Let f (n) be the smallest positive integer which is not a factor of n. Continue the series
f (n), f (f (n)), f (f (f (n))), . . . until you reach 2. What is the maximum length of the series?
(b) Let g(n) be the second smallest positive integer which is not a factor of n. Continue the
series g(n), g(g(n)), g(g(g(n))), . . . until you reach 3. What is the maximum length of the series?

1120F . Proposed by D. S. Mitrinović, University of Belgrade, Belgrade, Yugoslavia.


(a) Determine a positive number λ so that

(a + b + c)2 (abc) ≥ λ(bc + ca + ab)(b + c − a)(c + a − b)(a + b − c)

holds for all real numbers a, b, c.


(b) As above, but a, b, c are assumed to be positive.
(c) As above, but a, b, c are assumed to satisfy

b + c − a > 0, c + a − b > 0, a + b − c > 0.

1125F . Proposed by Jack Garfunkel, Flushing, N. Y.


If A, B, C are the angles of an acute triangle ABC, prove that
A B C 3
cot + cot + cot ≤ (csc 2A + csc 2B + csc 2C)
2 2 2 2
with equality when triangle ABC is equilateral.

1126. Proposed by Péter Ivády, Budapest, Hungary.


For 0 < x ≤ 1, show that
3x
sinh x < √ < tan x.
2 + 1 − x2

1127F . Proposed by D. S. Mitrinović, University of Belgrade, Belgrade, Yugoslavia.


(a) Let a, b, c and r be real numbers > 1. Prove or disprove that

(loga bc)r + (logb ca)r + (logc ab)r ≥ 3 · 2r .

(b) Find an analogous inequality for n numbers a1 , a2 , . . . , an rather than three numbers a, b, c.

34
1129. Proposed by Donald Cross, Exeter, England.
(a) Show that every positive whole number ≥ 84 can be written as the sum of three positive
whole numbers in at least four ways (all twelve numbers different) such that the sum of the
squares of the three numbers in any group is equal to the sum of the squares of the three
numbers in each of the other groups.
(b) Same as part (a), but with “three” replaced by “four” and “twelve” by “sixteen”.
(c)F Is 84 minimal in (a) and/or (b)?

1130. Proposed by George Tsintsifas, Thessaloniki, Greece.


Show that
3 3 3 7 3
a2 + b2 + c2 ≤ 34 R2

where a, b, c are the sides of a triangle and R is the circumradius.

1131. Proposed by Murray S. Klamkin, University of Alberta, Edmonton, Alberta.


Let A1 A2 A3 be a triangle with sides a1 , a2 , a3 labelled as usual, and let P be a point in or out
of the plane of the triangle. It is a known result that if R1 , R2 , R3 are the distances from P to
the respective vertices A1 , A2 , A3 , then a1 R1 , a2 R2 , a3 R3 satisfy the triangle inequality, i. e.

a1 R1 + a2 R2 + a3 R3 ≥ 2ai Ri , i = 1, 2, 3. (1)

For the ai Ri to form a non-obtuse triangle, we would have to satisfy

a21 R12 + a22 R22 + a23 R32 ≥ 2a2i Ri2

which, however, need not be true. Show that nevertheless



a21 R12 + a22 R22 + a23 R32 ≥ 2a2i Ri2

which is a stronger inequality than (1).

1137F . Proposed by Walther Janous, Ursulinengymnasium, Innsbruck, Austria.


Prove or disprove the triangle inequality
1 1 1 5
+ + > ,
ma mb mc s
where ma , mb , mc are the medians of a triangle and s is its semiperimeter.

1142. Proposed by J. T. Groenman, Arnhem, The Netherlands.


Suppose ABC is a triangle whose median point lies on its inscribed circle.
(a) Find an equation relating the sides a, b, c of 4ABC.
(b) Assume a ≥ b ≥ c. Find an upper bound for a/c.
(c) Give an example of a triangle with integral sides having the above property.

1144. Proposed by George Tsintsifas, Thessaloniki, Greece.


Let ABC be a triangle and P an interior point at distances x1 , x2 , x3 from the vertices A, B,
C and distances p1 , p2 , p3 from the sides BC, CA, AB, respectively. Show that
x1 x2 x2 x3 x3 x1 ³p p p2 p3 p3 p1 ´
1 2
+ + ≥4 + + .
ab bc ca ab bc ca

35
1145. Proposed by Walther Janous, Ursulinengymnasium, Innsbruck, Austria.
Given a plane convex figure and a straight line l (in the same plane) which splits the figure into
two parts whose areas are in the ratio 1 : t (t ≥ 1). These parts are then projected orthogonally
onto a straight line n perpendicular to l. Determine, in terms of t, the maximum ratio of the
lengths of the two projections.

1148. Proposed by Stanley Rabinowitz, Digital Equipment Corp., Nashua, New Hampshire.
Find the triangle of smallest area that has integral sides and integral altitudes.

1150F . Proposed by Jack Garfunkel, Flushing, N. Y.


M1

In the figure, 4M1 M2 M3 and the three circles


G2
with centers O1 , O2 , O3 represent the Malfatti G3 O
configuration. Circle O is externally tangent O1
to these three circles and the sides of triangle
G1 G2 G3 are each tangent to O and one of the
O2 O3
smaller circles. Prove that M2
M3
P(4G1 G2 G3 ) ≥ P(4M1 M2 M3 ) + P(4O1 O2 O3 ),
G1
where P stands for perimeter. Equality is attained when 4O1 O2 O3 is equilateral.

1151F . Proposed by Jack Garfunkel, Flushing, N. Y.


Prove (or disprove) that for an obtuse triangle ABC,

ma + mb + mc ≤ s 3,
where ma , mb , mc denote the medians to sides a, b, c and s denotes the semiperimeter of 4ABC.
Equality is attained in the equilateral triangle.

1152. Proposed by J. T. Groenman, Arnhem, The Netherlands.


Prove that

X α 3X 1
cos ≤ cos (β − γ),
2 2 4
where α, β, γ are the angles of a triangle and the sums are cyclic over these angles.

1154. Proposed by Walther Janous, Ursulinengymnasium, Innsbruck, Austria.


Let A, B, and C be the angles of an arbitrary triangle. Determine the best lower and upper
bounds of the function
X A XA B
f (A, B, C) = sin − sin
2 2 2
(where the summations are cyclic over A, B, C) and decide whether they are attained.

1156. Proposed by Hidetosi Fukagawa, Aichi, Japan.


At any point P of an ellipse with semiaxes a and b (a > b), draw a normal line and let Q be the
other meeting point. Find the least value of length P Q, in terms of a and b.

1158. Proposed by Svetoslav Bilchev, Technical University, Russe, Bulgaria.


Prove that
q
X 1 √
√ ≥ 6 − 3 2,
( 2 + 1) cos A8 − sin A8
where the sum is cyclic over the angles A, B, C of a triangle. When does equality occur?

36
1159. Proposed by George Tsintsifas, Thessaloniki, Greece.
Let ABC be a triangle and P some interior point with distances AP = x1 , BP = x2 , CP = x3 .
Show that

(b + c)x1 + (c + a)x2 + (a + b)x3 ≥ 8F,

where a, b, c are the sides of 4ABC and F is its area.

1162. Proposed by George Tsintsifas, Thessaloniki, Greece. (Dedicated to Léo Sauvé.)


Let G = {A1 , A2 , . . . , An+1 } be a point set of diameter D (that is, max Ai Aj = D) in En . Prove
that G can be obtained in a slab of width d, where
 2D
 √2n+2 for n odd
d≤ q
 D · 2(n+1) for n even.
n(n+2)

(A slab is a closed connected region in En bounded by two parallel hyperplanes. Its width is the
distance between these hyperplanes.)

1165F . Proposed by Murray S. Klamkin, University of Alberta, Edmonton, Alberta. (Dedica-


ted to Léo Sauvé.)
For fixed n ≥ 5, consider an n-gon P imbedded in a unit cube.

(i) Determine the maximum perimeter of P if n is odd.


(ii) Determine the maximum perimeter of P if it is convex (which implies it is planar).
(iii) Determine the maximum volume of the convex hull of P if also n < 8.

1166. Proposed by Kenneth S. Williams, Carleton University, Ottawa, Ontario. (Dedicated


to Léo Sauvé.)
Let A and B be positive integers such that the arithmetic progression {An + B : n = 0, 1, 2,√
. . .}
contains at least one square. If M 2 (M > 0) is the smallest such square, prove that M < A+ B.

1167. Proposed by Jordan B. Tabov, Sofia, Bulgaria. (Dedicated to Léo Sauvé.)


Determine the greatest real number r such that for every acute triangle ABC of area 1 there
exists a point whose pedal triangle with respect to ABC is right-angled and of area r.

1169. Proposed by Andy Liu, University of Alberta, Edmonton, Alberta; and Steve Newman,
University of Michigan, Ann Arbor, Michigan. [To Léo Sauvé who, like J. R. R. Tolkien,
created a fantastic world.]
(i) The fellowship of the Ring. Fellows of a society wear rings formed of 8 beads, with two of
each of 4 colours, such that no two adjacent beads are of the same colour. No two members wear
indistinguishable rings. What is the maximum number of fellows of this society?
(ii) The Two Towers. On two of three pegs are two towers, each of 8 discs of increasing size from
top to bottom. The towers are identical except that their bottom discs are of different colours.
The task is to disrupt and reform the towers so that the two largest discs trade places. This is
to be accomplished by moving one disc at a time from peg to peg, never placing a disc on top
of a smaller one. Each peg is long enough to accommodate all 16 discs. What is the minimum
number of moves required?
(iii) The Return of the King. The King is wandering around his kingdom, which is an ordinary
8 by 8 chessboard. When he is at the north-east corner, he receives an urgent summons to return
to his summer palace at the south-west corner. He travels from cell to cell but only due south,
west, or south-west. Along how many different paths can the return be accomplished?

37
1171F . Proposed by D. S. Mitrinović and J. E. Pecaric, University of Belgrade, Belgrade,
Yugoslavia. (Dedicated to Léo Sauvé.)
(i) Determine all real numbers λ so that, whenever a, b, c are the lengths of three segments
which can form a triangle, the same is true for

(b + c)λ , (c + a)λ , (a + b)λ .

(For λ = −1 we have Crux 14 [1975: 281].)


(ii) Determine all pairs of real numbers λ, µ so that, whenever a, b, c are the lengths of three
segments which can form a triangle, the same is true for

(b + c + µa)λ , (c + a + µb)λ , (a + b + µc)λ .

1172. Proposed by Walther Janous, Ursulinengymnasium, Innsbruck, Austria.


Show that for any triangle ABC, and for any real λ ≥ 1,
µ ¶
X
λ C 2 λ
(a + b) sec ≥4 √ s,
2 3
where the sum is cyclic over 4ABC and s is the semiperimeter.

1175. Proposed by J. T. Groenman, Arnhem, The Netherlands.


Prove that if α, β, γ are the angles of a triangle,
3√
−2 < sin 3α + sin 3β + sin 3γ ≤ 3.
2
1181. Proposed by D. S. Mitrinović and J. E. Pecaric, University of Belgrade, Belgrade,
Yugoslavia. (Dedicated to Léo Sauvé.)
Let x, y, z be real numbers such that

xyz(x + y + z) > 0,

and let a, b, c be the sides, ma , mb , mc the medians and F the area of a triangle. Prove that
p
(a) |yza2 + zxb2 + xyc2 | ≥ 4F xyz(x + y + z);
p
(b) |yzm2a + zxm2b + xym2c | ≥ 3F xyz(x + y + z).

1182. Proposed by Peter Andrews and Edward T. H. Wang, Wilfrid Laurier University, Wa-
terloo, Ontario. (Dedicated to Léo Sauvé.)
Let a1 , a2 , . . . , an denote positive reals where n ≥ 2. Prove that
π a1 a2 an (n − 1)π
≤ tan−1 + tan−1 + · · · + tan−1 ≤
2 a2 a3 a1 2
and for each inequality determine when equality holds.

38
1186. Proposed by Svetoslav Bilchev, Technical University, and Emilia Velikova, Mathemati-
kalgymnasium, Russe, Bulgaria.
If a, b, c are the sides of a triangle and s, R, r the semiperimeter, circumradius, and inradius,
respectively, prove that
r
X √ 4R + r
(b + c − a) a ≥ 4r(4R + r)
3Rs
where the sum is cyclic over a, b, c.

1194. Proposed by Richard I. Hess, Rancho Palos Verdes, California.


My uncle’s ritual for dressing each morning except Sunday includes a trip to the sock drawer
where he (1) picks out three socks at random, (2) wears any matching pair and returns the
third sock to the drawer, (3) returns the three socks to the drawer if he has no matching pair
and repeats steps (1) and (3) until he completes step (2). The drawer starts with 16 socks each
Monday morning (8 blue, 6 black, 2 brown) and ends up with 4 socks each Saturday evening.
(a) On which day of the week does he average the longest time at the sock drawer?
(b) On which day of the week is he least likely to get a matching pair from the first three socks
chosen?

1199F . Proposed by D. S. Mitrinović and J. E. Pecaric, University of Belgrade, Belgrade,


Yugoslavia. (Dedicated to Léo Sauvé.)
Prove that for acute triangles,
27R2
s2 ≤ (2R + r)2 ,
27R2 − 8r 2
where s, r, R are the semiperimeter, inradius, and circumradius, respectively.

1200. Proposed by Murray S. Klamkin, University of Alberta, Edmonton, Alberta.


In a certain game, the first player secretly chooses an n-dimensional vector a = (a 1 , a2 , . . . , an )
all of whose components are integers. The second player is to determine a by choosing any
n-dimensional vectors xi , all of whose components are also integers. For each xi chosen, and
before the next xi is chosen, the first player tells the second player the value of the dot product
xi · a. What is the least number of vectors xi the second player has to choose in order to be able
to determine a? [Warning: this is somewhat “tricky”!]

1201F . Proposed by D. S. Mitrinović and J. E. Pecaric, University of Belgrade, Belgrade,


Yugoslavia. (Dedicated to Léo Sauvé.)
Prove that µ 2 ¶ µ ¶
xc ya2 zb2 1 1 1
(x + y + z) + 2 + 2 ≥ + + (a2 yz + b2 zx + c2 xy),
a2 b c a2 b2 c2
where a, b, c are the sides of a triangle and x, y, z are real numbers.

1203. Proposed by Milen N. Naydenov, Varna, Bulgaria.


A quadrilateral inscribed in a circle of radius R and circumscribed around a circle of radius r
has consecutive sides a, b, c, d, semiperimeter s and area F . Prove that
√ p
(a) 2 F ≤ s ≤ r + r2 + 4R2 ;
p
(b) 6F ≤ ab + ac + ad + bc + bd + cd ≤ 4r 2 + 4R2 + 4r r2 + 4R2 ;
³ p ´2
(c) 2sr 2 ≤ abc + abd + acd + bcd ≤ 2r r + r2 + 4R2 ;
16 2 2
(d) 4F r 2 ≤ abcd ≤ r (r + 4R2 ).
9

39
1209. Proposed by Edward T. H. Wang, Wilfrid Laurier University, Waterloo, Ontario.
Characterize all positive integers a and b such that

a + b + (a, b) ≤ [a, b],

and find when equality holds. Here (a, b) and [a, b] denote respectively the g.c.d. and l.c.m. of a
and b.

1210. Proposed by Curtis Cooper, Central Missouri State University, Warrensburg, Missouri.
If A, B, C are the angles of an acute triangle, prove that

(tan A + tan B + tan C)2 ≥ (sec A + 1)2 + (sec B + 1)2 + (sec C + 1)2 .

1212. Proposed by Svetoslav Bilchev, Technical University, and Emilia Velikova, Mathemati-
kalgymnasium, Russe, Bulgaria.
Prove that
u bc v ca w ab
· + · + · ≥a+b+c
v+w s−a w+u s−b u+v s−c
where a, b, c are the sides of a triangle and s is its semiperimeter, and u, v, w are arbitrary
positive real numbers.

1213F . Proposed by Murray S. Klamkin, University of Alberta, Edmonton, Alberta.


In Math. Gazette 68 (1984) 222, P. Stanbury noted the two close approximations e 6 ≈ π 5 + π 4
and π 9 /e8 ≈ 10. Can one show without a calculator that (i) e6 > π 5 + π 4 and (ii) π 9 /e8 < 10?

1214. Proposed by J. T. Groenman, Arnhem, The Netherlands.


Let A1 A2 A3 be an equilateral triangle and let P be an interior point. Show that there is a
triangle with side lengths P A1 , P A2 , P A3 .

1215. Proposed by Edward T. H. Wang, Wilfrid Laurier University, Waterloo, Ontario.


Let a, b, c be nonnegative real numbers with a + b + c = 1. Show that

ab + bc + ca ≤ a3 + b3 + c3 + 6abc ≤ a2 + b2 + c2 ≤ 2 (a3 + b3 + c3 ) + 3abc,

and for each inequality determine all cases when equality holds.

1216F . Proposed by Walther Janous, Ursulinengymnasium, Innsbruck, Austria.


Prove or disprove that

sin A sin B sin C 9 3
2< + + ≤ ,
A B C 2π
where A, B, C are the angles (in radians) of a triangle.

1218F . Proposed by D. S. Mitrinović and J. E. Pecaric, University of Belgrade, Belgrade,


Yugoslavia.
Let F1 be the area of the orthic triangle of an acute triangle of area F and circumradius R.
Prove that
4F 3
F1 ≤ .
27R4

40
1221F . Proposed by D. S. Mitrinović and J. E. Pecaric, University of Belgrade, Belgrade,
Yugoslavia.
Let u, v, w be nonnegative numbers and let 0 < t ≤ 2. If a, b, c are the sides of a triangle and if
F is its area, prove that
µ ¶
u t v t w t 3 4F t
(bc) + (ca) + (ab) ≥ √ .
v+w w+u u+v 2 3
[See Solution II of Crux 1051 [1986: 252].]

1224. Proposed by George Tsintsifas, Thessaloniki, Greece.


A1 A2 A3 is a triangle with circumcircle Ω. Let i < Xi be the radii of the two circles tangent to
A1 A2 , A1 A3 , and arc A2 A3 of Ω. Let 2 , X2 , 3 , X3 be defined analogously. Prove that:
3
X 3
X 3
X
i
(a) = 1; (b) Xi ≥ 3 xi ≥ 12r,
Xi
i=1 i=1 i=1

where r is the inradius of 4A1 A2 A3 .

1225F . Proposed by David Singmaster, The Polytechnic of the South Bank, London, England.
What convex subset S of a unit cube gives the maximum value for V /A, where V is the volume
of S and A is its surface area? (For the two-dimensional case, see Crux 870 [1986: 180].)

1228. Proposed by J. Garfunkel, Flushing, New York and C. Gardner, Austin, Texas.
If QRS is the equilateral triangle of minimum perimeter that can be inscribed in a triangle
ABC, show that the perimeter of QRS is at most half the perimeter of ABC, with equality
when ABC is equilateral.

1229. Proposed by Edward T. H. Wang, Wilfrid Laurier University, Waterloo, Ontario.


Characterize all positive integers a and b such that
(a, b)[a,b] ≤ [a, b](a,b)
and determine when equality holds. (As usual, (a, b) and [a, b] denote respectively the g.c.d. and
l.c.m. of a and b.)

1234F . Proposed by Jack Garfunkel, Flushing, N. Y. A


Given the Malfatti configuration of three circles inscribed
in triangle ABC as shown, let A0 , B 0 , C 0 be the centers A′
of the three circles, and let r and r 0 be the inradii of
triangles ABC and A0 B 0 C 0 respectively. Prove that
B′ C′
√ B C
r ≤ (1 + 3 )r0 .

Equality is attained when ABC is equilateral.

1236. Proposed by Gordon Fick, University of Calgary, Calgary, Alberta.


Prove without calculus that if 0 ≤ θ ≤ 1, and 0 ≤ y ≤ n where y and n are integers, then
³ y ´y ³ y ´n−y
θy (1 − θ)n−y ≤ 1− .
n n
In statistics, this says that the sample proportion is the maximum likelihood estimator of the
population proportion. To the best of my knowledge, all mathematical statistics texts prove this
result with calculus.

41
1237F . Proposed by Niels Bejlegaard, Stavanger, Norway.
If ma , mb , mc denote the medians to the sides a, b, c of a triangle ABC, and s is the semiperimeter
of ABC, show that
X 2X
a cos A ≤ ma sin A ≤ s,
3
where the sums are cyclic.

1242. Proposed by Murray S. Klamkin, University of Alberta, Edmonton, Alberta.


Pn
The following problem appears in a book on matrix analysis: “Show that aij xi xj is positive
i,j=1
P P
definite if aii x2i + i6=j |aij |xi xj is positive definite.” Give a counterexample!
i

1243. Proposed by George Tsintsifas, Thessaloniki, Greece.


Let ABC be a triangle and M an interior point with barycentric coordinates (λ 1 , λ2 , λ3 ). The
distances of M from the vertices A, B, C are x1 , x2 , x3 and the circumradii of the triangles
M BC, M CA, M AB, ABC are R1 , R2 , R3 , R. Show that

λ1 R1 + λ2 R2 + λ3 R3 ≥ R ≥ λ1 x1 + λ2 x2 + λ3 x3 .

1245. Proposed by Walther Janous, Ursulinengymnasium, Innsbruck, Austria. (Dedicated to


Léo Sauvé.)
Let ABC be a triangle, and let H be a hexagon created A
by drawing tangents to the incircle of ABC parallel to
the sides of ABC. Prove that
2
perimeter(H) ≤ perimeter(ABC).
3
B C
When does equality occur?

1247. Proposed by Robert E. Shafer, Berkeley, California.


Prove that for 0 ≤ φ < θ ≤ π/2,

φ φ φ φ θ θ θ θ
cos2 log cos2 + sin2 log sin2 − cos2 log cos2 − sin2 log sin2
2 2 2 2 2 2 2 2
3³ 4 4
´
< sin 3 θ − sin 3 φ .
4

1249F . Proposed by D. S. Mitrinović and J. E. Pecaric, University of Belgrade, Belgrade,


Yugoslavia.
Prove the triangle inequalities
X 1 ³ r ´2 ³ r ´4 ³ r ´4
(a) sin4 A ≤ 2 − −3 ≤2−5 ;
2 R R R
X ³ r ´2 ³ r ´4 ³ r ´4
(b) sin2 2A ≥ 6 + 12 ≥ 36 ;
R R R
X ³ r ´2 ³ r ´3 ³ r ´2
(c) sin 2B sin 2C ≤ 5 +8 ≤9 ,
R R R
where the sums are cyclic over the angles A, B, C of a triangle, and r, R are the inradius and
circumradius respectively.

42
1251. Proposed by Stanley Rabinowitz, Alliant Computer Systems Corp., Littleton, Massa-
chusetts. (Dedicated to Léo Sauvé.)
(a) Find all integral n for which there exists a regular n-simplex with integer edge and
integer volume.
(b)F Which such n-simplex has the smallest volume?
1252. Proposed by George Tsintsifas, Thessaloniki, Greece.
Let ABC be a triangle and M an interior point with barycentric coordinates λ 1 , λ2 , λ3 . We
denote the pedal triangle and the Cevian triangle of M by DEF and A0 B 0 C 0 respectively. Prove
that
[DEF ] ³ s ´2
≥ 4λ 1 λ 2 λ 3 ,
[A0 B 0 C 0 ] R
where s is the semiperimeter and R the circumradius of 4ABC, and [X] denotes the area of
figure X.

1254. Proposed by Walther Janous, Ursulinengymnasium, Innsbruck, Austria.


Let ABC be a triangle and n ≥ 1 a natural number. Show that

 <1 if n = 1,
¯X ¯ √
¯ ¯ 3 3
¯ sin n(B − C)¯ < 2 if n = 2,

 √
 ≤ 3 3 if n ≥ 3,
2

where the sum is cyclic.

1256. Proposed by D. J. Smeenk, Zaltbommel, The Netherlands.


Let ABC be a triangle with sides satisfying a3 = b3 + c3 . Determine the range of angle A.

1258. Proposed by Ian Witten, University of Calgary, Calgary, Alberta.


Think of a picture as an m × n matrix A of real numbers between 0 and 1 inclusive, where
aij represents the brightness of the picture at the point (i, j). To reproduce the picture on a
computer we wish to approximate it by an m × n matrix B of 0’s and 1’s, such that every “part”
of the original picture is “close” to the corresponding part of the reproduction. These are the
ideas behind the following definitions:
A subrectangle of an m × n grid is a set of positions of the form
{(i, j) | r1 ≤ i ≤ r2 , s1 ≤ j ≤ s2 }
where 1 ≤ r1 ≤ r2 ≤ m and 1 ≤ s1 ≤ s2 ≤ n are constants. For any subrectangle R, let
¯ ¯
¯ X ¯
¯ ¯
d(R) = ¯
¯ (aij − bij )¯¯ ,
¯(i,j)∈R ¯

where A and B are as given above, and define


d(A, B) = max d(R),
the maximum taken over all subrectangles R.
(a) Show that there exist matrices A such that d(A, B) > 1 for every 0-1 matrix B of
the same size.
(b)F Is there a constant c such that for every matrix A of any size, there is some 0-1
matrix B of the same size such that d(A, B) < c?

43
1259. Proposed by Murray S. Klamkin, University of Alberta, Edmonton, Alberta.
If x, y, z ≥ 0, disprove the inequality

(yz + zx + xy)2 (x + y + z) ≥ 9xyz(x2 + y 2 + z 2 ).

Determine the largest constant one can replace the 9 with to obtain a valid inequality.

1265. Proposed by George Tsintsifas, Thessaloniki, Greece.


Let ABC be a triangle with area F and exradii ra , rb , rc , and let A0 B 0 C 0 be a triangle with area
F 0 and altitudes h0a , h0b , h0c . Show that
r
ra rb rc F
0
+ 0 + 0 ≥3 .
ha hb hc F0

1266. Proposed by Themistocles M. Rassias, Athens, Greece.


n
Q
Let a1 , a2 , . . . , an be distinct odd natural numbers, and let ai be divisible by exactly k primes,
i=1
of which p is the smallest. Prove that

1 1 1 Ip−2
+ + ··· + <
a1 a2 an Ip+2k−2
where
2m(2m − 2) · · · 4 · 2
I2m+1 = .
(2m + 1)(2m − 1) · · · 3 · 1

1267. Proposed by J. T. Groenman, Arnhem, The Netherlands.


Let A1 A2 A3 be a triangle with inscribed circle I of radius r. Let Ii and Ji , of radii λi and µi ,
be the two circles tangent to I and the lines A1 A2 and A1 A3 . Analogously define circles I2 , J2 ,
I3 , J3 of radii λ2 , µ2 , λ3 , µ3 , respectively.

(a) Prove that λ1 µ1 = λ2 µ2 = λ3 µ3 = r2 .


3
X 3
X
(b) Prove that λi + µi ≥ 10r.
i=1 i=1

1269F . Proposed by Walther Janous, Ursulinengymnasium, Innsbruck, Austria.


Let ABC be a non-obtuse triangle with circumcenter M and circumradius R. Let u 1 , u2 , u3 be
the lengths of the parts of the cevians (through M ) between M and the sides opposite to A, B,
C respectively. Prove or disprove that

R u1 + u 2 + u 3
≤ < R.
2 3

1270. Proposed by Péter Ivády, Budapest, Hungary.


Prove the inequality

x p
√ < tanh x < 1 − e−x2
1 + x2

for x > 0.

44
1271. Proposed by Murray S. Klamkin, University of Alberta, Edmonton, Alberta. (Dedicated
in memoriam to Léo Sauvé.)
Prove that
√ X Ai X A2 A3
3 sin ≥4 sin Bi sin sin ,
2 2 2
where A1 A2 A3 and B1 B2 B3 are two triangles and the sums are cyclic over their angles.

1273. Proposed by George Tsintsifas, Thessaloniki, Greece.


Let ABC be a triangle, M an interior point, and A0 B 0 C 0 its pedal triangle. Denote the sides of
the two triangles by a, b, c and a0 , b0 , c0 respectively. Prove that

a0 b0 c0
+ + < 2.
a b c
1277. Proposed by Zun Shan and Edward T. H. Wang, Wilfrid Laurier University, Waterloo,
Ontario.
Determine all possible values of the expression

x1 x2 + x 2 x3 + · · · + x n x1

where n ≥ 2 and xi = 1 or −1 for each i.

1280. Proposed by Walther Janous, Ursulinengymnasium, Innsbruck, Austria.


Let ABC be a triangle and let A1 , B1 , C1 be points on BC, CA, AB, respectively, such that
A1 C B1 A C1 B
= = = k > 1.
BA1 CB1 AC1
Show that
k2 − k + 1 perimeter(A1 B1 C1 ) k
< < ,
k(k + 1) perimeter(ABC) k+1

and that both bounds are best possible.

1281F . Proposed by Stanley Rabinowitz, Alliant Computer Systems Corp., Littleton, Massa-
chusetts.
Find the area of the largest triangle whose vertices lie in or on a unit n-dimensional cube.

1282. Proposed by George Tsintsifas, Thessaloniki, Greece.


Let ABC be a triangle, I the incenter, and A0 , B 0 , C 0 the intersections of AI, BI, CI with the
circumcircle. Show that

IA0 + IB 0 + IC 0 − (IA + IB + IC) ≤ 2(R − 2r)

where R and r are the circumradius and inradius of 4ABC.

1283. Proposed by Murray S. Klamkin, University of Alberta, Edmonton, Alberta.


Show that the polynomial

P (x, y, z) = (x2 + y 2 + z 2 )3 − (x3 + y 3 + z 3 )2 − (x2 y + y 2 z + z 2 x)2 − (xy 2 + yz 2 + zx2 )2

is nonnegative for all real x, y, z.

45
1284. Proposed by J. T. Groenman, Arnhem, The Netherlands.
Let A1 A2 A3 A4 be a cyclic quadrilateral with A1 A2 = a1 , A2 A3 = a2 , A3 A4 = a3 , A4 A1 = a4 .
Let ρ1 be the radius of the circle outside the quadrilateral, tangent to the segment A 1 A2 and
the extended lines A2 A3 and A4 A1 . Define ρ2 , ρ3 , ρ4 analogously. Prove that
1 1 1 1 8
+ + + ≥ √ .
ρ1 ρ2 ρ3 ρ4 4 a 1 a2 a3 a4

When does equality hold?

1286. Proposed by Walther Janous, Ursulinengymnasium, Innsbruck, Austria.


Let x, y, z be positive real numbers. Show that
" P #x+y+z
Y · x(x + y + z) ¸x ( yz)2
≤ ,
(x + y)(x + z) 4xyz(x + y + z)
Q P
where and are to be understood cyclically.

1288. Proposed by Len Bos, University of Calgary, Calgary, Alberta.


Show that for x1 , x2 , . . . , xn > 0,

n(xn1 + xn2 + · · · + xnn ) ≥ (x1 + x2 + · · · + xn )(xn−1


1 + xn−1
2 + · · · + xnn−1 ).

1289. Proposed by Carl Friedrich Sutter, Viking, Alberta.


“To reward you for slaying the dragon”, the Queen said to Sir George, “I grant you all the land
you can walk around in a day.”
She pointed to a pile of wooden stakes. “Take some of these stakes with you”, she continued.
“Pound them into the ground along the way, and be back at your starting point in 24 hours.
All the land in the convex hull of your stakes will then be yours.” (The Queen had read a little
mathematics.)
Assume that it takes Sir George 1 minute to pound in a stake, and that he walks with constant
speed between stakes. How many stakes should he use, to get as much land as possible?

1292F . Proposed by Jack Garfunkel, Flushing, N. Y.


It has been shown (see Crux 1083 [1987: 96]) that if A, B, C are the angles of a triangle,
µ ¶
2 X X B−C 2 X A
√ sin A ≤ cos ≤√ cos ,
3 2 3 2

where the sums are cyclic. Prove that


µ ¶ µ ¶
X B−C 1 X X A
cos ≤√ sin A + cos ,
2 3 2

which if true would imply the right hand inequality above.

1296. Proposed by Murray S. Klamkin, University of Alberta, Edmonton, Alberta.


Let r1 , r2 , r3 be the distances from an interior point of a triangle to its sides a1 , a2 , a3 , respec-
tively, and let R be the circumradius of the triangle. Prove that

a1 r1n + a2 r2n + a3 r3n ≤ (2R)n−2 a1 a2 a3

for all n ≥ 1, and determine when equality holds.

46
1297. Proposed by Walther Janous, Ursulinengymnasium, Innsbruck, Austria. (To the me-
mory of Léo.)
(a) Let C > 1 be a real number. The sequence z1 , z2 , . . . of real numbers satisfies 1 < zn and
z1 + · · · + zn < Czn+1 for n ≥ 1. Prove the existence of a constant a > 1 such that zn > an ,
n ≥ 1.
(b)F Let conversely z1 < z2 < · · · be a strictly increasing sequence of positive real numbers
satisfying zn ≥ an , n ≥ 1, where a > 1 is a constant. Does there necessarily exist a constant C
such that z1 + · · · + zn < Czn+1 for all n ≥ 1?

1302. Proposed by Mihály Bencze, Brasov, Romania.


n
P
Suppose ak > 0 for k = 1, 2, . . . , n and tanh2 ak = 1. Prove that
k=1
n n
X 1 X sinh ak
≥n .
k=1
sinh ak cosh2 ak
k=1

1303. Proposed by George Tsintsifas, Thessaloniki, Greece.


Let ABC and A1 B1 C1 be two triangles with sides a, b, c and a1 , b1 , c1 and inradii r and r1 , and
let P be an interior point of ABC. Set AP = x, BP = y, CP = z. Prove that
a1 x2 + b 1 y 2 + c 1 z 2
≥ 4rr1 .
a+b+c
1305. Proposed by J. T. Groenman, Arnhem, The Netherlands.
Let A1 A2 A3 be an acute triangle with circumcenter O. Let P1 , Q1 (Q1 6= A1 ) denote the
intersection of A1 O with A2 A3 and with the circumcircle, respectively, and define P2 , Q2 , P3 ,
Q3 analogously. Prove that
OP1 · OP2 · OP3
(a) ≥ 1;
P 1 Q1 · P 2 Q2 · P 3 Q3
OP1 OP2 OP3
(b) + + ≥ 3;
P 1 Q1 P 2 Q2 P 3 Q3
A1 P1 · A 2 P2 · A 3 P3
(c) ≥ 27.
P 1 Q1 · P 2 Q2 · P 3 Q3
1313. Proposed by Wendel Semenko, Snowflake, Manitoba.
Show that any triangular piece of paper √ of area 1 can be folded once so that when placed on a
table it will cover an area of less than 5−1
2 .

1315. Proposed by J. T. Groenman, Arnhem, The Netherlands.


Let ABC be a triangle with medians AD, BE, CF and median point G. We denote 4AGF =
41 , 4BGF = 42 , 4BGD = 43 , 4CGD = 44 , 4CGE = 45 , 4AGE = 46 , and let Ri and
ri denote the circumradius and inradius of 4i (i = 1, 2, . . . , 6). Prove that
(i) R1 R3 R5 = R2 R4 R6 ;
15 1 1 1 1 1 1 9
(ii) < + + = + + < ,
2r r1 r3 r5 r2 r4 r6 r
where r is the inradius of 4ABC.

1318. Proposed by R. S. Luthar, University of Wisconsin Center, Janesville.


Find, without calculus, the largest possible value of
sin 5x + cos 3x
.
sin 4x + cos 4x

47
1320. Proposed by Themistocles M. Rassias, Athens, Greece.
Asumme that a1 , a2 , a3 , . . . are real numbers satisfying the inequality

|am+n − am − an | ≤ C

for all m, n ≥ 1 and for some constant C. Prove that there exists a constant k such that

|an − nk| ≤ C

for all n ≥ 1.

1327. Proposed by George Tsintsifas, Thessaloniki, Greece.


Let x1 , x2 , x3 be the distances of the vertices of a triangle from a point P in the same plane.
Let r be the inradius of the triangle, and p be the power of the point P with respect to the
circumcircle of the triangle. Prove that

x1 x2 x3 ≥ 2rp.

1332. Proposed by Murray S. Klamkin, University of Alberta.


It is known that if A, B, C are the angles of a triangle,
A B C
sin + sin + sin ≥ 1,
2 2 2
with equality if and only if the triangle is degenerate with angles π, 0, 0. Establish the related
non-comparable inequality
A B C 5r
sin + sin + sin ≥ − 1,
2 2 2 R
where r and R are the inradius and circumradius respectively.

1333. Proposed by George Tsintsifas, Thessaloniki, Greece.


If a, b, c and a0 , b0 , c0 are the sides of two triangles and F , F 0 are their areas, show that
X √ √ √
a [a0 − ( b0 − c0 )2 ] ≥ 4 3F F 0 ,

where the sum is cyclic. (This improves Crux 1114 [1987: 185].)

1338. Proposed by Jean Doyen, Université Libre de Bruxelles, Brussels, Belgium; J. Chris
Fisher, University of Regina, Regina, Saskatchewan; and Günter Kist, Technische Universität,
Munich, Federal Republic of Germany.
In a theoretical version of the Canadian lottery “Lotto 6–49“, a ticket consists of six distinct
integers chosen from 1 to 49 (inclusive). A t-prize is awarded for any ticket having t or more
numbers in common with a designated “winning“ ticket. Denote by f (t) the smallest¡49¢ number of
tickets required to be certain of winning a t-prize. Clearly f (1) = 8 and f (6) = 6 . Show that
f (2) ≤ 19. Can you do better?

1339. Proposed by Weixuan Li, Changsha Railway Institute, Changsha, Hunan, China, and
Edward T. H. Wang, Wilfrid Laurier University, Waterloo, Ontario.
Let a, b, m, n denote positive real numbers such that a ≤ b and m ≤ n. Show that

(bm − am )n ≤ (bn − an )m

and determine all cases when equality holds.

48
1341. Proposed by Stanley Rabinowitz, Alliant Computer Systems Corp., Littleton, Massa-
chusetts. √
An ellipse has center O and the ratio of the lengths of the axes is 2 + 3. If P is a point on
the ellipse, prove that the (acute) angle between the tangent to the ellipse at P and the radius
vector P O is at least 30 ◦.

1344. Proposed by Florentin Smarandache, Craiova, Romania.


There are given mn + 1 points such that among any m + 1 of them there are two within distance
1 from each other. Prove that there exists a sphere of radius 1 containing at least n + 1 of the
points.

1345. Proposed by P. Erdos, Hungarian Academy of Sciences, and Esther Szekeres, University
of New South Wales, Kensington, Australia.
Given a convex n-gon X1 X2 . . . Xn of perimeter p, denote by f (Xi ) the sum of the distances of
Xi to the other n − 1 vertices.

(a) Show that if n ≥ 6, there is a vertex Xi such that f (Xi ) > p.


(b) Is it true that for n large enough, the average value of f (Xi ), 1 ≤ i ≤ n, is greater
than p?

1348F . Proposed by Murray S. Klamkin, University of Alberta.


Two congruent convex centrosymmetric planar figures are inclined to each other (in the same
plane) at a given angle. Prove or disprove that their intersection has maximum area when the
two centers coincide.

1352. Proposed by Murray S. Klamkin, University of Alberta.


Determine lower and upper bounds for

Sr = cosr A + cosr B + cosr C

where A, B, C are the angles of a non-obtuse triangle, and r is a positive real number, r 6=
1, 2. (The cases r = 1 and 2 are known; see items 2.16 and 2.21 of Bottema et al, Geometric
Inequalities.)

1356F . Proposed by Walther Janous, Ursulinengymnasium, Innsbruck, Austria.


Show that
√ √
x x2 xn x1 + · · · + xn
√ 1 +√ + ··· + √ ≥ √
1 − x1 1 − x2 1 − xn n−1

for positive real numbers x1 , . . . , xn (n ≥ 2) satisfying x1 + · · · + xn = 1.

1357F . Proposed by Jack Garfunkel, Flushing, N. Y.


Isosceles right triangles AA0 B, BB 0 C, CC 0 A are constructed outwardly on the sides of a triangle
ABC, with the right angles at A0 , B 0 , C 0 , and triangle A0 B 0 C 0 is drawn. Prove or disprove that

A B C
sin A0 + sin B 0 + sin C 0 ≥ cos + cos + cos ,
2 2 2

where A0 , B 0 , C 0 are the angles of 4A0 B 0 C 0 .

49
1361. Proposed by J. T. Groenman, Arnhem, The Netherlands.
Let ABC be a triangle with sides a, b, c and angles α, β, γ, and let its circumcenter lie on the
escribed circle to the side a.

(i) Prove that − cos α + cos β + cos γ = 2.
(ii) Find the range of α.

1363F . Proposed by P. Erdos, Hungarian Academy of Sciences.


Let there be given n points in the plane, no three on a line and no four on a circle. Is it true
that these points must determine at least n distinct distances, if n is large enough? I offer $25
U.S. for the first proof of this.

1365. Proposed by George Tsintsifas, Thessaloniki, Greece.


Prove that

3 sin A sin B sin C 3 3
< + + <
π π−A π−B π−C π
where A, B, C are the angles (in radians) of an acute triangle.

1366F . Proposed by Walther Janous, Ursulinengymnasium, Innsbruck, Austria.


Prove or disprove that
√ √ √
x y z x+ y+ z
√ +√ +√ ≥ √
x+y y+z z+x 2
for all positive real numbers x, y, z.

1369. Proposed by G. R. Veldkamp, De Bilt, The Netherlands.


The perimeter of a triangle is 24 cm and its area is 24 cm2 . Find the maximal length of a side
and write it in a simple form.

1371F . Proposed by Murray S. Klamkin, University of Alberta.


In Math. Gazette 68 (1984) 222, P. Stanbury noted the close approximation

π 9 /e8 ≈ 9.999838813 ≈ 10.

Are there positive integers l, m such that π l /em is closer to a positive integer than for the case
given? (See Crux 1213 [1988: 116] for a related problem.)

1377. Proposed by Colin Springer, student, Waterloo, Ontario.


In right triangle ABC, hypotenuse AC has length
√ 2. Let O be the midpoint of AC and let I be
the incentre of the triangle. Show that OI ≥ 2 − 1.

1380. Proposed by Kee-Wai Lau, Hong Kong.


Prove the inequality

sin(tan x) < tan(sin x)

for 0 < x < π, x 6= π/2.

50
1384. Proposed by Stanley Rabinowitz, Alliant Computer Systems Corp., Littleton, Massa-
chusetts.
If the center of curvature of every point√ on an ellipse lies inside the ellipse, prove that the
eccentricity of the ellipse is at most 1/ 2.

1386. Proposed by George Tsintsifas, Thessaloniki, Greece.


Let A1 A2 . . . An be a polygon inscribed in a circle and containing the centre of the circle. Prove
that
n
4 X ai n2 π
n−2+ < ≤ sin ,
π âi π n
i=1

where ai is the side Ai Ai+1 and âi is the arc Ai Ai+1 .

1389. Proposed by Derek Chang, California State University, Los Angeles, and Raymond Kill-
grove, Indiana State University, Terre Haute.
Find
Xn
max |i − π(i)|,
π∈Sn
i=1

where Sn is the set of all permutations of {1, 2, . . . , n}.

1390. Proposed by Hidetosi Fukagawa, Aichi, Japan.


A, B, C are points on a circle Γ such that CM is the perpendicular bisector of AB. P is a point
on CM and AP meets Γ again at D. As P varies over segment CM , find the largest radius of
the inscribed circle tangent to segments P D, P B, and arc DB of Γ , in terms of the length of
CM .

1391. Proposed by George Tsintsifas, Thessaloniki, Greece.


Let ABC be a triangle and D the point on BC so that the incircle of 4ABD and the excircle
(to side DC) of 4ADC have the same radius ρ1 . Define ρ2 , ρ3 analogously. Prove that
9
ρ1 + ρ2 + ρ3 ≥ r,
4
where r is the inradius of 4ABC.

1392. Proposed by Angel Dorito, Geld, Ontario.


An immense spherical balloon is being inflated so that it constantly touches the ground at a
fixed point A. A boy standing at a point at unit distance from A fires an arrow at the balloon.
The arrow strikes the balloon at its nearest point (to the boy) but does not penetrate it, the
balloon absorbing the shock and the arrow falling vertically to the ground. What is the longest
distance through which the arrow can fall, and how far from A will it land in this case?

1394. Proposed by Murray S. Klamkin, University of Alberta.


If x, y, z > 0, prove that
p p p √
y 2 + yz + z 2 + z 2 + zx + x2 + x2 + xy + y 2 ≥ 3 yz + zx + xy.

51
1399. Proposed by Sydney Bulman-Fleming and Edward T. H. Wang, Wilfrid Laurier Uni-
versity, Waterloo, Ontario.
Prove that
(n + 1)!
σ(n!) ≤
2
for all natural numbers n and determine all cases when equality holds. (Here σ(k) denotes the
sum of all positive divisors of k.)

1400. Proposed by Robert E. Shafer, Berkeley, California.


In a recent issue of the American Mathematical Monthly (June-July 1988, page 551), G. Klam-
bauer showed that if xs e−x = y s e−y (x, y, s > 0, x 6= y) then x + y > 2s. Show that if
xs e−x = y s e−y where x 6= y and x, y, s > 0 then xy(x + y) < 2s3 .

1401. Proposed by P. Penning, Delft, The Netherlands.


Given are a circle C and two straight lines l and m in the plane of C that intersect in a point S
inside C. Find the tangent(s) to C intersecting l and m in points P and Q so that the perimeter
of 4SP Q is a minimum.

1402. Proposed by George Tsintsifas, Thessaloniki, Greece.


Let M be an interior point of the triangle A1 A2 A3 and B1 , B2 , B3 the feet of the perpendiculars
from M to sides A2 A3 , A3 A1 , A1 A2 respectively. Put ri = Bi M , i = 1, 2, 3. R0 is the circumradius
of 4B1 B2 B3 , and R, r the circumradius and inradius of 4A1 A2 A3 . Prove that

R0 Rr ≥ 2r1 r2 r3 .

1403F . Proposed by Walther Janous, Ursulinengymnasium, Innsbruck, Austria.


For n ≥ 2, prove or disprove that
x1 + · · · + x n
1< ≤2
n
for all natural numbers x1 , x2 , . . . , xn satisfying

x1 + x 2 + · · · + x n = x 1 · x 2 · . . . · x n .

1406. Proposed by R. S. Luthar, University of Wisconsin Center, Janesville.


If 0 < θ < π, prove without calculus that

θ
cot − cot θ > 2.
4
1413. Proposed by Walther Janous, Ursulinengymnasium, Innsbruck, Austria.
For 0 < x, y, z < 1 let

u = z(1 − y), v = x(1 − z), w = y(1 − x).

Prove that
µ ¶
1 1 1
(1 − u − v − w) + + ≥ 3.
u v w

When does equality occur?

52
1414. Proposed by Murray S. Klamkin, University of Alberta.
Determine the maximum value of the sum
r r r
B C C A A B
tan tan + λ + tan tan + λ + tan tan + λ
2 2 2 2 2 2
where A, B, C are the angles of a triangle and λ is a nonnegative constant. (The case λ = 5 is
item 2.37 of O. Bottema et al, Geometric Inequalities.)

1416. Proposed by Hidetosi Fukagawa, Aichi, Japan.


Y R
In the figure, the unit square ABCD and the line l
A D
are fixed, and the unit square P QRS rotates with P S
and Q lying on l and AB respectively. X is the foot
of the perpendicular from S to l. Find the position
of point Q so that the length XY is a maximum. Q
X P B C l
1420. Proposed by Shailesh Shirali, Rishi Valley School, India.
If a, b, c are positive integers such that

0 < a2 + b2 − abc ≤ c,

show that a2 + b2 − abc is a perfect square. (This is a generalization of problem 6 of the 1988
I.M.O. [1988: 197].)

1421. Proposed by J. T. Groenman, Arnhem, The Netherlands, and D. J. Smeenk, Zaltbom-


mel, The Netherlands.
ABC is a triangle with sides a, b, c. The escribed circle to the side a has centre I a and touches
a, b, c (produced) at D, E, F respectively. M is the midpoint of BC.

(a) Show that the lines Ia D, EF and AM have a common point Sa .


(b) In the same way we have points Sb and Sc . Prove that
area(4Sa Sb Sc ) 3
> .
area(4ABC) 2

1422. Proposed by George Tsintsifas, Thessaloniki, Greece.


Let A1 A2 A3 be a triangle and M an interior point; λ1 , λ2 , λ3 the barycentric coordinates of
M ; and r1 , r2 , r3 its distances from the sides A2 A3 , A3 A1 , A1 A2 respectively. Set Ai M = Ri ,
i = 1, 2, 3. Prove that
3 · ¸
X r2 r3 r3 r1 r1 r2
λi Ri > 2 λ1 · + λ2 · + λ3 · .
r1 r2 r3
i=1

1424. Proposed by Walther Janous, Ursulinengymnasium, Innsbruck, Austria.


Show that
X
a tan A ≥ 10R − 2r

for any acute triangle ABC, where a, b, c are its sides, R its circumradius, and r its inradius,
and the sum is cyclic.

53
1428F . Proposed by Svetoslav Bilchev and Emilia Velikova, Technical University, Russe, Bul-
garia.
A1 A2 A3 is a triangle with sides a1 , a2 , a3 , and P is an interior point with distances Ri and ri
(i = 1, 2, 3) to the vertices and sides, respectively, of the triangle. Prove that
³X ´ ³X ´ X
a1 R1 r1 ≥ 6 a 1 r2 r3

where the sums are cyclic.

1429F . Proposed by D. S. Mitrinović, University of Belgrade, and J. E. Pecaric, University


of Zagreb.

(a) Show that


X x21
sup = n − 1,
x21 + x2 x3
where x1 , x2 , . . . , xn are n positive real numbers (n ≥ 3), and the sum is cyclic.
(b) More generally, what is
X x1r+s
sup ,
x1r+s + xr2 xs3
for natural numbers r and s?

1430. Proposed by Mihály Bencze, Brasov, Romania.


AD, BE, CF are (not necessary concurrent) Cevians in triangle ABC, intersecting the circum-
circle of 4ABC in the points P , Q, R. Prove that
AD BE CF
+ + ≥ 9.
DP EQ F R
When does equality hold?

1440F . Proposed by Jack Garfunkel, Flushing, N. Y.


Prove or disprove that if A, B, C are the angles of a triangle,
sin A sin B sin C 3 √
4
√ +√ +√ ≤ · 3.
sin A + sin B sin B + sin C sin C + sin A 2

1443. Proposed by Walther Janous, Ursulinengymnasium, Innsbruck, Austria.


Given an integer n ≥ 2, determine the minimum value of
X µ x2 ¶
i

1≤i,j≤n
x j
i6=j

over all positive real numbers x1 , . . . , xn such that x21 + · · · + x2n = 1.

1445. Proposed by Murray S. Klamkin and Andy Liu, University of Alberta.


Determine the minimum value of
x3 y3 z3
+ +
1 − x8 1 − y 8 1 − z 8
where x, y, z ≥ 0 and x4 + y 4 + z 4 = 1.

54
1448. Proposed by Jack Garfunkel, Flushing, N. Y.
If A, B, C are the angles of a triangle, prove that
µX ¶2
2 A X
sin ≥ cos A,
3 2

with equality when A = B = C.

1449. Proposed by David C. Vaughan, Wilfrid Laurier University.


Prove that for all x ≥ y ≥ 1,

x y 1 y x 1
√ +√ +√ ≥√ +√ +√ .
x+y y+1 x+1 x + y x+1 y +1

1452. Proposed by Walther Janous, Ursulinengymnasium, Innsbruck, Austria.


Let x1 , x2 , x3 be positive reals satisfying x1 + x2 + x3 = 1, and consider the inequality

(1 − x1 )(1 − x2 )(1 − x3 ) ≥ cr (x1 x2 x3 )r . (1)

For each real r, find the greatest constant cr such that (1) holds for all choices of the xi , or prove
that no such constant cr exists.

1454. Proposed by Marcin E. Kuczma, Warszawa, Poland.


Given a convex pentagon of area S, let S1 , . . . , S5 denote the areas of the five triangles cut off
by the diagonals (each triangle is spanned by three consecutive vertices of the pentagon). Prove
that the sum of some four of the Si ’s exceeds S.

1457. Proposed by Colin Springer, student, University of Waterloo.


In 4ABC, the sides are a, b, c, the perimeter is p and the circumradius is R. Show that

a2 b2
R2 p ≥ .
a+b−c
Under what conditions does equality hold?

1460. Proposed by Mihály Bencze, Brasov, Romania.


P is an interior point of a convex n-gon A1 A2 . . . An . For each i = 1, . . . , n let Ri = P Ai and wi
be the length of the bisector of ÁP in 4Ai P Ai+1 (An+1 = A1 ). Also let c1 , . . . , cn be positive
real numbers. Prove that
n n µ ¶
πX 2 X 1 1
2 cos ci ≥ ci ci+1 wi +
n Ri Ri+1
i=1 i=1

(Rn+1 = R1 ).

1461. Proposed by Walther Janous, Ursulinengymnasium, Innsbruck, Austria.


Let a, b, c, r, R, s be the sides, inradius, circumradius, and semiperimeter of a triangle and let
a0 , b0 , c0 , r0 , R0 , s0 be similarly defined for a second triangle. Show that
³ X ´2
4ss0 − aa0 ≥ 4 (s2 + r2 + 4Rr)(s02 + r02 + 4R0 r0 ),

where the sum is cyclic.

55
1462F . Proposed by Jack Garfunkel, Flushing, N. Y.
If A, B, C are the angles of a triangle, prove or disprove that
µ ¶ r r r
√ A B C A B C
2 sin + sin + sin ≥ sin + sin + sin ,
2 2 2 2 2 2
with equality when A = B = C.

1472. Proposed by Walther Janous, Ursulinengymnasium, Innsbruck, Austria.


For each integer n ≥ 2, find the largest constant cn such that
n
X X
cn |ai | ≤ |ai − aj |
i=1 i<j
Pn
for all real numbers a1 , . . . , an satisfying i=1 ai = 0.

1473F . Proposed by Murray S. Klamkin, University of Alberta.


Given is a unit circle and an interior point P . Find the convex n-gon of largest area and/or
perimeter which is inscribed in the circle and passes through P .

1478F . Proposed by D. M. Milošević, Pranjani, Yugoslavia.


A circle of radius R is circumscribed about a regular n-gon. A point on the circle is at distances
a1 , a2 , . . . , an from the vertices of the n-gon. Prove that
n
X √
a3i ≥ 2R3 n 2.
i=1

1479. Proposed by Vedula N. Murty, Pennsylvania State University at Harrisburg.


Given x > 0, y > 0 satisfying x2 + y 2 = 1, show without calculus that

x3 + y 3 ≥ 2xy.

1484. Proposed by Walther Janous, Ursulinengymnasium, Innsbruck, Austria.


Let 0 < r, s, t ≤ 1 be fixed. Show that the relation

r cot rA = s cot sB = t cot tC

holds for exactly one triangle ABC, and that this triangle maximizes the expression

sin rA sin sB sin tC

over all triangles ABC.

1487. Proposed by Kee-Wai Lau, Hong Kong.


Prove the inequality

x + sin x ≥ 2 log(1 + x)

for x > −1.

1488. Proposed by Avinoam Freedman, Teaneck, N. J.


Prove that in any acute triangle, the sum of the circumradius and the inradius is less than the
length of the second-longest side.

56
1490F . Proposed by Jack Garfunkel, Flushing, N. Y.
This was suggested by Walther Janous’ problem Crux 1366 [1989: 271].
Find the smallest constant k such that
x y z √
√ +√ +√ ≤k x+y+z
x+y y+z z+x

for all positive x, y, z.

1493. Proposed by Toshio Seimiya, Kawasaki, Japan.


Two squares ABDE and ACF G are described on AB and AC outside the triangle ABC. P
and Q are on line EG such that BP and CQ are perpendicular to BC. Prove that

BP + CQ ≥ BC + EG.

When does equality hold?

1498F . Proposed by D. M. Milošević, Pranjani, Yugoslavia.


Show that
3
Y
hai i ≤ (3r)2s ,
i=1

where a1 , a2 , a3 are the sides of a triangle, h1 , h2 , h3 its altitudes, r its inradius, and s its semi-
perimeter.

1504. Proposed by Walther Janous, Ursulinengymnasium, Innsbruck, Austria.


Let A1 A2 . . . An be a circumscribable n-gon with incircle of radius 1, and let F1 , F2 , . . . , Fn be
the areas of the n corner regions inside the n-gon and outside the incircle. Show that

1 1 n2
+ ··· + ≥ .
F1 Fn n tan πn − π

Equality holds for the regular n-gon.

1508. Proposed by Edward T. H. Wang, Wilfrid Laurier University, Waterloo, Ontario.


Let a ≤ b < c be the lengths of the sides of a right triangle. Find the largest constant K such
that
a2 (b + c) + b2 (c + a) + c2 (a + b) ≥ Kabc

holds for all right triangles and determine when equality holds. It is known that the inequality
holds when K = 6 (problem 351 of the College Math. Journal ; solution on p. 259 of Volume 20,
1989).

1509. Proposed by Carl Friedrich Sutter, Viking, Alberta.


Professor Chalkdust teaches two sections of a mathematics course, with the same material taught
in both sections. Section 1 runs Mondays, Wednesdays, and Fridays for 1 hour each day, and
Section 2 runs Tuesdays and Thursdays for 1.5 hours each day. Normally Professor Chalkdust
covers one unit of material per hour, but if she is teaching some material for the second time
she teaches twice as fast. The course began a Monday. In the long run (i. e. after N weeks as
N → ∞) will one section be taught more material than the other? If so, which one, and how
much more?

57
1510F . Proposed by Jack Garfunkel, Flushing, N. Y.
P is any point inside a triangle ABC. Lines P A, P B, P C are drawn and angles P AC, P BA,
P CB are denoted by α, β, γ respectively. Prove or disprove that
A B C
cot α + cot β + cot γ ≥ cot + cot + cot ,
2 2 2
with equality when P is the incenter of 4ABC.

1512F . Proposed by Walther Janous, Ursulinengymnasium, Innsbruck, Austria.


Given r > 0, determine a constant C = C(r) such that

(1 + z)r (1 + z r ) ≤ C (1 + z 2 )r

for all z > 0.

1515. Proposed by Marcin E. Kuczma, Warszawa, Poland.


We are given a finite collection of segments in the plane, of total length 1. Prove that there
exists a straight line l such that the sum of lengths of projections of the given segments to line
l is less than 2/π.

1516. Proposed by Toshio Seimiya, Kawasaki, Japan.


ABC is an isosceles triangle in which AB = AC and ÁA < 90 ◦. Let D be any point on segment
BC. Draw CE parallel to AD meeting AB produced in E. Prove that CE > 2CD.

1523. Proposed by Walther Janous, Ursulinengymnasium, Innsbruck, Austria.


Let 0 < t ≤ 21 be fixed. Show that
µ ¶
X √ 1 X
cos tA ≥ 2 + 2 cos t + π+ sin tA,
4
where the sums are cyclic over the angles A, B, C of a triangle. [This generalizes Murray Klam-
kin’s problem E3180 in the Amer. Math. Monthly (solution p. 771, October 1988.]

1524. Proposed by George Tsintsifas, Thessaloniki, Greece.


ABC is a triangle with sides a, b, c and area F , and P is an interior point. Put R1 = AP ,
R2 = BP
√ , R3 = CP . Prove that the triangle with sides aR1 , bR2 , cR3 has circumradius at least
4F/(3 3).

1528F . Proposed by Ji Chen, Ningbo University, China.


If a, b, c, d are positive real numbers such that a + b + c + d = 2, prove or disprove that
a2 b2 c2 d2 16
+ + + ≤ .
(a2 + 1)2 (b2 + 1)2 (c2 + 1)2 (d2 + 1)2 25

1530F . Proposed by D. S. Mitrinović, University of Belgrade, and J. E. Pečarić, University


of Zagreb.
Let
R π/2
0 sin2k x dx
Ik = R π/2
0 sin2k+1 x dx
where k is a natural number. Prove that
1
1 ≤ Ik ≤ 1 + .
2k

58
1531. Proposed by J. T. Groenman, Arnhem, The Netherlands.
Prove that
v+w bc w+u ca u+v ab
· + · + · ≥ 4 (a + b + c),
u s−a v s−b w s−c
where a, b, c, s are the sides and semiperimeter of a triangle, and u, v, w are positive real numbers.
(Compare with Crux 1212 [1988: 115].)

1533. Proposed by Marcin E. Kuczma, Warszawa, Poland.


For any integers n ≥ k ≥ 0, n ≥ 1, denote by p(n, k) the probability that a randomly chosen
permutation of {1, 2, . . . , n} has exactly k fixed points, and let

P (n) = p(n, 0)p(n, 1) · · · p(n, n).

Prove that

P (n) ≤ exp(−2n n!).

1534. Proposed by Jack Garfunkel, Flushing, N. Y.


Triangle H1 H2 H3 is formed by joining the feet of the altitudes of an acute triangle A1 A2 A3 .
Prove that
s s0
≤ 0,
r r
where s, s0 and r, r 0 are the semiperimeters and inradii of A1 A2 A3 and H1 H2 H3 respectively.

1539F . Proposed by D. M. Milošević, Pranjani, Yugoslavia.


If α, β, γ are the angles, s the semiperimeter, R the circumradius and r the inradius of a triangle,
prove or disprove that
µ ¶2
X α
2 β 2R − r
tan tan2 ≤ ,
2 2 s

where the sum is cyclic.

1542F . Proposed by Murray S. Klamkin, University of Alberta.


For fixed n, determine the minimum value of

Cn = | cos θ| + | cos 2θ| + · · · + | cos nθ|.

It is conjectured that min Cn = [n/2] for n > 2.

1543. Proposed by George Tsintsifas, Thessaloniki, Greece.


Show that the circumradius of a triangle is at least four times the inradius of the pedal triangle
of any interior point.

1546. Proposed by Graham Denham, student, University of Alberta.


Prove that for every positive integer n and every positive real x,
n 2
X xk n(n+1)
≥x 2 .
k
k=1

59
1550. Proposed by Mihály Bencze, Brasov, Romania.
Let A = [−1, 1]. Find all functions f : A → A such that

|xf (y) − yf (x)| ≥ |x − y|

for all x, y ∈ A.

1553. Proposed by Murray S. Klamkin, University of Alberta.


It has been shown by Oppenheim that if ABCD is a tetrahedron of circumradius R, a, b, c are
the edges of face ABC, and p, q, r are the edges AD, BD, CD, then

64R4 ≥ (a2 + b2 + c2 )(p2 + q 2 + r2 ).

Show more generally that, for n-dimensional simplexes,

(n + 1)4 R4 ≥ 4E0 E1 ,

where E0 is the sum of the squares of all edges emanating from one of the vertices and E 1 is the
sum of the squares of all the other edges.

1558. Proposed by George Tsintsifas, Thessaloniki, Greece.


Let P be an interior point of a triangle ABC and let AP , BP , CP intersect the circumcircle
of 4ABC again in A0 , B 0 , C 0 , respectively. Prove that the power p of P with respect to the
circumcircle satisfies

|p| ≥ 4rr 0 ,

where r, r 0 are the inradii of triangles ABC and A0 B 0 C 0 .

1562. Proposed by Toshio Seimiya, Kawasaki, Japan.


Let M be the midpoint of BC of a triangle ABC such that ÁB = 2ÁC, and let D be the
intersection of the internal bisector of angle C with AM . Prove that ÁM DC ≤ 45 ◦.

1567. Proposed by Seung-Jin Bang, Seoul, Republic of Korea.


Let

x1 x 1 + · · · + x n
f (x1 , x2 , . . . , xn ) = .
(x1 + · · · + xn−1 )2 + xn

Prove that f (x1 , x2 , . . . , xn ) ≤ 2 under the condition that x1 + · · · + xn ≥ 2 and all xi ≥ 0.

1568. Proposed by Jack Garfunkel, Flushing, N. Y.


Show that
X 2 ³X ´2
sin A ≥ √ cos A
3
where the sums are cyclic over the angles A, B, C of an acute triangle.

1571. Proposed by George Tsintsifas, Thessaloniki, Greece.


Let ABC be a triangle with circumradius R and area F , and let P be a point in the same plane.
Put AP = R1 , BP = R2 , CP = R3 , R0 the circumradius of the pedal triangle of P , and p the
power of P relative to the circumcircle of 4ABC. Prove that
p
18R2 R0 ≥ a2 R1 + b2 R2 + c2 R3 ≥ 4F 3 |p|.

60
1574. Proposed by Murray S. Klamkin, University of Alberta.
Determine sharp upper and lower bounds for the sum of the squares of the sides of a quadrilateral
with given diagonals e and f . For the upper bound, it is assumed that the quadrilateral is convex.

1578. Proposed by O. Johnson and C. S. Goodlad, students, King Edward’s School, Birming-
ham, England.
For each fixed positive real number an , maximise
a1 a2 · · · an−1
(1 + a1 )(a1 + a2 )(a2 + a3 ) · · · (an−1 + an )
over all positive real numbers a1 , a2 , . . . , an−1 .

1580F . Proposed by Ji Chen, Ningbo University, China.


For every convex n-gon, if one circle with centre O and radius R contains it and another circle
with centre I and radius r is contained in it, prove or disprove that
π 2
R2 ≥ r2 sec2 + IO .
n

1581F . Proposed by Murray S. Klamkin and Andy Liu, University of Alberta.


If T1 and T2 are two triangles with equal circumradii, it is easy to show that if the angles of
T2 majorize the angles of T1 , then the area and perimeter of T2 is not greater than the area
and perimeter, respectively, of T1 . (One uses the concavity of sin x and log sin x in (0, π).) If T1
and T2 are two tetrahedra with equal circumradii, and the solid angles of T2 majorize the solid
angles of T1 , is it true that the volume, the surface area, and the total edge length of T 2 are not
larger than the corresponding quantities for T1 ?

1584. Proposed by Walther Janous, Ursulinengymnasium, Innsbruck, Austria.


Prove that for λ > 1
µ ¶
ln λ 3 2
< .
λ−1 λ(λ + 1)

1586. Proposed by Jack Garfunkel, Flushing, N. Y.


Let ABC be a triangle with angles A ≥ B ≥ C and sides a ≥ b ≥ c, and let A0 B 0 C 0 be a triangle
with sides

a0 = a + λ, b0 = b + λ, c0 = c + λ

where λ is a positive constant. Prove that A − C ≥ A0 − C 0 (i. e., 4A0 B 0 C 0 is in a sense “more
equilateral“ than 4ABC).

1588. Proposed by D. M. Milošević, Pranjani, Yugoslavia.


Show that
a2
sin B sin C ≤ 1 − ,
(b + c)2
where a, b, c are the sides of the triangle ABC.

1589. Proposed by Mihály Bencze, Brasov, Romania.


Prove that, for any natural number n,

n
p p
n! + n+2 (n + 2)! < 2 · n+1 (n + 1)!.

61
1592. Proposed by Marcin E. Kuczma, Warszawa, Poland.
If P is a monic polynomial of degree n > 1, having n negative roots (counting multiplicities),
show that
P 0 (0)P (1) ≥ 2n2 P (0),
and find conditions for equality.

1598F . Proposed by Walther Janous, Ursulinengymnasium, Innsbruck, Austria.


Let λ > 0. Determine the maximum constant C = C(λ) such that for all non-negative real
numbers x1 , x2 there holds
x21 + x22 + λx1 x2 ≥ C (x1 + x2 )2 .
1599. Proposed by Milen N. Naydenov, Varna, Bulgaria.
A convex quadrilateral with sides a, b, c, d has both an incircle and a circumcircle. Its circumra-
dius is R and its area F . Prove that

abc + abd + acd + bcd ≤ 2 F (F + 2R2 ).
1601. Proposed by Toshio Seimiya, Kawasaki, Japan.
ABC is a right-angled triangle with the right angle at A. Let D be the foot of the perpendicular
from A to BC, and let E and F be the intersections of the bisector of ÁB with AD and AC
respectively. Prove that DC > 2 EF .

1602. Proposed by Marcin E. Kuczma, P Warszawa, Poland.


Suppose x1 , x2 , . . . , xn ∈ [0, 1] and ni=1 xi = m + r where m is an integer and r ∈ [0, 1). Prove
that
Xn
x2i ≤ m + r 2 .
i=1

1606F . Proposed by Walther Janous, Ursulinengymnasium, Innsbruck, Austria.


For integers n ≥ k ≥ 1 and real x, 0 ≤ x ≤ 1, prove or disprove that
³ k−1 µ ¶µ ¶
x ´n X j n j
1− ≥ 1− x (1 − x)n−j .
k k j
j=0

1609. Proposed by John G. Heuver, Grande Prairie Composite H. S., Grande Prairie, Alberta.
P is a point in the interior of a tetrahedron ABCD of volume V , and Fa , Fb , Fc , Fd are the areas
of the faces opposite vertices A, B, C, D, respectively. Prove that
P A · Fa + P B · Fb + P C · Fc + P D · Fd ≥ 9V.
1610. Proposed by P. Penning, Delft, The Netherlands.
Consider the multiplication d × dd × ddd, where d < b − 1 is a nonzero digit in base b, and the
product (base b) has six digits, all less than b − 1 as well. Suppose that, when d and the digits
of the product are all increased by 1, the multiplication is still true. Find the lowest base b in
which this can happen.

1611. Proposed by George Tsintsifas, Thessaloniki, Greece.


Let ABC be a triangle with angles A, B, C (measured in radians), sides a, b, c, and semiperimeter
s. Prove that
Xb+c−a 6s Xb+c−a 9
(i) ≥ ; (ii) ≥ .
A π aA π

62
1612F . Proposed by Walther Janous, Ursulinengymnasium, Innsbruck, Austria.
Let x, y, z be positive real numbers. Show that
X y 2 − x2
≥ 0,
z+x
where the sum is cyclic over x, y, z, and determine when equality holds.

1613. Proposed by Murray S. Klamkin, University of Alberta.


Prove that
µ ¶ µ ¶
sin x 2p tan x p
+ ≥2
x x
for p ≥ 0 and 0 < x < π/2. (The case p = 1 is problem E3306, American Math. Monthly,
solution in March 1991, pp. 264–267.)

1619. Proposed by Hui-Hua Wan and Ji Chen, Ningbo University, Zhejiang, China.
Let P be an interior point of a triangle ABC and let R1 , R2 , R3 be the distances from P to the
vertices A, B, C, respectively. Prove that, for 0 < k < 1,
1
R1k + R2k + R3k < (1 + 2 k−1 )1−k (ak + bk + ck ).
1621F . Proposed by Murray S. Klamkin, University of Alberta. (Dedicated to Jack Garfun-
kel.)
Let P be a point within or on an equilateral triangle and let c1 ≤ c2 ≤ c3 be the lengths of the
three concurrent cevians through P . Determine the minimum value of c2 /c3 over all P .

1622. Proposed by Marcin E. Kuczma, Warszawa, Poland.


Let n be a positive integer.
(a) Prove the inequality
õ ¶ µ ¶ !n
a2n + b2n a+b 2 a−b 2
≤ + (2n − 1)
2 2 2
for real a, b, and find conditions for equality.
(b) Show that the constant 2n − 1 in the right-hand expression is the best possible, in the sense
that on replacing it by a smaller one we get an inequality which fails to hold for some a, b.

1627. Proposed by George Tsintsifas, Thessaloniki, Greece. (Dedicated to Jack Garfunkel.)


Two perpendicular chords M N and ET partition the circle (O, R) into four parts Q 1 , Q2 , Q3 , Q4 .
We denote by (Oi , ri ) the incircle of Qi , 1 ≤ i ≤ 4. Prove that

r1 + r2 + r3 + r4 ≤ 4( 2 − 1)R.
1629. Proposed by Rossen Ivanov, student, St. Kliment Ohridsky University, Sofia, Bulgaria.
In a tetrahedron x and v, y and u, z and t are pairs of opposite edges, and the distances
between the midpoints of each pair are respectively l, m, n. The tetrahedron has surface area S,
circumradius R, and inradius r. Prove that, for any real number a with 0 ≤ a ≤ 1,
à √ !1−a
2a 2a 2 2a 2a 2 2a 2a 2 3
x v l +y u m +z t n ≥ (2S)1+a (Rr)a .
4
1630. Proposed by Isao Ashiba, Tokyo, Japan.
Maximize
a1 a2 + a3 a4 + · · · + a2n−1 a2n
over all permutations a1 , a2 , . . . , a2n of the set {1, 2, . . . , 2n}.

63
1631F . Proposed by Murray S. Klamkin, University of Alberta. (Dedicated to Jack Garfun-
kel.)
Let P be a point within or on an equilateral triangle and let c1 , c2 , c3 be the lengths of the three
concurrent cevians through P . Determine the largest constant λ such that c λ1 , cλ2 , cλ3 are the sides
of a triangle for any P .

1633. Proposed by Toshio Seimiya, Kawasaki, Japan.


In triangle ABC, the internal bisectors of ÁB and ÁC meet AC and AB at D and E, respectively.
We put ÁBDE = x, ÁCED = y. Prove that if ÁA > 60 ◦ then cos 2x + cos 2y > 1.

1634. Proposed by F. F. Nab, Tunnel Mountain, Alberta.


A cafeteria at a universtity has round tables (of various sizes) and rectangular trays (all the same
size). Diners place their trays of food on the table in one of two ways, depending on whether
the short or long sides of the trays point toward the centre of the table:

or

Moreover, at the same table everybody aligns their trays the same way. Suppose n mathematics
students come in to eat together. How should they align their trays so that the table needed is
as small as possible?

1636F . Proposed by Walther Janous, Ursulinengymnasium, Innsbruck, Austria.


Determine the set of all real exponents r such that
|x − y|
dr (x, y) =
(x + y)r
satisfies the triangle inequality

dr (x, y) + dr (y, z) ≥ dr (x, z) for all x, y, z > 0

(and thus induces a metric on R+ – see Crux 1449, esp. [1990: 224]).

1637. Proposed by George Tsintsifas, Thessaloniki, Greece.


Prove that
X sin B + sin C 12
>
A π
where the sum is cyclic over the angles A, B, C (measured in radians) of a nonobtuse triangle.

1639. Proposed by K. R. S. Sastry, Addis Ababa, Ethiopia.


ABCD is a convex cyclic quadrilateral. Prove that

(AB + CD)2 + (AD + BC)2 ≥ (AC + BD)2 .

1642. Proposed by Murray S. Klamkin, University of Alberta.


Determine the maximum value of

x(1 − y 2 )(1 − z 2 ) + y(1 − z 2 )(1 − x2 ) + z(1 − x2 )(1 − y 2 )

subject to yz + zx + xy = 1 and x, y, z ≥ 0.

64
1649F . Proposed by D. M. Milošević, Pranjani, Yugoslavia.
Prove or disprove that
X α X √
cot − 2 cot α ≥ 3,
2
where the sums are cyclic over the angles α, β, γ of a triangle.

1651. Proposed by George Tsintsifas, Thessaloniki, Greece.


Let ABC be a triangle and A1 , B1 , C1 the common points of the inscribed circle with the sides
BC, CA, AB, respectively. We denote the length of the arc B1 C1 (not containing A1 ) of the
incircle by Sa , and similarly define Sb and Sc . Prove that

a b c 9 3
+ + ≥ .
Sa Sb Sc π

1652. Proposed by Murray S. Klamkin, University of Alberta.


Given fixed constants a, b, c > 0 and m > 1, find all positive values of x, y, z which minimize
µ ¶
xm + y m + z m + a m + b m + c m x+y+z+a+b+c m
− .
6 6

1654F . Proposed by Walther Janous, Ursulinengymnasium, Innsbruck, Austria.


Let x, y, z be positive real numbers. Show that
X x
p ≤ 1,
x + (x + y)(x + z)

where the sum is cyclic over x, y, z, and determine when equality holds.

1656. Proposed by Hidetosi Fukagawa, Aichi, Japan.


Given a triangle ABC, we take variable points P on segment AB and Q on segment AC. CP
meets BQ in T . Where should P and Q be located so that the area of 4P QT is maximized?

1662. Proposed by Murray S. Klamkin, University of Alberta.


Prove that
x2r+1 x2r+1 x2r+1 4r
1
+ 2 + ··· + n ≥ (x1 x2 + x2 x3 + · · · + xn x1 )r ,
s − x1 s − x2 s − xn (n − 1)n2r−1

where n > 3, r ≥ 1/2, xi ≥ 0 for all i, and s = x1 + x2 + · · · + xn . Also, find some values of n
and r such that the inequality is sharp.

1663F . Proposed by Walther Janous, Ursulinengymnasium, Innsbruck, Austria.


Let A, B, C be the angles of a triangle, r its inradius and s its semiperimeter. Prove that
r r
X A 3 r X A
cot ≤ csc ,
2 2 s 2
where the sums are cyclic over A, B, C.

65
1664. Proposed by Iliya Bluskov, Technical University, Gabrovo, Bulgaria. (Dedicated to Jack
Garfunkel.)
Consider two concentric circles with radii R1 and R (R1 > R) and a triangle ABC inscribed
in the inner circle. Points A1 , B1 , C1 on the outer circle are determined by extending BC, CA,
AB, respectively. Prove that
F1 F
≥ 2,
R12 R

where F1 and F are the areas of triangles A1 B1 C1 and ABC respectively, with equality when
ABC is equilateral.

1666. Proposed by Marcin E. Kuczma, Warszawa, Poland.


(a) How many ways are there to select and draw a triangle T and a quadrilateral Q around a
common incircle of unit radius so that the area of T ∩ Q is as small as possible? (Rotations and
reflections of the figure are not considered different.)
(b)F The same question, with the triangle and the quadrilateral replaced by an m-gon and an
n-gon, where m, n ≥ 3.

1672. Proposed by Walther Janous, Ursulinengymnasium, Innsbruck, Austria.


Show that for positive real numbers a, b, c, x, y, z,
µ ¶
a b c xy + yz + zx
(y + z) + (z + x) + (x + y) ≥ 3 ,
b+c c+a a+b x+y+z
and determine when equality holds.

1674. Proposed by Murray S. Klamkin, University of Alberta.


Given positive real numbers r, s and an integer n > r/s, find positive x1 , x2 , . . . , xn so as to
minimize
µ ¶
1 1 1
+ + · · · + r (1 + x1 )s (1 + x2 )s · · · (1 + xn )s .
xr1 xr2 xn

1676. Proposed by K. R. S. Sastry, Addis Ababa, Ethiopia.


OA is a fixed radius and OB a variable radius of a unit circle, such that ÁAOB ≤ 90 ◦. P QRS
is a square inscribed in the sector OAB so that P Q lies along OA. Determine the minimum
length of OS.

1678. Proposed by George Tsintsifas, Thessaloniki, Greece.


Show that
√ √ √ √ √
s ( a + b + c) ≤ 2 (ra + rb + rc ),

where a, b, c are the sides of a triangle, s the semiperimeter, and ra , rb , rc the exradii.

1680. Proposed by Zun Shan and Ji Chen, Ningbo University, China.


If ma , mb , mc are the medians and ra , rb , rc the exradii of a triangle, prove that
rb rc rc ra ra rb
+ + ≥ 3.
mb mc mc ma ma mb

66
1691F . Proposed by Walther Janous, Ursulinengymnasium, Innsbruck, Austria.
Let n ≥ 2. Determine the best upper bound of
x1 x2 xn
+ + ··· + ,
x2 x3 · · · x n + 1 x1 x3 · · · x n + 1 x1 x2 · · · xn−1 + 1
over all x1 , . . . , xn with 0 ≤ xi ≤ 1 for i = 1, 2, . . . , n.

1695. Proposed by Seung-Jin Bang, Seoul, Republic of Korea.


Let p(x) = a0 + a1 x + a2 x2 + a3 x3 + a4 x4 + a5 x5 with a0 > 0 and
a0 + a 2 a2 + a 4 a4
a0 + + + < 0.
3 5 7
Prove that there exists at least one zero of p(x) in the interval (−1, 1).

1696. Proposed by Ed Barbeau, University of Toronto.


A B
An 8 21
by 11 sheet of paper is folded along a line AE through
the corner A so that the adjacent corner B on the longer side
lands on the opposite longer side CD at F . Determine, with E
a minimum of measurement or computation, whether triangle D C
F
AEF covers more than half the quadrilateral AECD.

1698. Proposed by Hidetosi Fukagawa, Aichi, Japan. A


ABC is an equilateral triangle of area 1. DEF is an equilateral
triangle of variable size, placed so that the two triangles over- F E
lap, with DE k AB, EF k BC, F D k CA, and D, E, F not
in 4ABC, as shown. The corners of 4DEF sticking outside
4ABC are then folded over. Find the maximum possible area B C
of the uncovered (shaded) part of 4DEF . D

1699. Proposed by Xue-Zhi Yang and Ji Chen, Ningbo University, China.


Let R, r, ha , hb , hc , ra , rb , rc be the circumradius, inradius, altitudes, and exradii of a triangle.
Prove that
r r r r r
2R ra rb rc 4R
+5≤ + + ≤ + 1.
r ha hb hc r

1701F . Proposed by Walther Janous, Ursulinengymnasium, Innsbruck, Austria.


If ABC is a triangle, prove or disprove that
½ ¾
ha cos A hb cos B hc cos C
R ≥ 4 max , , ,
1 + 8 cos2 A 1 + 8 cos2 B 1 + 8 cos2 C
where ha , hb , hc are the altitudes of the triangle and R is its circumradius.

1703. Proposed by Murray S. Klamkin, University of Alberta.


Determine the maximum and minimum values of
x2 + y 2 + z 2 + λxyz,
where x + y + z = 1, x, y, z ≥ 0, and λ is a given constant.

1707. Proposed by Allan Wm. Johnson Jr., Washington, D. C.


What is the largest integer m for which an m × m square can be cut up into 7 rectangles whose
dimensions are 1, 2, . . . , 14 in some order?

67
1712. Proposed by Murray S. Klamkin, University of Alberta.
Determine the minimum value of
16 sin2 (A/2) sin2 (B/2) sin2 (C/2) + 1
tan(A/2) tan(B/2) tan(C/2)

where A, B, C are the angles of a triangle.

1713. Proposed by Jeremy Bern, student, Ithaca H. S., Ithaca, N. Y.


For a fixed positive integer n, let K be the area of the region
( n ¯ ¯ )
X ¯ 1 ¯
z: ¯z − k¯ ≥ 1
¯ ¯
k=1

in the complex plane. Prove that K ≥ π(11n2 + 1)/12.

1730. Proposed by George Tsintsifas, Thessaloniki, Greece.


Prove that
X sabc
bc(s − a)2 ≥ ,
2
where a, b, c, s are the sides and semiperimeter of a triangle, and the sum is cyclic over the sides.

1734. Proposed by Murray S. Klamkin, University of Alberta.


Determine the minimum value of
p p p
(1 − ax)2 + (ay)2 + (az)2 + (1 − by)2 + (bz)2 + (bx)2 + (1 − cz)2 + (cx)2 + (cy)2

for all real values of a, b, c, x, y, z.

1742. Proposed by Murray S. Klamkin, University of Alberta.


Let 1 ≤ r < n be integers and xr+1 , xr+2 , . . . , xn be given positive real numbers. Find positive
x1 , x2 , . . . , xr so as to minimize the sum
X xi
S=
xj

taken over all i, j ∈ {1, 2, . . . , n} with i 6= j.


(This problem is due to Byron Calhoun, a high school student in McLean, Virginia. It appeared,
with solution, in a science project of his.)

1743F . Proposed by Walther Janous, Ursulinengymnasium, Innsbruck, Austria.


Let 0 < γ < 180 ◦ be fixed. Consider the set 4(γ) of all triangles ABC having angle γ at C, whose
altitude through C meets AB in an interior point D such that the line through the incenters of
4ADC and 4BCD meets the sides AC and BC in interior points E and F respectively. Prove
or disprove that
µ ¶ µ ¶2
area(4EF C) cos(γ/2) − sin(γ/2) + 1
sup = .
4(γ) area(4ABC) 2 cos(γ/2)

(This would generalize problem 5 of the 1988 IMO [1988: 197].)

68
1749. Proposed by D. M. Milošević, Pranjani, Yugoslavia.
Let ABC be a triangle with external angle-bisectors wa0 , wb0 , wc0 , inradius r, and circumradius R.
Prove that
Ãs s s !2
1 1 1 2
(i) 0
+ 0 + 0
< ;
wa wb wc r
µ ¶
1 1 1 2 R
(ii) 0
+ 0 + 0 < 2.
wa wb wc 3r

1750. Proposed by Iliya Bluskov, Technical University, Gabrovo, Bulgaria.


Pairs of numbers from the set {11, 12, . . . , n} are adjoined to each of the 45 different (unordered)
pairs of numbers from the set {1, 2, . . . , 10}, to obtain 45 4-element sets A 1 , A2 , . . . , A45 . Suppose
that |Ai ∩ Aj | ≤ 2 for all i 6= j. What is the smallest n possible?

1754F . Proposed by Walther Janous, Ursulinengymnasium, Innsbruck, Austria.


Let n and k be positiveP integers such that 2 ≤ k < n, and let x1 , x2 , . . . , xn be nonnegative real
numbers satisfying ni=1 xi = 1. Prove or disprove that
½ ¾
X 1 1
x1 x2 · · · xk ≤ max , ,
k k nk−1

where the sum is cyclic over x1 , x2 , . . . , xn . [The case k = 2 is known – see inequality (1) in the
solution of Crux 1662, this issue.]

1756. Proposed by K. R. S. Sastry, Addis Ababa, Ethiopia.


For positive integers n ≥ 3 and r ≥ 1, the n-gonal number of rank r is defined as

r2 r
P (n, r) = (n − 2) − (n − 4) .
2 2
Call a triple (a, b, c) of natural numbers, with a ≤ b < c, an n-gonal Pythagorean triple if
P (n, a) + P (n, b) = P (n, c). When n = 4, we get the usual Pythagorean triple.
(i) Find an n-gonal Pythagorean triple for each n.
(ii) Consider all triangles ABC whose sides are n-gonal Pythagorean triples for some n ≥ 3.
Find the maximum and the minimum possible values of angle C.

1757. Proposed by Avinoam Freedman, Teaneck, N. J.


Let A1 A2 A3 be an acute triangle with sides a1 , a2 , a3 and area F , and let 4B1 B2 B3 (with sides
b1 , b2 , b3 ) be inscribed in 4A1 A2 A3 with B1 ∈ A2 A3 , etc. Show that for any x1 , x2 , x3 > 0,

(x1 a21 + x2 a22 + x3 a23 )(x1 b21 + x2 b22 + x3 b23 ) ≥ 4F 2 (x2 x3 + x3 x1 + x1 x2 ).

1759. Proposed by Isao Ashiba, Tokyo, Japan.


A is a fixed point on a circle, and P and Q are variable points on the circle so that AP + P Q
equals the diameter of the circle. Find P and Q so that the area of 4AP Q is as large as possible.

1761. Proposed by Toshio Seimiya, Kawasaki, Japan.


ABC is an isosceles triangle with AB = AC. Let D be the foot of the perpendicular from C to
AB, and let M be the midpoint of CD. Let E be the foot of the perpendicular from A to BM ,
and let F be the foot of the perpendicular from A to CE. Prove that AF ≤ AB/3.

69
1762. Proposed by Steven Laffin, student, École J. H. Picard and Andy Liu, University of Al-
berta, Edmonton. (Dedicated to Professor David Monk, University of Edinburgh, on his sixtieth
birthday.)
Starship Venture is under attack from a Zokbar fleet, and its Terrorizer is destroyed. While it
can hold out, it needs a replacement to drive off the Zokbars. Starbase has spare Terrorizers,
which can be taken apart into any number of components, and enough scout ships to provide
transport. However, the Zokbars have n Space Octopi, each of which can capture one scout ship
at a time. Starship Venture must have at least one copy of each component to reassemble a
Terrorizer, but it is essential that the Zokbars should not be able to do the same. Into how many
components must each Terrorizer be taken apart (assuming all are taken apart in an identical
manner), and how many scout ships are needed to transport them? Give two answers:
(a) assuming that the number of components per Terrorizer is as small as possible, minimize
the number of scout ships;
(b) assuming instead that the number of scout ships is as small as possible, minimize the number
of components per Terrorizer.

1763. Proposed by Walther Janous, Ursulinengymnasium, Innsbruck, Austria.


Let t ≥ 0, and for each integer n ≥ 1 define

1 + t + t2 + · · · + t n
xn = .
n+1
√ √ √
Prove that x1 ≤ x2 ≤ 3 x3 ≤ 4 x4 ≤ · · ·.

1764. Proposed by Murray S. Klamkin, University of Alberta.


(a) Determine the extreme values of a2 b + b2 c + c2 a, where a, b, c are sides of a triangle of
semiperimeter 1.
(b)F What are the extreme values of a21 a2 + a22 a3 + · · · + a2n a1 , where a1 , a2 , . . . , an are the
(consecutive) sides of an n-gon of semiperimeter 1?

1765. Proposed by Kyu Hyon Han, student, Seoul, South Korea.


There are four circles piled up, making a total of 10 regions.
The outer circles each have 5 regions and the central circle
has 7 regions. You put one of the numbers 0, 1, . . . , 9 in each
region, without reusing any number, so that the sum of the
numbers in any circle is always the same value, say S. What
is the smallest and the largest possible value of S?

1766F . Proposed by Jun-hua Huang, The 4th Middle School of Nanxian, Hunan, China.
The sequence x1 , x2 , . . . is defined by x1 = 1, x2 = x, and

xn+2 = xxn+1 + nxn

for n ≥ 0. Prove or disprove that for each n ≥ 2, the coefficients of the polynomial x n−1 xn+1 −x2n
are all nonnegative, except for the constant term when n is odd.

70
1771F . Proposed by Walther Janous, Ursulinengymnasium, Innsbruck, Austria.
Let a, b, c be the sides of a triangle and u, v, w be non-negative real numbers such that u+v+w =
1. Prove that
X X
ubc − s vwa ≥ 3Rr,

where s, R, r are the semiperimeter, circumradius and inradius of the triangle, and the sums are
cyclic.

1772. Proposed by Iliya Bluskov, Technical University, Gabrovo, Bulgaria.


The equation x3 + ax2 + (a2 − 6)x + (8 − a2 ) = 0 has only positive roots. Find all possible values
of a.

1774F . Proposed by Murray S. Klamkin, University of Alberta.


Determine the smallest λ ≥ 0 such that

2(x3 + y 3 + z 3 ) + 3xyz ≥ (xλ + y λ + z λ )(x3−λ + y 3−λ + z 3−λ )

for all non-negative x, y, z.

1775. Proposed by P. Penning, Delft, The Netherlands.


Find the radius of the smallest sphere (in three-dimensional space) which is tangent to the three
lines y = 1, z = −1; z = 1, x = −1; x = 1, y = −1; and whose centre does not lie on the line
x = y = z.

1776. Proposed by David Doster, Choate Rosemary Hall, Wallingford, Connecticut.


Given 0 < x0 < 1, the sequence x0 , x1 , . . . is defined by
¯ ¯
3 3 ¯¯ 1 ¯¯
xn+1 = − ¯xn − ¯
4 2 2

for n ≥ 0. It is easy to see that 0 < xn < 1 for all n. Find the smallest closed interval J in [0, 1]
so that xn ∈ J for all sufficient large n.

1780. Proposed by Jordan Stoyanov, Queen’s University, Kingston, Ontario.


Prove that, for any natural number n and real numbers α1 , α2 , . . . , αn ,

(1 − sin2 α1 sin2 α2 · · · sin2 αn )n + (1 − cos2 α1 cos2 α2 · · · cos2 αn )n ≥ 1.

1781. Proposed by Walther Janous, Ursulinengymnasium, Innsbruck, Austria.


Let a > 0 and x1 , x2 , . . . , xn ∈ [0, a] (n ≥ 2) such that

x1 x2 · · · xn = (a − x1 )2 (a − x2 )2 · · · (a − xn )2 .

Determine the maximum possible value of the product x1 x2 · · · xn .

1784. Proposed by Murray S. Klamkin, University of Alberta, and Dale Varberg, Hamline
University, St. Paul, Minnesota.
A point in 3-space is at distances 9, 10, 11 and 12 from the vertices of a tetrahedron. Find the
maximum and minimum possible values of the sum of the squares of the edges of the tetrahedron.

71
1788. Proposed by Christopher J. Bradley, Clifton College, Bristol, England.
A pack of cards consists of m red cards and n black cards. The pack is thoroughly shuffled and
the cards are then laid down in a row. The number of colour changes one observes in moving
from left to right along the row is k. (For example, for m = 5 and n = 4 the row RRBRBBRBR
exhibits k = 6.) Prove that k is more likely to be even than odd if and only if

|m − n| > m + n.

1789F . Proposed by D. M. Milošević, Pranjani, Yugoslavia.


Let a1 , a2 , a3 be the sides of a triangle, w1 , w2 , w3 the angle bisectors, F the area, and s the
semiperimeter. Prove or disprove that

w1a1 + w2a2 + w3a3 ≤ (F 3)s .

1792. Proposed by Walther Janous, Ursulinengymnasium, Innsbruck, Austria.


Let x, y ≥ 0 such that x + y = 1, and let λ > 0. Determine the best lower and upper bounds (in
terms of λ) for

(λ + 1)(xλ + y λ ) − λ(xλ+1 + y λ+1 ).

1793. Proposed by Murray S. Klamkin, University of Alberta.


Prove that in any n-dimensional simplex there is at least one vertex such that the n edges
emanating from that vertex are possible sides of an n-gon.

1794. Proposed by Iliya Bluskov, Technical University, Gabrovo, Bulgaria.


Pairs of numbers from the set {7, 8, . . . , n} are adjoined to each of the 20 different (unordered)
triples of numbers from the set {1, 2, . . . , 6}, to obtain twenty 5-element sets A 1 , A2 , . . . , A20 .
Suppose that |Ai ∩ Aj | ≤ 2 for all i 6= j. What is the smallest n possible?

1796. Proposed by Ji Chen, Ningbo University, China.


If A, B, C are the angles of a triangle, prove that
X X
sin B sin C ≤ 3 sin(B/2) sin(C/2),

where the sums are cyclic.

1801. Proposed by Murray S. Klamkin, University of Alberta. (Dedicated to O. Bottema.)


If A1 , A2 , A3 are angles of a triangle, prove that
X
(1 + 8 cos A1 sin A2 sin A3 )2 sin A1 ≥ 64 sin A1 sin A2 sin A3 ,

where the summation is cyclic over the indices 1, 2, 3.

1802. Proposed by Walther Janous, Ursulinengymnasium, Innsbruck, Austria.


Prove that, for any real numbers x and y,

x4 + y 4 + (x2 + 1)(y 2 + 1) ≥ x3 (1 + y) + y 3 (1 + x) + x + y,

and determine when equality holds.

1808. Proposed by George Tsintsifas, Thessaloniki, Greece.


Three congruent circles that pass through a common point meet again in points A, B, C. A 0 B 0 C 0
is the triangle containing the three circles and whose sides are each tangent to two of the circles.
Prove that [A0 B 0 C 0 ] ≥ 9[ABC], where [XY Z] denotes the area of triangle XY Z.

72
1813F . Proposed by D. N. Verma, Bombay, India.
Suppose that a1 > a2 > a3 and r1 > r2 > r3 are positive real numbers. Prove that the
determinant
¯ r1 ¯
¯ a1 ar12 ar13 ¯
¯ r ¯
¯ a 1 ar2 ar3 ¯
¯ r2 2 2 ¯
¯ a 1 ar2 ar3 ¯
3 3 3
is positive.

1816. Proposed by Marcin E. Kuczma, Warszawa, Poland.


Given a finite set S of n + 1 points in the plane, with two distinguished points B and E in S,
consider all polygonal paths P = P0 P1 . . . Pn whose vertices are all points of S, in any order
except that P0 = B and Pn = E. For such a path P define l(P) to be the length of P and
n−1
X −−−−→ −−−−→
a(P) = θ(Pi−1 Pi , Pi Pi+1 ),
i=1
where θ(v, w) is the angle between the vectors v and w, 0 ≤ θ(v, v) ≤ π. Prove or disprove that
the minimum values of l(P) and of a(P) are attained for the same path P.

1818. Proposed by Ed Barbeau, University of Toronto.


Prove that, for 0 ≤ x ≤ 1 and a positive integer k,
(1 + x)k [x + (1 − x)k+1 ] ≥ 1.
1823. Proposed by G. P. Henderson, Campbellcroft, Ontario.
A rectangular box is to be decorated with a ribbon that goes across the faces and makes various
angles with the edges. If possible, the points where the ribbon crosses the edges are chosen so
that the length of the closed path is a local minimum. This will ensure that the ribbon can be
tightened and tied without slipping off. Is there always a minimal path that crosses all six faces
just once?

1824. Proposed by George Tsintsifas, Thessaloniki, Greece.


Let ABC be a triangle and M a point in its plane. We consider the circles with diameters AM ,
BM , CM and the circle containing and internally tangent to these three circles. Show that the
radius P of this large circle satisfies P ≥ 3r, where r is the inradius of 4ABC.

1825. Proposed by Marcin E. Kuczma, Warszawa, Poland.


Suppose that the real polynomial x4 + ax3 + bx2 + cx + d has four positive roots. Prove that
abc ≥ a2 d + 5c2 .

1827. Proposed by Šefket Arslanagić, Trebinje, Yugoslavia, and D. M. Milošević, Pranjani,


Yugoslavia.
Let a, b, c be the sides, A, B, C the angles (measured in radians), and s the semi-perimeter of
a triangle.
(i) Prove that
X bc 12 s
≥ ,
A(s − a) π
where the sums here and below are cyclic.
(ii)F It follows easily from the proof of Crux 1611 (see [1992: 62] and the correction in this issue)
that also
Xb+c 12 s
≥ .
A π
Do the two summations above compare in general?

73
1830. Proposed by P. Tsaoussoglou, Athens, Greece.
If a > b > c > 0 and a−1 + b−1 + c−1 = 1, prove that
4 1 1 4
2
+ + ≥ .
c (a − b)b (b − c)c 3

1831. Proposed by Walther Janous, Ursulinengymnasium, Innsbruck, Austria.


Let x, y, z be any real numbers and let λ be an odd positive integer. Prove or disprove that

x(x + y)λ + y(y + z)λ + z(z + x)λ ≥ 0.

1834. Proposed by Marcin E. Kuczma, Warszawa, Poland.


Given positive numbers A, G and H, show that they are respectively the arithmetic, geometric
and harmonic means of some three positive numbers x, y, z if and only if
µ ¶
A3 G3 3 A 2
+ + 1 ≤ 1 + .
G3 H 3 4 H

1837. Proposed by Andy Liu, University of Alberta.


A function f : R → R+ is said to be strictly log-convex if
µ µ ¶¶
x1 + x 2 2
f (x1 )f (x2 ) ≥ f
2
for all x1 , x2 ∈ R, with equality if and only if x1 = x2 . f is said to be strictly log-concave if the
inequlaity is reversed.
(a) Prove that if f and g are strictly log-convex functions, then so is f + g.
(b)F Does the same conclusion hold for strictly log-concave functions?

1840. Proposed by Jun-hua Huang, The 4th Middle School of Nanxian, Hunan, China.
Let 4ABC be an acute triangle with area F and circumcenter O. The distances from O to
BC, CA, AB are denoted da , db , dc respectively. 4A1 B1 C1 (with sides a1 , b1 , c1 ) is inscribed in
4ABC, with A1 ∈ BC etc. Prove that

da a1 + db b1 + dc c1 ≥ F.

1843. Proposed by Šefket Arslanagić, Trebinje, Yugoslavia, and D. M. Milošević, Pranjani,


Yugoslavia.
Let a, b, c be the sides, A, B, C the angles (measured in radians), and s the semi-perimeter of
a triangle.
(i) Prove that
X a 9
≥ .
2A(s − a) π

(ii)F It is obvious that also


X 1 9
≥ .
A π
Do the two summations compare in general?

74
1845. Proposed by Christopher J. Bradley, Clifton College, Bristol, U. K.
Suppose that x1 , x2 , x3 , x4 , x5 are real numbers satisfying x1 < x2 < x3 < x4 < x5 and
X X X X
xi = 10, xi xj = 35, xi xj xk = 50, xi xj xk xl = 25.
i i<j i<j<k i<j<k<l

Prove that

5+ 5
< x5 < 4.
2
1846. Proposed by George Tsintsifas, Thessaloniki, Greece.
Consider the three excircles of a given triangle ABC. Let A0 B 0 C 0 be the triangle containing
these three circles and whose sides are each tangent to two of the circles. Prove that

[A0 B 0 C 0 ] ≥ 25[ABC],

where [XY Z] denotes the area of triangle XY Z.

1849. Proposed by Shi-Chang Shi and Ji Chen, Ningbo University, China.


Let three points P, Q, R be on the sides BC, CA, AB, respectively, of a triangle ABC, such that
they cut the perimeter of 4ABC into three equal parts; i. e. QA + AR = RB + BP = P C + CQ.
(a) Prove that

1
RP · P Q + P Q · QR + QR · RP ≥ (a + b + c)2 .
12

(b)F Prove or disprove that the circumradius of 4P QR is at least half the circumradius of
4ABC.

1851. Proposed by Walther Janous, Ursulinengymnasium, Pn Innsbruck, Austria.


2
Let x1 , x2 , . . . , xn (n ≥ 2) be real numbers such that i=1 xi = 1. Prove that
√ n √
2 n−1 1 X xi + 2 2 n+1
√ ≤ ≤ √ .
5 n−1 n xi + 5 5 n+1
i=1

1853. Proposed by Iliya Bluskov, Technical University, Gabrovo, Bulgaria.


Let {bb }∞
n=1 be a sequence of positive real numbers which satisfies the condition

3bn+2 ≥ bn+1 + 2bn

for every n ≥ 1. Prove that either the sequence converges or limn→∞ bn = ∞.

1854. Proposed by K. R. S. Sastry, Addis Ababa, Ethiopia.


In any convex pentagon prove that the sum of the squares of the diagonals is less than three
times the sum of the squares of the sides.

1855. Proposed by Bernardo Recamán, United World College of Southern Africa, Mbabane,
Swaziland.
Twelve friends agree to eat out once a week. Each week they will divide themselves into 3 groups
of 4 each, and each of these groups will sit together at a separate table. They have agreed to
meet until any two of the friends will have sat at least once at the same table at the same time.
What is the minimum number of weeks this requires?

75
1856. Proposed by Jisho Kotani, Akita, Japan.
Find the rectangular brick of largest volume that can be completely wrapped in a square piece
of paper of side 1 (without cutting the paper).

1857. Proposed by Gottfried Perz, Pestalozzigymnasium, Graz, Austria.


Prove that, for any positive integer n,

27n (n!)3 √
1< < 2.
(3n + 1)!

1860F . Proposed by Jun-hua Huang, The 4th Middle School of Nanxian, Hunan, China.
Prove or disprove that
X cos[(A − B)/4]
≥ 4,
cos(A/2) cos(B/2)
where the sum is cyclic over the angles A, B, C of a triangle.

1861. Proposed by Walther Janous, Ursulinengymnasium, Innsbruck, Austria.


Let f : R+ → R be an increasing and concave function from the positive real numbers to the
reals. Prove that if 0 < x ≤ y ≤ z and n is a positive integer then

(z n − xn )f (y) ≥ (z n − y n )f (x) + (y n − xn )f (z).

1864. Proposed by George Tsintsifas, Thessaloniki, Greece.


Consider the three excircles of a given triangle ABC. Let P be the radius of the circle containing
and internally tangent to these three circles. Prove that P ≥ 7r, where r is the inradius of 4ABC.

1868. Proposed by De-jun Zhao, Chengtun High School, Xingchang, China.


Let n ≥ 3, a1 > a2 > · · · > an > 0, and p > q > 0. Show that

ap1 aq2 + ap2 aq3 + · · · + apn−1 aqn + apn aq1 > aq1 ap2 + aq2 ap3 + · · · + aqn−1 apn + aqn ap1 .

1870F . Proposed by K. R. S. Sastry, Addis Ababa, Ethiopia.


In any convex pentagon ABCDE prove or disprove that

AC · BD + BD · CE + CE · DA + DA · EB + EB · AC
> AB · CD + BC · DE + CD · EA + DE · AB + EA · BC.

(Note: the first sum involves diagonals, the second sum involves sides.)

1874. Proposed by Pedro Melendez, Belo Horizonte, Brazil.


Find the smallest positive integer n such that n! is divisible by 19931994 .

1877. Proposed by Iliya Bluskov, Technical University, Gabrovo, Bulgaria.


Let B1 , B2 , . . . , Bb be k-element subsets of {1, 2, . . . , n} such that |Bi ∩ Bj | ≤ 1 for all i 6= j.
Show that
· · ¸¸
n n−1
b≤ ,
k k−1

where [x] denotes the greatest integer ≤ x.

76
1878F . Proposed by Jun-hua Huang, The 4th Middle School of Nanxian, Hunan, China.
Given two triangles ABC and A0 B 0 C 0 , prove or disprove that

sin A0 sin B 0 sin C 0 R


+ + ≤1+ ,
sin A sin B sin C r
where r, R are the inradius and circumradius of triangle ABC.

1882. Proposed by Christopher J. Bradley, Clifton College, Bristol, U. K.


Arthur tosses a fair coin until he obtains two heads in succession. Betty tosses another fair coin
until she obtains a head and a tail in succession, with the head coming immediately prior to the
tail.
(i) What is the average number of tosses each has to make?
(ii) What is the probability that Betty makes fewer tosses than Arthur (rather than the same
number or more than Arthur)?

1883. Proposed by George Tsintsifas, Thessaloniki, Greece.


Let ABC be a triangle and construct the circles with sides AB, BC, CA as diameters. A 0 B 0 C 0
is the triangle containing these three circles and whose sides are each tangent to two of these
circles. Prove that
µ ¶
0 0 0 13 √
[A B C ] ≥ + 3 [ABC],
4

where [XY Z] denotes the area of triangle XY Z.

1887. Proposed by Václav Konec̆ný, Ferris State University, Big Rapids, Michigan.
Given an acute triangle ABC, form the hexagon A1 C2 B1 A2 C1 B2 B1 C2
as shown, where
A
BC = BC1 = CB2 ,
CA = CA1 = AC2 , A2 A1
B C
and
C1 B2
AB = AB1 = BA2 .
Prove that the area of the hexagon is at least 13 times the area of 4ABC, with equality when
ABC is equilateral.

1890. Proposed by Federico Ardila, student, Colegio San Carlos, Bogotá, Colombia.
Let n be a positive integer and let

f (x) = an xn + an−1 xn−1 + · · · + a1 x + a0 ,


k n k n−1 k k
g(x) = x + x + ··· + x + ,
an an−1 a1 a0

where k and the ai ’s are positive real numbers. Prove that

f (g(1))g(f (1)) ≥ 4k.

When does equality hold?

77
1892. Proposed by Marcin E. Kuczma, Warszawa, Poland.
Let n ≥ 4 be an integer. Find the exact upper and lower bounds for the cyclic sum
n
X xi
xi−1 + xi + xi+1
i=1

(where of course x0 = xn , xn+1 = x1 ), over all n-tupels of nonnegative numbers (x1 , . . . , xn )


without three zeros in cyclic succession. Characterize all cases in which either one of these
bounds is attained.

1895. Proposed by Ji Chen and Gang Yu, Ningbo University, China.


Let P be an interior point of a triangle A1 A2 A3 ; R1 , R2 , R3 the distances from P to A1 , A2 , A3 ;
and R the circumradius of 4A1 A2 A3 . Prove that
32 3
R1 R2 R3 ≤ R ,
27
with equality when A2 = A3 and P A2 = 2P A1 .

1901. Proposed by Marcin E. Kuczma, Warszawa, Poland.


Suppose f : R → R is a continuous even function such that f (0) = 0 and f (x + y) ≤ f (x) + f (y)
for all x, y ∈ R. Must f be monotonic on R+ ?

1904. Proposed by Kee-Wai Lau, Hong Kong.


If ma , mb , mc are the medians of a triangle with sides a, b, c, prove that

ma (bc − a2 ) + mb (ca − b2 ) + mc (ab − c2 ) ≥ 0.

1907. Proposed by Gottfried Perz, Pestalozzigymnasium, Graz, Austria.


Find the largest constant k such that
kabc
≤ (a + b)2 + (a + b + 4c)2
a+b+c
for all a, b, c > 0.

1913. Proposed by N. Kildonan, Winnipeg, Manitoba.


I was at a restaurant for lunch the other day. The bill came, and I wanted to give the waiter a
whole number of dollars, with the difference between what I give him and the bill being the tip.
I always like to tip between 10 and 15 percent of the bill. But if I gave him a certain number
of dollars, the tip would have been less than 10 % of the bill, and if instead I gave him one
dollar more, the tip would have been more than 15 % of the bill. What was the largest possible
amount of the bill? [Editor’s note to non-North American readers: your answer should be in
dollars and cents, where there are (reasonably enough) 100 cents in a dollar.]

1914. Proposed by K. R. S. Sastry, Addis Ababa, Ethiopia.


Let A1 A2 . . . An be a regular n-gon, with M1 , M2 , . . . , Mn the midpoints of the sides. Let P be
a point in the plane of the n-gon. Prove that
n
X n
X
P Mi ≥ cos(180 ◦/n) P Ai .
i=1 i=1

78
1920. Proposed by Walther Janous, Ursulinengymnasium, Innsbruck, Austria.
Let a, b, c be the sides of a triangle.
(a) Prove that, for any 0 < λ ≤ 2,
µ ¶3
1 (a + b)(b + c)(c + a) 2
< ≤ ,
(1 + λ)2 (λa + b + c)(a + λb + c)(a + b + λc) 2+λ
and that both bounds are best possible.
(b)F What are the bounds for λ > 2?

1924. Proposed by Jisho Kotani, Akita, Japan.


A large sphere of radius 1 and a smaller sphere of radius r < 1 overlap so that their intersection
is a circle of radius r, i. e., a great circle of the small sphere. Find r so that the volume inside
the small sphere and outside the large sphere is as large as possible.

1933. Proposed by George Tsintsifas, Thessaloniki, Greece.


Two externally tangent circles of radii R1 and R2 are
internally tangent to a semicircle of radius 1, as in the
figure. Prove that

R1 + R2 ≤ 2 ( 2 − 1).

1940. Proposed by Ji Chen, Ningbo University, China.


Show that if x, y, z > 0,
µ ¶
1 1 1 9
(xy + yz + zx) 2
+ 2
+ 2
≥ .
(x + y) (y + z) (z + x) 4

1942. Proposed by Paul Bracken, University of Waterloo.


Prove that, for any a ≥ 1,
Ã∞ !2 ∞
X 1 X 1
2
> 2 .
(a + k) (a + k)3
k=0 k=0

1944. Proposed by Paul Yiu, Florida Atlantic University, Boca Raton.


Find the smallest positive integer n so that

(n + 1)2000 > (2n + 1)1999 .

1945. Proposed by Murray S. Klamkin, University of Alberta.


Let A1 A2 . . . An be a convex n-gon.
(a) Prove that

A1 A2 + A 2 A3 + · · · + A n A1 ≤ A 1 A3 + A 2 A4 + · · · + A n A2 .

(b)F Prove or disprove that


³π ´
2 cos (A1 A2 + A2 A3 + · · · + An A1 ) ≥ A1 A3 + A2 A4 + · · · + An A2 .
n
1948. Proposed by Marcin E. Kuczma, Warszawa, Poland.
Are there any nonconstant differentiable functions f : R → R such that

f (f (f (x))) = f (x) ≥ 0

for all x ∈ R?

79
1949. Proposed by Federico Ardila, student, Colegio San Carlos, Bogotá, Colombia.
Let D, E, F be points on the sides BC, CA, AB respectively of triangle ABC, and let R be the
circumradius of ABC. Prove that
µ ¶
1 1 1 AB + BC + CA
+ + (DE + EF + F D) ≥ .
AD BE CF R

1953. Proposed by Murray S. Klamkin, University of Alberta.


Determine a necessary and sufficient condition on real constants r1 , r2 , . . . , rn such that

x21 + x22 + · · · + x2n ≥ (r1 x1 + r2 x2 + · · · + rn xn )2

holds for all real x1 , x2 , . . . , xn .

1956. Proposed by George Tsintsifas, Thessaloniki, Greece.


In a semicircle of radius 4 there are three tangent circles
as in the figure. √Prove that the radius of the smallest
circle is at most 2 − 1.

1957. Proposed by William Soleau, New York.


A 9 by 9 board is filled with 81 counters, each being green on one side
and yellow on the other. Initially, all have their green sides up, except the
31 marked with circles in the diagram. In one move, we can flip over a
block of adjacent counters, vertically or horizontally only, provided that
at least one of the counters at the ends of the block is on the edge of the
board. Determine a shortest sequence of moves which allows us to flip all
counters to their green sides.

1958. Proposed by Marcin E. Kuczma, Warszawa, Poland.


Find the tetrahedron of maximum volume given that the sum of the lengths of some four edges
is 1.

1961. Proposed by Toshio Seimiya, Kawasaki, Japan.


ABC is an isosceles triangle with AB = AC. We denote the circumcircle of 4ABC by Γ . Let
D be the point such that DA and DC are tangent to Γ at A and C respectively. Prove that
ÁDBC ≤ 30 ◦.

1962. Proposed by Murray S. Klamkin, University of Alberta.


If A, B, C, D are non-negative angles with sum π, prove that
(i) cos2 A + cos2 B + cos2 C + cos2 D ≥ 2 sin A sin C + 2 sin B sin D;
(ii) 1 ≥ sin A sin C + sin B sin D.

80
1965F . Proposed by Ji Chen, Ningbo University, China.
Let P be a point in the interior of the triangle ABC, and let the lines AP , BP , CP intersect
the opposite sides at D, E, F respectively.
(a) Prove or disprove that

R3
PD · PE · PF ≤ ,
8
where R is the circumradius of 4ABC. Equality holds when ABC is equilateral and P is its
centre.
(b) Prove or disprove that
1
PE · PF + PF · PD + PD · PE ≤ max{a2 , b2 , c2 },
4
where a, b, c are the sides of the triangle. Equality holds when ABC is equilateral and P is its
centre, and also when P is the midpoint of the longest side of ABC.

1972. Proposed by Marcin E. Kuczma, Warszawa, Poland.


Define a sequence a0 , a1 , a2 , . . . of nonnegative integers by: a0 = 0 and

a2n = 3an , a2n+1 = 3an + 1 for n = 0, 1, 2, . . . .

(a) Characterize all nonnegative integers n so that there is exactly one pair (k, l) satisfying

k>l and ak + al = n. (1)

(b) For each n, let f (n) be the number of pairs (k, l) satisfying (1). Find

max f (n).
n<31972

1976. Proposed by Gottfried Perz, Pestalozzigymnasium, Graz, Austria.


If a, b and c are positive numbers, prove that

a(3a − b) b(3b − c) c(3c − a) a3 + b3 + c 3


+ + ≤ .
c(a + b) a(b + c) b(c + a) abc

1985. Proposed by Murray S. Klamkin and Andy Liu, University of Alberta.


Let A1 A2 . . . A2n be a regular 2n-gon, n > 1. Translate every even-numbered vertex A2 , A4 , . . .,
A2n by an equal (nonzero) amount to get new vertices A02 , A04 , . . . , A02n , and so that the new
2n-gon A1 A02 A3 A04 . . . A2n−1 A02n is still convex. Prove that the perimeter of A1 A02 . . . A2n−1 A02n
is greater than the perimeter of A1 A2 . . . A2n .

1990. Proposed by Leng Gangsong, Hunan Educational Institute, Changsha, China.


Let r be the inradius of a tetrahedron A1 A2 A3 A4 , and let r1 , r2 , r3 , r4 be the inradii of triangles
A2 A3 A4 , A1 A3 A4 , A1 A2 A4 , A1 A2 A3 respectively. Prove that
1 1 1 1 2
2 + 2 + 2 + 2 ≤ 2,
r1 r2 r3 r4 r

with equality if the tetrahedron is regular.

81
1994. Proposed by N. Kildonan, Winnipeg, Manitoba.
This problem marks the one and only time that the number of a Crux problem is equal to the
year in which it is published. In particular this is the first time that

a problem number in an integer multiple of its publication year. (1)

Assuming that Crux continues indefinitely to publish 10 problems per issue and 10 issues per
year, will there be a last time (1) happens? If so, when will this occur?

2000. Proposed by Marcin E. Kuczma, Warszawa, Poland.


A 1000-element set is randomly chosen from {1, 2, . . . , 2000}. Let p be the probability that the
sum of the chosen numbers is divisible by 5. Is p greater than, smaller than, or equal to 1/5?

2006. Proposed by John Duncan, University of Arkansas, Fayetteville; Dan Velleman, Am-
herst College, Amherst, Massachusetts; and Stan Wagon, Macalester College, St. Paul, Minne-
sota.
Suppose we are given n ≥ 3 disks, of radii a1 ≥ a2 ≥ · · · ≥ an . We wish to place them in
some order around an interior disk so that each given disk touches the interior disk and its two
immediate neighbors. If the given disks are of widely different sizes (such as 100, 100, 100, 100,
1), we allow a disk to overlap other given disks that are not immediate neighbors. In what order
should the given disks be arranged so as to maximize the radius of the interior disk?
[Editor’s note. Readers may assume that for any ordering of the given disks the configuration of
the problem exists and that the radius of the interior disk is unique, though, as the proposers
point out, this requires a proof (which they supply).]

2009. Proposed by Bill Sands, University of Calgary.


Sarah got a good grade at school, so I gave her N two-dollar bills. Then, since Tim got a better
grade, I gave him just enough five-dollar bills so that he got more money than Sarah. Finally,
since Ursula got the best grade, I gave her just enough ten-dollar bills so that she got more
money than Tim. What is the maximum amount of money that Ursula could have received?
(This is a variation of problem 11 on the 1994 Alberta High School Mathematics Contest, First
Part; see Skoliad Corner, this issue.)

2015. Proposed by Shi-Chang Shi and Ji Chen, Ningbo University, China.


Prove that
µ ¶ √
1 1 1 27 3
(sin A + sin B + sin C) + + ≥ ,
A B C 2π

where A, B, C are the angles (in radians) of a triangle.

2018. Proposed by Marcin E. Kuczma, Warszawa, Poland.


How many permutations (x1 , . . . , xn ) of {1, . . . , n} are there such that the cyclic sum
n
X
|xi − xi+1 |
i=1

(with xn+1 = x1 ) is (a) a minimum, (b) a maximum?

82
2020. Proposed by Christopher J. Bradley, Clifton College, Bristol, U. K.
Let a, b, c, d be distinct real numbers such that
a b c d
+ + + = 4 and ac = bd.
b c d a
Find the maximum value of
a b c d
+ + + .
c d a b
2022. Proposed by K. R. S. Sastry, Dodballapur, India.
Find the smallest integer of the form
A?B
,
B
where A and B are three-digit positive integers and A ? B denotes the six-digit integer formed
by placing A and B side by side.

2023. Proposed by Waldemar Pompe, student, University of Warsaw, Poland.


Let a, b, c, d, e be positive numbers with abcde = 1.
(a) Prove that

a + abc b + bcd c + cde


+ +
1 + ab + abcd 1 + bc + bcde 1 + cd + cdea
d + dea e + eab 10
+ + ≥ .
1 + de + deab 1 + ea + eabc 3
(b) Find a generalization!

2029F . Proposed by Jun-hua Huang, The Middle School Attached To Hunan Normal Univer-
sity, Changsha, China.
ABC is a triangle with area F and internal angle bisectors wa , wb , wc . Prove or disprove that

wb wc + wc wa + wa wb ≥ 3 3 F.

2032. Proposed by Tim Cross, Wolverley High School, Kidderminster, U. K.


Prove that, for nonnegative real numbers x, y and z,
p p p p
x2 + 1 + y 2 + 1 + z 2 + 1 ≥ 6 (x + y + z).

When does equality hold?

2039F . Proposed by Dong Zhou, Fudan University, Shang-hai, China, and Ji Chen, Ningbo
University, China.
Prove or disprove that

sin A sin B sin C 9 3
+ + ≥ ,
B C A 2π
where A, B, C are the angles (in radians) of a triangle. [Compare with Crux 1216 [1988: 120]
and this issue!]

83
2044. Proposed by Murray S. Klamkin, University of Alberta.
Suppose that n ≥ m ≥ 1 and x ≥ y ≥ 0 are such that

xn+1 + y n+1 ≤ xm − y m .

Prove that xn + y n ≤ 1.

2048. Proposed by Marcin E. Kuczma, Warszawa, Poland.


Find the least integer n so that, for every string of length n composed of the letters a, b, c, d,
e, f , g, h, i, j, k (repititions allowed), one can find a nonempty block of (consecutive) letters in
which no letter appears an odd number of times.

2049F . Proposed by Jan Ciach, Ostrowiec Świȩtokrzyski, Poland.


Let a tetrahedron ABCD with centroid G be inscribed in a sphere of radius R. The lines AG,
BG, CG, DG meet the sphere again at A1 , B1 , C1 , D1 respectively. The edges of the tetrahedron
are denoted a, b, c, d, e, f . Prove or disprove that
√ µ ¶
4 1 1 1 1 4 6 1 1 1 1 1 1
≤ + + + ≤ + + + + + .
R GA1 GB1 GC1 GD1 9 a b c d e f

Equality holds if ABCD is regular. (This inequality, if true, would be a three-dimensional version
of problem 5 of the 1991 Vietnamese Olympiad; see [1994: 41].)

2053. Proposed by Jisho Kotani, Akita, Japan.


A figure consisting of two equal and externally tangent circles is inscribed in an ellipse. Find the
eccentricity of the ellipse of minimum area.

2057F . Proposed by Jan Ciach, Ostrowiec Świȩtokrzyski, Poland.


Let P be a point inside an equilateral triangle ABC, and let Ra , Rb , Rc and ra , rb , rc denote
the distances of P from the vertices and edges, respectively, of the triangle. Prove or disprove
that
µ ¶µ ¶µ ¶
ra rb rc 27
1+ 1+ 1+ ≥ .
Ra Rb Rc 8

Equality holds if P is the centre of the triangle.

2064. Proposed by Murray S. Klamkin, University of Alberta.


Show that
½ ¾ µ ¶
a b c b c a 1 1 1
3 max + + , + + ≥ (a + b + c) + +
b c a a b c a b c

for arbitrary positive real numbers a, b, c.

2073F . Proposed by Jan Ciach, Ostrowiec Świȩtokrzyski, Poland.


Let P be an interior point of an equilateral triangle A1 A2 A3 with circumradius R, and let
R1 = P A1 , R2 = P A2 , R3 = P A3 . Prove or disprove that
9 3
R1 R2 R3 ≤ R .
8
Equality holds if P is the midpoint of a side. [Compare this problem with Crux 1895 [1995: 204].]

84
2078F . Proposed by Šefket Arslanagić, Berlin, Germany.
Prove or disprove that
√ √ √ p
a − 1 + b − 1 + c − 1 ≤ c (ab + 1)

for a, b, c ≥ 1.

2084. Proposed by Murray S. Klamkin, University of Alberta, Edmonton, Alberta.


Prove that
B C C A A B A B C
cos cos + cos cos + cos cos ≥ 1 − 2 cos cos cos ,
2 2 2 2 2 2 2 2 2
where A, B, C are the angles of a triangle.

2090. Proposed by Peter Ivády, Budapest, Hungary.


For 0 < x < π/2 prove that
µ ¶
sin x 2 π 2 − x2
< 2 .
x π + x2

2093F . Proposed by Walther Janous, Ursulinengymnasium, Innsbruck, Austria.


Let A, B, C be the angles (in radians) of a triangle. Prove or disprove that
µ ¶ √
1 1 1 27 3
(sin A + sin B + sin C) + + ≤ .
π−A π−B π−C 4π

2095. Proposed by Murray S. Klamkin, University of Alberta.


Prove that

ax (y − z) + ay (z − x) + az (x − y) ≥ 0

where a > 0 and x > y > z.

2099. Proposed by Proof, Warszawa, Poland.


The tetrahedron T is contained inside the tetrahedron W . Must the sum of the lengths of the
edges of T be less than the sum of the lengths of the edges of W ?

2100. Proposed by Iliya Bluskov, student, Simon Fraser University, Burnaby, B. C.


Find 364 five-element subsets A1 , A2 , . . . , A364 of a 17-element set such that |Ai ∩ Aj | ≤ 3 for all
1 ≤ i < j ≤ 364.

2101. Proposed by Ji Chen, Ningbo University, China.


Let a, b, c be the sides and A, B, C the angles of a triangle. Prove that for any k ≤ 1,
X ak 3X k
≥ a ,
A π
where the sums are cyclic.

2105. Proposed by Christopher J. Bradley, Clifton College, Bristol, U. K.


Find all values of λ for which the inequality

2(x3 + y 3 + z 3 ) + 3(1 + 3λ)xyz ≥ (1 + λ)(x + y + z)(yz + zx + xy)

holds for all positive real numbers x, y, z.

85
2106. Proposed by Yang Kechang, Yueyang University, Hunan, China.
A quadrilateral has sides a, b, c, d (in that order) and area F . Prove that

2a2 + 5b2 + 8c2 − d2 ≥ 4F.

When does equality hold?

2108. Proposed by Vedula N. Murty, Andhra University, Visakhapatnam, India.


Prove that
r
a+b+c 1 3 (b + c)2 (c + a)2 (a + b)2
≤ ,
3 4 abc
where a, b, c > 0. Equality holds if a = b = c.

2113. Proposed by Marcin E. Kuczma, Warszawa, Poland.


Prove the inequality
à n !à n ! à n !à n !
X X X X ai bi
ai bi ≥ (ai + bi )
ai + bi
i=1 i=1 i=1 i=1

for any positive numbers a1 , . . . , an , b1 , . . . , bn .

2116. Proposed by Yang Kechang, Yueyang University, Hunan, China.


A triangle has sides a, b, c and area F . Prove that

3 4 5 25 5(2F )6
a b c ≥ .
27
When does equality hold?

2117. Proposed by Toshio Seimiya, Kawasaki, Japan.


ABC is a triangle with AB > AC, and the bisector of ÁA meets BC at D. Let P be an interior
point of the side AC. Prove that ÁBP D < ÁDP C.

2128. Proposed by Toshio Seimiya, Kawasaki, Japan.


ABCD is a square. Let P and Q be interior points on the sides BC and CD respectively, and
let E and F be the intersections of P Q with AB and AD respectively. Prove that

π ≤ ÁP AQ + ÁECF < .
4
2136. Proposed by G. P. Henderson, Campbellcroft, Ontario. P P
Let a, b, c be the lengths of the sides of a triangle. Given the values of p = a and q = ab,
prove that r = abc can be estimated with an error of at most r/26.

2138. Proposed by Christopher J. Bradley, Clifton College, Bristol, U. K.


ABC is an acute angle triangle with circumcentre O. AO meets the circle BOC again at A 0 ,
BO meets the circle COA again at B 0 , and CO meets the circle AOB again at C 0 . Prove that
[A0 B 0 C 0 ] ≥ 4 [ABC], where [XY Z] denotes the area of triangle XY Z.

86
2139. Proposed by Waldemar Pompe, student, University of Warsaw, Poland.
Point P lies inside triangle ABC. Let D, E, F be the orthogonal projections from P onto the
lines BC, CA, AB, respectively. Let O 0 and R0 denote the circumcentre and circumradius of the
triangle DEF , respectively. Prove that
√ p
[ABC] ≥ 3 3R0 R02 − (O0 P )2 ,

where [XYZ] denotes the area of triangle XY Z.

2145. Proposed by Robert Geretschläger, Bundesrealgymnasium, Graz, Austria.


n
Y Yn
k−1
Prove that (ak + b ) ≤ (ak + bn−k ) for all a, b > 1.
k=1 k=1

2146. Proposed by Toshio Seimiya, Kawasaki, Japan.


ABC is a triangle with AB > AC, and the bisector of ÁA meets BC at D. Let P be an interior
point on the segment AD, and let Q and R be the points of intersection of BP and CP with
sides AC and AB respectively. Prove that P B − P C > RB − QC > 0.

2153. Proposed by Šefket Arslanagić, Berlin, Germany.


Suppose that a, b, c ∈ R. If, for all x ∈ [−1, 1], |ax2 + bx + c| ≤ 1, prove that

|cx2 + bx + a| ≤ 2.

2163. Proposed by Theodore Chronis, student, Aristotle University of Thessaloniki, Greece.


Prove that if n, m ∈ N and n ≥ m2 ≥ 16, then 2n ≥ nm .

2167. Proposed by Šefket Arslanagić, Berlin, Germany.


Prove, without the aid the differential calculus, the inequality, that in a right triangle

a2 (b + c) + b2 (a + c) √
≥ 2 + 2,
abc
where a and b are the legs and c the hypotenuse of the triangle.

2172. Proposed by Walther Janous, Ursulinengymnasium, Innsbruck, Austria.


Let x, y, z ≥ 0 with x + y + z = 1. For fixed real numbers a and b, determine the maximum
c = c(a, b) such that

a + bxyz ≥ c(yz + zx + xy).

87
2173. Proposed by Walther Janous, Ursulinengymnasium, Innsbruck, Austria.
Let n ≥ 2 and x1 , . . . , xn > 0 with x1 + . . . + xn = 1. Consider the terms
n r
X 1 − xk
ln = (1 + xk )
xk
k=1

and
n
Y 1 + xk
rn = C n √
1 − xk
k=1

where
√ √
Cn = ( n − 1)n+1 ( n)n /(n + 1)n−1 .

1. Show l2 ≤ r2 .
2. Prove or disprove: ln ≥ rn for n ≥ 3.

2176. Proposed by Šefket Arslanagić, Berlin, Germany.


Prove that
v v v
u n u n u n
uY uY uY
n
t (ak + bk ) ≥ n
t ak + t
n
bk
k=1 k=1 k=1

where a1 , a2 , . . . , an > 0 and n ∈ N.

2178. Proposed by Christopher J. Bradley, Clifton College, Bristol, U. K.


If A, B, C are the angles of a triangle, prove that

sin A sin B sin C ≤ 8(sin3 A cos B cos C + sin3 B cos C cos A + sin3 C cos A cos B)

≤ 3 3 (cos2 A + cos2 B + cos2 C).

2180. Proposed by Juan-Bosco Romero Márquez, Universidad de Valladolid, Valladolid, Spain.


Prove that if a > 0, x > y > z > 0, n ≥ 0 (natural), then

1. ax (yz)n (y − z) + ay (xz)n (z − x) + az (xy)n (x − y) ≥ 0,


2. ax cosh x(y − z) + ay cosh y(z − x) + az cosh z(x − y) ≥ 0.

2183. Proposed by Václav Konec̆ný, Ferris State University, Big Rapids, Michigan, USA.
Suppose that A, B, C are the angles of a triangle and that k, l, m ≥ 1. Show that

0 < sink A · sinl B · sinm C


S
h k
ih l
ih m
i
≤ k k ll mm S 2 (Sk 2 + P )− 2 (Sl2 + P )− 2 (Sm2 + P )− 2 ,

where S = k + l + m and P = klm.

2188. Proposed by Victor Oxman, University of Haifa, Haifa, Israel.


Suppose that a, b, c are the sides of a triangle with semi-perimeter s and area ∆. Prove that
1 1 1 s
+ + < .
a b c ∆

88
2190. Proposed by Walther Janous, Ursulinengymnasium, Innsbruck, Austria.
Determine the range of

sin2 A sin2 B sin2 C


+ +
A B C
where A, B, C are the angles of a triangle.

2191. Proposed by Šefket Arslanagić, Berlin, Germany.


Find all positive integers n, that satisfy the inequality
1 π 1
< sin < √ .
3 n 2 2

2192. Proposed by Theodore Chronis, student, Aristotle University of Thessaloniki, Greece.


Let {an } be a sequence defined as follows:
µ ¶
a2
an+1 + an−1 = an , n ≥ 1.
a1
¯ ¯ ¯ ¯
¯ a2 ¯ ¯ an ¯
Show that if ¯ ¯ ≥ 2, then ¯¯ ¯¯ ≥ n.
¯ ¯
a1 a1

2198. Proposed by Vedula N. Murty, Andhra University, Visakhapatnam, India.


Prove that, if a, b, c are the lengths of the sides of a triangle
µ ¶ µ ¶ µ ¶
2 2 1 2 2 1 2 2 1
(b − c) − + (c − a) − + (a − b) − ≥ 0,
bc a2 ca b2 ab c2

with equality if and only if a = b = c.

2199. Proposed by David Doster, Choate Rosemary Hall, Wallingford, Connecticut, USA.
Find the maximum value of c for which (x + y + z)2 > cxz for all 0 ≤ x < y < z.

220AF . Proposed by Ji Chen, Ningbo University, China.


Let P be a point in the interior of √
the triangle ABC, and let α1 = ÁP AB, β1 = ÁP BC,
γ1 = ÁP CA. Prove or disprove that 3 α1 β1 γ1 ≤ π/6.

2202. Proposed by Walther Janous, Ursulinengymnasium, Innsbruck, Austria.


Suppose that n ≥ 3. Let A1 , . . . , An be a convex n-gon (as usual with interior angles A1 , . . . , An ).
Determine the greatest constant Cn such that
n n
X 1 X 1
≥ Cn .
Ak π − Ak
k=1 k=1

Determine when equality occurs.

2204. Proposed by Šefket Arslanagić, Berlin,


√ Germany.
For triangle ABC such that R(a + b) = c ab, prove that
3
r< a.
10
Here, a, b, c, R, and r are the three sides, the circumradius and the inradius of 4ABC.

89
2206. Proposed by Heinz-Jürgen Seiffert, Berlin, Germany.
Let a and b denote distinct positive real numbers.
(a) Show that if 0 < p < 1, p 6= 12 , then

1 p 1−p √ a+b
(a b + a1−p bp ) < 4p(1 − p) ab + [1 − 4p(1 − p)] .
2 2
(b) Use (a) to deduce Pólya’s inequality:
µ ¶
a−b 1 √ a+b
< 2 ab + .
ln a − ln b 3 2

2213. Proposed by Victor Oxman, University of Haifa, Haifa, Israel.


Suppose that the function f (u) has a second derivative in the interval (a, b), and that f (u) ≥ 0
for all u ∈ (a, b). Prove that

1. (y − z)f (x) + (z − x)f (y) + (x − y)f (z) > 0 for all x, y, z ∈ (a, b), z < y < x
if and only if f 00 (u) > 0 for all u ∈ (a, b);
2. (y − z)f (x) + (z − x)f (y) + (x − y)f (z) = 0 for all x, y, z ∈ (a, b), z < y < x
if and only if f (u) is a linear function on (a, b).

2214. Proposed by Walther Janous, Ursulinengymnasium, Innsbruck, Austria.


Let n ≥ 2 be a natural number. Show that there exists a constant C = C(n) such that for all
real x1 , . . . , xn ≥ 0 we have
v
n u n
X √ uY
xk ≤ t (xk + C).
k=1 k=1

Determine the minimum C(n) for some values of n.


(For example, C(2) = 1.)

2232. Proposed by Šefket Arslanagić, University of Sarajevo, Sarajevo, Bosnia and Herzego-
vina.
Find all solutions of the inequality:
jnk ¹n + 1º ¹n + 2º
2
n +n−5< + + < n2 + 2n − 2, (n ∈ N).
3 3 3

2233. Proposed by Walther Janous, Ursulinengymnasium, Innsbruck, Austria.


Let x, y, z be non-negative real numbers such that x + y + z = 1, and let p be a positive real
number.
(a) If 0 < p ≤ 1, prove that

xp + y p + z p ≥ Cp [(xy)p + (yz)p + (zx)p ],

where
( ln 2
3p if p ≤ ln 3−ln 2 ,
Cp = ln 2
2p+1 if p ≥ ln 3−ln 2 .

(b)F Prove the same inequality for p > 1.

90
2236. Proposed by Victor Oxman, University of Haifa, Haifa, Israel.
Let ABC be an arbitrary triangle and let P be an arbitrary point in the interior of the circum-
circle of 4ABC. Let K, L, M , denote the feet of the perpendiculars from P to the lines AB,
[ABC]
BC, CA, respectively. Prove that [KLM ] ≤ .
4
Note: [XY Z] denotes the area of 4XY Z.

2240. Proposed by Victor Oxman, University of Haifa, Haifa, Israel.


Let ABC be an arbitrary triangle with the points D, E, F on the sides BC, CA, AB respectively,
BD BF AE AF [ABC]
so that ≤ ≤ 1 and ≤ . Prove that [DEF ] ≤ with equality if and only
DC FA EC FB 4
if two of the three points D, E, F , (at least) are mid-points of the corresponding sides.
Note: [XY Z] denotes the area of 4XY Z.

2256. Proposed by Russell Euler and Jawad Sadek, Department of Mathematics and Statistics,
Northwest Missouri State University, Maryville,µMissouri,
¶ USA.
ln(x) − ln(y) 1
If 0 < y < x ≤ 1, prove that > ln .
x−y y

2260. Proposed by Vedula N. Murty, Andhra University, Visakhapatnam, India.


Let n be a positive integer and x > 0. Prove that

(n + 1)n+1
(1 + x)n+1 ≥ x.
nn
2262. Proposed by Juan-Bosco Romero Márquez, Universidad de Valladolid, Valladolid, Spain.
Consider two triangles 4ABC and 4A0 B 0 C 0 such that ÁA ≥ 90 ◦ and ÁA0 ≥ 90 ◦ and whose
sides satisfy a > b ≥ c and a0 > b0 ≥ c0 . Denote the altitudes to sides a and a0 by ha and h0a .
1 1 1 1 1 1
Prove that (a) ≥ 0 + 0 , (b) ≥ 0+ 0 .
ha h0a bb cc ha h0a bc bc

2275. Proposed by M. Perisastry, Vizianagaram, Andhra Pradesh, India.


Let b > 0 and ba ≥ ba for all a > 0. Prove that b = e.

2282. Proposed by D. J. Smeenk, Zaltbommel, The Netherlands.


A line, l, intersects the sides BC, CA, AB, of 4ABC at D, E, F respectively such that D is
the mid-point of EF . Determine the minimum value of |EF | and express its length as elements
of 4ABC.

2290. Proposed by Panos E. Tsaoussoglou, Athens, Greece.


For x, y, z ≥ 0, prove that

[(x + y)(y + z)(z + x)]2 ≥ xyz(2x + y + z)(2y + z + x)(2z + x + y).

2296. Proposed by Vedula N. Murty, Andhra University, Visakhapatnam, India.


Show that
πx 2x2
sin2 > for 0 < x < 1.
2 1 + x2
Hence or otherwise, deduce that
sin πx
π< <4 for 0 < x < 1.
x(1 − x)

91
2299. Proposed by Walther Janous, Ursulinengymnasium, Innsbruck, Austria.
Let x, y, z > 0 be real numbers such that x + y + z = 1. Show that
Y · (1 − y)(1 − x) ¸(1−y)(1−z)/x 256
≥ .
x 81
cyclic

Determine the cases of equality.

2300. Proposed by Christopher J. Bradley, Clifton College, Bristol, U. K.


Suppose that ABC is a triangle with circumradius R. The circle passing through A and touching
BC at its mid-point has radius R1 . Define R2 and R3 similarly.
Prove that
27 2
R12 + R22 + R32 ≥ R .
16
2301. Proposed by Christopher J. Bradley, Clifton College, Bristol, U. K.
Suppose that ABC is a triangle with sides a, b, c, that P is a point in the interior of 4ABC, and
that AP meets the circle BP C again at A0 . Define B 0 and C 0 similarly. Prove that the perimeter
P of the hexagon AB 0 CA0 BC 0 satisfies
³√ √ √ ´
P≥2 ab + bc + ca .

2306. Proposed by Vedula N. Murty, Andhra University, Visakhapatnam, India.


(a) Give an elementary proof of the inequality
³ πx ´2 2x2
sin > ; (0 < x < 1).
2 1 + x2
(b) Hence (or otherwise) show that
(
< πx(1−x) 1
1−2x ; (0 < x < 2 ,
tan πx
> πx(1−x) 1
1−2x ; ( 2 < x < 1).

sin πx
(c) Find the maximum value of f (x) = x(1−x) on the interval (0, 1).

2326F . Proposed by Walther Janous, Ursulinengymnasium, Innsbruck, Austria.


Prove or disprove that if A, B and C are the angles of a triangle, then

2 X (1 − sin A )(1 + 2 sin A ) 9


2 2
< ≤ .
π π−A π
cyclic

2340. Proposed by Walther Janous, Ursulinengymnasium, Innsbruck, Austria.


Let λ > 0 be a real number and a, b, c be the sides of a triangle. Prove that
Y s + λa
≥ (2λ + 3)3 .
s−a
cyclic

[As usual s denotes the semiperimeter.]

92
2345. Proposed by Vedula N. Murty, Andhra University, Visakhapatnam, India.
Suppose that x > 1.
(a) Show that

3(x2 − 1)
ln x > .
x2 + 4x + 1
(b) Show that
µ ¶
a−b 1 √ a+b
< 2 ab + ,
ln a − ln b 3 2
where a > 0, b > 0 and a 6= b.

2349. Proposed by Václav Konec̆ný, Ferris State University, Big Rapids, Michigan, USA.
Suppose that 4ABC has acute angles such that A < B < C. Prove that
µ ¶ µ ¶
A B B A
sin2 B sin sin A + > sin2 A sin sin B + .
2 2 2 2

2362. Proposed by Mohammed Aassila, Université Louis Pasteur, Strasbourg, France.


Suppose that a, b, c > 0. Prove that
1 1 1 3
+ + ≥ .
a(1 + b) b(1 + c) c(1 + a) 1 + abc

2365. Proposed by Victor Oxman, University of Haifa, Haifa, Israel.


Triangle DAC is equilateral. B is on the line DC so that ÁBAC = 70 ◦. E is on the line AB
so that ÁECA = 55 ◦. K is the mid-point of ED. Without the use of a computer, calculator or
protractor, show that 60 ◦ > ÁAKC > 57.5 ◦.

2374. Proposed by Toshio Seimiya, Kawasaki, Japan.


Given triangle ABC with ÁBAC > 60 ◦. Let M be the mid-point of BC. Let P be any point in
the plane of 4ABC. Prove that AP + BP + CP ≥ 2 AM .

2382. Proposed by Mohammed Aassila, Université Louis Pasteur, Strasbourg, France.


If 4ABC has inradius r and circumradius R, show that
µ ¶
2 B−C 2r
cos ≥ .
2 R

2384. Proposed by Paul Bracken, CRM, Université de Montréal, Québec.


Prove that

2(3n − 1)n ≥ (3n + 1)n for all n ∈ N.

2389. Proposed by Nikolaos Dergiades, Thessaloniki, Greece.


Suppose that f is continuous on Rn and satisfies the condition that when any two of its variables
are replaced by their arithmetic mean, the value of the function increases; for example:
µ ¶
a1 + a 3 a1 + a 3
f (a1 , a2 , a3 , . . . , an ) ≤ f , a2 , , a4 , . . . , an .
2 2
a1 + a 2 + . . . + a n
Let m = . Prove that
n
f (a1 , a2 , a3 , . . . , an ) ≤ f (m, m, m, . . . , m).

93
2392. Proposed by George Tsintsifas, Thessaloniki, Greece.
Suppose that xi , yi , (1 ≤ i ≤ n) are positive real numbers. Let
µ n
¶µ n

P P
n xi yi
X x i yi i=1 i=1
An = , Bn = n ,
xi + y i P
i=1 (xi + yi )
i=1
µ n
¶2 µ n
¶2
P P
xi + yi n
i=1 i=1
X x2 + y 2
i i
Cn = n , Dn = .
P xi + y i
(xi + yi ) i=1
i=1

Prove that

1. An ≤ Cn ,
2. Bn ≤ Dn ,
3. 2An ≤ 2Bn ≤ Cn ≤ Dn .

2393. Proposed by George Tsintsifas, Thessaloniki, Greece.


Suppose that a, b, c and d are positive real numbers. Prove that

1. [(a + b)(b + c)(c + d)(d + a)]3/2 ≥ 4abcd(a + b + c + d)2 ,


Y
2. [(a + b)(b + c)(c + d)(d + a)]3 ≥ 16(abcd)2 (2a + b + c).
a,b,c,d
cyclic

2394. Proposed by Vedula N. Murty, Visakhapatnam, India.


µ ¶
a + b a+b
The inequality aa bb ≥ , where a, b > 0, is usally proved using Calculus. Give a proof
2
without the aid of Calculus.

2400. Proposed by Václav Konec̆ný, Ferris State University, Big Rapids, Michigan, USA.
cos(πx)
(a) Show that 1 + (π − 2)x < < 1 + 2x for 0 < x < 1/2.
1 − 2x
[Proposed by Bruce Shawyer, Editor-in-Chief.]
µ ¶
F cos(πx) π 1 2
(b) Show that < − 2(π − 2) x − for 0 < x < 1/2.
1 − 2x 2 2

2401. Proposed by D. J. Smeenk, Zaltbommel, The Netherlands.


In triangle ABC, CD is the altitude from C to AB. E and F are the mid-points of AB and CD
respectively. P and Q are points on line segments BC and AC respectively, and are such that
P Q k BA. The projection of Q onto AB is R. P R and EF intersect at S. Prove that
(a) S is the mid-point of line segment P R,
1 1 1
(b) 2
≤ 2
+ .
PR AB CD2

2414. Proposed by Wu Wei Chao, Guang Zhou Normal University, Guang Zhou City, Dong
Province, China, and Edward T. H. Wang, Wilfrid Laurier University, Waterloo, Ontario.
y x
For 1 < x ≤ e ≤ y or e ≤ x < y, prove that xx y x > xy y x .

94
2422F . Proposed by Walther Janous, Ursulinengymnasium, Innsbruck, Austria.
Let A, B, C be the angles of an arbitrary triangle. Prove or disprove that

1 1 1 9 3
+ + ≥ .
A B C 2π(sin A sin B sin C)1/3

2423. Proposed by Walther Janous, Ursulinengymnasium, Innsbruck, Austria.


Let x1 , x2 , . . . , xn > 0 be real numbers such that x1 + x2 + . . . + xn = 1, where n > 2 is a natural
number. Prove that
n µ ¶ Y n µ ¶
Y 1 n − xk
1+ ≥ .
xk 1 − xk
k=1 k=1

Determine the cases of equality.

2439. Proposed by Toshio Seimiya, Kawasaki, Japan.


Suppose that ABCD is a square with side a. Let P and Q be points on sides BC and CD
respectively, such that ÁP AQ = 45 ◦. Let
√ E and F be the intersections of P Q with AB and AD
respectively. Prove that AE + AF ≥ 2 2 a.

2443. Proposed by Michael Lambrou, University of Crete, Crete, Greece.


Without the use of any calculating device, find an explicit example of an integer, M , such that
sin(M ) > sin(33)(≈ 0.99991). (Of course, M and 33 are in radians.)

2468. Proposed by Walther Janous, Ursulinengymnasium, Innsbruck, Austria.


For c > 0, let x, y, z > 0 satisfy

xy + yz + zx + xyz = c. (1)

Determine the set of all c > 0 such that whenever (1) holds, then we have

x + y + z ≥ xy + yz + zx.

2472. Proposed by Václav Konec̆ný, Ferris State University, Big Rapids, Michigan, USA.
If A, B, C are the angles of a triangle, prove that
µ ¶ µ ¶ µ ¶ · ¸
A−B B−C C −A A B C 3
cos2 cos2 cos2 ≥ 8 sin sin sin .
2 2 2 2 2 2

2477. Proposed by Walther Janous, Ursulinengymnasium, Innsbruck, Austria.


Given a non-degenerate 4ABC with circumcircle Γ , let rA be the inradius C
of the region bounded by BA, AC and arc(CB) (so that the region includes
the triangle). Similarly, define rB and rC . Aus usual, r and R are the
inradius and circumradius of 4ABC. A
Prove that
B
64 3 32 2
(a) r ≤ r A rB rC ≤ Rr ;
27 27
16 2 8
(b) r ≤ rB rC + rC rA + rA rB ≤ Rr;
3 3
4
(c) 4r ≤ rA + rB + rC ≤ (R + r),
3
with equality occuring in all cases if and only if 4ABC is equilateral.

95
2481. Proposed by Mihály Bencze, Brasov, Romania.
Suppose that A, B, C are 2 × 2 commutative matrices. Prove that

det((A + B + C)(A3 + B 3 + C 3 − 3ABC)) ≥ 0.

2482. Proposed by Mihály Bencze, Brasov, Romania.


Suppose that p, q, r are complex numbers. Prove that

|p + q| + |q + r| + |r + p| ≤ |p| + |q| + |r| + |p + q + r|.

2483. Proposed by Mihály Bencze, Brasov, Romania.


Suppose that 0 ≤ A, B, C and A + B + C ≤ π. Show that

0 ≤ A − sin A − sin B − sin C + sin(A + B) + sin(A + C) ≤ π.

There are, of course, similar inequalities with the angles permuted cyclically.

2497. Proposed by Nikolaos Dergiades, Thessaloniki, Greece.


Given 4ABC and a point D on AC, let ÁABD = δ and ÁDBC = γ. Find all values of ÁBAC
AD
for which γδ > DC .

2502. Proposed by Toshio Seimiya, Kawasaki, Japan.


In 4ABC, the internal bisectors of ÁBAC, ÁABC and ÁBCA meet BC, AC and AB at D,
E and F respectively. Let p and q be the perimeters of 4ABC and 4DEF respectively. Prove
that p ≥ 2q, and that equality holds if and only if 4ABC is equilateral.

2504. Proposed by Hayo Ahlburg, Benidorm, Spain, and Walther Janous, Ursulinengymnasi-
um, Innsbruck, Austria.
Suppose that
Y A, B and C are the angles of a triangle. Determine the best lower and upper
bounds of cos(B − C).
cyclic

2505. Proposed by Hayo Ahlburg, Benidorm, Spain, and Walther Janous, Ursulinengymnasi-
um, Innsbruck, Austria.
Suppose that
Y A, B and C are the angles of a triangle. Determine the best lower and upper
bounds of sin(B − C).
cyclic

2507. Proposed by Ice B. Risteski, Skopje, Macedonia.


Show that there are infinitely many pairs of distinct natural numbers, n and k such that
gcd(n! + 1, k! + 1) > 1.

2509. Proposed by Ice B. Risteski, Skopje, Macedonia.


Show that there are infinitely many pairs of distinct natural numbers, n and k such that
gcd(n! − 1, k! − 1) > 1.

2512. Proposed by Juan-Bosco Romero Márquez, Universidad de Valladolid, Valladolid, Spain.


In 4ABC, the sides satisfy a ≥ b ≥ c. Let R and r be the circumradius and the inradius
respectively. Prove that

bc ≤ 6Rr ≤ a2 ,

with equality if and only if a = b = c.

96
2516. Proposed by Toshio Seimiya, Kawasaki, Japan.
In isosceles 4ABC (with AB = AC), let D and E be points on sides AB and AC respectively
such that AD < AE. Suppose that BE and CD meet at P . Prove that AE + EP < AD + DP .

2522F . Proposed by Walther Janous, Ursulinengymnasium, Innsbruck, Austria.


Suppose that a, b and c are positive real numbers. Prove that
µ ¶µ ¶
1 1 1 1 1 1 9
+ + + + ≥ .
a b c 1+a 1+b 1+c 1 + abc

2523. Proposed by Walther Janous, Ursulinengymnasium, Innsbruck, Austria.


Prove that, if t ≥ 1, then
à r !
t−1 2t2 + 5t + 2
ln t ≤ 1+ .
2(t + 1) t

Also, prove that, if 0 < t ≤ 1, then


à r !
t−1 2t2 + 5t + 2
ln t ≥ 1+ .
2(t + 1) t

2527. Proposed by K. R. S. Sastry, Dodballapur, India.


Let AD, BE and CF be concurrent cevians of 4ABC. Assume that:
(a) AD is a median; (b) BE bisects ÁABC; (c) BE bisects AD.
Prove that CF > BE.

2529. Proposed by George Tsintsifas, Thessaloniki, Greece.


z }| {
Let G = {A1 , A2 , . . . , An } be a set of points on a unit hemisphere. Let Ai Aj be the spherical
z }| {
distance between the points Ai and Aj . Suppose that Ai Aj ≥ d. Find max d.

2531. Proposed by George Tsintsifas, Thessaloniki, Greece.


Let F be a convex plane set and AB its diameter. The points A and B divide the perimeter of
F into two parts, L1 and L2 , say. Prove that
1 L1
< < π − 1.
π−1 L2

2532. Proposed by Ho-joo Lee, student, Kwangwoon University, Kangwon-Do, South Korea.
Suppose that a, b and c are positive real numbers satisfying a2 + b2 + c2 = 1. Prove that

1 1 1 2(a3 + b3 + c3 )
+ + ≥ 3 + .
a2 b2 c2 abc
2536. Proposed by Cristinel Mortici, Ovidius University of Constanta, Romania.
Let f : R → R be a continuous and periodic function such that for all positive integers n the
following inequality holds:
|f (1)| |f (2)| |f (n)|
+ + ··· + ≤ 1.
1 2 n
Prove that there exists c ∈ R such that f (c) = 0 and f (c + 1) = 0.

97
2539. Proposed by Ho-joo Lee, student, Kwangwoon University, Kangwon-Do, South Korea.
Let ABCD be a convex quadrilateral with vertices oriented in the clockwise sense. Let X and
Y be interior points on AD and BC, respectively. Suppose that P is a point between X and Y
such that ÁAXP = ÁBY P = ÁAP B = θ and ÁCP D = π − θ for some θ.
(a) Prove that AD · BC ≥ 4P X · P Y .
(b)F Find the case(s) of equality.

2542. Proposed by Hassan Ali Shah Ali, Tehran, Iran.


Suppose that k is a natural number and αi ≥ 0, i = 1, . . . , n, and αn+1 = α1 . Prove that
 k−1
X k X
j−1
αik−j αi+1 ≥  αi  .
1≤i≤n nk−2
1≤i≤n
1≤j≤k

Determine the necessary and sufficient conditions for equality.

2551. Proposed by Panos E. Tsaoussoglou, Athens, Greece.


Suppose that ak (1 ≤ k ≤ n) are positive real numbers. Let ej,k = (n − 1) if j = k and
ej,k = (n − 2) otherwise. Let dj,k = 0 if j = k and dj,k = 1 otherwise. Prove that

n
n X n
à n !2
Y Y X
ej,k a2k ≥ dj,k ak .
j=1 k=1 j=1 k=1

2552. Proposed by Aram Tangboondouangjit, Carnegie Mellon University, Pittsburgh, PA,


USA.
Suppose that a, b, c > 0. If x ≥ a+b+c

3 3
− 1, prove that

(b + cx)2 (c + ax)2 (a + bx)2


+ + ≥ abc.
a b c
2554. Proposed by Aram Tangboondouangjit, Carnegie Mellon University, Pittsburgh, PA,
USA.
In triangle ABC, prove that at least one of the quantities
µ ¶ µ ¶
2 A B
(a + b − c) tan tan ,
2 2
µ ¶ µ ¶
B C
(−a + b + c) tan2 tan ,
2 2
µ ¶ µ ¶
2 C A
(a − b + c) tan tan ,
2 2
2r
is greater than or equal to 3 , where r is the radius of the incircle of 4ABC.

2555. Proposed by Aram Tangboondouangjit, Carnegie Mellon University, Pittsburgh, PA,


USA.
In any triangle ABC, show that

X 1 4 3
¡ ¢3 < .
tan3 A + tan B + tan C 3
cyclic 2 2 2

98
2557. Proposed by Gord Sinnamon, University of Western Ontario, London, Ontario, and
Hans Heinig, McMaster University, Hamilton, Ontario.
(a) Show that for all positive sequences {xi } and all integers n > 0,
 2
Xn Xk X j X n Xk
xi ≤ 2  xj  x−1
k .
k=1 j=1 i=1 k=1 j=1

(b)F Does the above inequality remain true without the factor 2?
(c)F What is the minimum constant c that can replace the factor 2 in the above inequality?

2571. Proposed by Ho-joo Lee, student, Kwangwoon University, Seoul, South Korea.
Suppose that a, b and c are the sides of a triangle. Prove that
√ √ √
1 1 1 3( a + b + c)
√ √ √ +√ √ √ +√ √ √ ≥ .
a+ b− c b+ c− a c+ a− b a+b+c

2572. Proposed by José Luis Dı́az, Universitat Politècnica Catalunya, Terrassa, Spain.
Let a, b, c be positive real numbers. Prove that
µ ¶
a + b + c a+b+c
ab bc ca ≤ .
3

2575. Proposed by H. Fukagawa, Kani, Gifu, Japan.


Suppose that 4ABC has a right angle at C. The circle, centre A and radius AC meets the
hypotenuse AB at D. In the region bounded by the arc DC and the line segments BC and BD,
draw a square EF GH of side y, where E lies on arc DC, F lies on DB and G and H lie on BC.
Assume that BC is constant and that AC = x is variable. Find max y and the corresponding
value of x.

2580. Proposed by Ho-joo Lee, student, Kwangwoon University, Seoul, South Korea.
Suppose that a, b and c are positive real numbers. Prove that
b+c c+a a+b 1 1 1
2
+ 2 + 2 ≤ + + .
a + bc b + ac c + ab a b c
2581. Proposed by Ho-joo Lee, student, Kwangwoon University, Seoul, South Korea.
Suppose that a, b and c are positive real numbers. Prove that

ab + c2 bc + a2 ca + b2
+ + ≥ a + b + c.
a+b b+c c+a
2585. Proposed by Vedula N. Murty, Visakhapatnam, India.
Prove that, for 0 < θ < π/2,

tan θ + sin θ > 2θ.

99
2588. Proposed by Niels Bejlegaard, Stavanger, Norway.
Each positive whole integer ak (1 ≤ k ≤ n) is less than a given positive integer N . The least
common multiple of any two of the numbers ak is geater than N .
n
X 1
(a) Show that < 2.
ak
k=1
n
X 1 6
(b)F Show that < .
ak 5
k=1
n
F
X 1
(c) Find the smallest real number γ such that < γ.
ak
k=1

2590. Proposed by Joaquı́n Gómez Rey, IES Luis Buñuel, Alcorcón, Spain.
n ³ ´ µ µ ¶¶n
Y n 2 1 2n
For n = 1, 2, . . ., prove that ≤ .
k n+1 n
k=1

2594. Proposed by Nairi M. Sedrakyan, Yerevan, Armenia.


Given a point M inside the triangle ABC, prove that

min(M A, M B, M C) + M A + M B + M C < AB + BC + CA.

2596. Proposed by Clark Kimberling, University of Evansville, Evansville, IN, USA.


Write r ¿ s if there is an integer k satisfying r < k < s. Find, as a function of n (n ≥ 2) the
least positive integer k satisfying
k k k k
¿ ¿ ¿ · · · ¿ ¿ k.
n n−1 n−2 2
2597. Proposed by Michael Lambrou, University of Crete, Crete, Greece.
Let P be an arbitrary interior point of an equilateral triangle ABC. Prove that
· µ ¶¸
|ÁP AB − ÁP AC|
|ÁP BC − ÁP CB| ≤ arcsin 2 sin −
2
µ ¶
|ÁP AB − ÁP AC|
≤ |ÁP AB − ÁP AC|.
2

Show that the left inequality cannot be improved in the sense that there is a position Q of P on
the ray AP giving an equality.

2603. Proposed by Ho-joo Lee, student, Kwangwoon University, Kangwon-Do, South Korea.
Suppose that A, B and C are the angles of a triangle. Prove that
r
15
sin A + sin B + sin C ≤ + cos(A − B) + cos(B − C) + cos(C − A).
4

100
2604. Proposed by Walther Janous, Ursulinengymnasium, Innsbruck, Austria.
(a) Determine the upper and lower bounds of
a b a
+ −
a+b b+c a+c
for all positive real numbers a, b and c.
(b)F Determine the upper and lower bounds (as functions of n) of
n−1
X xj x1

xj + xj+1 x1 + xn
j=1

for all positive real numbers x1 , x2 , . . . , xn .

2608F . Proposed by Faruk Zejnulahi and Šefket Arslanagić, University of Sarajevo, Sarajevo,
Bosnia and Herzegovina.
Suppose that x, y, z ≥ 0 and x2 + y 2 + z 2 = 1. Prove or disprove that

x y z 3 3
(a) 1 ≤ + + ≤ ;
1 − yz 1 − zx 1 − xy 2
x y z √
(b) 1 ≤ + + ≤ 2.
1 + yz 1 + zx 1 + xy

2615. Proposed by Murray S. Klamkin, University of Alberta, Edmonton, Alberta.


Suppose that x1 , x2 , . . . , xn are non-negative numbers such that
X X n(n + 1)
x21 + (x1 x2 )2 = ,
2
where the sums here and subsequently
P are symmetric over the subscripts 1, 2, . . . , n.
(a) Determine the maximum of x1 . q
P
F
(b) Prove or disprove that the minimum of x1 is n(n+1)
2 .

2623F . Proposed by Hassan Ali Shah Ali, Tehran, Iran.


Suppose that x1 , x2 , . . . , xn > 0. Let xn+1 = x1 , xn+2 = x2 , etc.
For k = 0, 1, . . . , n − 1, let
n
à P !
k
X x j+i
Sk = Pk i=0 .
j=1 i=0 xj+1+i

Prove or disprove that Sk ≥ Sk+1 .

2625. Proposed by Ho-joo Lee, student, Kwangwoon University, Kangwon-Do, South Korea.
If R denotes the circumradius of triangle ABC, prove that

18R3 ≥ (a2 + b2 + c2 ) R + 3 abc.

2627. Proposed by Walther Janous, Ursulinengymnasium, Innsbruck, Austria.


Let x1 , . . . , xn be positive real numbers and let sn = x1 + · · · + xn (n ≥ 2). Let a1 , . . . , an be
non-negative real numbers. Determine the optimum constant C(n) such that
 1
n n n
X aj (sn − xj ) Y
≥ C(n)  aj  .
xj
j=1 j=1

101
2628. Proposed by Victor Oxman, University of Haifa, Haifa, Israel.
Four points, X, Y , Z and W are taken inside or on triangle ABC. Prove that there exists a set
of three of these points such that the area of the triangle formed by them is less than 38 of the
area of the given triangle.

2629. Proposed by Christopher J. Bradley, Clifton College, Bristol, U. K.


In triangle ABC, the symmedian point is denoted by S. Prove that

1 BC 2 AS 2 + CA2 BS 2 + AB 2 CS 2
(AS 2 + BS 2 + CS 2 ) ≥ .
3 BC 2 + CA2 + AB 2
2633. Proposed by Mihály Bencze, Brasov, Romania.
Prove that
n
n(n + 1) X 1 31 n(n + 1)
< (k!) k < + .
2e 20 4
k=1

2635. Proposed by Toshio Seimiya, Kawasaki, Japan.


Consider triangle ABC, and three squares BCDE, CAF G and ABHI constructed on its sides,
outside the triangle. √
Let XY Z be the triangle enclosed by the lines EF , DI and GH. Prove
that [XY Z] ≤ (4 − 2 3) [ABC], where [P QR] denotes the area of 4P QR.

2637. Proposed by Toshio Seimiya, Kawasaki, Japan.


Suppose that ABC is an isosceles triangle with AB = AC. Let D be a point on side AB, and let
E be a point on AC produced beyond C. The line DE meets BC at P . The incircle of 4ADE
touches DE at Q.
Prove that BP · P C ≤ DQ · QE, and that equality holds if and only if BD = CE.

2641. Proposed by George Tsintsifas, Thessaloniki, Greece.


Let H be a centrosymmetric convex hexagon, with area h, and let P be its minimal circumscribed
parallelogram, with area p. Prove that

3 p ≤ 4 h.

2645. Proposed by Ho-joo Lee, student, Kwangwoon University, Kangwon-Do, South Korea.
Suppose that a, b and c are positive real numbers. Prove that

2(a3 + b3 + c3 ) 9(a + b + c)2


+ 2 ≥ 33.
abc a + b2 + c2
2650. Proposed by Juan-Bosco Romero Márquez, Universidad de Valladolid, Valladolid, Spain.
In 4ABC, let a denote the side BC, and ha , the corresponding altitude. Let r and R be the
radii of the inscribed and circumscribed circles, respectively. Prove that ra < h a R.

2651F . Proposed by Murray S. Klamkin, University of Alberta, Edmonton, Alberta.


Let P be a non-exterior point of a regular n-dimensional simplex A0 A1 A2 . . . An of edge length
e. If
n
X n
X
F = P Ak + min P Ak , F0 = P Ak + max P Ak ,
0≤k≤n 0≤k≤n
k=0 k=0

determine the maximum and minimum values of F and F 0 . (Professor Klamkin offers a prize of
$100 for the first correct solution received by the Editor-in-Chief.)

102
2652F . Proposed by Walther Janous, Ursulinengymnasium, Innsbruck, Austria.
Let d, e and f be the sides of the triangle determined by the three points at which the internal
angle-bisectors of given 4ABC meet the opposite sides. Prove that

s2
d2 + e 2 + f 2 ≤ ,
3
where s is the semiperimeter of 4ABC.

2656F . Proposed by Vedula N. Murty, Dover, PA, USA.


For positive real numbers a, b and c, show that

(1 − b)(1 − bc) (1 − c)(1 − ca) (1 − a)(1 − ab)


+ + ≥ 0.
b(1 + a) c(1 + b) a(1 + c)

2662. Proposed by Christopher J. Bradley, Clifton College, Bristol, U. K.


Suppose that 4ABC is acute-angled, has inradius r and has area ∆. Prove that
³√ √ √ ´2 ∆
cot A + cot B + cot C ≤ 2 .
r

2664. Proposed by Aram Tangboondouangjit, Carnegie Mellon University, Pittsburgh, PA,


USA.
Let a, b and c be positive real numbers such that a + b + c = abc. Prove that

a5 (bc − 1) + b5 (ca − 1) + c5 (ab − 1) ≥ 54 3.

2665. Proposed by Aram Tangboondouangjit, Carnegie Mellon University, Pittsburgh, PA,


USA.
In 4ABC, we have ÁACB = 90¡◦ and ¢ sides AB = c, BC = ¡ a¢and CA = b. In 4DEF , we have
ÁEF D = 90 ◦, EF = (a + c) sin B2 and F D = (b + c) sin A2 . Show that DE ≥ c.

2667. Proposed by Walther Janous, Ursulinengymnasium, Innsbruck, Austria.


You are given a circle Γ and two points A and B outside of Γ such that the line through A and
B does not intersect Γ . Let X be any point on Γ . Determine at which point X on Γ the sum
AX + XB attains its minimum value.

2668F . Proposed by Vedula N. Murty, Dover, PA, USA.


Suppose that 0 < r < q < 1 and that 0 < m < ∞. Show that
p p
(1 − q)(q + r − qr) 1 + m2 + q(1 − r) (q − 2)2 + m2 q 2
p p
> (1 − r)(q + r − qr) 1 + m2 + r(1 − q) (r − 2)2 + m2 r2 .

2669F . Proposed by Murray S. Klamkin, University of Alberta, Edmonton, Alberta.


Let A1 , A2 , . . . , A2n , be any 2n points in E m . Determine the largest kn such that
¡ ¢
A1 A22 + A2 A23 + · · · + A2n A21 ≥ kn A1 A2n+1 + A2 A2n+2 + · · · + An A22n .

For n = 2, it is easily shown that k2 = 1. That k3 = 12 is an Armenian Olympiad problem.


(Professor Klamkin offers a prize of $50 for the first correct solution received by the Editor-in-
Chief.)

103
2672. Proposed by Vedula N. Murty, Dover, PA, USA.
n
X (n + 1)α+1 − 1
(a) Suppose that α > 0. Prove that kα < .
α+1
k=1
n
(n + 1)α+1 − 1 X α
(b) Suppose that −1 < α < 0. Prove that < k .
α+1
k=1
[These two inequalities appear differently in “Analytic Inequalities“ by Nicolas D. Kazarinoff, Holt Rine-
hart and Winston, p. 24. The term “−1“ is missing from the numerators.]

2673. Proposed by George Baloglou, SUNY Oswego, Oswego, NY, USA.


Let n ≥ 2 be an integer.
(a) Show that
¡ ¢¡ ¢ ¡ ¢
(1 + a1 · · · an )n ≥ (a1 · · · an ) 1 + an−2
1 1 + an−2
2 · · · 1 + an−2
n

for all a1 ≥ 1, a2 ≥ 1, . . . , an ≥ 1, if and only if n ≤ 4.


(b) Show that
1 1 1 n
n−2 + n−2 + · · · + n−2 ≥ 1 + a · · · a
¡ ¢ ¡ ¢ ¡ ¢
a1 1 + a2 a2 1 + a 3 an 1 + a 1 1 n

for all a1 > 0, a2 > 0, . . . , an > 0, if and only if n ≤ 3.


(c) Show that
1 1 1 n
n−2 + n−2 + · · · + n−2 ≥ 1 + a · · · a
¡ ¢ ¡ ¢ ¡ ¢
a1 1 + a1 a2 1 + a 2 an 1 + a n 1 n

for all a1 > 0, a2 > 0, . . . , an > 0, if and only if n ≤ 8.


(d)F Show that
µ ¶µ ¶
1 1 1 1 1 1 n2
+ + ··· + + + · · · + ≥
a1 a2 an 1 + an−2
1 1 + an−2
2 1 + an−2
n 1 + a1 · · · a n
for all a1 > 0, a2 > 0, . . . , an > 0, if and only if n ≤ 5.

2676. Proposed by Vedula N. Murty, Dover, PA, USA.


Let A, B and C be the angles of a triangle. Show that

(sin A + sin B + sin C)2 ≤ 6 (1 + cos A cos B cos C).

When does equality occur?

2677. Proposed by Péter Ivády, Budapest,


µ Hungary.

π 2 − x2 x
π
For 0 < x < 2 , show that 2 < cos √ .
π + x2 3

2685. Proposed by Mohammed Aassila, Strasbourg, France.


(a) Let C be a bounded, closed and convex domain in the plane. Construct a parallelogram P
contained in C such that A(P) ≥ 12 A(C), where A denotes area.
(b)F Prove that if, further, C is centrally symmetric, then one can construct a parallelogram P
such that A(P) ≥ π2 A(C).

2686F . Proposed by Mohammed Aassila, Strasbourg, France.


Let C be a bounded, closed and convex domain in space. Construct a parallelepiped P contained
in C such that V(P) ≥ 49 V(C), where V denotes volume.

104
2690. Proposed by Juan-Bosco Romero Márquez, Universidad de Valladolid, Valladolid, Spain.
Let 4ABC be such that ÁA is the largest angle. Let r be the inradius and R the circumradius.
Prove that
b+c
A ≷ 90 ◦ ⇐⇒ R + r ≷ .
2
2693. Proposed by Paul Yiu, Florida Atlantic University, Boca Raton, FL, USA.
Given triangle ABC and a point P , the line through P parallel to BC, intersects AC, AB at
Y1 , Z1 respectively. Similarly, the parallel to CA intersects BC, AB at X2 , Z2 , and the parallel
to AB intersects BC, AC at X3 , Y3 . Locate the point P for which the sum

Y1 P · P Z 1 + Z 2 P · P X 2 + X 3 P · P Y 3

of products of signed lengths is maximal.

2700. Proposed by José Luis Dı́az-Barrero, Universitat Politècnica de Catalunya, Terrassa,


Spain.
Let n be a positive integer. Show that
n µ ¶ X n µ ¶ µ ¶
X k n n n+k
< log < 2n−1 .
n+k k k n
k=1 k=1

[Ed. “log“ is, of course, the natural logarithm.]

2702. Proposed by Walther Janous, Ursulinengymnasium, Innsbruck, Austria.


Let λ be an arbitrary real number. Show that
³ s ´2λ
s2 ≥ 33λ+1 (s2 − 8Rr − 2r 2 ),
r
where R, r and s are the circumradius, the inradius and the semi-perimeter of a triangle,
respectively. Determine the cases of equality.

2704. Proposed by Mihály Bencze, Brasov, Romania.


Prove that
 
1 X p 2 s 2 + r 2 + 4Rr
R − 2r ≥ 2(b + c2 ) − a2 −  ≥ 0,
12 R
cyclic

where a, b and c are the sides of a triangle, and R, r and s are the circumradius, the inradius
and the semi-perimeter of a triangle, respectively.

2707. Proposed by Walther Janous, Ursulinengymnasium, Innsbruck, Austria.


Let ABC be a triangle and P a point in its plane. The feet of the perpendiculars from P to the
lines BC, CA and AB are D, E and F respectively. Prove that

AB 2 + BC 2 + CA2
≤ AF 2 + BD2 + CE 2 ,
4
and determine the cases of equality.

105
2709. Proposed by Toshio Seimiya, Kawasaki, Japan.
Suppose that

1. P is an interior point of 4ABC,


2. AP , BP and CP meet BC, CA and AB at D, E and F , respectively,
3. A0 is a point on AD produced beyond D such that DA0 : AD = κ : 1, where κ is a fixed
positive number,
4. B 0 is a point on BE produced beyond E such that EB 0 : BE = κ : 1, and
5. C 0 is a point on CF produced beyond F such that F C 0 : CF = κ : 1.
(3κ+1)2
Prove that [A0 B 0 C 0 ] ≤ 4 [ABC], where [P QR] denotes the area of 4P QR.

2710. Proposed by Jaroslav S̆vrc̆ek, Palacký University, Olomouc, Czech Republic.


Determine the point P on the semicircle Γ , constructed externally over the side AB of the square
ABCD, such that AP 2 + CP 2 is maximal.

2717. Proposed by Mihály Bencze, Brasov, Romania.


For any triangle ABC, prove that
µ ¶ µ ¶ µ ¶
A B C A−B B−C C −A
8 sin sin sin ≤ cos cos cos .
2 2 2 2 2 2

2718. Proposed by Mihály Bencze, Brasov, Romania.


Let Ak ∈ Mm (R) with Ai Aj = Om , i, j ∈ {1, 2, . . . , n}, with i < j and xk ∈ R∗ , (k = 1, 2, . . . , n).
Prove that
à n
!
X
2 2
det Im + (xk Ak + xk Ak ) ≥ 0.
k=1

2723. Proposed by Walther Janous, Ursulinengymnasium, Innsbruck, Austria.


For 1 ≤ k ≤ N , let n1 , n2 , . . . , nk be non-negative integers such that n1 + n2 + · · · + nk = N .
Determine the minimum value of the sum
k µ ¶
X nj
when (a) m = 2; (b)F m ≥ 3.
m
j=1

2724F . Proposed by Walther Janous, Ursulinengymnasium, Innsbruck, Austria.


Let a, b, c be the sides of a triangle and ha , hb , hc , respectively, the corresponding altitudes.
Prove that the maximum range of validity of the inequality
µ t ¶1/t √ µ t ¶1/t
ha + htb + htc 3 a + bt + ct
≤ ,
3 2 3
− ln 4 ln 4
where t 6= 0 is <t< .
ln 4 − ln 3 ln 4 − ln 3

2729. Proposed by Václav Konec̆ný, Ferris State University, Big Rapids, Michigan, USA.
Let Z(n) denote the number of trailing zeros of n!, where n ∈ N.
Z(n) 1
(a) Prove that < .
n 4
Z(n) 1
(b)F Prove or disprove that lim = .
n→∞ n 4

106
2730. Proposed by Peter Y. Woo, Biola University, La Mirada, CA, USA.
Let AM(x1 , x2 , . . . , xn ) and GM(x1 , x2 , . . . , xn ) denote the arithmetic mean and the geometric
mean of the real numbers x1 , x2 , . . . , xn , respectively. Given positive real numbers a1 , a2 , . . . , an ,
b1 , b2 , . . . , bn , prove that

(a) GM(a1 + b1 , a2 + b2 , . . . , an + bn ) ≥ GM(a1 , a2 , . . . , an ) + GM(b1 , b2 , . . . , bn ).

For each real number t ≥ 0, define f (t) = GM(t + b1 , t + b2 , . . . , t + bn ) − t.


(b) Prove that f (t) is a monotonic increasing function of t, and that

lim f (t) = AM(b1 , b2 , . . . , bn ).


t→∞

2732. Proposed by Mihály Bencze, Brasov, Romania.


Let ABC be a triangle with sides a, b, c, medians ma , mb , mc , altitudes ha , hb , hc , and area ∆.
Prove that
½ ¾
√ ma mb mc
a2 + b2 + c2 ≥ 4 3 ∆ max , , .
ha hb hc

2734. Proposed by Murray S. Klamkin, University of Alberta, Edmonton, Alberta.


Prove that

(bc)2n+3 + (ca)2n+3 + (ab)2n+3 ≥ (abc)n+2 (an + bn + cn ),

where a, b, c are non-negative reals, and n is a non-negative integer.

2738. Proposed by Šefket Arslanagić, University of Sarajevo, Sarajevo, Bosnia and Herzego-
vina.
Let x, y and z be positive real numbers satisfying x2 + y 2 + z 2 = 1. Prove that

x y z 3 3
+ + ≥ .
1 − x2 1 − y 2 1 − z 2 2

2739. Proposed by Šefket Arslanagić, University of Sarajevo, Sarajevo, Bosnia and Herzego-
vina.
Suppose that a, b and c are positive real numbers. Prove that
√ √ √ √ √ √ √
a+b+c+ a a+b+c+ b a+b+c+ c 9+3 3
+ + ≥ √ .
b+c c+a a+b 2 a+b+c
2743. Proposed by ¡Péter¢ Ivády, Budapest, Hungary.
Show that, for x, y ∈ 0, π2 ,
µ ¶ ³x´ ³y ´
x y
+ cos cos < 2.
sin x sin y 2 2

2747. Proposed by K. R. S. Sastry, Bangalore, India.


Prove that the orthocentre of a triangle lies inside or on the incircle if and only if the inradius
is a mean proportional to the two segments of an altitude, sectioned by the orthocentre.

2748. Proposed by Walther Janous, Ursulinengymnasium, Innsbruck, Austria.


Let a1 , a2 , . . . , an (n ≥ 1) be non-negative real numbers such that a1 ≤ a2 ≤ · · · ≤ an and
Xn Xn
ak = 1. Determine the least upper bound of an (n + 1 − k)ak .
k=1 k=1

107
2749. Proposed by Christopher J. Bradley, Clifton College, Bristol, U. K.
Suppose that P is an interior point of 4ABC. The line through P parallel to AB meets BC at
L and CA at M 0 . The line through P parallel to BC meets CA at M and AB at N 0 . The line
through P parallel to CA meets AB at N and BC at L0 . Prove that
µ ¶µ ¶µ ¶µ ¶µ ¶µ ¶
BL CM AN BL0 CM 0 AN 0
(a) = 1;
LC MA NB L0 C M 0A N 0B
µ ¶µ ¶µ ¶
BL CM AN 1
(b) ≤ ;
LC MA NB 8
(c) [LM N ] = [L0 M 0 N 0 ]; [Note: [XY Z] denotes the area of 4XY Z.]
[ABC]
(d) [LM N ] ≤ .
3
Locate the point P when equality holds in part (b) and (d).

2757F . Proposed by Walther Janous, Ursulinengymnasium, Innsbruck, Austria.


Let A, B and C be the angles of a triangle. Show that

X 1 9 3
¡ ¢ ¡ ¢3 ≤ .
tan A + 8 tan π−A 11
cyclic 2 4

2760. Proposed by Michel Bataille, Rouen, France.


Suppose that A, B, C are the angles of a triangle. Prove that

8(cos A + cos B + cos C) ≤ 9 + cos(A − B) + cos(B − C) + cos(C − A)


≤ csc2 (A/2) + csc2 (B/2) + csc2 (C/2).

2768. Proposed by Mohammed Aassila, Strasbourg, France.


Let x1 , x2 , . . . , xn be n positive real numbers. Prove that
x1 x2 xn n
p
2
+p 2
+ ··· + p 2
≥√ .
x 1 x2 + x 2 x 2 x3 + x 3 x n x1 + x 1 2

2769. Proposed by Aram Tangboondouangjit, student, University of Maryland, College Park,


Maryland, USA.
In 4ABC, suppose that cos B − cos C = cos A − cos B ≥ 0. Prove that

(b2 + c2 ) cos A − (a2 + b2 ) cos C ≤ (c2 − a2 ) sec B.

2770. Proposed by Aram Tangboondouangjit, student, University of Maryland, College Park,


Maryland, USA.
In 4ABC, suppose that a ≤ b ≤ c and ∠ABC 6= π2 . Prove that
µ ¶µ ¶
b b
2 + sec B ≤ 1 + 1+ .
a c

108
2774. Proposed by Wu Wei Chao, Guang Zhou University (New), Guang Zhou City, Guang
Dong Province, China.
Let x be a real number such that 0 < x ≤ 29 π. Prove that

(sin x)sin x < cos x.

(This is a generalization of Problem 10261 in the American Mathematical Monthly [1992 : 872,
1994 : 690]).

2775. Proposed by Li Zhou, Polk Community College, Winter Haven, FL, USA.
In 4ABC, let M be the mid-point of BC. Prove that
µ ¶ µ ¶ µ ¶ µ ¶
B−C A B C
cos ≥ sin(∠AM B) ≥ 8 sin sin sin .
2 2 2 2

2778. Proposed by Mihály Bencze, Brasov, Romania.


Suppose that z 6= 1 is a complex number such that z n = 1 (n ≥ 1). Prove that

(n + 1)(2n + 1)
|nz − (n + z)| ≤ |z − 1|2 .
6

2786F . Proposed by Šefket Arslanagić, University of Sarajevo, Sarajevo, Bosnia and Herze-
govina.
Prove or disprove the inequality
1 1 1 27
3≤ + + ≤ ,
1 − xy 1 − yz 1 − zx 8
where x + y + z = 1 and x, y, z ≥ 0.

2787F . Proposed by Šefket Arslanagić, University of Sarajevo, Sarajevo, Bosnia and Herze-
govina.
Prove or disprove the inequality
27 1 1 1 11
≤ ¡ x+y ¢2 + ¡ y+z ¢2 + ¡ z+x ¢2 ≤ ,
8 1− 2 1− 2 1− 2 3

where x + y + z = 1 and x, y, z ≥ 0.

2791. Proposed by Mihály Bencze, Brasov, Romania.


Suppose that f : [0, 1] → (0, ∞) is a continuous function. Prove that if there exists α > 0 such
that, for n ∈ N,
Z 1 Z 1
α n 1
x (f (x)) dx ≥ ≥ (f (x))n+1 dx,
0 (n + 1)α + 1 0

then α is unique.

109
2792. Proposed by Mihály Bencze, Brasov, Romania.
Let Ak ∈ Mn (R) (k = 1, 2, . . . , m ≥ 2) for which
X
(Ai Aj + Aj Ai ) = 0n .
1≤i<j≤m

Prove that
à m
!
X
det (In + Ak )2 − (m − 2)In ≥ 0.
k=1

2794. Proposed by Mihály Bencze, Brasov, Romania.


Suppose that zk ∈ C∗ (k = 1, 2, . . . , n) such that

|z1 + z2 + · · · + zn | + |z2 + z3 + · · · + zn | + · · · + |zn−1 + zn | + |zn |


= |z1 + 2z2 + · · · + nzn |.

Prove that the zk are collinear.

2795. Proposed by Mihály Bencze, Brasov, Romania.


A convex polygon with sides a1 , a2 , . . . , an , is inscribed in a circle of radius R. Prove that
n q µ ¶
X
2 2 (n − 2)π
4R − ak ≤ 2nR sin .
n
k=1

2796F . Proposed by Fernando Castro G., Matirín Estado Monagas, Vénézuéla.


Let {pn } be the sequence of prime numbers. Prove that, for each n ≥ 2, the set I = {1, 2, . . . , n}
can be partitioned into two sets A and B, where A ∪ B = I, in such a way that
Q
pi
1 ≤ Q i∈A ≤ 2.
j∈B pj

2798F . Proposed by Šefket Arslanagić, University of Sarajevo, Sarajevo, Bosnia and Herze-
govina.
Prove or disprove the inequality
n
X 1 n
P
≤ ¡ 1 ¢n−1 ,
j=1
1 − xj 1− n
n
P n
Q
where xj = 1, xj ≥ 0 (j = 1, 2, . . . , n), and P = xj .
j=1 j=1

2799F . Proposed by Šefket Arslanagić, University of Sarajevo, Sarajevo, Bosnia and Herze-
govina.
Prove or disprove the inequality
µ ¶
X 1 n 1
≤ ,
1 − x i xj 2 1 − n12
i,j∈{1,2,...,n}
1≤i<j≤n

n
P
where xj = 1, xj ≥ 0.
j=1

110
2801. Proposed by Heinz-Jürgen Seiffert, Berlin, Germany.
Suppose that 4ABC is not obtuse. Denote (as usual) the sides by a, b, and c and the circum-
radius by R. Prove that

µ ¶1 µ ¶1 µ ¶1 µ ¶ 3
2A a 2B b 2C c 2 R
≤ .
π π π 3

When does equality hold?

2803. Proposed by I. C. Draghicescu, Bucharest, Romania.


Suppose that x1 , x2 , . . . , xn (n > 2) are real numbers such that the sum of any n − 1 of them is
n
P
greater than the remaining number. Let s = xk . Prove that
k=1

n
X x2k s
≥ .
s − 2xk n−2
k=1

2806. Proposed by Mihály Bencze, Brasov, Romania.


Suppose that x, y, z > 0, α ∈ R and xα + y α + z α = 1. Prove that

a) x2 + y 2 + z 2 ≥ xα+2 + y α+2 + z α+2 + 2xyz(xα−1 + y α−1 + z α−1 ),


1 1 1 α−2 α−2 α−2 2 (xα+1 + y α+1 + z α+1 )
b) + + ≥ x + y + z + .
x2 y 2 z 2 xyz

2807. Proposed by Aram Tangboondouangjit, student, University of Maryland, College Park,


Maryland, USA.
In 4ABC, denote its area by [ABC] (and its semi-perimeter by s). Show that
½ ¾
2s4 − (a4 + b4 + c4 )
min = 38.
[ABC]2

2810. Proposed by I. C. Draghicescu, Bucharest, Romania.


n
P
Suppose that a, b and x1 , x2 , . . . , xn (n ≥ 2) are positive real numbers. Let s = xk . Prove
k=1
that
n µ ¶ µ ¶
Y b nb n
a+ ≥ a+ .
xk s
k=1

2811. Proposed by Mihály Bencze, Brasov, Romania.


Determine all functions f : R → R which satisfy, for all real x,

f (x3 + x) ≤ x ≤ f 3 (x) + f (x).

2812. Proposed by Mihály Bencze, Brasov, Romania.


Determine all injective functions f : R → R which satisfy
µ ¶ µ ¶
2 1 1
(2a + b)f (ax + b) ≥ af + bf +a
x x

for all positive real x, where a, b ∈ R, a > 0, a2 + 4b > 0 and 2a + b > 0.

111
2814. Proposed by Juan José Egozcue and José Luis Dı́az-Barrero, Universitat Politècnica de
Catalunya, Terrassa, Spain.
Let a, b, and c be positive real numbers such that a + b + c = abc. Find the minimum value of
r r r
1 1 1
1 + 2 + 1 + 2 + 1 + 2.
a b c
2819. Proposed by Mihály Bencze, Brasov, Romania.
³ ´ ³p ´
Let f : R → R satisfy, for all real x and y, f 2x+y
3 ≥ f 3 x2 y . Prove that f is decreasing
on (−∞, 0] and increasing on [0, ∞).

2821. Proposed by Walther Janous, Ursulinengymnasium, Innsbruck, Austria.


In triangle 4ABC, let wa , wb , wc be the lengths of the interior angle bisectors, and r the
inradius. Prove that
1 1 1 1
2
+ 2 + 2 ≤ 2,
wa wb wc 3r

with equality if and only if 4ABC is equilateral.

2829. Proposed by George Tsintsifas, Thessaloniki, Greece.


Given 4ABC with sides a, b, c, prove that

3 (a4 + b4 + c4 ) ab + bc + ca
+ 2 ≥ 2.
(a2 + b2 + c2 )2 a + b2 + c2

2831. Proposed by Achilleas Pavlos Porfyriadis, Student, American College of Thessaloniki


Anatolia“, Thessaloniki, Greece.

For a convex polygon, prove that it is impossible for two sides without a common vertex to be
longer than the longest diagonal.

2833F . Proposed by Walther Janous, Ursulinengymnasium, Innsbruck, Austria.


Let a be a positive real number, and let n ≥ 2 be an integer. For each k = 1, 2, . . . , n, let x k be
x
a non-negative real number, λk be a positive real number, and let yk = λk xk + λk+1 k+1
. Here and
elsewhere, indices greater than n are to be reduced modulo n.

(a) If a > 1, prove that


n
X n
X n
X n
X
n+ a yk ≥ 2 a xk and 3n + ayk +yk+1 ≥ (1 + axk )2 .
k=1 k=1 k=1 k=1

(b) If 0 < a < 1, prove that the opposite inequalities hold.

[The proposer has proofs for the cases n = 3 and n = 4.]

2835. Proposed by George Tsintsifas, Thessaloniki, Greece.


For non-negative real numbers x and y, not both equal to 0, prove that

x4 + y 4 xy 5
4
+ ≥ .
(x + y) x+y 8

112
2839. Proposed by Murray S. Klamkin, University of Alberta, Edmonton, Alberta.
Suppose that x, y, and z are real numbers. Prove that

(x3 + y 3 + z 3 )2 + 3 (xyz)2 ≥ 4 (y 3 z 3 + z 3 x3 + x3 y 3 ).

Determine the cases of equality.

2841. Proposed by Mihály Bencze, Brasov, Romania.


Prove the following inequalities:
µ ¶
π 1 3 11
1− + −
2 4n 32n2 128n3
µ ¶2
(2n)!! 1

(2n − 1)!! 2n + 1
µ ¶
π 1 3 11 83
≤ 1− + − + .
2 4n 32n2 128n3 2048n4

2842. Proposed by George Tsintsifas, Thessaloniki, Greece.


Let x1 , x2 , . . . , xn be positive real numbers. Prove that

n
µ n
¶1
P Q n
xnk n xk
k=1 k=1
(a) n + n ≥ 2,
Q P
n xk xk
k=1 k=1

n
µ n
¶1
P Q n
xnk xk
k=1 k=1
(b) n + n ≥ 1.
Q P
xk xk
k=1 k=1

2843. Proposed
³ by Bektemirov
´ Baurjan, student, Aktobe, Kazakstan.
1 1 1 x y z
Suppose that 2 x + y + z = 4 + yz + zx + xy for positive real x, y, z. Prove that

1
(1 − x)(1 − y)(1 − z) ≤ .
64
2846. Proposed by George Tsintsifas, Thessaloniki, Greece.
A regular simplex Sn = A1 A2 A3 . . . An+1 is inscribed in the unit sphere Σ in En . Let O be the
−−→ −−→
origin in En , M ∈ Σ, uk = OAk and v = OM .
n+1
P
Find the maximum value of |uk · v|.
k=1

2852. Proposed by Toshio Seimiya, Kawasaki, Japan.


In 4ABC, we have AB < AC. The internal bisector of ÁBAC meets BC at D. Let P be an
interior point of the line segment AD, and let E and F be the intersections of BP and CP with
AC and AB, respectively. Prove that
PE AC
< .
PF AB

113
2859F . Proposed by Mohammed Aassila, Université Louis Pasteur, Strasbourg, France.
Prove that
X ab X a
≥ ,
c(c + a) c+a
cyclic cyclic

where a, b, c represent the three sides of a triangle.

2860. Proposed by Juan-Bosco Romero Márquez, Universidad de Valladolid, Valladolid, Spain.


In 4ABC and 4A0 B 0 C 0 , the lengths of the sides satisfy a ≥ b ≥ c and a0 ≥ b0 ≥ c0 . Let ha and
ha0 denote the lengths of the altitudes to the opposite sides from A and A0 , respectively. Prove
that
(a) bb0 + cc0 ≥ aha0 + a0 ha ;
(b) bc0 + b0 c ≥ aha0 + a0 ha .

2863. Proposed by Mihály Bencze, Brasov, Romania.


Suppose that a, b, c are complex numbers such that |a| = |b| = |c|. Prove that
¯ ¯ ¯ ¯ ¯ ¯
¯ ab ¯ ¯ bc ¯ ¯ ca ¯ √
¯ a2 − b2 ¯ + ¯ b2 − c2 ¯ + ¯ c2 − a2 ¯ ≥ 3.
¯ ¯ ¯ ¯ ¯ ¯

2864. Proposed by Panos E. Tsaoussoglou, Athens, Greece.


If a, b, c are the sides of an acute angled triangle, prove that
Xp p
a2 + b2 − c2 a2 − b2 + c2 ≤ ab + bc + ca.
cyclic

2865. Proposed by George Baloglou, SUNY Oswego, Oswego, NY.


Suppose that D, E, F are the points at which the concurrent lines AD, BE, CF meet the sides
of a given triangle ABC. Let p1 and p2 be the perimeters and δ1 and δ2 the areas of 4ABC
and 4DEF , respectively. Prove that

(a) 2p2 ≤ p1 if AD, BE, and CF are angle bisectors;


(b) 2p2 ≤ p1 if AD, BE, and CF are altitudes;
(c) 3p2 ≤ 2p1 for all D, E, F if and only if 4ABC is equilateral;
(d) 4p2 ≤ p1 for all D, E, F and arbitrary 4ABC.

2869. Proposed by Toshio Seimiya, Kawasaki, Japan.


Given rectangle ABCD with area S, let E and F be points on sides AB and AD, respectively,
such that [CEF ] = 31 S, where [P QR] denotes the area of 4P QR. Prove that ÁECF ≤ π6 .

2871. Proposed by Mihály Bencze, Brasov, Romania.


In 4ABC, denote the sides by a, b, c, the symmedians by sa , sb , sc , and the circumradius by
R. Prove that
bc ca ab
+ + ≤ 6R.
sa sb sc

114
2874. Proposed by Vedula N. Murty, Dover, PA, USA.
Let a, b and c denote the side lengths BC, CA, and AB, respectively, of triangle ABC, and let
s, r, and R denote the semi-perimeter, inradius, and circumradius of the triangle, respectively.
Let y = s/R and x = r/R. Show that
X
1. sin2 A = 2 ⇐⇒ y − x = 2 ⇐⇒ 4ABC is right-angled;
cyclic
X
2. sin2 A > 2 ⇐⇒ y − x > 2 ⇐⇒ 4ABC is acute-angled;
cyclic
X
3. sin2 A < 2 ⇐⇒ y − x < 2 ⇐⇒ 4ABC is obtuse-angled.
cyclic

2875. Proposed by Michel Bataille, Rouen, France.


Suppose that the incircle of 4ABC is tangent to the sides BC, CA, AB, at D, E, F , respectively.
Prove that
s2
EF 2 + F D 2 + DE 2 ≤ ,
3
where s is the semiperimeter of 4ABC.

2880. Proposed by Mihály Bencze, Brasov, Romania.


1. If x, y, z > 1, prove that

(a) (logyz x4 yz)(logzx xy 4 z)(logxy xyz 4 ) > 25,


(b)F (logyz x4 yz)(logzx xy 4 z)(logxy xyz 4 ) > 27.

2.F If xk > 1 (k = 1, 2, . . . , n) and α ≥ −1, prove that


n µ ¶
Y n+α n
logbk bk xα+1
k ≥ ,
n−1
k=1

where bk = x1 · · · xk−1 xk+1 · · · xn .

2882. Proposed by Mihály Bencze, Brasov, Romania.


If x ∈ (0, π2 ), 0 ≤ a ≤ b, and 0 ≤ c ≤ 1, prove that
µ ¶b µ ¶a
c + cos x sin x
< .
c+1 x

2883. Proposed by Šefket Arslanagić and Faruk Zejnulahi, University of Sarajevo, Sarajevo,
Bosnia and Herzogovina.
Suppose that x, y, z ∈ [0, 1) and that x + y + z = 1. Prove that
r r r
xy yz zx 3
+ + ≤ .
z + xy x + yz y + zx 2

2884. Proposed by Niels Bejlegaard, Copenhagen, Denmark.


Suppose that a, b, c are the sides of a non-obtuse triangle. Give a geometric proof and hence, a
geometric interpretation of the inequality
Xp
a+b+c≥ a 2 + b2 − c2 .
cyclic

115
2886. Proposed by Panos E. Tsaoussoglou, Athens, Greece.
If a, b, c are positive real numbers such that abc = 1, prove that

ab2 + bc2 + ca2 ≥ ab + bc + ca.

2887. Proposed by Vedula N. Murty, Dover, PA, USA.


If a, b, c are the sides of 4ABC in which at most one angle exceeds π3 , and if R is its circumradius,
prove that
X
a2 + b2 + c2 ≤ 6R2 cos A.
cyclic

2888F . Proposed by Vedula N. Murty, Dover, PA, USA.


π
Let a, b, c be the sides of 4ABC, in which at most one angle exceeds 3. Give an algebraic proof
of
Y X
8a2 b2 c2 + (b2 + c2 − a2 ) ≤ 3abc a (b2 + c2 − a2 ).
cyclic cyclic

2889. Proposed by Vedula N. Murty, Dover, PA, USA.


Suppose that A, B, C are the angles of 4ABC, and that r and R are its inradius and circum-
radius, respectively. Show that
³ r ´2
4 cos(A) cos(B) cos(C) ≤ 2 .
R
2890. Proposed by José Luis Dı́az-Barrero, Universitat Politècnica de Catalunya, Barcelona,
Spain.
n−1
P n
Q
Suppose that the polynomial A(z) = z n + ak z k can be factored into A(z) = (z − zk ),
k=0 k=1
where the zk are positive real numbers. Prove that, for k = 1, 2, . . . , n − 1,
¯ ¯1 ¯ ¯ 1
¯ an−k ¯ k
¯ ≥ ¯ an−k−1 ¯
¯ ¯ k+1
¯
¯ C(n, k) ¯ ¯ C(n, k + 1) ¯ ,

¡n¢
where C(n, k) denotes the binomial coefficient k . When does equality occur?

2891. Proposed by Vedula N. Murty, Dover, PA, USA, adapted by the editors.
Two proofreaders, Chris and Pat, were asked to read a manuscript and find the errors. Let B
be the number of errors which both Chris and Pat found, C the number of errors found only
by Chris, and P the number found only by Pat; lastly, let N be the number of errors found by
neither of them. Prove that
p
(B + P )(C + N )(B + C)(P + N ) ≥ |BN − CP |.

2893. Proposed by Vedula N. Murty, Dover, PA, USA.


In [2001: 45–47], we find three proofs of the classical inequality
X 3
1≤ cos(A) ≤ .
2
cyclic

In [2002: 86–87], we find Klamkin’s illustrations of the Majorization (or Karamata) Inequality.
Prove the above “classical inequality” using the Majorization Inequality.

116
2894. Proposed by Vedula N. Murty, Dover, PA, USA.
Suppose that 4ABC is acute-angled. With the standard notation, prove that
9
4abc < (a2 + b2 + c2 )(a cos A + b cos B + c cos C) ≤ abc.
2
2895. Proposed by Vedula N. Murty, Dover, PA, USA.
Suppose that A and B are two events with probabilities P (A) and P (B) such that 0 < P (A) < 1
and 0 < P (B) < 1. Let

2 [P (A ∩ B) − P (A)P (B)]
K= .
P (A) + P (B) − 2P (A)P (B)

Show that |K| < 1, and interpret the value K = 0.

2899. Proposed by Hiroshi Kotera, Nara City, Japan.


Find the maximum area of a pentagon ABCDE inscribed in a unit circle such that the diagonal
AC is perpendicular to the diagonal BD.

2900F . Proposed by Stanley Rabinowitz, Westford, MA, USA.


Let I be the incentre of 4ABC, r1 the inradius of 4IAB and r2 the inradius of 4IAC.
Computer experiments using Geometer’s Sketchpad suggest that r2 < 45 r1 .

(a) Prove or disprove this conjecture.


5
(b) Can 4 be replaced by a smaller constant?

2904. Proposed by Mohammed Aassila, Strasbourg, France.


Suppose that x1 > x2 > · · · > xn are real numbers. Prove that
n n
X X n(n − 1) 1X
x2k − ln(xj − xk ) ≥ (1 + 2 ln 2) − k ln k.
4 2
k=1 1≤j<k≤n k=1

2906. Proposed by Titu Zvonaru,


³ Bucharest,
´ Romania.
2 n2
Suppose that k ∈ N. Find min n + k .
n∈N

2911. Proposed by Mihály Bencze, Brasov, Romania.


(a) If z, w ∈ C and |z| = 1, prove that
n
X n−1
X
(n − 1) |w + z k | ≥ (n − k)|1 − z k |.
k=1 k=1

(b) If x ∈ R, prove that


n
X n−1
X
(n − 1) | cos(kx)| ≥ (n − k)| sin(kx)|.
k=1 k=1

2913. Proposed by Mihály Bencze, Brasov, Romania.


If a, b, c > 1 and α > 0, prove that
√ √ √ √ √ √
a α loga b+ α loga c + b α logb a+ α logb c + c α logc a+ α logc b
√ ³ 1 1 1
´
≤ abc aα− 2 + bα− 2 + cα− 2 .

117
2916. Proposed by George Tsintsifas, Thessaloniki, Greece.
Let S = A1 A2 A3 A4 be a tetrahedron and let M be the Steiner point; that is, the point M is
4
P
such that Aj M is minimized. Assuming that M is an interior point of S, and denoting by
j=1
A0j the intersection of Aj M with the opposite face, prove that

4
X 4
X
Aj M ≥ 3 A0j M.
j=1 j=1

2917F . Proposed by Šefket Arslanagić and Faruk Zejnulahi, University of Sarajevo, Sarajevo,
Bosnia and Herzogovina.
If x1 , x2 , x3 , x4 , x5 ≥ 0 and x1 + x2 + x3 + x4 + x5 = 1, prove or disprove that
x1 x2 x3 x4 x5 5
+ + + + ≥ .
1 + x2 1 + x3 1 + x4 1 + x5 1 + x1 6

2918. Proposed by Šefket Arslanagić and Faruk Zejnulahi, University of Sarajevo, Sarajevo,
Bosnia and Herzogovina.
Let a1 , a2 , . . . , a100 be real numbers satisfying:

a1 ≥ a2 ≥ · · · ≥ a100 ≥ 0;
a21 + a22 ≥ 200;
a23 + a24 + · · · + a2100 ≥ 200.

What is the minimum value of a1 + a2 + · · · + a100 ?

2919F . Proposed by Ross Cressman, Wilfrid Laurier University, Waterloo, ON.


Let n ∈ N with n > 1, and let
 
 n
X 
Tn = x = (x1 , . . . , xn ) ∈ Rn xj > 0 for j = 1, . . . , n, and xj = 1 .
 
j=1

n n
P √ P √
Let p, q, r ∈ Tn such that q j rj < pj rj . Prove or disprove:
j=1 j=1

n q
X n q
X
(a) qj (rj + pj ) < pj (rj + pj ),
j=1 j=1

(b) for all λ ∈ [0, 1],


X n q n q
X
qj (λrj + (1 − λ)pj ) < pj (λrj + (1 − λ)pj ).
j=1 j=1

[Proposer’s remarks: (a) is the special case of (b) with λ = 12 . This question is connected with
properties of the Shahshahani metric on Tn , a metric important for population genetics.]

2920. Proposed by Simon Marshall, student, Onslow College, Wellington, New Zealand.
Let a, b, and c be positive real numbers. Prove that

a4 + b4 + c4 + 2(a2 b2 + b2 c2 + c2 a2 ) ≥ 3(a3 b + b3 c + c3 a).

118
2923. Proposed by Šefket Arslanagić, University of Sarajevo, Sarajevo, Bosnia and Herzego-
vina.
Suppose that x, y ≥ 0 (x, y ∈ R) and x2 + y 3 ≥ x3 + y 4 . Prove that x3 + y 3 ≤ 2.

2924. Proposed by Todor Mitev, University of Rousse, Rousse, Bulgaria.


Suppose that x1 , . . . , xn (n ≥ 3) are positive real numbers satisfying
1 1 1
+ + ··· + ≥ α,
1+ x22 x3 · · · xn 2
1 + x 1 x3 · · · x n 2
1 + x1 x2 · · · xn−1
for some α > 0. Prove that
x1 x2 xn nα
+ + ··· + ≥ x1 x2 · · · x n .
x2 x3 x1 n−α

2927F . Proposed by Šefket Arslanagić, University of Sarajevo, Sarajevo, Bosnia and Herze-
govina.
Suppose that a, b and c are positive real numbers. Prove that
a3 b3 c3 3(ab + bc + ca)
2 2
+ 2 2
+ 2 2
≥ .
b − bc + c c − ca + a a − ab + b a+b+c
2928. Proposed by Christopher J. Bradley, Bristol, UK.
Suppose that ABC is an equilateral triangle and that P is a point in the plane of 4ABC. The
perpendicular from P to BC meets AB at X, the perpendicular from P to CA meets BC at Y ,
and the perpendicular from P to AB meets CA at Z.

1. If P is in the interior of 4ABC, prove that [XY Z] ≤ [ABC].


2. If P lies on the circumcircle of ABC, prove that X, Y , and Z are collinear.

2930. Proposed by José Luis Dı́az-Barrero, Universitat Politècnica de Catalunya, Barcelona,


Spain.
Suppose that a, b, and c are positive real numbers. Prove that
µ ¶
1 1 1 ab bc ca −2
+ + − 27 + +
a2 b2 c2 c a b
"µ ¶2 µ ¶ µ ¶#
1 1 1 1 1 2 1 1 2
≥ − + − + − .
3 a b b c c a

2933. Proposed by Titu Zvonaru, Bucharest, Romania. q


Prove, without the use of a calculator, that sin(40 ◦) < 37 .

2935. Proposed by Titu Zvonaru, Bucharest, Romania.


Suppose that a, b, and c are positive real numbers which satisfy a2 + b2 + c2 = 1, and that n > 1
is a positive integer. Prove that
1
a b c (n + 1)1+ n
+ + ≥ .
1 − a n 1 − bn 1 − cn n
2937. Proposed by Todor Mitev, University of Rousse, Rousse, Bulgaria.
Suppose that x1 , . . . , xn (n ≥ 2) are positive real numbers. Prove that
µ ¶
2 2 1 1 n2
(x1 + · · · + xn ) + · · · + ≥ .
x21 + x1 x2 x2n + xn x1 2

119
2938. Proposed by Todor Mitev, University of Rousse, Rousse, Bulgaria.
Suppose that x1 , . . . , xn , α are positive real numbers. Prove that
p √
(a) n (x1 + α) · · · (xn + α) ≥ α + n x1 · · · xn ;
p x1 + · · · + x n
(b) n (x1 + α) · · · (xn + α) ≤ α + .
n
2946. Proposed by Panos E. Tsaoussoglou, Athens, Greece.
Let x, y, z be positive real numbers satisfying x2 + y 2 + z 2 = 1. Prove that
µ ¶
1 1 1 √
(a) + + − (x + y + z) ≥ 2 3,
x y z
µ ¶
1 1 1 √
(b) + + + (x + y + z) ≥ 4 3.
x y z

2949F . Proposed by Walther Janous, Ursulinengymnasium, Innsbruck, Austria.


Let n ≥ 3 be an odd natural number. Determine the smallest number µ = µ(n) such that the
entries of any row and of any column of the matrix
 
1 a1,2 · · · a1,µ
 2 a2,2 · · · a2,µ 
 
 .. .. .. .. 
 . . . . 
n an,2 · · · an,µ

are distinct numbers from the set {1, 2, . . . , n − 1, n}, and the numbers in each row sum to the
same value.

2950F . Proposed by Walther Janous, Ursulinengymnasium, Innsbruck, Austria.


Let ABC be a triangle whose largest angle does not exceed 2π/3. For λ, µ ∈ R, consider inequa-
lities of the form
µ ¶ µ ¶ µ ¶ µ ¶ µ ¶ µ ¶
A B C A B C
cos · cos · cos ≥ λ + µ · sin · sin · sin .
2 2 2 2 2 2

2 3−1
(a) Prove that λmax ≥ .
8
(b) Prove or disprove that

2 3−1 √
λ= and µ = 1 + 3
8
yield the best inequality in the sense that λ cannot be increased. Determine also the cases of
equality.

2953. Proposed by Titu Zvonaru, Bucharest, Romania.


Let m, n be positive integers with n > 1, and let a, b, c be positive real numbers satisfying
am+1 + bm+1 + cm+1 = 1. Prove that
m
a b c (m + n)1+ n
+ + ≥ .
1 − man 1 − mbn 1 − mcn n

120
2955. Proposed by José Luis Dı́az-Barrero, Universitat Politècnica de Catalunya, Barcelona,
Spain.
Let n be a positive integer. For each positive integer k, let fk be the k th Fibonacci number; that
is, f1 = 1, f2 = 1, and fk+2 = fk+1 + fk for all k ≥ 1. Prove that
à n !à n !
X X 1
2
fk+1 ≥ n2 .
f2k
k=1 k=1

2956. Proposed by David Loeffler, student, Trinity College, Cambridge, UK.


Let A, B, C be the angles of a triangle. Prove that
µ ¶ µ ¶ µ ¶
A B C
tan2 + tan2 + tan2 <2
2 2 2
if and only if
µ ¶ µ ¶ µ ¶
A B C
tan + tan + tan < 2.
2 2 2

2959. Proposed by Peter Y. Woo, Biola University, La Mirada, CA, USA.


Given a non-isosceles triangle ABC, prove that there exists a unique inscribed equilateral tri-
angle P QR of minimal area, with P , Q, R on BC, CA, and AB, respectively. Construct it by
straightedge and compass.

2961. Proposed by Juan-Bosco Romero Márquez, Universidad de Valladolid, Valladolid, Spain.


Let ABC and A0 B 0 C 0 be two right triangles with right angles at A and A0 . If wa and wa0 are
the interior angle bisectors of angles A and A0 , respectively, prove that awa a0 wa0 ≥ bcb0 c0 , with
equality if and only if both ABC and A0 B 0 C 0 are isosceles.

2962. Proposed by Juan-Bosco Romero Márquez, Universidad de Valladolid, Valladolid, Spain.


Let ABC and A0 B 0 C 0 be two triangles satisfying a ≥ b ≥ c and a0 ≥ b0 ≥ c0 . If ha , ha0 are the
altitudes from the vertices A, A0 , respectively, to the opposite sides, prove that

(i) bb0 + cc0 ≥ aha0 + a0 ha , (ii) bc0 + b0 c ≥ aha0 + a0 ha .

Remark: Since this problem is identical to problem 2860, it is closed and no solutions will be
accepted.

2963. Proposed by Mihály Bencze, Brasov, Romania.


Let ABC be any acute-angled triangle. Let r and R be the inradius and circumradius, respec-
tively, and let s be the semiperimeter; that is, s = 12 (a + b + c). Let ma be the length of the
median from A to BC, and let wa be the length of the internal bisector of ∠A from A to the
side BC. We define mb , mc , wb and wc similarly. Prove that

3s2 − r2 − 4Rr X ma s2 − r2 − 4Rr


(a) ≤ ≤ ;
8sRr awa 7sRr
cyclic

3 X m2 4R + r
a
(b) ≤ 2 2
≤ .
4 b +c 4R
cyclic

121
2964. Proposed by Joe Howard, Portales, NM, USA.
(Inspired by Problem 80.D, Math. Gazette 80 (489) (1996) p. 606.)
¡ ¢
Let x ∈ 0, π2 . Show that:
· ¸· ¸
2 + cos x 2(1 − cos x) 1 + cos x
(a) 2
> ;
3 x 2
r
2 + cos x 1 + cos x 2(1 − cos x)
(b) < < .
3 2 x2

2967. Proposed by Vasile Cı̂rtoaje, University of Ploiesti, Romania.


Let a1 , a2 , . . . , an be positive real numbers, and let
 −1
X n n−1
X j
En =  ai  .
i=1 j=0

à !−1
√ n−1
P
If r = n a1 a2 · · · an ≥ 1, prove that En ≥ n rj for:
j=0

(a) n = 2, (b) n = 3, (c)F n ≥ 4.

2968. Proposed by Vasile Cı̂rtoaje, University of Ploiesti, Romania.


Let a1 , a2 , . . . , an be positive real numbers, and let
1 + a 1 a2 1 + a 2 a3 1 + a n a1
En = + + ··· + .
1 + a1 1 + a2 1 + an

Let r = n a a · · · a ≥ 1.
1 2 n
n(1+r 2 )
(a) Prove that En ≥ 1+r for n = 3 and n = 4.
n(1+r 2 )
(b)F Prove or disprove that En ≥ 1+r for n = 5.

2969. Proposed by Vasile Cı̂rtoaje, University of Ploiesti,√Romania.


Let a, b, c, d, and r be positive real numbers such that r = 4 abcd ≥ 1. Prove that
1 1 1 1 4
2
+ 2
+ 2
+ 2
≥ .
(1 + a) (1 + b) (1 + c) (1 + d) (1 + r)2

2970. Proposed by Titu Zvonaru, Bucharest, Romania.


If m and n are positive integers such that m ≥ n, and if a, b, c > 0, prove that
am bm cm an bn cn
+ + ≥ + + .
bm + cm cm + am am + bm bn + cn cn + an an + bn
2971. Proposed by Michel Bataille, Rouen, France.
For a, b, c ∈ (0, 1), find the least upper bound and the greatest lower bound of a + b + c + abc,
subject to the constraint ab + bc + ca = 1.

122
2972. Proposed by Vasile Cı̂rtoaje, University of Ploiesti, Romania.
(a) Prove that if 0 ≤ λ ≤ 4, then, for all positive real numbers x, y, z, t,

(t2 + 1)(x3 + y 3 + z 3 ) + 3(1 − t2 )xyz


≥ (1 + λt)(x2 y + y 2 z + z 2 x) + (1 − λt)(xy 2 + yz 2 + zx2 ).
1
(b) For t = 4 and λ = 4, the above inequality becomes

17(x3 + y 3 + z 3 ) + 45xyz ≥ 32(x2 y + y 2 z + z 2 x).

Find all positive values of δ such that the inequality

x3 + y 3 + z 3 + 3δxyz ≥ (1 + δ)(x2 y + y 2 z + z 2 x)

holds for all x, y, z which are: (i) positive real numbers: (ii) side lengths of a triangle.

2975. Proposed by Juan-Bosco Romero Márquez, Universidad de Valladolid, Valladolid, Spain.


Given an inscribed convex quadrilateral with sides of length m, n, p, q, taken in order around

the quadrilateral, and diagonals of length d and d0 , prove that mp + nq ≤ 12 (d + d0 ).

2976. Proposed by Šefket Arslanagić, University of Sarajevo, Sarajevo, Bosnia and Herzego-
vina.
Let a, b, c ∈ R. Prove that

(a2 + ab + b2 )(b2 + bc + c2 )(c2 + ca + a2 ) ≥ (ab + bc + ca)3 .

2977. Proposed by Vasile Cı̂rtoaje, University of Ploiesti, Romania.



Let a1 , a2 , . . . , an be positive real numbers, let r = n a1 a2 · · · an , and let
1 1 1 n
En = + + ··· + − .
a1 (1 + a2 ) a2 (1 + a3 ) an (1 + a1 ) r(1 + r)
(a) Prove that En ≥ 0 for

(a1 ) n = 3;
(a2 ) n = 4 and r ≤ 1;
1
(a3 ) n = 5 and ≤ r ≤ 2;
2
(a4 ) n = 6 and r = 1.

(b)F Prove or disprove that En ≥ 0 for

(b1 ) n = 5 and r > 0;


(b2 ) n = 6 and r ≤ 1.

2983. Proposed by Vasile Cı̂rtoaje, University of Ploiesti, Romania.


Let a1 , a2 , . . . , an < 1 be non-negative real numbers satisfying
r √
a21 + a22 + · · · + a2n 3
a= ≥ .
n 3
Prove that
a1 a2 an na
2 + 2 + ··· + 2
≥ .
1 − a1 1 − a2 1 − an 1 − a2

123
2988F . Proposed by Faruk Zejnulahi and Šefket Arslanagić, University of Sarajevo, Sarajevo,
Bosnia and Herzegovina.
Let x, y, z be non-negative real numbers satisfying x + y + z = 1. Prove or disprove:
1
(a) xy 2 + yz 2 + zx2 ≥ (xy + yz + zx);
3
2
(b) xy 2 + yz 2 + zx2 ≥ xy + yz + zx − .
9
How do the right sides of (a) and (b) compare?

2989. Proposed by Mihály Bencze, Brasov, Romania.


Prove that if 0 < a < b < d < π and a < c < d satisfy a + d = b + c, then
cos(a − d) − cos(b + c) ad
< .
cos(b − c) − cos(a + d) bc
2991. Proposed by Mihály Bencze, Brasov, Romania.
Let n be an integer, n ≥ 3. For all zi ∈ C, i = 1, 2, . . . , n, prove
¯ ¯ ¯ ¯
¯ n 3 ¯ ¯ n ¯¯ X ¡
¯X X ¯ ¯X
¢
(n − 1) ¯¯ zi − 3 zi zj zk ¯¯ ≤ ¯ zi ¯ |zi − zj |2 + (n − 3)|zi + zj | .
¯ i=1 1≤i<j<k≤n ¯ ¯ i=1 ¯ 1≤i<j≤n

2992. Proposed by Pham Van Thuan, Hanoi City, Viet Nam.


Let Q be a point interior to 4ABC. Let M , N , P be points on the sides BC, CA, AB,
respectively, such that M N k AQ, N P k BQ, and P M k CQ. Prove that
1
[M N P ] ≤ [ABC],
3
where [XY Z] denotes the area of triangle XY Z.

2993F . Proposed by Faruk Zejnulahi and Šefket Arslanagić, University of Sarajevo, Sarajevo,
Bosnia and Herzegovina.
Let x, y, z be non-negative real numbers satisfying x + y + z = 1. Prove or disprove:
x y z 9
(a) + + ≥ ;
xy + 1 yz + 1 zx + 1 10
x y z 9
(b) 2 + 2 + 2 ≥ .
y +1 z +1 x +1 10
How do the left sides of (a) and (b) compare?

2994. Proposed by Faruk Zejnulahi and Šefket Arslanagić, University of Sarajevo, Sarajevo,
Bosnia and Herzegovina.
Let a, b, c be non-negative real numbers satisfying a + b + c = 3. Show that
a2 b2 c2 3
(a) + + ≥ ;
b+1 c+1 a+1 2
a b c 3
(b) + + ≥ ;
b+1 c+1 a+1 2
a2 b2 c2 3
(c) 2
+ 2
+ 2
≥ ;
b +1 c +1 a +1 2
a b c 3
(d) 2 + + ≥ .
b + 1 c2 + 1 a 2 + 1 2

124
2999. Proposed by José Luis Dı́az-Barrero and Juan José Egozcue, Universitat Politècnica de
Catalunya, Barcelona, Spain.
Let m, n be positive integers. Prove that
à n
!m
m+1X k 1
m+2
(nm − k m ) < .
m n m+1
k=1

3000. Proposed by Paul Dayao, Ateneo de Manila University, The Philippines.


Let f be a continuous, non-negative, and twice-differentiable function on [0, ∞). Suppose that
xf 00 (x) + f 0 (x) is non-zero and does not change sign on [0, ∞). If x1 , x2 , . . . , xn are non-negative
real numbers and c is their geometric mean, show that

f (x1 ) + f (x2 ) + · · · + f (xn ) ≥ nf (c),

with equality if and only if x1 = x2 = · · · = xn .

to be continued . . .

125
126
Inequalities proposed in
“The American Mathematical Monthly”
Last update: November 24, 2004

Please visit http://www.maa.org/pubs/monthly.html

(An asterisk (F) after a number indicates that a problem was proposed without a solution.)

10354. Proposed by Hassan Ali Shah Ali, Tehran, Iran.


Determine the least natural
√ √number N such that, for all n ≥ N , there exist natural numbers
a, b with n = lf loora 2 + b 3c.

10371. Proposed by Emil Yankov Stoyanov, Antiem I Mathematical School, Vidin, Bulgaria.
Let B 0 and C 0 be points on the sides AB and AC, respectively, of a given triangle ABC, and
let P be a point on the segment B 0 C 0 . Determine the maximum value of

min{[BP B 0 ], [CP C 0 ]}
[ABC]

where [F ] denotes the area of F .

10374. Proposed by David L. Bock, University of Maryland, College Park, MD.


Given an integer N , characterize the smallest square in the plane containing N lattice points.

10383. Proposed by Kevin Ford (student), University of Illinois, Urbana, IL.


Let B1 , B2 , . . . , Bs denote subsets of a finite set B, and let λi = #(Bi )/#(B) and λ = λ1 +· · ·+λs .
Show that, for every integer t satisfying 1 ≤ t ≤ λ, there exist r1 , r2 , . . . , rt with r1 < r2 < · · · < rt
and
µ ¶−1
s
#(br1 ∩ br2 ∩ · · · ∩ Brt ) ≥ (λ − t + 1) #(B).
t

10384. Proposed by Franklin Kemp, East Texas State University, Commerce, TX.
Suppose x1 < x2 < · · · < xn and y1 < y2 < · · · < yn . Define the correlation coefficient r in the
usual way:
P
(xi − x)(yi − y)
r = pP i P
2 2
i (xi − x) · i (yi − y)

where x and y are the average values of the xi and yi , respectively, and the sums run from 1 to
n. Show that r ≥ 1/(n − 1).

10391. Proposed by Emre Alkan (student), Bosphorus University, İstanbul, Turkey, and the
editors.
If a1 , a2 , . . . , an are real numbers with a1 ≥ a2 ≥ · · · ≥ an , and if φ is a convex function defined
on the closed interval [an , a1 ], then
n
X n
X
φ(ak )ak+1 ≥ φ(ak+1 ak
k=1 k=1

with the convention that an+1 = a1 .

127
10392. Proposed by Murray S. Klamkin, University of Alberta, Edmonton, Alberta, Canada.
Determine the extreme values of
1 1 1
+ +
1+x+u 1+y+v 1+z+w
where xyz = a3 , uvw = b3 , and x, y, z, u, v, w > 0.

10400. Proposed by Itshak Borosh, Douglas Hensley, and Arthur M. Hobbs, Texas A&M
University College, College Station, TX, and Anthony Evans, Write State University, Dayton,
OH.
Determine the set of all pairs (n, t) of integers with 0 ≤ t ≤ n and
t µ ¶
X n nt
< .
k t!
k=0

10404. Proposed by Behzad Djafari Rouhani, Shahid Beheshti University and Islamic Azad
University, Tehran, Iran.
Let x1 , x2 , . . . be a sequence of real numbers such that

|xi − xj | ≥ |xi+1 − xj+1 |

for all positive integers i, j with |i − j| ≤ 2. Prove that hxn /ni converges to a finite limit as
n → ∞.

10413. Proposed by Mirel Mocanu, University of Craiova, Craiova, Romania.


Four disjoint (except for boundary points) equilateral triangles of sides a, b, c and d, are enclosed
ina regular hexagon of unit side. √
(a) Prove that 3a + b + c + d ≤√4 3.
(b) When is 3a + b + c + d = 4 3? √
(c)F Prove or disprove that a + b + c + d ≤ 2 3.

10417. Proposed by Răzvan Satnoianu, A. S. E., Bucharest, Romania.


Given the acute triangle ABC, let ha , hb , and hc denote the altitudes and s the semiperimeter.
Show that

3 max{ha , hb , hc } ≥ s.

10419. Proposed by Bill Correll, Jr. (student), Denison University, Granville, OH.
Let k be an integer greater than or equal to 3. Let S(k) be the set of nonnegative real numbers
x for which
¹ º¹ º j k ¹ º¹ º ¹ º
x+k−2 x+k−1 x x+k−2 x+k−1 x
+ = + .
k k−1 k k−1 k k−1
(a) Determine the largest integer in S(k).
(b) Show that S(k) is the union of a finite number of intervals with the sum of the lengths of
those intervals equal to (k 2 − 3k + 6)/2.

10421. Proposed by Gigel Militaru, University of Bucharest, Bucharest, Romania.


Let n be an integer, n ≥ 3, and let z1 , . . . , zn and t1 , . . . , tn be complex numbers. Prove that
there exists an integer i, 1 ≤ i ≤ n with
n
X
4|zi ti | ≤ |zi tj + zj ti |.
j=1

128
10422. Proposed by Adam Fieldsteel, Wesleyan University, Middletown, CT.
Let f : [0, 1] → R be a C 1 strictly increasing function with f (1) = L, where L is the length of
the graph of f . R
1
(a) Show that 0 f (x) dx ≥ π/4.
R1
(b) Show that 0 f (x) dx = π/4 only if the graph of f is a quarter circle.

10709. Proposed by Zoltán Sasvári, Technical University of Dresden, Dresden, Germany.


Let X be a standard normal random variable, and choose y > 0. Show that

P r(a ≤ X ≤ a + y) 3
e−ay < < e−ay+(1/2)ay
P r(a − y ≤ X ≤ a)

when a > 0. Show that the reversed inequalities hold when a < 0.

10713. Proposed by Juan-Bosco Romero Márquez, Universidad de Valladolid, Valladolid,


Spain.
Given a triangle with angles A ≥ B ≥ C, let a, b, and c be the lengths of the corresponding oppo-
site sides, let r be the radius of the inscribed circle, and let R be the radius of the circumscribed
circle. Show that A is acute if and only if

b+c
R+r < .
2

10716. Proposed by Michael L. Catalano-Johnson and Danial Loeb, Daniel Wagner Associa-
tes, Malvern, PA.
What is the largest cubical present that can be completely wrapped (without cutting) by a unit
square of wrapping paper?

10725. Proposed by Vasile Mihai, Toronto, ON, Canada.


Fix a positive integer n. Given a permutation α of {1, 2, . . . , n}, let

n
X
f (α) = (α(i) − α(i + 1))2 ,
i=1

where α(n + 1) = α(1). Find the extreme values of f (α) as α ranges over all permutations of
{1, 2, . . . , n}.

10730. Proposed by Walther Janous, Ursulinengymnasium, Innsbruck, Austria.


Fix an integer n ≥ 2. Determine the largest constant C(n) such that
X
(xj − xi )2 ≥ C(n) · min (xi+1 − xi )2
1≤i<n
1≤i<j≤n

for all real numbers x1 < x2 < · · · < xn .

10944. Proposed by Marcin Mazur, University of Illinois, Urbana, IL.


Prove that if a, b, c are positive real numbers such that abc ≥ 29 , then

1 1 1 3
√ +√ +√ ≥p √ .
1+a 1+b 1 + c 1 + 3 abc

129
11055. Proposed by Razvan Satnoianu, City University, London, U. K.
Let ABC be an acute triangle, with semiperimeter p and with inscribed and circumscribed
circles of radius r and R, respectively.

a) Show that ABC has a median of length at most p/ 3.
b) Show that ABC has a median of length at most R + r.
c) Show that ABC has an altitude of length at least R + r.

11069. Proposed by Péter Ivády, Budapest, Hungary.


Show that for 0 < x < 1
1 − x2 £ ¤ sin πx
2
1 + x3 (1 − x)3 < .
1+x πx
11075. Proposed by Götz Trenkler, University of Dortmund, Dortmund, Germany.
Let a, b, and c be complex numbers. Show that
¯p ¯
¯ ¯
¯ a2 + b2 + c2 ¯ ≤ max{|a| + |b|, |b| + |c|, |a| + |c|}.

to be continued . . .

130
131
Inequalities proposed in
“The Mathematical Gazette”
Last update: November 25, 2004

Please visit http://www.m-a.org.uk/resources/periodicals/the_mathematical_gazette/

(An asterisk (F) after a number indicates that a problem was proposed without a solution.)

87.C. Proposed by Nick Lord.


Find the smallest value of α for which
· ¸
1 1
− xyz ≤ α − (xy + yz + zx)
27 3
holds for all non-negative x, y, z satisfying x + y + z = 1.
(That α = 79 works in teh substance of BMO2 (1999) qn. 3.)

87.I. Proposed by Michel Bataille.


Let A, B, C and D be distinct points on a circle with radius r. Show that

AB 2 + BC 2 + CD2 + DA2 + AC 2 + BD2 ≤ 16r 2 .

When does equality occur?

88.D. Proposed by H. A. Shah Ali.


Consider the m×n rectangular plan of rooms shown in the diagram: on
each inner wall there could be a door. What is the minimum number of m rooms
inner doors needed to allow entry into every room?
n rooms
88.F. Proposed by D. Mărghidanu.
Let a, b, c, d be real numbers strictly between 0 and 1. Prove the inequality:
µ ¶ (c+d) µ ¶ (d+a) µ ¶ (a+b) µ ¶ (b+c)
a+b 2 b+c 2 c+d 2 d+a 2
+ + + > 2.
2 2 2 2

88.J. Proposed by Péter Ivády.


Show that, for 0 < x < π4 and 0 < y < π4 , the following inequality holds:
4 cos x cos y
cos(x − y) ≤ .
(cos x + cos y)2

to be continued . . .

132
133
Inequalities proposed in

“Die WURZEL”
Last update: September 1, 2004

The best problem solving journal in Germany; visit http://www.wurzel.org

ζ11 Proposed by Prof. Šefket Arslanagić, Sarajevo, Bosnia and Herzegovina


Given the function
3
F (x) = sin 3x sin3 x + cos 3x cos3 x − cos 2x,
4
prove that
1 1
− ≤ F (x) ≤
4 4
for all real x.

ζ13 Proposed by Michael Möbius, Sulzbach, Germany


Let a, b, c, d be real numbers satisfying a2 + b2 ≤ 1 and c2 + d2 ≤ 1. Prove that
p p √
(a + c)2 + (b + d)2 + (a − c)2 + (b − d)2 ≤ 2 2.

When does equality hold?

ζ21 Proposed by Heinz-Jürgen Seiffert, Berlin, Germany


Prove that for all real numbers x, y with xy > 0 the inequality
r
2xy x2 + y 2 √ x+y
+ ≥ xy +
x+y 2 2
holds. When does exactly equality hold?

ζ23 Proposed by Prof. Šefket Arslanagić, Sarajevo, Bosnia and Herzegovina


Let R, r be the circumradius and inradius, respectively, in a right-angled triangle with hypote-
nuse c and legs a, b. Find the maximum of the value Rr .

ζ37 Proposed by Prof. Šefket Arslanagić, Sarajevo, Bosnia and Herzegovina


Let ABC a triangle with sides a, b, c and altitudes ha , hb , hc . Prove the inequality
3
h2a + h2b + h2c ≤ (a2 + b2 + c2 ).
4
When does equality hold?

ζ38 Proposed by Michael Heerdegen, Apolda, Germany


Prove that
n ¡ ¢
X (−1)n−i · 2i+1 · ni
≥0
i+1
i=0

for all natural numbers n. When does equality hold?

134
ζ39 Proposed by Zdravko F. Starc, Vršac, Yugoslavia
Let a, b and c be positive real numbers. Prove that
a5 + b5 + a 2 + b2 b5 + c5 + b2 + c2 c5 + a 5 + c 2 + a 2
+ + < 2 (a2 + b2 + c2 ).
(a + b)(a2 + b2 ) + 1 (b + c)(b2 + c2 ) + 1 (c + a)(c2 + a2 ) + 1
η41 Proposed by Hans Rudolf Moser, Bürglen, Switzerland
The bottom face of a pyramid is a regular n-gon and its edges have all the same constant length
s. Prove that the height of such a pyramid with maximum volume is independent of n.

ζ44 Proposed by Prof. Šefket Arslanagić, Sarajevo, Bosnia and Herzegovina


Prove that
(y 3 + x)(z 3 + y)(x3 + z) ≥ 125xyz,
where x ≥ 2, y ≥ 2, z ≥ 2 are real numbers.

ζ47 Proposed by Thomas Fischer, Jena, Germany


For all integers n ≥ 2 prove that
n2n < (n − 1)n−1 · (n + 1)n+1 .
ζ56 Proposed by Heinz-Jürgen Seiffert, Berlin, Germany
Let n be a positive integer and a1 , . . . , an positive real numbers with a1 + · · · + an = 1. Prove
that
n µ ¶
Y 1
n−2+ ≥ (2n − 2)n .
ak
k=1

When does equality hold?

ζ58 Proposed by Prof. Šefket Arslanagić, Sarajevo, Bosnia and Herzegovina


For n ∈ N prove that
nn ≤ n! · en−1 .
η44 Proposed by Prof. Šefket Arslanagić, Sarajevo, Bosnia and Herzegovina
Prove that, in an isosceles triangle ABC with AC = BC = a, AB = c, and the angle-bisector
AD = w, the inequalities
2ac ac √
>w> 2
a+c a+c
hold.

η45 Proposed by Prof. Walther Janous, Innsbruck, Austria


Let x, y, z be nonnegative real numbers with x + y + z = 1. Prove that
(1 − x2 )2 + (1 − y 2 )2 + (1 − z 2 )2 ≤ (1 + x)(1 + y)(1 + z).
When does equality hold?

η48 Proposed by Heinz-Jürgen Seiffert, Berlin, Germany


Let f : [a, b] → R be a twice continuously differentiable and strictly convex function. Further-
Rb
more, a f (x) dx = 0. Prove that
¡ 2 ¢2 Z b
f (b) − f 2 (a)
< f 3 (x) dx.
4(f 0 (b) − f 0 (a))
a

135
η49 Proposed by Dr. Roland Mildner, Leipzig, Germany
In a Cartesian coordinate system a circle K1 (radius 2a, centre M1 (0, a)) and a circle K2 (radius
a, centre M2 (2a, 0)) are drawn with a > 0. Determine the smallest value of a such that the
coordinates of the intersection points of K1 and K2 are integers.

η50 Proposed by Prof. Šefket Arslanagić, Sarajevo, Bosnia and Herzegovina


Prove that
r r r
a+b b+c c+a √
+ + ≥ 3 2,
c a b
where a, b, c are positive real numbers.

η51 Proposed by Hans Rudolf Moser, Bürglen, Switzerland


Given the linear system of equations in variables x, y, z with parameter p
px + y + z = p + 1
x + py + z = p
x + y + pz = p − 1.
For which values of p the solutions satisfy the inequalities x < y < z? When x > y > z holds?

η52 Proposed by Oleg Faynshteyn, Leipzig, Germany


A sphere is inscribed in a (nonregular) tetrahedron with surface area A. Let ² be a plane parallel
to one of the faces which touches the sphere. Determine the maximal area of a triangle that is
formed by the intersection of ² and the tetrahedron.

η57 Proposed by Prof. Šefket Arslanagić, Sarajevo, Bosnia and Herzegovina


Let a1 , a2 , a3 , . . . be a sequence of real numbers with a1 = 0, |a2 | = |a1 + 1|, |a3 | = |a2 + 1|, . . .,
|an | = |an−1 + 1|. Prove that, for each n ∈ N,
a1 + a 2 + · · · a n 1
≥− .
n 2
ι7 Proposed by Astrid Baumann, Friedberg, Germany
Prove the inequality
(1 + xn )(1 + x)n ≥ (1 + x2 )n + 2n xn
for all n ∈ N and x ≥ 0. In which cases equality holds?

κ47 Proposed by Prof. Walther Janous, Innsbruck, Austria


Prove that
r
4 4 3yz + 1 3 3yz + 1
y +z +3≥y+z+3· ·
4 4
for all real x, y.

λ31 Proposed by Prof. Šefket Arslanagić, Sarajevo, Bosnia and Herzegovina


Prove that a3 + b3 + c3 ≥ 3abc for any a, b, c ≥ 0.

λ32 Proposed by after Mihály Bencze, Kronstadt, Romania


Prove the inequality
às !
n 2 + n + 1)
X 1 n n+1 2 (n
< − n .
k3 + 1 2 n 3 (n2 + n)
k=1

136
λ34 Proposed by Dr. Roland Mildner, Leipzig, Germany
A buoy-similar solid consists of a circular cylinder (diameter d, height H), a circular cone
(diameter d, height h) on the top of the cylinder and a half sphere (diameter d) on the bottom
of the cylinder. How must the values of d, h and H be choosen to get the least surface area with
given fixed volume V of the solid?

to be continued . . .

137
Inequalities proposed in
“Elemente der Mathematik”
Last update: October 8, 2004

Please visit http://www.birkhauser.ch/journals/1700/1700_tit.htm

830. Proposed by S. Gabler, Mannheim, BRD.


Let x1 , x2 , . . . , xn (n ≥ 3) be positive real numbers satisfying x1 + x2 + · · · + xn = 1. Then
µ ¶−1 X
n xi xj 1

2 1 − xi 1 − xj (n − 1)2
1≤i<j≤n

with equality if and only if x1 = x2 = · · · = xn = n1 . Prove this.

845. Proposed by W. Janous, Innsbruck, A.


For non-negative reals x1 , . . . , xn satisfying x1 + · · · + xn = s (n ∈ N) prove that
n
ns X 1 s
n− ≤ ≤n− .
s+n 1 + xi s+1
i=1

When does exactly equality occur?

846. Proposed by P. Erdös.


Let
µ ¶
n
n−1≤k ≤ , n, k ∈ N.
2

Then always exist n points along the x-axis with x1 < x2 < · · · < xn , that determine exactly k
different distances xi − xj (i > j). Prove this.

849. Proposed by M. Bencze, Brasov, Romania.


For natural numbers n prove that

n(n − 1) n(n − 1)(2n + 5)


exp ≤ 11 · 22 · · · nn ≤ exp .
2 12

1084. Proposed by Walther Janous, Innsbruck, A. √


Let a, b, λ be real numbers such that λ > 0 and b − a ≥ π/ λ. The function f : [a, b] → R is
contigouos differentiable. Prove the existence of t ∈ (a, b) such that f 0 (t) < λ + f 2 (t).

1091. Proposed by Hansjürg Stocker, Wädenswil, CH; Jany Binz, Bolligen, CH.
How many non-decreasing sequences of natural numbers with length n · p (n, p ∈ N) exist which
members ak (k, ak ∈ N) satisfy the constraints

apn = apn−1 = apn−2 = · · · = apn−p+1 = n

and

api+1 ≥ api ≥ api−1 ≥ · · · ≥ api−p+1 ≥ i (i = n − 1, n − 2, . . . , 1)?

138
1094. Proposed by R. Bil, Kiel, D.
Prove that for all natural numbers n
µ ¶4 µ ¶3
4 n+ √
√ 4 n+1 3 n+ √
√ 3 n+1
µ ¶ µ ¶
n+1 2 n+1 2
< e<
n n
(e is as usual the Euler number.)

1126. Proposed by Rolf Rose, Magglingen, CH.


The sum of the surface areas of two solids with given shape is constant. Prove that the volumes
of these solids are proportional to their surface areas if the sum of the volumes is a minimum.
Furthermore, calculate this ratio of surface area to volume of two arbitrary solids with the same
shape und determine this value if one solid is a cube and the other a regular tetrahedron.

1128. Proposed by Wolfgang Moldenhauer, Erfurt, D.


Let p be a polynomial of degree ≤ 3 and q a polynomial of degree ≤ 5 with

p(0) = q(0), p(1) = q(1),


0 0
p (0) = q (0), p0 (1) = q 0 (1),
q 00 (0) = 0, q 00 (1) = 0.

Determine a constant C > 0 such that for all pairs (p, q) the inequality
Z 1 Z 1
p(t)q(t) dt ≥ C · (p(t))2 dt
0 0

holds.

1128. Proposed by Wolfgang Moldenhauer, Erfurt, D.


Let p be a polynomial of degree ≤ 3 and q a polynomial of degree ≤ 5 with

p(0) = q(0), p(1) = q(1),


p0 (0) = q 0 (0), p0 (1) = q 0 (1),
q 00 (0) = 0, q 00 (1) = 0.

Determine a constant C > 0 such that for all pairs (p, q) the inequality
Z 1 Z 1
p(t)q(t) dt ≥ C · (p(t))2 dt
0 0

holds.

1146. Proposed by Šefket Arslanagić, Sarajevo, BIH.


Prove or disprove: In each convex pentagon there are three diagonals from which one can con-
struct a triangle.

1147. Proposed by Zdravko F. Starc, Vršac, YU.


Prove the following inequalities:
µ ¶
1 2 n n(n − 1)
1 · 2 · · · n ≤ 1! · 2! · · · n! · exp , (1)
2
f1f1 · f2f2 · · · fnfn ≤ f1 ! · f2 ! · · · fn ! · exp(fn+2 − n − 1). (2)

139
Here fn denotes the Fibonacci numbers: f1 = f2 = 1, fn+2 = fn+1 + fn for n = 1, 2, 3, . . ..

1157. Proposed by Roland Wyss, Flumenthal, CH.


Given an ellipse with the equation 25x2 + 9y 2 = 900 and the points O(0|0) and C(1|0) on its
minor axis. For which points P on the periphery is ∠OP C a maximum?

1164. Proposed by Jany C. Binz, Bolligen, CH.


Three circles are inscribed in an isosceles triangle with base b, inradius % and circumradius r:
two of them with radius t touch each one of the legs, the base and the incircle; the other with
radius u touches both legs and the incircle. Determine the smallest triangle such that % is an
integer multiple of t and b, %, r, t, u are all integers.

1169. Proposed by Péter Ivády, Budapest, H.


Let 0 < x < π2 . Prove the inequality
µ ¶3 µ ¶2
2 + cos x sin x
< .
3 x

1174. Proposed by Peter Hohler, Aarburg, CH.


We consider sequences of k > 2 consecutive numbers:

n, n + 1, n + 2, . . . , n + k − 2, n + k − 1.

Most of such sequences contain at least one number which is coprime to all other numbers of
the sequence. Find the smallest sequence (that is, nk is minimum) with no number therein that
is coprime to all other numbers of the sequence.

1190. Proposed by Mihály Bencze, Sacele, RO.


Let f : R → R be an increasing differentiable function. Prove that
n Z nxl −(x1 +x2 +···+xn ) µ ¶
X t
f xl − dt ≥ 0,
0 n−1
l=1

where x1 , x2 , . . . , xn (n ≥ 2) are arbitrary real numbers.

1198. Proposed by Götz Trenkler, Dortmund, D.


Let a, b, c and d be complex numbers. Prove that
p
|ab + cd| ≤ max{|a|, |b| + |c|, |d|}.

140
1200. Proposed by Matthias Müller, Bad Saulgau, D.
A “Ulam sequence” is defined recursively as follows: Two natural numbers u 1 , u2 are given with
u1 < u2 . For n ≥ 3, let un be the smallest integer that is greater than un−1 and that can be
represented in the form un = uk + ul with 0 < k < l < n exactly once. Let xN be the number
of terms of these Ulam sequence which are less than or equal to N .
Prove:
xN 1
lim sup ≤ .
N →∞ N 2

1201. Proposed by Mihály Bencze, Sacele, RO.


Let 1 ≤ a < b. Prove the following inequalities:
µ ¶ µ ¶
x a x b
(a) cos √ < cos √ for 0 < x < π2 ,
a b
µ ¶a µ ¶
x x b
(b) cos √ > cos √
3
for sufficient small positive x.
3
a b

1205. Proposed by Roland Wyss, Flumenthal, CH.


The following problem is well known from the classroom: “Which rectangle with fixed perimeter
has maximum area?”. This will be generalized as follows: From a rectangular plate with sides
ax and y (a > 1), m ≥ 0 squares of side x and n ≥ 0 discs with diameter x should be cut. How
x and y must be selected to maximize the area of the rest piece while the perimeter u remains
constant? Prove also that a non-overlapping cutting of these m + n pieces is actually possible.

1207. Proposed by Šefket Arslanagić, Sarajevo, BIH.


Prove that for positive numbers x, y, z the following inequality holds:
x y z x+y+z
+ + ≥ √ .
y z x 3 xyz

to be continued . . .

141
Inequalities proposed at
International Mathematical Olympiads
Complete and up-to-date: September 10, 2004

2nd IMO 1960, Sinaia, Romania. Problem 2.


For what values of the variable x does the following inequality hold:
4x2
√ < 2x + 9?
(1 − 1 + 2x)2
2nd IMO 1960, Sinaia, Romania. Problem 6.
Consider a cone of revolution with an inscribed sphere tangent to the base of the cone. A
cylinder is circumscribed about this sphere so that one of its bases lies in the base of the
cone. Let V1 be the volume of the cone and V2 the volume of the cylinder.
(a) Prove that V1 6= V2 .
(b) Find the smallest number k for which V1 = kV2 , for this case, construct the angle
subtended by a diameter of the base of the cone at the vertex of the cone.

3rd IMO 1961, Veszprem, Hungaria. Problem 2.


Let a, b, c be the sides of a triangle, and T its area. Prove:

a2 + b2 + c2 ≥ 4 3 T.
In what case does equality hold?

3rd IMO 1961, Veszprem, Hungaria. Problem 4.


Consider triangle P1 P2 P3 and a point P within the triangle. Lines P1 P , P2 P , P3 P intersect
the opposite sides in points Q1 , Q2 , Q3 respectively. Prove that, of the numbers
P1 P P2 P P3 P
, ,
P Q1 P Q2 P Q3
at least one is ≤ 2 and at least one is ≥ 2.

3rd IMO 1961, Veszprem, Hungaria. Problem 5.


Construct triangle ABC if AC = b, AB = c and ∠AM B = ω, where M is the midpoint
of segment BC and ω < 90 ◦. Prove that a solution exists if and only if
ω
b tan ≤ c < b.
2
In what case does the equality hold?

4th IMO 1962, České Budejovice, Czechoslovakia. Problem 2 (Proposed by ).


Determine all real numbers x which satisfy the inequality:
√ √ 1
3−x− x+1> .
2
5th IMO 1963, Warsaw, Poland. Problem 3.
In an n-gon all of whose interior angles are equal, the lengths of consecutive sides satisfy
the relation a1 ≥ a2 ≥ · · · ≥ an . Prove that a1 = a2 = · · · = an .

142
6th IMO 1964, Moscow, U. S. S. R.. Problem 2.
Suppose a, b, c are the sides of a triangle. Prove that

a2 (b + c − a) + b2 (c + a − b) + c2 (a + b − c) ≤ 3abc.

6th IMO 1964, Moscow, U. S. S. R.. Problem 5.


Suppose five points in a plane are situated so that no two of the straight lines joining them
are parallel, perpendicular, or coincident. From each point perpendiculars are drawn to all
the lines joining the other four points. Determine the maximum number of intersections
that these perpendiculars can have.

7th IMO 1965, Berlin, German Democratic Republic. Problem 1.


Determine all values x in the interval 0 ≤ x ≤ 2π which satisfy the inequality
√ √ √
2 cos x ≤ | 1 + sin 2x − 1 − sin 2x| ≤ 2.

7th IMO 1965, Berlin, German Democratic Republic. Problem 2.


Consider the system of equations

a11 x1 + a12 x2 + a13 = 0


a21 x1 + a22 x2 + a23 = 0
a31 x1 + a32 x2 + a33 = 0

with unknowns x1 , x2 , x3 . The coefficients satisfy the conditions:


(a) a11 , a22 , a33 are positive numbers;
(b) the remaining coefficients are negative numbers;
(c) in each equation, the sum of the coefficients is positive.
Prove that the given system has only the solution x1 = x2 = x3 = 0.

7th IMO 1965, Berlin, German Democratic Republic. Problem 6.


In a plane a set of n points (n ≥ 3) is given. Each pair of points is connected by a segment.
Let d be the length of the longest of these segments. We define a diameter of the set to
be any connecting segment of length d. Prove that the number of diameters of the given
set is at most n.

8th IMO 1966, Sofia, Bulgaria. Problem 3.


Prove: The sum of the distances of the vertices of a regular tetrahedron from the center
of its circumscribed sphere is less than the sum of the distances of these vertices from any
other point in space.

8th IMO 1966, Sofia, Bulgaria. Problem 6.


In the interior of sides BC, CA, AB of triangle ABC, any points K, L, M , respectively,
are selected. Prove that the area of at least one of the triangles AM L, BKM , CLK is
less than or equal to one quarter of the area of triangle ABC.

9th IMO 1967, Cetinje, Yugoslavia. Problem 1.


Let ABCD be a parallelogram with side lengths AB = a, AD = 1, and with ∠BAD = α.

143
If 4ABD is acute, prove that the four circles of radius 1 with centers A, B, C, D cover
the parallelogram if and only if

a ≤ cos α + 3 sin α.

9th IMO 1967, Cetinje, Yugoslavia. Problem 2.


Prove that if one and only one edge of a tetrahedron is greater than 1, then its volume is
≤ 81 .

9th IMO 1967, Cetinje, Yugoslavia. Problem 4.


Let A0 B0 C0 and A1 B1 C1 be any two acute-angled triangles. Consider all triangles ABC
that are similar to 4A1 B1 C1 (so that vertices A1 , B1 , C1 correspond to vertices A, B, C,
respectively) and circumscribed about triangle A0 B0 C0 (where A0 lies on BC, B0 on CA,
and AC0 on AB). Of all such possible triangles, determine the one with maximum area,
and construct it.

10th IMO 1968, Moscow, U. S. S. R.. Problem 4.


Prove that in every tetrahedron there is a vertex such that the three edges meeting there
have lengths which are the sides of a triangle.

11th IMO 1969, Bucharest, Romania. Problem 5.


Given¡ n >¢ 4 points in the plane such that no three are collinear. Prove that there are at
least n−3
2
convex quadrilaterals whose vertices are four of the given points.

11th IMO 1969, Bucharest, Romania. Problem 6.


Prove that for all real numbers x1 , x2 , y1 , y2 , z1 , z2 with x1 > 0, x2 > 0, x1 y1 − z12 > 0,
x2 y2 − z22 > 0, the inequality
8 1 1
≤ +
(x1 + x2 )(y1 + y2 ) − (z1 + z2 ) 2 x1 y1 − z1 x2 y2 − z22
2

is satisfied. Give necessary and sufficient conditions for equality.

12th IMO 1970, Keszthely, Hungaria. Problem 2.


Let a, b and n be integers greater than 1, and let a and b be the bases of two number
systems. An−1 and An are numbers in the system with base a, and Bn−1 and Bn are
numbers in the system with base b; these are related as follows:

An = xn xn−1 · · · x0 , An−1 = xn−1 xn−2 · x0 ,


Bn = xn xn−1 · · · x0 , Bn−1 = xn−1 xn−2 · x0 ,
xn 6= 0, xn−1 6= 0.
An−1 Bn−1
Prove that < if and only if a > b.
An Bn

12th IMO 1970, Keszthely, Hungaria. Problem 3.


The real numbers a0 , a1 , . . . , an , . . . satisfy the condition:

1 = a 0 ≤ a1 ≤ a2 ≤ · · · ≤ a n ≤ · · · .

144
The numbers b1 , b2 , . . . , bn , . . . are defined by
n µ ¶
X ak−1 1
bn = 1− √ .
k=1
ak ak

(a) Prove that 0 ≤ bn < 2 for all n.


(b) Given c with 0 ≤ c < 2, prove that there exist numbers a0 , a1 , . . . with the above
properties such that bn > c for large enough n.

12th IMO 1970, Keszthely, Hungaria. Problem 5.


In the tetrahedron ABCD, angle BCD is a right angle. Suppose that the foot H of the
perpendicular from d to the plane ABC is the intersection of the altitudes of 4ABC.
Prove that

(AB + BC + CA)2 ≤ 6 (AD 2 + BD2 + CD2 ).

For what tetrahedra does equality hold?

12th IMO 1970, Keszthely, Hungaria. Problem 6.


In a plane there are 100 points, no three of which are collinear. Consider all possible
triangles having these points as vertices. Prove that no more than 70 % of these triangles
are acute-angled.
13th IMO 1971, Žilina, Czechoslovakia. Problem 1.
Prove that the following assertion is true for n = 3 and n = 5, and that it is false for
every other natural number n > 2: If a1 , a2 , . . . , an are arbitrary real numbers, then

(a1 − a2 )(a1 − a3 ) · · · (a1 − an ) + (a2 − a1 )(a2 − a3 ) · · · (a2 − an ) + · · · +


(an − a1 )(an − a2 ) · · · (an − an−1 ) ≥ 0.

13th IMO 1971, Žilina, Czechoslovakia. Problem 4.


All the faces of tetrahedron ABCD are acute-angled triangles. We consider all closed
polygonal paths of the form XY ZT X defined as follows: X is a point on edge AB distinct
from A and B; similarly, Y, Z, T are interior points of edges BC, CD, DA, respectively.
Prove that
(a) If ∠DAB + ∠BCD 6= ∠CDA + ∠ABC, then among the polygonal paths, there is
none of minimal length.
(b) If ∠DAB + ∠BCD = ∠CDA + ∠ABC, then they are infinitely many shortest
polygonal paths, their common length being 2AC sin(α/2), where α = ∠BAC + ∠CAD +
∠DAB.

13th IMO 1971, Žilina, Czechoslovakia. Problem 6.


Let A = (aij ) (i, j = 1, 2, . . . , n) be a square matrix whose elements are non-negative
integers. Suppose that whenever an element aij = 0, the sum of the elements in the ith
row and the jth column is ≥ n. Prove that the sum of all the elements of the matrix is
≥ n2 /2.

145
14th IMO 1972, Torun, Poland. Problem 4.
Find all solutions (x1 , x2 , x3 , x4 , x5 ) of the system of inequalities

(x21 − x3 x5 )(x22 − x3 x5 ) ≤ 0, (x24 − x1 x3 )(x25 − x1 x3 ) ≤ 0,


(x22 − x4 x1 )(x23 − x4 x1 ) ≤ 0, (x25 − x2 x4 )(x21 − x2 x4 ) ≤ 0,
(x23 − x5 x2 )(x24 − x5 x2 ) ≤ 0,

where x1 , x2 , x3 , x4 , x5 are positive real numbers.

14th IMO 1972, Torun, Poland. Problem 5.


Let f and g be real-valued functions defined for all real values of x and y, and satisfying
the equation

f (x + y) + f (x − y) = 2f (x)g(y)

for all x, y. Prove that if f (x) is not identically zero, and if |f (x)| ≤ 1 for all x, then
|g(y)| ≤ 1 for all y.

15th IMO 1973, Moscow, U. S. S. R.. Problem 1.


−−→ −−→ −−→
Point O lies on line g; OP1 , OP2 , . . . , OPn are unit vectors such that points P1 , P2 , . . . , Pn
all lie in a plane containing g and on one side of g. Prove that if n is odd,
−−→ −−→ −−→
|OP1 + OP2 + · · · + OPn | ≥ 1.
−−→ −−→
Here |OM | denotes the length of vector OM .

15th IMO 1973, Moscow, U. S. S. R.. Problem 3.


Let a and b be real numbers for which the equation

x4 + ax3 + bx2 + ax + 1 = 0

has at least one real solution. For all such pairs (a, b), find the minimum value of a 2 + b2 .

15th IMO 1973, Moscow, U. S. S. R.. Problem 4.


A soldier needs to check on the presence of mines in a region having the shape of an
equilateral triangle. The radius of action of his detector is equal to half the altitude of the
triangle. The soldier leaves from one vertex of the triangle. What path should he follow
in order to travel the least possible distance and still accomplish his mission?

15th IMO 1973, Moscow, U. S. S. R.. Problem 6.


Let a1 , a2 , . . . , an be n positive numbers, and let q be a given real number such that
0 < q < 1. Find n numbers b1 , b2 , . . . , bn for which

(a) ak < bk for k = 1, 2, . . . , n,


bk+1 1
(b) q < < for k = 1, 2, . . . , n − 1,
bk q
1+q
(c) b1 + b2 + · · · + bn < (a1 + a2 + · · · + an ).
1−q

146
16th IMO 1974, Erfurt, German Democratic Republic. Problem 2.
In the triangle ABC, prove that there is a point D on side AB such that CD is the
geometric mean of AD and DB if and only if
C
sin A sin B ≤ sin2 .
2
16th IMO 1974, Erfurt, German Democratic Republic. Problem 4.
Consider decompositions of an 8 × 8 chessboard into p non-overlapping rectangles subject
to the following conditions:
(i) Each rectangle has as many white squares as black squares.
(ii) If ai is the number of white squares in the i-th rectangle, then a1 < a2 < · · · < ap .
Find the maximum value of p for which such a decomposition is possible. For this value
of p, determine all possible sequences a1 , a2 , . . . , ap .

16th IMO 1974, Erfurt, German Democratic Republic. Problem 5.


Determine all possible values of
a b c d
S= + + +
a+b+d a+b+c b+c+d a+c+d
where a, b, c, d are arbitrary positive numbers.

17th IMO 1975, Burgas, Bulgaria. Problem 1.


Let xi , yi (i = 1, 2, . . . , n) be real numbers such that

x1 ≥ x 2 ≥ · · · ≥ x n and y1 ≥ y2 ≥ · · · ≥ yn .

Prove that, if z1 , z2 , . . . , zn is any permutation of y1 , y2 , . . . , yn , then


n
X n
X
2
(xi − yi ) ≤ (xi − zi )2 .
i=1 i=1

18th IMO 1976, Lienz, Austria. Problem 1.


In a plane convex quadrilateral of area 32, the sum of the lengths of two opposite sides
and one diagonal is 16. Determine all possible lengths of the other diagonal.

18th IMO 1976, Lienz, Austria. Problem 3.


A rectangular box can be filled completely with unit cubes. If one places as many cubes
as possible, each with volume 2, in the box, so that their edges are parallel to the edges
of the box, one can fill exactly 40 % of the box. Determine the possible dimensions of all
such boxes.

18th IMO 1976, Lienz, Austria. Problem 4.


Determine, with proof, the largest number which is the product of positive integers whose
sum is 1976.

19th IMO 1977, Beograd, Yugoslavia. Problem 2.


In a finite sequence of real numbers the sum of any seven successive terms is negative,

147
and the sum of any eleven successive terms is positive. Determine the maximum number
of terms in the sequence.

19th IMO 1977, Beograd, Yugoslavia. Problem 4.


Four real constants a, b, A, B are given, and

f (θ) = 1 − a cos θ − b sin θ − A cos 2θ − B sin 2θ.

Prove that if f (θ) ≥ 0 for all real θ, then

a2 + b2 ≤ 2 and A2 + B 2 ≤ 1.

19th IMO 1977, Beograd, Yugoslavia. Problem 6.


Let f (n) be a function defined on the set of all positive integers and having all its values
in the same set. Prove that if

f (n + 1) > f (f (n))

for each positive integer n, then

f (n) = n for each n.

20th IMO 1978, Bucharest, Romania. Problem 1.


m and n are natural numbers with 1 ≤ m < n. In their decimal representations, the last
three digits of 1978m are equal, respectively, to the last three digits of 1978n . Find m and
n such that m + n has its least value.

20th IMO 1978, Bucharest, Romania. Problem 5.


Let {ak } (k = 1, 2, 3, . . . , n, . . .) be a sequence of distinct positive integers. Prove that for
all natural numbers n,
n n
X ak X 1
≥ .
k=1
k2 k=1
k

20th IMO 1978, Bucharest, Romania. Problem 6.


An international society has its members from six different countries. The list of members
contains 1978 names, numbered 1, 2, . . . , 1978. Prove that there is at least one member
whose number is the sum of the numbers of two members from his own country, or twice
as large as the number of one member from his own country.

21st IMO 1979, London, United Kingdom. Problem 4.


Given a plane π, a point P in this plane and a point Q not in π, find all points R in π
such that the ratio (QP + P R)/QR is a maximum.

22nd IMO 1981, Washington D.C., U.S.A. Problem 1.


P is a point inside a given triangle ABC. D, E, F are the feet of the perpendiculars from
P to the lines BC, CA, AB respectively. Find all P for which
BC CA AB
+ +
PD PE PF

148
is least.

22nd IMO 1981, Washington D.C., U.S.A. Problem 3.


Determine the maximum value of m2 + n2 , where m and n are integers satisfying m, n ∈
{1, 2 . . . , 1981} and (n2 − mn − m2 )2 = 1.

23rd IMO 1982, Budapest, Hungary. Problem 3.


Consider the infinite sequences {xn } of positive real numbers with the following properties:
x0 = 1, and for all i ≥ 0, xi+1 ≤ xi .
(a) Prove that for every such sequence, there is an n ≥ 1 such that
x20 x21 x2
+ + · · · + n−1 ≥ 3.999.
x1 x2 xn
(b) Find such a sequence for which
x20 x21 x2
+ + · · · + n−1 < 4 for all n.
x1 x2 xn
23rd IMO 1982, Budapest, Hungary. Problem 6.
Let S be a square with sides of length 100, and let L be a path within S which does
not meet itself and which is composed of line segments A0 A1 , A1 A2 , . . ., An−1 An with
A0 6= An . Suppose that for every point P of the boundary of S there is a point of L at a
distance from P not greater than 1/2. Prove that there are two points X and Y in L such
that the distance between X and Y is not greater than 1, and the length of that part of
L which lies between X and Y is not less than 198.

24th IMO 1983, Paris, France. Problem 3.


Let a, b and c be positive integers, no two of which have a common divisor greater than
1. Show that 2abc − ab − bc − ca is the largest integer which cannot be expressed in the
form xbc + yca + zab, where x, y and z are non-negative integers.

24th IMO 1983, Paris, France. Problem 6.


Let a, b and c be the lengths of the sides of a triangle. Prove that
a2 b(a − b) + b2 c(b − c) + c2 a(c − a) ≥ 0.
Determine when equality occurs.

25th IMO 1984, Prague, Czechoslovakia. Problem 1.


Prove that
7
0 ≤ yz + zx + xy − 2xyz ≤ ,
27
where x, y and z are non-negative real numbers for which x + y + z = 1.

25th IMO 1984, Prague, Czechoslovakia. Problem 5.


Let d be the sum of the lengths of all the diagonals of a plane convex polygon with n
vertices (n > 3), and let p be its perimeter. Prove that
· ¸
2d h n i n + 1
n−3< < − 2,
p 2 2

149
where [x] denotes the greatest integer not exceeding x.

26th IMO 1985, Joutsa, Finland. Problem 3.


For any polynomial P (x) = a0 + a1 x + · · · + ak xk with integer coefficients, the number
of coefficients which are odd is denoted by w(P ). For i = 0, 1, . . ., let Qi (x) = (1 + x)i .
Prove that if i1 , i2 , . . . , in are integers such that 0 ≤ i1 < i2 < · · · < in , then
w(Qi1 + Qi2 + · · · + Qin ) ≥ w(Qi1 ).

26th IMO 1985, Joutsa, Finland. Problem 6.


For every real number x1 , construct the sequence x1 , x2 , . . . by setting
µ ¶
1
xn+1 = xn xn + for each n ≥ 1.
n
Prove that there exists exactly one value of x1 for which
0 < xn < xn+1 < 1
for every n.

27th IMO 1986, Warsaw, Poland. Problem 6.


One is given a finite set of points in the plane, each point having integer coordinates. Is it
always possible to color some of the points in the set red and the remaining points white
in such a way that for any straight line L parallel to either one of the coordinate axes the
difference (in absolute value) between the numbers of white points and red points on L
is not greater than 1? Justify your answer.

28th IMO 1987, Havana, Cuba. Problem 3.


Let x1 , x2 , . . . , xn be real numbers satisfying the equation x21 +x22 +· · ·+x2n = 1. Prove that
for every integer k ≥ 2 there are integers a1 , a2 , . . . , an , not all 0, such that |ai | ≤ k − 1
for all i and

(k − 1) n
|a1 x1 + a2 x2 + · · · + an xn | ≤ .
kn − 1
29th IMO 1988, Canberra, Australia. Problem 4.
Show that the set of real numbers x that satisfy the inequality
70
X k 5

k=1
x−k 4

is a union of disjoint intervals, the sum of whose lengths is 1988.

29th IMO 1988, Canberra, Australia. Problem 5.


ABC is a triangle right-angled at A, and D is the foot of the altitude from A. The
straight line joining the incenters of the triangles ABD, ACD intersects the sides AB,
AC at the points K, L respectively. S and T denote the areas of the triangles ABC and
AKL respectively. Show that
S ≥ 2T.

150
30th IMO 1989, Braunschweig, Germany. Problem 2.
In an acute-angled triangle ABC the internal bisector of angle A meets the circumcircle
of the triangle again at A1 . Points B1 and C1 are defined similarly. Let A0 be the point
of intersection of the line AA1 with the external bisectors of angles B and C. Points B0
and C0 are defined similarly. Prove that
(a) the area of the triangle A0 B0 C0 is twice the area of the hexagon AC1 BA1 CB1 ;
(b) the area of the triangle A0 B0 C0 is at least four times the area of the triangle ABC.

30th IMO 1989, Braunschweig, Germany. Problem 3.


Let n and k be positive integers and let S be a set of n points in the plane such that
(a) no three points of S are collinear, and
(b) for every point P of S there are at least k points of S equidistant from P .
Prove that
1 √
k < + 2n.
2
30th IMO 1989, Braunschweig, Germany. Problem 4.
Let ABCD be a convex quadrilateral such that the sides AB, AD, BC satisfy AB =
AD + BC. There exists a point P inside the quadrilateral at a distance h from the line
CD such that AP = h + AD and BP = h + BC. Show that
1 1 1
√ ≥√ +√ .
h AD BC
31st IMO 1990, Beijing, China. Problem 2.
Let n ≥ 3 and consider a set E of 2n − 1 distinct points on a circle. Suppose that exactly
k of these points are to be colored black. Such a coloring is good if there is at least one pair
of black points such that the interior of one of the arcs between them contains exactly n
points from E. Find the smallest value of k so that every such coloring of k points of E
is good.

32nd IMO 1991, Sigtuna, Sweden. Problem 1.


Given a triangle ABC, let I be the center of its inscribed circle. The internal bisectors of
the angles A, B, C meet the opposite sides in A0 , B 0 , C 0 , respectively. Prove that
1 AI · BI · CI 8
< 0 0 0
≤ .
4 AA · BB · CC 27
32nd IMO 1991, Sigtuna, Sweden. Problem 3.
Let S = {1, 2, 3, . . . , 280}. Find the smallest integer n such that each n-element subset of
S contains five numbers that are pairwise relatively prime.

32nd IMO 1991, Sigtuna, Sweden. Problem 5.


Let ABC be a triangle and P an interior point in ABC. Show that at least one of the
angles ∠P AB, ∠P BC, ∠P CA is less than or equal to 30 ◦.

32nd IMO 1991, Sigtuna, Sweden. Problem 6.


An infinite sequence x0 , x1 , x2 , . . . of real numbers is said to be bounded if there is a

151
constant C such that |xi | ≤ C for every i ≥ 0.
Given any real number a > 1, construct a bounded infinite sequence x0 , x1 , x2 , . . . such
that

|xi − xj ||i − j|a ≥ 1

for every pair of distinct nonnegative integers i, j.

33rd IMO 1992, Moscow, Russia. Problem 5.


Let S be a finite set of points in three-dimensional space. Let Sx , Sy , Sz be the sets
consisting of the orthogonal projections of the points of S onto the yz-plane, zx-plane,
xy-plane, respectively. Prove that

|S|2 ≤ |Sx | · |Sy | · |Sz |,

where |A| denotes the number of elements in the finite set A. (Note: the orthogonal
projection of a point onto a plane is the foot of the perpendicular from the point to the
plane.)

33rd IMO 1992, Moscow, Russia. Problem 6.


For each positive integer n, S(n) is defined to be the greatest integer such that, for every
positive integer k ≤ S(n), n2 can be written as the sum of k positive square integers.
(a) Prove that S(n) ≤ n2 − 14 for each n ≥ 4.
(b) Find an integer n such that S(n) = n2 − 14.
(c) Prove that there exist infinitely many positive integers n such that S(n) = n 2 − 14.

34th IMO 1993, Istanbul, Turkey. Problem 4 (Proposed by Macedonia).


For three points P, Q, R in the plane, we define m(P QR) to be the minimum of the lengths
of the altitudes of the triangle P QR (where m(P QR) = 0 when P , Q R are collinear).
Let A, B, C be given points in the plane. Prove that, for any point X in the plane,

m(ABC) ≤ m(ABX) + m(AXC) + m(XBC).

35th IMO 1994, Hong Kong, H. K.. Problem 1 (Proposed by France).


Let m and n be positive integers. Let a1 , a2 , . . . , am be distinct elements of {1, 2, . . . , n}
such that whenever ai + aj ≤ n for some i, j, 1 ≤ i ≤ j ≤ m, there exists k, 1 ≤ k ≤ m,
with ai + aj = ak . Prove that

a1 + a 2 + · · · + a m n+1
≥ .
m 2

36th IMO 1995, Toronto, Canada. Problem 2 (Proposed by Russia).


Let a, b, c be positive real numbers such that abc = 1. Prove that

1 1 1 3
+ 3 + 3 ≥ .
a3 (b+ c) b (c + a) c (a + b) 2

152
36th IMO 1995, Toronto, Canada. Problem 4 (Proposed by Poland).
Find the maximum value of x0 for which there exists a sequence of positive real numbers
x0 , x1 , . . . , x1995 satisfying the two conditions:

(a) x0 = x1995 ;
2 1
(b) xi−1 + = 2xi + for each i = 1, 2, . . . , 1995.
xi−1 xi

36th IMO 1995, Toronto, Canada. Problem 5 (Proposed by New Zealand).


Let ABCDEF be a convex hexagon with

AB = BC = CD, DE = EF = F A and ∠BCD = ∠EF A = 60 ◦.

Let G and H be two points in the interior of the hexagon such that ∠AGB = ∠DHE =
120 ◦. Prove that

AG + GB + GH + DH + HE ≥ CF.

37th IMO 1996, Mumbai, India. Problem 4 (Proposed by Russia).


The positive integers a and b are such that the numbers 15a + 16b and 16a − 15b are
both squares of positive integers. Find the least possible value that can be taken by the
minimum of these two squares?

37th IMO 1996, Mumbai, India. Problem 5 (Proposed by Armenia).


Let ABCDEF be a convex hexagon such that AB is parallel to ED, BC is parallel to
F E, and CD is parallel to AF . Let RA , RC , RE denote the circumradii of triangles F AB,
BCD, DEF respectively, and let p denote the perimeter of the hexagon. Prove that
p
RA + R C + R E ≥ .
2
38th IMO 1997, Mar del Plata, Argentina. Problem 1 (Proposed by Belarus).
In the plane the points with integer coordinates are the vertices of unit squares. The
squares are colored alternately black and white (as on a chessboard).
For any pair of positive integers m and n, consider a right-angled triangle whose vertices
have integer coordinates and whose legs, of lengths m and n, lie along the edges of the
squares.
Let S1 be the total area of the black part of the triangle and S2 be the total area of the
white part. Let f (m, n) = |S1 − S2 |.
(a) Calculate f (m, n) for all positive integers m and n that are either both even or both
odd.
(b) Prove that f (m, n) ≤ 12 max{m, n} for all m and n.
(c) Show that there is no constant C such that f (m, n) < C for all m and n.

153
38th IMO 1997, Mar del Plata, Argentina. Problem 3 (Proposed by Russia).
Let x1 , x2 , . . . , xn be real numbers satisfying the conditions
|x1 + x2 + · · · + xn | = 1
and
n+1
|xi | ≤ for i = 1, 2, . . . , n.
2
Show that there exists a permutation y1 , y2 , . . . , yn of x1 , x2 , . . . , xn such that
n+1
|y1 + 2y2 + · · · + nyn | ≤ .
2
38th IMO 1997, Mar del Plata, Argentina. Problem 6 (Proposed by Lithuania).
For each positive integer n, let f (n) denote the number of ways of representing n as a
sum of powers of 2 with nonnegative integer exponents. Representations that differ only
in the ordering of their summands are considered to be the same. For instance, f (4) = 4,
because the number 4 can be represented in the following four ways:
4; 2 + 2; 2 + 1 + 1; 1 + 1 + 1 + 1.
Prove that, for any integer n ≥ 3,
2 /4 2 /2
2n < f (2n ) < 2n .
39th IMO 1998, Taipei, Taiwan. Problem 2 (Proposed by India).
In a competition, there are a contestants and b examiners, where b ≥ 3 is an odd integer.
Each examiner rates each contestant as either “pass” or “fail”. Suppose k is a number
such that, for any two examiners, their ratings coincide for at most k contestants. Prove
that
k b−1
≥ .
a 2b
39th IMO 1998, Taipei, Taiwan. Problem 6 (Proposed by Bulgaria).
Consider all functions f from the set N of all positive integers into itself satisfying the
equation
f (t2 f (s)) = s(f (t))2
for all s and t in N . Determine the least possible value of f (1998).

40th IMO 1999, Bucharest, Romania. Problem 2 (Proposed by Poland).


Let n be a fixed integer, with n ≥ 2.
(a) Determine the least constant C such that the inequality
X
xi xj (x2i + x2j ) ≤ C (x1 + x2 + · · · + xn )4
1≤i<j≤n

holds for all real numbers x1 , . . . , xn ≥ 0.


(b) For this constant C, determine when equality holds.
(For a solution, see BS00, p. 23.)

154
40th IMO 1999, Bucharest, Romania. Problem 3 (Proposed by Belarus).
Consider an n × n square board, where n is a fixed even positive integer. The board is
divided into n2 unit squares. We say that two different squares on the board are adjacent
if they have a common side.
N unit squares on the board are marked in such a way that every square (marked or
unmarked) on the board is adjacent to at least one marked square.
Determine the smallest possible value of N .

41st IMO 2000, Taejon, Republic of Korea. Problem 2 (Proposed by U. S. A.).


Let a, b, c be positive real numbers such that abc = 1. Prove that
µ ¶µ ¶µ ¶
1 1 1
a−1+ b−1+ c−1+ ≤ 1.
b c a

42nd IMO 2001, Washington D.C., U.S.A. Problem 1 (Proposed by South Korea).
Let ABC be an acute-angled triangle with circumradius O. Let P on BC be the foot of the
altitude from A. Suppose that ∠BCA ≥ ∠ABC +30 ◦. Prove that ∠CAB +∠COP < 90 ◦.

42nd IMO 2001, Washington D.C., U.S.A. Problem 2 (Proposed by South Korea).
Prove that
a b c
√ +√ +√ ≥1
a2 + 8bc 2
b + 8ca 2
c + 8ab

for all positive real numbers a, b and c.

42nd IMO 2001, Washington D.C., U.S.A. Problem 3 (Proposed by Germany).


Twenty-one girls and twenty-one boys took part in a mathematical contest. Each conte-
stant solved at most six problems. For each girl and each boy, at least one problem was
solved by both of them. Prove that there was a problem that was solved by at least three
girls and at least three boys.

43rd IMO 2002, Glasgow, United Kingdom. Problem 4 (Proposed by Romania).


Let n be an integer greater than 1. The positive divisors of n are d1 , d2 , . . . , dk , where
1 = d1 < d2 < · · · < dk = n. Define D = d1 d2 + d2 d3 + · · · + dk−1 dk .
(a) Prove that D < n2 .
(b) Determine all n for which D is a divisor of n2 .

43rd IMO 2002, Glasgow, United Kingdom. Problem 6 (Proposed by Ukraine).


Let Γ1 , Γ2 , . . . , Γn be circles of radius 1 in the plane, where n ≥ 3. Denote their centres
by O1 , O2 , . . . , On , respectively. Suppose that no line meets more than two of the circles.
Prove that
X 1 (n − 1)π
≤ .
1≤i<j≤n
O i O j 4

155
44th IMO 2003, Tokyo, Japan. Problem 5 (Proposed by Ireland).
Let n be a positive integer and x1 , x2 , . . . , xn be real numbers with x1 ≤ x2 ≤ · · · ≤ xn .
(a) Prove that

à n X
n
!2 n n
X 2(n2 − 1) X X
|xi − xj | ≤ (xi − xj )2 .
i=1 j=1
3 i=1 j=1

(b) Show that equality holds if and only if x1 , x2 , . . . , xn is an arithmetic sequence.

156
45th IMO 2004, Athens, Greece. Problem 4.
Let n ≥ 3 be an integer. Let t1 , t2 , . . . , tn be positive real numbers such that
µ ¶
2 1 1 1
n + 1 > (t1 + t2 + · · · + tn ) + + ··· + .
t1 t2 tn
Show that ti , tj , tk are side lengths of a triangle for all i, j, k with 1 ≤ i < j < k ≤ n.

to be continued in 2005 . . .

157
Inequalities proposed in
“More Mathematical Morsels”
by R. Honsberger

1, p. 20
If a, b, c, are nonnegative real numbers such that

(1 + a)(1 + b)(1 + c) = 8,

prove that the product abc cannot exceed 1.

2, p. 26
Suppose S is a set of n odd positive integers a1 < a2 < · · · < an such that no two of the
differences |ai − aj | are the same. Prove, then, that the sum Σ of all the integers must be at
least n(n2 + 2)/3.

3, p. 33
For every integer n > 1, prove that
1 1 1 3n
1+ 2
+ 2 + ··· + 2 > .
2 3 n 2n + 1
4, p. 48
A and B play a game on a given triangle P QR as follows. First A chooses a point X on QR;
then B takes his choice of Y on RP , and finally, A chooses Z on P Q. A’s object is to make the
inscribed 4XY Z as large as possible (in area) while B is trying to make it as small as possible.
What is the greatest area that A can be sure of getting?

5, p. 75
S is a set of 1980 points in the plane such that the distance between any two of them is at least
1. Prove that S must√ contain a subset T of 220 points such that the distance between each two
of them is at least 3.

6, p. 86
S is a collection of disjoint intervals in the unit interval [0, 1]. If no two points of S are 1/10th
of a unit apart, prove that the sum of the lengths of all the intervals in S cannot exceed 1/2.

7, p. 87
M is a set of 3n points in the plane such that the maximum distance between any two of the
points is 1 unit. Prove that

(a) for any 4 points of M , the√distance between some two of them is less than or at most 1/ 2,
(b) some circles of radius ≤ 3/2 encloses the entire set M √

(d) there is some pair of the 3n points of M whose distance apart is at most 4/(3 n − 3).

8, p. 119
Suppose x and y vary over the nonnegative real numbers. If the value of
p
x + y + 2x2 + 2xy + 3y 2

is always 4, prove that x2 y is always less than 4.

158
9, p. 125
In the plane, n circles of unit radius are drawn with different centers. Of course, overlapping
circles partly cover each other’s circumferences. A given circle could be so overlaid that any
uncovered parts of its circumference are all quite small; that is, it might have no sizable uncovered
arcs at all. However, this can’t be true of every circle; prove that some circle must have a
continuously uncovered arc which is at least 1/nth of its circumference.

10, p. 147
The first n positive integers (1, 2, 3, . . . , n) are spotted around a circle in any order you wish and
the positive differences d1 , d2 , . . . , dn between consecutive pairs are determined. Prove that, no
matter how the integers might be jumbled up around the circle, the sum of these n differences,

S = d 1 + d2 + · · · + d n ,

will always amount to at least 2n − 2.

11, p. 149
Prove that a regular hexagon H = ABCDEF of side 2 can be covered with 6 disks of unit
radius, but not by 5.

12, p. 153
If 10 points are chosen in a circle C of diameter 5, prove that the distance between some pair of
them is less than 2.

13, p. 163
If a, b, c, d are positive real numbers that add up to 1, prove that
√ √ √ √
S = 4a + 1 + 4b + 1 + 4c + 1 + 4d + 1 < 6.

14, p. 187
Between what two integers does the sum S lie, where S is the unruly sum

10 9
X 2 1 1 1
S= n− 3 = 1 + √
3 2
+√
3 2
+ ··· + p ?
2 3 3
(10 9 )2
n=1

15, p. 195
Prove that the positive root of

x(x + 1)(x + 2) · · · (x + 1981) = 1

is less than 1/1981!.

16, p. 195
Let S be a collection of positive integers, not necessarily distinct, which contains the number
68. The average of the numbers in S is 56; however, if a 68 is removed, the average would drop
to 55. What is the largest number that S can possibly contain?

17, p. 198
Prove that, among any seven real numbers y1 , y2 , . . . , y7 , some two, yi and yj , are such that

yi − y j 1
0≤ ≤√ .
1 + y i yj 3

159
18, p. 199
A unit square is to be covered by 3 congruent circular disks.
(a) Show that there are disks of diameter less tahn the diagonal of the square that provide a
covering.
(b) Determine the smallest possible diameter.

19, p. 203
Let x1 , x2 , . . . , xn , where n ≥ 2, be positive numbers that add up to 1. Prove that
x1 x2
S= + + ··· +
1 + x 2 + x3 + · · · + x n 1 + x 1 + x3 + · · · + x n
xn n
≥ .
1 + x1 + x2 + · · · + xn−1 2n − 1
20, p. 205
If the positive real numbers x1 , x2 , . . . , xn+1 are such that
1 1 1
+ + ··· + = 1,
1 + x1 1 + x2 1 + xn+1
prove that

x1 x2 · · · xn+1 ≥ nn+1 .

21, p. 244
If 0 ≤ a, b, c, d ≤ 1, prove that

(1 − a)(1 − b)(1 − c)(1 − d) + a + b + c + d ≥ 1.

22, p. 246
Determine an experiment in probability to justify the inequality

(1 − pm )n + (1 − q n )m > 1

for all positive integers m and n greater than 1 and all positive real numbers p and q such that
p + q ≤ 1.

23, p. 283
C1 C2
Let the fixed point P be taken anywhere inside the lens-
shaped region of intersection R of two given circles C1 and P U
C2 . Let U V be a chord of R through P . Determine how V
R
to construct the chord which makes the product P U · P V
a minimum.

160
161
Inequalities proposed in
“Old and New Inequalities”
by T. Andreescu, V. Cı̂rtoaje, G. Dospinescu, M. Lascu

1. Kömal
Prove the inequality

p p p 3 2
a2 + (1 − b)2 + b2 + (1 − c)2 + c2 + (1 − a)2 ≥
2
holds for arbitrary real numbers a, b.

2. Junior TST 2002, Romania, [Dinu Şerbănescu]


If a, b, c ∈ (0, 1) prove that
√ p
abc + (1 − a)(1 − b)(1 − c) < 1.
3. Gazeta Matematică, [Mircea Lascu]
Let a, b, c be positive real numbers such that abc = 1. Prove that
b+c c+a a+b √ √ √
√ + √ + √ ≥ a + b + c + 3.
a b c
4. Tournament of the Towns, 1993
If the equation x4 + ax3 + 2x2 + bx + 1 = 0 has at least one real root, then a2 + b2 ≥ 8.

5.
Find the maximum value of the expression x3 + y 3 + z 3 − 3xyz where x2 + y 2 + z 2 = 1
and x, y, z are real numbers.

6. Ukraine, 2001
Let a, b, c, x, y, z be positive real numbers such that x + y + z = 1. Prove that
p
ax + by + cz + 2 (xy + yz + zx)(ab + bc + ca) ≤ a + b + c.
7. [Darij Grinberg]
If a, b, c are three positive real numbers, then
a b c 9
+ + ≥ .
(b + c)2 (c + a)2 (a + b)2 4 (a + b + c)
8. Gazeta Matematică, [Hojoo Lee]
Let a, b, c ≥ 0. Prove that
√ √ √
a 4 + a 2 b 2 + b 4 + b 4 + b 2 c 2 + c 4 + c 4 + c 2 a2 + a 4 ≥
√ √ √
≥ a 2a2 + bc + b 2b2 + ca + c 2c2 + ab.

9. JBMO 2002 Shortlist


If a, b, c are positive real numbers such that abc = 2, then
√ √ √
a3 + b3 + c3 ≥ a b + c + b c + a + c a + b.
When does equality hold?

162
10. Gazeta Matematică, [Ioan Tomescu]
Let x, y, z > 0. Prove that
xyz 1
≤ 4.
(1 + 3x)(x + 8y)(y + 9z)(z + 6) 7
When do we have equality?

11. [Mihai Piticari, Dan Popescu]


Prove that
5 (a2 + b2 + c2 ) ≤ 6 (a3 + b3 + c3 ) + 1,
for all a, b, c > 0 with a + b + c = 1.

12. [Mircea Lascu]


Let x1 , x2 , . . . , xn ∈ R, n ≥ 2 and a > 0 such that
a2
x1 + x 2 + · · · + x n = a and x21 + x22 + · · · + x2n ≤ .
n−1
£ ¤
Prove that xi ∈ 0, 2a
n
, for all i ∈ {1, 2, . . . , n}.

13. [Adrian Zahariuc]


Prove that for any a, b, c ∈ (1, 2) the following inequality holds
√ √ √
b a c b a c
√ √ + √ √ + √ √ ≥ 1.
4b c − c a 4c a − a b 4a b − b c
14.
For positive real numbers a, b, c such that abc ≤ 1, prove that
a b c
+ + ≥ a + b + c.
b c a
15. [Vasile Cirtoaje, Mircea Lascu]
Let a, b, c, x, y, z be positive real numbers such that a + x ≥ b + y ≥ c + z and a + b + c =
x + y + z. Prove that ay + bx ≥ ac + xz.

16. Junior TST 2003, Romania, [Vasile Cirtoaje, Mircea Lascu]


Let a, b, c be positive real numbers so that abc = 1. Prove that
3 6
1+ ≥ .
a+b+c ab + ac + bc
17. JBMO 2002 Shortlist
Let a, b, c be positive real numbers. Prove that
a3 b 3 c 3 a2 b 2 c 2
+ + ≥ + + .
b 2 c 2 a2 b c a
18. Russia 2004
Prove that if n > 3 and x1 , x2 , . . . , xn > 0 have product 1, then
1 1 1
+ + ··· + > 1.
1 + x 1 + x 1 x2 1 + x 2 + x 2 x3 1 + x n + x n x1

163
19. [Marian Tetiva]
Let x, y, z be positive real numbers satisfying the condition

x2 + y 2 + z 2 + 2xyz = 1.
Prove that
1
(a) xyz ≤ ;
8
3
(b) xy + xz + yz ≤ ≤ x2 + y 2 + z 2 ;
4
1
(c) xy + xz + yz ≤ + 2xyz.
2
20. Gazeta Matematică, [Marius Olteanu]
Let x1 , x2 , x3 , x4 , x5 ∈ R so that x1 + x2 + x3 + x4 + x5 = 0. Prove that

| cos x1 | + | cos x2 | + | cos x3 | + | cos x4 | + | cos x5 | ≥ 1.

21. [Florina Cárlan, Marian Tetiva]


Prove that if x, y, z > 0 satisfy the condition x + y + z = xyz then
√ p √
xy + xz + yz ≥ 3 + x2 + 1 + y 2 + 1 + z 2 + 1.

22. JBMO, 2003, [Laurenţiu Panaitopol]


Prove that
1 + x2 1 + y2 1 + z2
+ + ≥ 2,
1 + y + z 2 1 + z + x2 1 + x + y 2
for any real numbers x, y, z > −1.

23.
Let a, b, c > 0 with a + b + c = 1. Show that
a2 + b b 2 + c c 2 + a
+ + ≥ 2.
b+c c+a a+b
24. Kvant, 1988
Let a, b, c ≥ 0 such that a4 + b4 + c4 ≤ 2(a2 b2 + b2 c2 + c2 a2 ). Prove that

a2 + b2 + c2 ≤ 2 (ab + bc + ca).

25. Vietnam, 1998


Let n ≥ 2 and x1 , . . . , xn be positive real numbers satisfying
1 1 1 1
+ + ··· + = .
x1 + 1998 x2 + 1998 xn + 1998 1998
Prove that

n
x 1 x2 · · · x n
≥ 1998.
n−1

164
26. [Marian Tetiva]
Consider positive real numbers x, y, z so that
x2 + y 2 + z 2 = xyz.
Prove the following inequalities

a) xyz ≥ 27;
b) xy + xz + yz ≥ 27;
c) x + y + z ≥ 9;
d) xy + xz + yz ≥ 2 (x + y + z) + 9.

27. Russia, 2002


Let x, y, z be positive real numbers with sum 3. Prove that
√ √ √
x + y + z ≥ xy + yz + zx.
28. Gazeta Matematică, [D. Olteanu]
Let a, b, c be positive real numbers. Prove that
a+b a b+c b c+a c 3
· + · + · ≥ .
b + c 2a + b + c c + a 2b + c + a a + b 2c + a + b 4
29. India, 2002
For any positive real numbers a, b, c show that the following inequality holds
a b c c+a a+b b+c
+ + ≥ + + .
b c a c+b a+c b+a
30. Proposed for the Balkan Mathematical Olympiad
Let a, b, c be positive real numbers. Prove that
a3 b3 c3 3 (ab + bc + ca)
2 2
+ 2 2
+ 2 2
≥ .
b − bc + c c − ac + a a − ab + b a+b+c
31. [Adrian Zahariuc]
Consider the pairwise distinct integers x1 , x2 , . . . , xn , n ≥ 0. Prove that
x21 + x22 + · · · + x2n ≥ x1 x2 + x2 x3 + · · · + xn x1 + 2n − 3.
32. Crux Mathematicorum, [Murray Klamkin]
Find the maximum value of the expression
x21 x2 + x22 x3 + · · · + x2n−1 xn + x2n x1
when x1 , x2 , . . . , xn−1 , xn ≥ 0 add up to 1 and n > 2.

33. IMO Shortlist, 1986


Find the maximum value of the constant c such that for any x1 , x2 , . . . , xn , . . . > 0 for
which xk+1 ≥ x1 + x2 + · · · + xk for any k, the inequality
√ √ √ √
x1 + x2 + · · · + xn ≤ c x1 + x 2 + · · · + x n
also holds for any n.

165
34. Russia, 2002
Given are positive real numbers a, b, c and x, y, z, for which a + x = b + y = c + z = 1.
Prove that µ ¶
1 1 1
(abc + xyz) + + ≥ 3.
ay bz cx
35. Gazeta Matematică, [Viorel Vâjâitu, Alexandru Zaharescu]
Let a, b, c be positive real numbers. Prove that
ab bc ca 1
+ + ≤ (a + b + c).
a + b + 2c b + c + 2a c + a + 2b 4
36.
Find the maximum value of the expression
a3 (b + c + d) + b3 (c + d + a) + c3 (d + a + b) + d3 (a + b + c),
where a, b, c, d are real numbers whose sum of squares is 1.

37. Crux Mathematicorum 1654, [Walther Janous]


Let x, y, z be positive real numbers. Prove that
x y z
p + p + p ≤ 1.
x + (x + y)(x + z) y + (y + z)(y + x) z + (z + x)(z + y)
38. Iran, 1999
Suppose that a1 < a2 < · · · < an are real numbers for some integer n ≥ 2. Prove that
a1 a42 + a2 a43 + · · · + an a41 ≥ a2 a41 + a3 a42 + · · · + a1 a4n .
39. [Mircea Lascu]
Let a, b, c be positive real numbers. Prove that
µ ¶
b+c c+a a+b a b c
+ + ≥4 + + .
a b c b+c c+a a+b
40.

Let a1 , a2 , . . . , an > 1 be positive integers. Prove
√ that at least one of the numbers a1 a2 ,
a√ √ √
2
a3 , . . . , an−1 an , an a1 is less than or equal 3 3.

41. [Mircea Lascu, Marian Tetiva]


Let x, y, z be positive real numbers which satisfy the condition
xy + xz + yz + 2xyz = 1.
Prove that the following inequalities hold
1
a) xyz ≤ ;
8
3
b) x + y + z ≥ ;
2
1 1 1
c) + + ≥ 4 (x + y + z);
x y z
1 1 1 (2z − 1)2
d) + + − 4 (x + y + z) ≥ , where z = max{x, y, z}.
x y z z(2z + 1)

166
42. [Manlio Marangelli]
Prove that for any positive real numbers x, y, z,
3 (x2 y + y 2 z + z 2 x)(xy 2 + yz 2 + zx2 ) ≥ xyz(x + y + z)3 .
43. [Gabriel Dospinescu]
Prove that if a, b, c are real numbers such that max{a, b, c} − min{a, b, c} ≤ 1, then
1 + a3 + b3 + c3 + 6abc ≥ 3a2 b + 3b2 c + 3c2 a.
44. [Gabriel Dospinescu]
Prove that for any positive real numbers a, b, c we have
µ ¶µ ¶µ ¶ µ ¶
a2 b2 c2 1 1 1
27 + 2 + 2+ 2+ ≥ 6 (a + b + c) + + .
bc ca ab a b c
45. TST Singapore
a2k
Let a0 = 21 and ak+1 = ak + n
. Prove that 1 − 1
n
< an < 1.

46. [Călin Popa]


Let a, b, c be positive real numbers, with a, b, c ∈ (0, 1) such that ab + bc + ca = 1. Prove
that
µ ¶
a b c 3 1 − a2 1 − b2 1 − c 2
+ + ≥ + + .
1 − a2 1 − b2 1 − c 2 4 a b c
47. [Titu Andreescu, Gabriel Dospinescu]
Let x, y, z ≤ 1 and x + y + z = 1. Prove that
1 1 1 27
2
+ 2
+ 2
≤ .
1+x 1+y 1+z 10
48. [Gabriel Dospinescu]
√ √ √
Prove that if x + y + z = 1, then
(1 − x)2 (1 − y)2 (1 − z)2 ≥ 215 xyz(x + y)(y + z)(z + x).
49.
Let x, y, z be positive real numbers such that xyz = x + y + z + 2. Prove that
(1) xy + yz + zx ≥ 2 (x + y + z);
√ √ √ 3√
(2) x + y + z ≤ xyz.
2
50. IMO Shortlist, 1987
Prove that if x, y, z are real numbers such that x2 + y 2 + z 2 = 2, then
x + y + z ≤ xyz + 2.
51. [Titu Andreescu, Gabriel Dospinescu]
Prove that for any x1 , x2 , . . . , xn ∈ (0, 1) and for any permutation σ of the set {1, 2, . . . , n},
we have the inequality
 n 
P
n xi  Ã X n
!
X 1  1
≥ 1 + i=1  · .
i=1
1 − x i
 n i=1
1 − x i · x σ(i)

167
52. Vojtech Jarnik
n
P 1
Let x1 , x2 , . . . , xn be positive real numbers such that 1+xi
= 1. Prove that
i=1

n n
X √ X 1
xi ≥ (n − 1) √ .
i=1 i=1
xi

53. USAMO, 1999, [Titu Andreescu]


Let n > 3 and a1 , a2 , . . . , an be real numbers such that

a1 + a 2 + · · · + a n ≥ n and a21 + a22 + · · · + a2n ≥ n2 .

Prove that max{a1 , a2 , . . . , an } ≥ 2.

54. [Vasile Cı̂rtoaje]


If a, b, c, d are positive real numbers, then
a−b b−c c−d d−a
+ + + ≥ 0.
b+c c+d d+a a+b
55. France, 1996
If x and y are positive real numbers, show that xy + y x > 1.

56. MOSP, 2001


Prove that if a, b, c > 0 have product 1, then

(a + b)(b + c)(c + a) ≥ 4 (a + b + c − 1).

57.
Prove that for any a, b, c > 0,

(a2 + b2 + c2 )(a + b − c)(b + c − a)(c + a − b) ≤ abc(ab + bc + ca).

58. Kvant, 1988, [D. P. Mavlo]


Let a, b, c > 0. Prove that
1 1 1 a b c (a + 1)(b + 1)(c + 1)
3+a+b+c+ + + + + + ≥3 .
a b c b c a 1 + abc
59. [Gabriel Dospinescu]
Prove that for any positive real numbers x1 , x2 , . . . , xn with product 1 we have the
inequality
n
à n n
!n
Y X X 1
nn · (xni + 1) ≥ xi + .
i=1 i=1 i=1
x i

60. Kvant, 1993


Let a, b, c, d > 0 such that a + b + c = 1. Prove that
½ ¾
3 3 3 1 1 d
a + b + c + abcd ≥ min , + .
4 9 27

168
61. AMM
Prove that for any real numbers a, b, c we have the inequality
X
(1 + a2 )2 (1 + b2 )2 (a − c)2 (b − c)2 ≥ (1 + a2 )(1 + b2 )(1 + c2 )(a − b)2 (b − c)2 (c − a)2 .
62. [Titu Andreescu, Mircea Lascu]
Let α, x, y, z be positive real numbers such that xyz = 1 and α ≥ 1. Prove that
xα yα zα 3
+ + ≥ .
y+z z+x x+y 2
63. Korea, 2001
Prove that for any real numbers x1 , . . . , xn , y1 , . . . , yn such that x21 + · · · + x2n = y12 + · · · +
yn2 = 1, Ã !
n
X
(x1 y2 − x2 y1 )2 ≤ 2 1 − xk yk .
k=1

64. TST Romania, [Laurenţiu Panaitopol]


Let a1 , a2 , . . . , an be pairwise distinct positive integers. Prove that
2n + 1
a21 + a22 + · · · + a2n ≥ (a1 + a2 + · · · + an ).
3
65. [Călin Popa]
Let a, b, c be positive real numbers such that a + b + c = 1. Prove that
√ √ √ √
b c c a a b 3 3
√ √ + √ √ + √ √ ≥ .
a( 3c + ab) b( 3a + bc) c( 3b + ca) 4
66. [Titu Andreescu, Gabriel Dospinescu]
Let a, b, c, d be real numbers such that (1 + a2 )(1 + b2 )(1 + c2 )(1 + d2 ) = 16. Prove that
−3 ≤ ab + bc + cd + da + ac + bd − abcd ≤ 5.
67. APMO, 2004
Prove that
(a2 + 2)(b2 + 2)(c2 + 2) ≥ 9(ab + bc + ca)
for any positive real numbers a, b, c.

68. [Vasile Cı̂rtoaje]


Prove that if 0 < x ≤ y ≤ z and x + y + z = xyz + 2, then
a) (1 − xy)(1 − yz)(1 − xz) ≥ 0;
32
b) x2 y ≤ 1, x3 y 2 ≤ .
27
69. [Titu Andreescu]
Let a, b, c be positive real numbers such that a + b + c ≥ abc. Prove that at least two of
the inequalities
2 3 6 2 3 6 2 3 6
+ + ≥ 6, + + ≥ 6, + + ≥6
a b c b c a c a b
are true.

169
70. [Gabriel Dospinescu, Marian Tetiva]
Let x, y, z > 0 such that x + y + z = xyz. Prove that

(x − 1)(y − 1)(z − 1) ≤ 6 3 − 10.

71. Moldava TST, 2004, [Marian Tetiva]


Prove that for any positive real numbers a, b, c,
¯ 3 ¯
¯ a − b3 b3 − c3 c3 − a3 ¯ (a − b)2 + (b − c)2 + (c − a)2
¯ a+b + b+c + c+a ¯≤ .
¯ ¯
4
72. USAMO, 2004, [Titu Andreescu]
Let a, b, c be positive real numbers. Prove that

(a5 − a2 + 3)(b5 − b2 + 3)(c5 − c2 + 3) ≥ (a + b + c)3 .

73. [Gabriel Dospinescu]


Let n > 2 and x1 , x2 , . . . , xn > 0 such that
à n !à n !
X X 1
xk = n2 + 1.
k=1 k=1
xk

Prove that
à n !à n
!
X X 1 2
x2k > n2 + 4 + .
k=1
x2
k=1 k
n(n − 1)

74. [Gabriel Dospinescu, Mircea Lascu, Marian Tetiva]


Prove that for any positive real numbers a, b, c,

a2 + b2 + c2 + 2abc + 3 ≥ (1 + a)(1 + b)(1 + c).

75. USAMO, 2003, [Titu Andreescu, Zuming Feng]


Let a, b, c be positive real numbers. Prove that
(2a + b + c)2 (2b + c + a)2 (2c + a + b)2
+ + ≤ 8.
2a2 + (b + c)2 2b2 + (c + a)2 2c2 + (a + b)2
76. Austrian-Polish Competition, 1995
Prove that for any positive real numbers x, y and any positive integers m, n,

(n − 1)(m − 1)(xm+n + y m+n ) + (m + n − 1)(xm y n + xn y m )


≥ mn(xm+n−1 y + y m+n−1 x).

77. Crux Mathematicorum 2023, [Waldemar Pompe]


Let a, b, c, d, e be positive real numbers such that abcde = 1. Prove that
a + abc b + bcd c + cde
+ +
1 + ab + abcd 1 + bc + bcde 1 + cd + cdea
d + dea e + eab 10
+ + ≥ .
1 + de + deab 1 + ea + eabc 3

170
78. TST 2003, USA, [Titu¡ Andreescu]
¢
Prove that for any a, b, c ∈ 0, π2 the following inequality holds

sin a · sin(a − b) · sin(a − c) sin b · sin(b − c) · sin(b − a)


+ +
sin(b + c) sin(c + a)
sin c · sin(c − a) · sin(c − b)
≥0
sin(a + b)
79. KMO Summer Program Test, 2001
Prove that if a, b, c are positive real numbers, then
√ √ √ √
a4 + b4 + c4 + a2 b2 + b2 c2 + c2 a2 ≥ a3 b + b3 c + c3 a + ab3 + bc3 + ca3 .
80. [Gabriel Dospinescu, Mircea Lascu]
For a given n > 2 find the smallest constant kn with the property: if a1 , . . . , an > 0 have
product 1, then
a1 a2 a2 a3 an a1
+ 2 + 2 ≤ kn .
(a1 + a2 )(a2 + a1 ) (a2 + a3 )(a3 + a2 ) (an + a1 )(a21 + an )
2 2 2

81. Kvant, 1989, [Vasile Cı̂rtoaje]


For any real numbers a, b, c, x, y, z prove that the inequality holds
p 2
ax + by + cz + (a2 + b2 + c2 )(x2 + y 2 + z 2 ) ≥ (a + b + c)(x + y + z).
3
82. [Vasile Cı̂rtoaje]
Prove that the sides a, b, c of a triangle satisfy the inequality
µ ¶ µ ¶
a b c b c a
3 + + −1 ≥2 + + .
b c a a b c
83. Crux Mathematicorum 2423, [Walther Janous]
Let n > 2 and let x1 , x2 , . . . , xn > 0 add up to 1. Prove that
n µ ¶ Y n µ ¶
Y 1 n − xi
1+ ≥ .
i=1
xi i=1
1 − xi

84. TST 1999, Romania, [Vasile Cı̂rtoaje, Gheorghe Eckstein]


Consider positive real numbers x1 , x2 , . . . , xn such that x1 x2 · · · xn = 1. Prove that
1 1 1
+ + ··· + ≤ 1.
n − 1 + x1 n − 1 + x2 n − 1 + xn
85. USAMO, 2001, [Titu Andreescu]
Prove that for any nonnegative real numbers a, b, c such that a2 + b2 + c2 + abc = 4 we
have
0 ≤ ab + bc + ca − abc ≤ 2.

86. TST 2000, USA, [Titu Andreescu]


Prove that for any positive real numbers a, b, c the following inequality holds
a+b+c √ 3 √ √ √ √ √ √
− abc ≤ max{( a − b)2 , ( b − c)2 , ( c − a)2 }.
3

171
87. [Kiran Kedlaya]
Let a, b, c be positive real numbers. Prove that
√ √ r
a + ab + 3 abc 3 a+b a+b+c
≤ a· · .
3 2 3
88. Vietnamese IMO Training Camp, 1995
Find the greatest constant k such that for any positive integer n which is not a square,
√ √
|(1 + n) sin(π n)| > k.

89. Vietnam, 2004, [Dung Tran Nam]


Let x, y, z > 0 such that (x + y + z)2 = 32xyz. Find the minimum and maximum of
x4 + y 4 + z 4
.
(x + y + z)4
90. Crux Mathematicorum 2393, [George Tsintifas]
Prove that for any a, b, c, d > 0,

(a + b)3 (b + c)3 (c + d)3 (d + a)3 ≥ 16a2 b2 c2 d2 (a + b + c + d)4 .

91. [Titu Andreescu, Gabriel Dospinescu]


Find the maximum value of the expression
(ab)n (bc)n (ca)n
+ + ,
1 − ab 1 − bc 1 − ca
where a, b, c are nonnegative real numbers which add up to 1 and n is some positive
integer.

92.
Let a, b, c be positive real numbers. Prove that
1 1 1 3
+ + ≥ √ √ .
a(1 + b) b(1 + c) c(1 + a) 3
abc (1 + 3 abc)
93. Vietnam, 2002, [Dung Tran Nam]
Prove that for any real numbers a, b, c such that a2 + b2 + c2 = 9,

2 (a + b + c) − abc ≤ 10.

94. [Vasile Cı̂rtoaje]


Let a, b, c be positive real numbers. Prove that
µ ¶µ ¶ µ ¶µ ¶
1 1 1 1
a+ −1 b+ −1 + b+ −1 c+ −1
b c c a
µ ¶µ ¶
1 1
c+ −1 a + − 1 ≥ 3.
a b

95. [Gabriel Dospinescu]


Let n be an integer greater than 2. Find the greatest real number mn and the least real

172
number Mn such that for any positive real numbers x1 , x2 , . . . , xn (with xn = x0 , xn+1 =
x1 ),
n
X xi
mn ≤ ≤ Mn .
i=1
xi−1 + 2 (n − 1)xi + xi+1

96. Gazeta Matematică, [Vasile Cı̂rtoaje]


If x, y, z are positive real numbers, then
1 1 1 9
+ 2 + 2 ≥ .
x2 + xy + y 2 y + yz + z 2 z + zx + x 2 (x + y + z)2
97. Gazeta Matematică, [Vasile Cı̂rtoaje]
For any a, b, c, d > 0 prove that

2 (a3 + 1)(b3 + 1)(c3 + 1)(d3 + 1) ≥ (1 + abcd)(1 + a2 )(1 + b2 )(1 + c2 )(1 + d2 ).

98. Vietnam TST, 1996


Prove that for any real numbers a, b, c,
4
(a + b)4 + (b + c)4 + (c + a)4 ≥ (a4 + b4 + c4 ).
7
99. Bulgaria, 1997
Prove that if a, b, c are positive real numbers such that abc = 1, then
1 1 1 1 1 1
+ + ≤ + + .
1+a+b 1+b+c 1+c+a 2+a 2+b 2+c
100. Vietnam, 2001, [Dung Tran Nam]
Find the minimum value of the expression a1 + 2b + 3c where a, b, c are positive real numbers
such that 21ab + 2bc + 8ca ≤ 12.

101. [Titu Andreescu, Gabriel Dospinescu]


Prove that for any x, y, z, a, b, c > 0 such that xy + yz + zx = 3,
a b c
(y + z) + (z + x) + (x + y) ≥ 3.
b+c c+a a+b
102. Japan, 1997
Let a, b, c be positive real numbers. Prove that
(b + c − a)2 (c + a − b)2 (a + b − c)2 3
+ + ≥ .
(b + c)2 + a2 (c + a)2 + b2 (a + b)2 + c2 5
103. [Vasile Cı̂rtoaje, Gabriel Dospinescu]
Prove that if a1 , a2 , . . . , an ≥ 0 then
µ ¶n
a1 + a2 + · · · + an−1
an1 + an2 + ··· + ann − na1 a2 · · · an ≥ (n − 1) − an
n−1
where an is the least among the numbers a1 , a2 , . . . , an .

173
104. Kvant, [Turkevici]
Prove that for all positive real numbers x, y, z, t,

x4 + y 4 + z 4 + t4 + 2xyzt ≥ x2 y 2 + y 2 z 2 + z 2 t2 + t2 x2 + x2 z 2 + y 2 t2 .

105.
Prove that for any real numbers a1 , a2 , . . . , an the following inequality holds
à n !2 n
X X ij
ai ≤ ai aj .
i=1 i,j=1
i + j − 1

106. TST Singapore


Prove that if a1 , a2 , . . . , an , b1 , b2 , . . . , bn are real numbers between 1001 and 2002, inclusi-
vely, such that a21 + a22 + · · · + a2n = b21 + b22 + · · · + b2n , then we have the inequality
a31 a32 a3 17
+ + · · · + n ≤ (a21 + a22 + · · · + a2n ).
b1 b2 bn 10
107. [Titu Andreescu, Gabriel Dospinescu]
Prove that if a, b, c are positive real numbers which add up to 1, then

(a2 + b2 )(b2 + c2 )(c2 + a2 ) ≥ 8 (a2 b2 + b2 c2 + c2 a2 )2 .

108. Gazeta Matematică, [Vasile Cı̂rtoaje]


If a, b, c, d are positive real numbers such that abcd = 1, then
1 1 1 1
2
+ 2
+ 2
+ ≥ 1.
(1 + a) (1 + b) (1 + c) (1 + d)2
109. Gazeta Matematică, [Vasile Cı̂rtoaje]
Let a, b, c be positive real numbers. Prove that
a2 b2 c2 a b c
2 2
+ 2 2
+ 2 2
≥ + + .
b +c c +a a +b b+c c+a a+b
110. TST 2004, Romania, [Gabriel Dospinescu]
Let a1 , a2 , . . . , an be real numbers and let S be a non-empty subset of {1, 2, . . . , n}. Prove
that
à !2
X X
ai ≤ (ai + · · · + aj )2 .
i∈S 1≤i≤j≤n

111. [Dung Tran Nam]


Let x1 , x2 , . . . , x2004 be real numbers in the interval [−1, 1] such that x31 +x32 +· · ·+x32004 = 0.
Find the maximal value of the x1 + x2 + · · · + x2004 .

112. [Gabriel Dospinescu, Călin Popa]


Prove that if n ≥ 2 and a1 , a2 , . . . , an are real numbers with product 1, then
2n √
a21 + a22 + · · · + a2n − n ≥ · n n − 1 (a1 + a2 + · · · + an − n).
n−1

174
113. Gazeta Matematică, [Vasile Cı̂rtoaje]
If a, b, c are positive real numbers, then
r r r
2a 2b 2c
+ + ≤ 3.
a+b b+c c+a
114. Iran, 1996
Prove the following inequality for positive real numbers x, y, z
µ ¶
1 1 1 9
(xy + yz + zx) 2
+ 2
+ 2
≥ .
(x + y) (y + z) (z + x) 4
115.
Prove that for any x, y in the interval [0, 1],
√ p p √
1 + x2 + 1 + y 2 + (1 − x)2 + (1 − y)2 ≥ (1 + 5)(1 − xy).

116. Miklos Schweitzer Competition, [Suranyi]


Prove that for any positive real numbers a1 , a2 , . . . , an the following inequality holds

(n − 1)(an1 + an2 + · · · + ann ) + na1 a2 · · · an ≥ (a1 + a2 + · · · + an )(an−1


1 + an−1
2 + · · · + ann−1 ).

117. A generalization of Turkevici’s inequality


Prove that for any x1 , x2 , . . . , xn > 0 with product 1,
X n
X
2
(xi − xj ) ≥ x2i − n.
1≤i<j≤n i=1

118. [Gabriel Dospinescu]


Find the minimum value of the expression
n r
X a1 a2 · · · a n
i=1
1 − (n − 1)ai
1
where a1 , a2 , . . . , an < n−1
add up to 1 and n > 2 is an integer.

119. [Vasile Cı̂rtoaje]


Let a1 , a2 , . . . , an < 1 be nonnegative real numbers such that
r √
a21 + a22 + · · · + a2n 3
a= ≥ .
n 3
Prove that
a1 a2 an na
2
+ 2
+ ··· + 2
≥ .
1 − a1 1 − a2 1 − an 1 − a2
120. [Vasile Cı̂rtoaje, Mircea Lascu]
Let a, b, c, x, y, z be positive real numbers such that

(a + b + c)(x + y + z) = (a2 + b2 + c2 )(x2 + y 2 + z 2 ) = 4.

175
Prove that
1
abcxyz < .
36
121. Mathlinks Contest, [Gabriel Dospinescu]
For a given n > 2, find the minimal value of the constant kn , such that if x1 , x2 , . . . , xn > 0
have product 1, then
1 1 1
√ +√ + ··· + √ ≤ n − 1.
1 + k n x1 1 + k n x2 1 + k n xn
122. [Vasile Cı̂rtoaje, Gabriel Dospinescu]
For a given n > 2, find the maximal value of the constant kn such that for any x1 , x2 , . . . , xn >
0 for which x21 + x22 + · · · + x2n = 1 we have the inequality

(1 − x1 )(1 − x2 ) · · · (1 − xn ) ≥ kn x1 x2 · · · xn .

176
Bài tập đa thức dành cho HS thi Quốc gia

1. (VMO 2002). Cho a,b,c laø ba nghieäm thöïc sao cho ña thöùc
Px  x3  ax2  bx  c coù ba nghieäm thöïc.
3


Chöùng minh raèng: 12ab  27 c  6a3  10 a2  2b 2 . 
2. (VMO 2003). Cho hai ña thöùc:
Px  4x3  2x2  15x  9 ; Qx  12 x3  6x2  7 x  1 .
a) Chöùng minh raèng moãi ña thöùc ñeàu coù ba nghieäm phaân bieät.
b) Kyù hieäu  , töông öùng laø nghieäm lôùn nhaát cuûa Px vaø Qx .
Chöùng minh raèng  2  3  2  4 .
x2 x3 xn
3. (Estonia 1996, Crux 2005). Cho fn x  1  x    ...  . Chöùng
2 6 n!
minh raèng fn  x voâ nghieäm neáu n chaün vaø coù duy nhaát moät
nghieäm neáu n leû.
4. (Yugolavia TST 1999, Crux 2005). Cho Px laø moät ña thöùc baäc 2n
sao cho P0  1 vaø Pk  2 k 1 ,k  1,2 n .
Chöùng minh raèng: 2P2n  1  P2n  2  1 .
5. (Crux 2006). Tìm taát caû caùc giaù trò a   ñeå ña thöùc
Px  x3  13x  a coù 3 nghieäm nguyeân.

Ñaùp soá: a  12 .


6. (Irish 13, Crux 2006). Cho ña thöùc Px  a0  a1x  a2 x2  ...  an xn coù
heä soá khoâng aâm. Giaû söû P4  2, P16  8 . Chöùng minh raèng:
P8  4 vaø tìm taát caû caùc ña thöùc khi P8  4 .
1
Ñaùp soá: Vôùi P8  4 thì P x  x .
2
7. (Spanish, Crux 2006). Cho hai ña thöùc:
Px  x4  ax3  bx2  cx  1 ; Qx  x4  cx3  bx2  ax  1 vôùi a,b,c   .

Tìm ñieàu kieän cuûa a,b,c ñeå Px,Qx coù nghieäm chung. Haõy xaùc
ñònh taát caû caùc nghieäm cuûa Px vaø Qx.

Văn Phú Quốc- GV.Trường THPT chuyên Nguyễn Bỉnh Khiêm


 190
Bài tập đa thức dành cho HS thi Quốc gia

Ñaùp soá: * Ñieàu kieän: b  2 ; a  c  0 .


* Nghieäm cuûa Px :
 a  a 2  4  a  a2  4
1,1, , .
2 2
a  a2  4 a  a2  4
Nghieäm cuûa Qx : 1,1, , .
2 2
8. (Singapore 2002, Crux 2006). Bieát ña thöùc Px  x3  ax2  bx  c coù 3
nghieäm nguyeân döông phaân bieät vaø P2002  2001 . Xeùt ña thöùc
 
Qx  x2  2x  2002 bieát P Qx voâ nghieäm. Xaùc ñònh taát caû caùc giaù
trò cuûa a.
Ñaùp soá: a  5951 .
x  y  z  0

9. (VMS 2009). Cho x,y ,z laø caùc soá thöïc thoûa maõn: x 2  y 2  z 2  2 .
x 3  y 3  z 3  0

Chöùng minh raèng vôùi moïi soá töï nhieân n ta luoân coù:
x 2 n 1  y 2 n  1  z 2 n  1  0 .
10. (Moscow 2011). Cho n  3 . Tìm taát caû caùc ña thöùc heä soá thöïc
Px  a0  a1x  ...  an xn , an  0 coù n nghieäm khoâng lôùn hôn 1 vaø
thoûa maõn ñieàu kieän a02  a1an  an2  a0 an1 .
n
Ñaùp soá: Px  an x  1  x  a vôùi a  1 .
11. (VMO 1979). Chöùng minh raèng x  1 toàn taïi tam giaùc maø soá ño 3
caïnh laø: P1 x  x4  x3  2x2  x  1;P2  x  2x3  x2  2x  1;P3  x  x4  1 .
Tìm goùc lôùn nhaát?
Ñaùp soá: Goùc lôùn nhaát laø 1200 .
12. (IMO Shortlist). Cho Px laø moät ña thöùc baäc 3 vôùi heä soá höõu tæ.
Cho daõy höõu tæ qn ñöôïc xaùc ñònh nhö sau: qn  Pqn1,n  1 .
Chöùng minh raèng toàn taïi soá töï nhieân k  1 sao cho vôùi moïi n  1 ta
coù: qn k  qn .
13. (IMO Shortlist). Cho ña thöùc Px thoûa maõn:

Văn Phú Quốc- GV.Trường THPT chuyên Nguyễn Bỉnh Khiêm  191
Bài tập đa thức dành cho HS thi Quốc gia

P0  0,P1  P0,P2  2P1  P0,P3  3 P2  3P1  P0


vaø Pn  4  4Pn  3  6 Pn  2  4Pn  1  Pn .
Chöùng minh raèng: Pn  0,n   .
14. (China TST 2006). Cho haøm soá f :    thoûa maõn ñieàu kieän:
 
f 0  0, f n  n  f f n  1 ,n  1 .

Tìm taát caû caùc ña thöùc g x vôùi heä soá thöïc thoûa maõn
f n   gn ,n   .
n1
Ñaùp soá: gn  vôùi  laø nghieäm döông cuûa phöông

trình x 2  x  1  0 .
15. (VMO 1968). Cho a  b  0 vaø a  b  1 .
Chöùng minh ña thöùc f x  x2  bn x  an coù hai nghieäm phaân bieät
thuoäc khoaûng 1;1 vôùi moïi n nguyeân döông.
16. (Bulgaria 1995). Cho ña thöùc f x  x2  m  4x  m2  3m  3 coù
mx12 mx22 49
nghieäm x1 ,x2 . Chöùng minh raèng: 1    .
1  x1 1  x2 9
17. (Japan 1993). Cho ña thöùc Hx  x3  3x  p coù nghieäm phuï thuoäc
p.
Xeùt haøm f p :
– Neáu H x coù 3 nghieäm thì f p laø tích cuûa 2 nghieäm lôùn nhaát vaø beù
nhaát.
– Neáu H x coù ñuùng 1 nghieäm thì f p laø bình phöông nghieäm ñoù.
Tìm giaù trò lôùn nhaát cuûa f p . Ñaùp soá:
max f p  3 taïi p  0 .
18. (VMS 2005).
a) Toàn taïi hay khoâng ña thöùc Px thoûa maõn:
Px  Px vaø Px  Px x ?

Văn Phú Quốc- GV.Trường THPT chuyên Nguyễn Bỉnh Khiêm


 192
Bài tập đa thức dành cho HS thi Quốc gia

b) Bieát raèng ña thöùc Qx coù tính chaát Qx  Qx,x   . Chöùng
minh raèng Qx  0 x   .
19. (USA TST 2010). Cho Px laø moät ña thöùc vôùi heä soá nguyeân
thoûa maõn P0  0 vaø  P0,P1,...  1 . Chöùng minh raèng coù voâ haïn
soá. Chöùng minh raèng coù voâ haïn soá n sao cho
Pn  P0,Pn  1  P1,...  n .
20. (Vietnam TST 1998). Cho haøm soá f :    sao cho moãi soá döông
c , toàn taïi ña thöùc Pc x sao cho f x  Pc  x  cx1998 ,x   . Chöùng
minh raèng f cuõng laø moät ña thöùc.
21. (Russia 2010). Cho hai soá a,b maø a  ab sao cho ña thöùc
  
Px  x 2  20 ax  10b x 2  20bx  10 a voâ nghieäm. Chöùng minh raèng
20b  a khoâng nguyeân.
22. (Vietnam TST 2009). Cho ña thöùc Px  rx3  qx2  px  1,r  0 chæ coù
moät nghieäm thöïc vaø nghieäm ñoù khoâng phaûi nghieäm boäi. daõy soá
an ñöôïc xaùc ñònh nhö sau:
a0  1, a1  p , a2  p 2  q vaø an 3   pan 2  qan1  ran .
Chöùng minh raèng daõy soá chöùa voâ haïn soá aâm.
23. (Putnam 2008). Cho n  3 . Xeùt hai ña thöùc f x, gx heä soá thöïc sao
cho n ñieåm  f 1;g1, f 2; g2,..., f n; gn trong maët phaúng laø n
ñænh cuûa n  giaùc ñeàu theo thöù töï ngöôïc chieàu kim ñoàng hoà.
Chöùng minh raèng baäc cuûa f x hay g x lôùn hôn n  2 .
24. (Bantic 1998). Cho ña thöùc Pk  x  1  x  x2  ...  xk 1 .
n
 x  1
Chöùng minh raèng: C P x  2
k
n k
n 1
Pn  .
k 1  2 
25. (Bantic 2003). Chöùng minh raèng vôùi moïi nghieäm x cuûa ña thöùc
x 3  px  q vôùi p ,q   ñeàu thoûa maõn: 4qx  p 2 .

Văn Phú Quốc- GV.Trường THPT chuyên Nguyễn Bỉnh Khiêm  193
Bài tập đa thức dành cho HS thi Quốc gia

26. (Bantic 2004). Cho ña thöùc Px vôùi heä soá khoâng aâm. Chöùng minh
 1
raèng neáu PxP   1 ñuùng vôùi moïi x  1 thì cuõng ñuùng vôùi moïi x
x  
döông.
27. (Japan 2008). Cho ña thöùc Px vôùi heä soá nguyeân sao cho Pn2   0 vôùi
moät soá soá n nguyeân khaùc 0. Chöùng minh raèng: Pk 2   1 vôùi moïi
k  * .
28. (IMO Shortlist 2002). Cho ña thöùc Px baäc ba coù heä soá nguyeân.
Giaû söû raèng xPx  yP y vôùi voâ soá caëp soá nguyeân  x,y,x  y .
Chöùng minh Px coù nghieäm nguyeân.
29. (IMO Shortlist 2009). Cho ña thöùc Px khoâng haèng vaø coù heä soá
nguyeân. Chöùng minh raèng khoâng toàn taïi haøm soá T :    sao cho






vôùi moïi soá nguyeân x thoûa maõn T T T ... x  x thì nhaän giaù trò
n

Pn .
30. (Bantic 2008). Cho ña thöùc Px vôùi heä soá nguyeân vaø Px  5 taïi 5
giaù trò nguyeân cuûa x . Chöùng minh raèng khoâng toàn taïi soá nguyeân
x sao cho 6  Px  4 hoaëc 6  P x  16 .
31. (Germany). Giaû söû An laø taäp caùc ña thöùc daïng:
Px  a0  a1x  ...  an xn
vôùi 0  a0  an  a1  an1  ...  an  a n1 .
   
 2  2 

Chöùng minh raèng neáu ña thöùc Px  An,Q x  Am thì ña thöùc
PxQx  An  m .
32. (IMO Shortlist). Cho n ña thöùc: P1 x,P2 x,...,Pn  x heä soá thöïc.
Chöùng minh raèng toàn taïi 6 ña thöùc Ak x,Bk x thoûa heä thöùc sau
ñaây:
n

P x  A x  B x  A x  xB x  A x  xB x .


k 1
k
2 2
1
2
1
2
2
2
2
2
3
2
2

Văn Phú Quốc- GV.Trường THPT chuyên Nguyễn Bỉnh Khiêm


 194
Bài tập đa thức dành cho HS thi Quốc gia

33. (Bulgaria 1998). Cho ña thöùc Pn  x,y,n  1 ñöôïc xaùc ñònh bôûi:
 
P1 x,y  1 vaø Pn 1  x,y   x  y  1y  1Pn x ,y  2  y  y 2 Pn x, y .
Chöùng minh raèng: Pn x, y  Pn  y ,x,x,y   ,n  1 .
34. (China 1996). Cho ña thöùc Px baäc 5 coù 5 nghieäm thöïc phaân bieät.
Tìm soá beù nhaát cuûa caùc heä soá khaùc 0.
Ñaùp soá: 3.
35. (IMO Shortlist). Cho 2n  2 soá ai ,bi thoûa maõn:
0  b0  a0 ,bi  ai ,i  1,n .
Chöùng minh raèng caùc nghieäm neáu coù cuûa ña thöùc
a0 x n  a1 x n1  ...  an coù giaù trò tuyeät ñoái khoâng vöôït quaù nghieäm
döông duy nhaát x0 cuûa phöông trình: b0 x n  b1 x n1  ...  bn  0 .
36. (IMO 1974). Cho ña thöùc Px coù baäc m  0 vaø coù caùc heä soá
nguyeân. Goïi n laø soá taát caû caùc nghieäm nguyeân phaân bieät cuûa hai
phöông trình Px  1 vaø Px  1 . Chöùng minh raèng: n  m  2 .
37. (Hungari 1979). Cho ña thöùc Px coù baäc  2n thoûa maõn ñieàu kieän:
P k  1,k  n,n  1,...,0,1,...,n .

Chöùng minh raèng: Px  2 2 n ,x   n,n .


38. (China TST 2009). Cho f x laø ña thöùc baäc n coù caùc heä soá baèng
1 . Bieát raèng ña thöùc x  1 laø nghieäm boäi caáp m vôùi
m  2k ,k  , k  2 . Chöùng minh n  2 k  1  1 .
39. (VMO 2011). Cho n laø soá nguyeân döông. Chöùng minh raèng ña
thöùc Px,y  xn  xy  y n khoâng theå phaân tích thaønh tích hai ña thöùc
hai bieán heä soá thöïc, khaùc haèng soá.
40. (Indonesia TST 2010). Giaû söû ña thöùc Px  ax3  bx2  cx  d coù 3
nghieäm döông vaø P0  0 . Chöùng minh raèng: 2b 3  9a2 d  7 abc  0 .

Văn Phú Quốc- GV.Trường THPT chuyên Nguyễn Bỉnh Khiêm  195
Bài tập dãy số và giới hạn dành cho hs thi Quốc gia

BÀI TẬP DÃY SỐ


1. Cho daõy soá un xaùc ñònh bôûi: un  n3  2n2  3n  1.n! .
Haõy tính toång S  u1  u2  ...  u2013 . Ñaùp soá:
S  2014 2.2014! 1 .
2n
2. Cho daõy soá un xaùc ñònh bôûi: un  arctan .
n4  n2  2
Haõy tính toång S  u1  u2  ...  u2013 .

Ñaùp soá: S  arctan2013 2  2014  .
4
3. (VMO 2001, baûng B). Cho daõy soá xn ñöôïc xaùc ñònh nhö sau:
2 xn
x1  vaø xn1  vôùi moïi n   * .
3 22n  1xn  1
Haõy tính toång cuûa 2001 soá haïng ñaàu tieân cuûa daõy xn . Ñaùp soá:
4002
.
4003
4. Giaû söû an  1,n vaø lim an  1 .
n

an  an2  ...  ank  k


Cho k    , haõy tính lim .
n an  1
kk  1
Ñaùp soá: .
2
2 2 2
1  1  2  n  1 
5. Vôùi    , haõy tính lim         ...     .
n n  n  n n  
  
1
Ñaùp soá:  2    .
3
6. Cho    . Haõy tính limsinn!  .
n

Ñaùp soá: 0.
7. Tìm giôùi haïn cuûa daõy soá xn bieát:
 1  2  n
xn   1  2 1  2 ...1  2  n  1 .
 n  n   n 
Ñaùp soá: e.

Văn Phú Quốc-GV. Trường THPT chuyên Nguyễn Bỉnh Khiêm 1


Bài tập dãy số và giới hạn dành cho hs thi Quốc gia

n
 1 k
8. Tìm giôùi haïn cuûa daõy soá xn bieát: xn   1   2. Ñaùp
k1  n n 
3
soá: e 2 .
n2
 1
9. Tính giôùi haïn: lim1   .e n .
n 
 n
1

Ñaùp soá: e 2 .
n
k
10. Cho daõy soá Sn xaùc ñònh bôûi: Sn  k cos . Tìm giôùi haïn:
k 1 n
Sn
lim .
n  n2
1
Ñaùp soá: .
2
 
1 1 1
11. Tính lim   ...  . Ñaùp
n 
nn  1 n  1n  2 2 n2 n  1 
 
soá: ln2 .
a1  0
12. Cho daõy soá an xaùc ñònh bôûi:  .
an1  ln1  an ,n  1
nnan  2
Haõy tính: a) lim nan ; b) lim .
n  n  lnn
2
Ñaùp soá: a) 2 b) .
3
1 1 1 1
13. Cho daõy soá an xaùc ñònh bôûi coâng thöùc: an  0
 1  2  ...  n ,
Cn C n Cn Cn
n  1 . Haõy tính lim an .
n 

Ñaùp soá: 2 .
n

14. Vôùi x   ñònh nghóa x  x  x . Haõy tính lim  2  3  .
n

   Ñaùp

soá: 1.
15. Cho hai daõy soá döông an vaø bn thoûa maõn ñieàu kieän:

Văn
 2 Phú Quốc-GV. Trường THPT chuyên Nguyễn Bỉnh Khiêm
Bài tập dãy số và giới hạn dành cho hs thi Quốc gia

an bn
lim  lim  0.
n a1  a2  ...  an n b1  b2  ...  bn
vaø cho daõy cn xaùc ñònh nhö sau: cn  a1bn  a2bn1  ...  anb1 , n   .
cn
Haõy tính lim .
n  c  c  ...  c
1 2 n

Ñaùp soá: 0.

16. Tính lim n!e  n!e .
n

Ñaùp soá: 0.
17. (VMO 1994, baûng A). Cho a  0,1 . Xeùt daõy soá xn ñöôïc xaùc ñònh
bôûi:
x0  a

 4  .
xn   2  arccosxn1  2 arcsin xn1 ,n  1
  
Chöùng minh raèng xn coù giôùi haïn höõu haïn khi n daàn ñeán voâ
cuøng vaø tìm giôùi haïn ñoù.
Ñaùp soá: 1.
18. (VMO 2000, baûng B). Cho soá thöïc c  2 . Daõy soá xn ñöôïc xaây
x  c
 0
döïng theo caùch sau:  neáu caùc bieåu thöùc
xn1  c  c  xn ,n  0
döôùi caên laø khoâng aâm.
Chöùng minh raèng: daõy xn ñöôïc xaùc ñònh vôùi moïi giaù trò n vaø toàn
taïi giôùi haïn höõu haïn vaø toàn taïi giôùi haïn höõu haïn lim xn .
n

1  4c  3
Ñaùp soá: lim xn  .
n 2
19. (VMO 2003, baûng B). Cho soá thöïc   0 vaø cho daõy soá thöïc xn
xaùc ñònh bôûi: x1  0 vaø xn1 xn       1,n  1 .
a) Haõy tìm soá haïng toång quaùt cuûa daõy xn .

Văn Phú Quốc-GV. Trường THPT chuyên Nguyễn Bỉnh Khiêm 3


Bài tập dãy số và giới hạn dành cho hs thi Quốc gia

b) Chöùng minh raèng daõy xn coù giôùi haïn höõu haïn khi n   vaø
tìm giôùi haïn ñoù.
n1
Ñaùp soá: a) – Vôùi   2 thì xn  .
n
 1 n 2    1 n 1    1
     
– Vôùi   2 thì xn   n 1 n
.
1    1
b) – Vôùi   1  1 thì lim xn  1 .
n

– Vôùi   1  1 thì lim    1 .


n 

20. (VMO 2005, baûng B). Cho daõy soá xn xaùc ñònh bôûi:
 4
x1  a  0;  vaø xn1  3 xn3  7 xn2  5 xn ,n  1 .
 3
 
Chöùng minh raèng: daõy xn coù giôùi haïn höõu haïn khi n   vaø
tìm giôùi haïn ñoù. Ñaùp soá: – Neáu a  0
thì lim xn  0
n

4 4
– Neáu a 
thì lim xn 
3 n 3
 4
– Neáu a   0;  thì lim xn  1
 3 n

21. (VMO 2004, baûng A). Xeùt daõy soá thöïc xn ñöôïc xaùc ñònh bôûi:

x1  1 vaø xn1 
2  cos2 x  cos  ,n  1
n
2

trong ñoù  laø moät


2  2cos2 x  2  cos2
n

tham soá thöïc. Haõy xaùc ñònh taát caû caùc giaù trò cuûa  ñeå daõy soá yn
n
1
vôùi y n   2x ,n  1 coù giôùi haïn höõu haïn khi n   vaø haõy tìm
k1 k
1
giôùi haïn cuûa noù. Ñaùp
1
soá:   k ,k   vaø lim yn  .
n 2
22. (VMO 1998, baûng B). Cho soá thöïc a . Xeùt daõy soá xn ñöôïc xaùc

ñònh bôûi: x1  a vaø xn1 



xn xn2  3  ,n  1 .
2
3x  1 n

Văn
 4 Phú Quốc-GV. Trường THPT chuyên Nguyễn Bỉnh Khiêm
Bài tập dãy số và giới hạn dành cho hs thi Quốc gia

Chöùng minh raèng daõy soá treân coù giôùi haïn höõu haïn vaø tìm giôùi haïn
ñoù.
Ñaùp soá: – Neáu a  0 thì lim xn  0
n

– Neáu a  0 thì lim xn  1


n

– Neáu a  0 thì lim xn  1


n

u1  1

23. Cho daõy soá un xaùc ñònh bôûi:  un .
u  ,n  1
 n 1
 u n2  1  2
un 1
Haõy tìm lim .
n un
Ñaùp soá: 2  1 .
24. (Moldova 2011). Cho daõy soá xn xaùc ñònh nhö sau:


x0  1,x1  41 vaø xn 2  3 xn  8 xn2  xn2 1 . 
Tìm soá haïng toång quaùt cuûa daõy soá ñaõ cho vaø chöùng minh raèng
moïi soá haïng cuûa daõy ñeàu laø soá töï nhieân.
n n
1 
    
Ñaùp soá: xn   25  17 3 2  3  25  17 3 2  3  441  .
6 
 n


25. Cho xn laø moät daõy soá bò chaën treân vaø thoûa maõn ñieàu kieän:
1 3
xn  2  xn 1  xn ,n  1 .
4 4
Chöùng minh raèng daõy xn coù giôùi haïn höõu haïn khi n   .
26. (Ñeà nghò OLP 30/4/2000). Cho daõy soá xn thoûa maõn:
0  xn m  xn  xm ,m,n  1 .
x 
Chöùng minh raèng daõy soá  n  coù giôùi haïn höõu haïn.
n
27. (OLP 30/4/2013). Cho daõy soá xn nhö sau:
14xn1  51
x1  1 vaø xn  ,n  2 .
5 xn1  18

Văn Phú Quốc-GV. Trường THPT chuyên Nguyễn Bỉnh Khiêm 5


Bài tập dãy số và giới hạn dành cho hs thi Quốc gia

34  33.3 2012
Tìm x2013 vaø tìm lim xn . Ñaùp soá: x2013  vaø
n 11.3 2012  10
lim xn  3 .
n 

28. (Ñeà nghò OLP 30/4/2000). Cho daõy soá un xaùc ñònh nhö sau:
n
un  4
a1  4 a2  ...  4 an ;0  an  2000.30 4 vôùi n   * .

Chöùng minh raèng daõy soá un hoäi tuï.


29. Cho daõy soá xn xaùc ñònh nhö sau:
1 2
x1  0, x2 
2 3

vaø xn1  1  xn  xn31 ,n  2 . 
Chöùng minh raèng daõy soá xn hoäi tuï vaø tìm giôùi haïn cuûa noù.
5 1
Ñaùp soá: .
2
30. Cho daõy soá xn xaùc ñònh bôûi:
5 20n  21
x1  vaø xn1  xn3  12 xn  ,n  1 .
2 n1
Chöùng minh raèng daõy soá ñaõ cho coù giôùi haïn höõu haïn vaø tìm giôùi haïn
ñoù.
Ñaùp soá: 2.
31. (Ñeà nghò OLP 30/4/2011). Cho daõy soá xn xaùc ñònh bôûi:
2011 2 n  4999
x1  vaø xn1  xn2  2 xn  ,n  1 .
2010 n  2499
Chöùng minh raèng daõy soá xn coù giôùi haïn höõu haïn vaø tìm giôùi haïn
ñoù.
Ñaùp soá: 1.
32. (Ñeà nghò OLP 30/4/2013). Cho daõy soá xn xaùc ñònh nhö sau:

x1  2 vaø xn1  4  8 xn  1 ,n  1 .


Chöùng minh raèng daõy soá ñaõ cho coù giôùi haïn höõu haïn. Tìm giôùi haïn ñoù.
Ñaùp soá: 3.
33. (VMO 2013). Cho daõy soá an xaùc ñònh nhö sau:

Văn
 6 Phú Quốc-GV. Trường THPT chuyên Nguyễn Bỉnh Khiêm
Bài tập dãy số và giới hạn dành cho hs thi Quốc gia

an  2
a1  1 vaø an1  3  ,n  1 .
2 an
Chöùng minh raèng daõy soá coù giôùi haïn höõu haïn vaø tìm giôùi haïn ñoù.
Ñaùp soá: 2.
34. (IMO Shortlist 1987). Cho ak,bk laø hai daõy soá döông thoûa maõn:
1
vôùi moïi n  1 ta coù an  bn vaø cosan x  cosbn x   ,x   . Chöùng
n
a  a
minh raèng daõy  n  coù giôùi haïn höõu haïn vaø tính lim n .
 bn  n  bn
Ñaùp soá: 0.
35. (Ñeà nghò OLP 30/4/2002). Cho a  0 vaø daõy xn xaùc ñònh nhö sau:
1
4

x1  a vaø xn1  log 3 xn3  1  3
 ,n   .
3
Tính lim xn .
n

Ñaùp soá: 2.
x0  2011
36. Cho daõy soá xn xaùc ñònh nhö sau:  .

x  ln 1  e
 n 1
 xn
,n  0
Chöùng minh raèng daõy soá ñaõ cho hoäi tuï vaø tính giôùi haïn cuûa noù.
 5  1
Ñaùp soá: ln .
 2 
 
37. (Vietnam TST 1985). Cho daõy soá xn xaùc ñònh nhö sau:
x1  2,9

x  3  xn
,n  1
 n 1
 xn2  1

Haõy tìm moät soá thöïc naèm beân traùi daõy con x1 , x3 ,x5 ,... vaø naèm beân
phaûi daõy con x2 ,x4 , x6 ,... cuûa daõy xn . Ñaùp soá:
3  5 1 .
2

Văn Phú Quốc-GV. Trường THPT chuyên Nguyễn Bỉnh Khiêm 7


Bài tập dãy số và giới hạn dành cho hs thi Quốc gia

38. (Ñeà nghò OLP 30/4/2006). Cho daõy soá un xaùc ñònh bôûi:

u1  1,u1  3 vaø un2 1  4un 1  u n  u n1  2 un  2 un1  2 ,n  1 .

Chöùng minh raèng daõy un coù giôùi haïn höõu haïn vaø tìm giôùi haïn
cuûa daõy ñoù.
Ñaùp soá: 2.
39. (Ñeà döï bò thi HSG tænh Gia Lai 2010, baûng A). Cho daõy soá xn
x0 , x1 , x2  
xaùc ñònh bôûi:  .
 x x

x  log 5 3 n  4 n 2 ,n  
 n  3
Chöùng minh raèng daõy soá xn coù giôùi haïn höõu haïn vaø tìm giôùi haïn
ñoù.
Ñaùp soá: 2.
40. (Ñeà nghò thi OLP 30/4/2004). Cho daõy soá un xaùc ñònh bôûi:
 1 1
u0  3 ,u1  2
 .
u  1 u 2  3 u ,n  
 n 1 4 n1 4 n

Tìm limun .
n

Ñaùp soá: 1.
41. Chöùng minh raèng vôùi n  3 , phöông trình ln n x  x  x  1 coù ñuùng
hai nghieäm laø un ,vn trong ñoù un  vn ; hôn nöõa limun  1 ,
n

lim vn   .
n

3
42. Cho phöông trình: x  2 x 2  ...  nxn  , n   * . Chöùng minh raèng
4
n   * , phöông trình ñaõ cho coù nghieäm duy nhaát un  0; .  
Chöùng minh raèng daõy un coù giôùi haïn höõu haïn vaø tính giôùi haïn
ñoù.

Văn
 8 Phú Quốc-GV. Trường THPT chuyên Nguyễn Bỉnh Khiêm
Bài tập dãy số và giới hạn dành cho hs thi Quốc gia

1
Ñaùp soá: .
3
1 1 1 1
43. Cho phöông trình:  1  2  ...  n  0, n   * .
x x 1 x 2 x n
a) Chöùng minh raèng vôùi moïi n   * , phöông trình treân luoân coù
nghieäm duy nhaát un  0;1 .

b) Chöùng minh raèng daõy un vôùi un xaùc ñònh ôû caâu a) coù giôùi haïn.
Tìm giôùi haïn ñoù.
Ñaùp soá: b) 0.
       
44. Cho phöông trình: tan x    tan x  2   ...  tan x  n   0 .
 2  2   2 

a) Chöùng minh raèng vôùi moãi n  2 , phöông trình coù moät nghieäm
 
duy nhaát trong khoaûng  0;  . Kyù hieäu nghieäm ñoù laø un .
4  
b) Chöùng minh raèng un coù giôùi haïn höõu haïn khi n   .

45. Cho daõy haøm un x xaùc ñònh nhö sau:

 
u1 x  ux  x2  2 ; un 1 x  u un x ; n  1 .

a) Chöùng minh phöông trình: un  x  0 coù ñuùng 2n nghieäm phaân


bieät.
b) Chæ ra nghieäm lôùn nhaát un cuûa phöông trình naøy vaø tìm limun .
n

Ñaùp soá: b) 2.
46. Cho phöông trình:
1 1 1
  ...   1 , n  *
x 22 1 x  1 33 2 x  n  1 n  1  n  1 n
a) Chöùng minh raèng vôùi n  23 thì phöông trình treân coù nghieäm
döông duy nhaát. Kyù hieäu nghieäm ñoù laø un .

Văn Phú Quốc-GV. Trường THPT chuyên Nguyễn Bỉnh Khiêm 9


Bài tập dãy số và giới hạn dành cho hs thi Quốc gia

b) Chöùng minh raèng daõy soá un coù giôùi haïn höõu haïn khi n  
vaø tìm giôùi haïn ñoù.
Ñaùp soá: b) 1.
47. (Vietnam TST 1990). Cho daõy soá xn thoûa maõn:
x1  x4  1 ; x2  x3  9 vaø xn 4  4 xn xn 1xn 2 xn 3 , n   *
Chöùng minh raèng daõy soá xn coù giôùi haïn höõu haïn vaø tìm giôùi haïn
ñoù.
Ñaùp soá: 3.
x  x2  0;x3  9
48. Cho daõy soá xn ñöôïc xaùc ñònh bôûi  1 .
xn  xn 1  xn 2  3 xn 3 n  1
Tìm lim xn .
n

9
Ñaùp soá: .
2
n
1
49. Daõy soá un ñöôïc xaùc ñònh bôûi coâng thöùc un   2
n  1 . Chöùng
k  1 ( k!)

minh raèng daõy un coù giôùi haïn höõu haïn vaø giôùi haïn ñoù laø moät soá voâ
tyû.
50. (The Winter mathematical competitions in Bulgaria 1999).
Cho daõy an  n  a n2  1,n  1,2,... vôùi a laø tham soá thöïc.
a) Tìm caùc giaù trò a sao cho an hoäi tuï.
b) Tìm caùc giaù trò a sao cho an taêng.
Ñaùp soá: a) a  1 ; b) a  1 .
51. (Balkan 2002). Cho an thoûa maõn ñieàu kieän:
a1  20,a2  30, an 2  3 an1  an ,n  1 .
Tìm taát caû caùc soá nguyeân döông n sao cho 1  5 an an1 laø moät soá
chính phöông.
Ñaùp soá: n  3 .
52. Cho daõy soá an xaùc ñònh bôûi: a0  2;an1  4 an  15 an2  60 ,n  0 .

Văn
 10Phú Quốc-GV. Trường THPT chuyên Nguyễn Bỉnh Khiêm
Bài tập dãy số và giới hạn dành cho hs thi Quốc gia

1
Chöùng minh raèng: soá b  a  8 coù theå bieåu dieãn thaønh toång cuûa
5 2n
ba soá nguyeân döông lieân tieáp vôùi n  1 .
53. (VMO 1989). Xeùt daõy soá Fibonacci 1,1,2,3,5,8,13,...
Ñaët f n  1985n2  1956n  1960 .
a) Chöùng raèng toàn taïi voâ haïn soá F cuûa daõy treân sao cho f F chia
heát cho 1989.
b) Toàn taïi hay khoâng moät soá G cuûa daõy sao cho f(G) + 2 chia heát cho
1989?
54. (VMO 1998 A). Cho daõy soá nguyeân döông ann0 xaùc ñònh bôûi:
a0  20, a1  100, an 2  4 an1  5 an  20 vôùi n  0 .
Tìm soá döông h nhoû nhaát coù tính chaát: an h  an chia heát cho 1998
vôùi moïi n   .
Ñaùp soá: h  108 .
55. (VMO 1995). Moät daõy soá an ñöôïc xaùc ñònh bôûi:
a  9an ,if n is even
a0  1,a1  3,an 2   n1
9 an 1  5 an ,if n is odd
2000
2
Chöùng minh raèng: a
k 1995
k
chia heát cho 20;b) a2 n1 khoâng phaûi laø

moät soá chính phöông n   *


56. (VMO 1987). Cho daõy soá xn,yn xaùc ñònh bôûi:
 
x0  365 ; xn 1  xn xn1986  1  1622 , n  0

vaø y 0  16 ; y n 1  y n  yn3  1  1952 , n  0 .


Chöùng minh raèng: xn  yk  0 , n,k  1 .
57. Cho daõy soá xn xaùc ñònh bôûi: x0  0;x1  1; x2  0 vaø
n
2

 n  1 n  1 n1
xn  3 
n n n

x . 
xn 2  n2  n  1 xn1 

Chöùng minh raèng xn laø soá chính phöông vôùi moïi n  0 .


58. (St. Petersburg City MO 2002).Cho daõy soá an xaùc ñònh bôûi:

Văn Phú Quốc-GV. Trường THPT chuyên Nguyễn Bỉnh Khiêm  11


Bài tập dãy số và giới hạn dành cho hs thi Quốc gia

 an  1
 , if an  1
 2
an  1 
 2 an , if a  1
n
1  an
Cho a0 laø moät soá nguyeân döông, an  2 vôùi moãi n  1;2;...;2011 vaø
a2002  2 . Tính a0 ?
Ñaùp soá: a0  3.22002  1 .
u  0;u2  1
59. Cho daõy soá un xaùc ñònh bôûi:  1 . Chöùng minh raèng:
un 2  un  un 1  1
 
Neáu p  5 laø soá nguyeân toá thì up up  1 chia heát cho p .
60. (BMO 1996, Round 4). Cho daõy soá an xaùc ñònh bôûi:
an n
a1  1;an 1   .
n an
Chöùng minh raèng: an2   n vôùi n  4 (kyù hieäu x laø phaàn nguyeân cuûa
x ).
61. (BMO 2001, Round 4). Cho daõy an thoûa maõn a0  4;a1  22 vaø
an  6 an1  an 2  0 vôùi n  2 . Chöùng minh raèng toàn taïi caùc daõy
yn2  7
xn,yn goàm caùc soá nguyeân döông sao cho an  xn  yn
vôùi moïi

n  0.
1
62. Cho daõy soá an ñöôïc xaùc ñònh bôûi an   1 vôùi moïi n  1,2,3,...
n2
Chöùng minh raèng toàn taïi soá nguyeân döông n0 sao cho vôùi moïi
a2 a3 a
n  n0 thì   ...  n1  n  2009 .
a1 a2 an
u1  2

63. Cho daõy soá un xaùc ñònh nhö sau:  1
un 1  1  u ;n  1,2,...
 n

Goïi p laø soá leû, q laø soá chaün baát kì. Chöùng minh raèng: up  uq .

Văn
 12Phú Quốc-GV. Trường THPT chuyên Nguyễn Bỉnh Khiêm
Bài tập dãy số và giới hạn dành cho hs thi Quốc gia

64. (Taiwan 2000). Cho daõy soá nguyeân n  3 , giaû söû raèng daõy soá thöïc
döông a1 ,a2 ,...,an thoûa maõn ai 1  ai 1  ki ai vôùi daõy k1 ,k2 ,...,kn laø daõy soá
nguyeân döông baát kyø (trong ñoù a0  an vaø an1  a1 ). Chöùng minh
raèng: 2n  k1  k2  ...  kn  3 n .
65. (China 2000). Cho a1 ,a2 ,...,an laø caùc soá khoâng aâm thoûa maõn:
n 
am  n  am  an m,n   .Chöùng minh raèng: am  ma1    1 am vôùi
m
 
moïi n  m .
66. (China MO 2006). Cho daõy soá an xaùc ñònh bôûi:
1 1
a1  , ak  1   ak  k  1,2,...
2 2  ak

Chöùng minh baát ñaúng thöùc sau:


n n
 n   a1  a2  ...  an   1  1  1 
  1      1  1...  1 .
 2a1  a2  ...  an    n   a1  a2   an 
 
67. Cho ba daõy soá nguyeân xn,yn,zn ñöôïc xaùc ñònh nhö sau:
x1  3,y1  4,z1  5

xn1  3 xn  2 zn  1
 ;n  1 .
y n1  3 xn  2 zn  2
z  4 x  3 z  2
 n 1 n n

a) Chöùng minh moãi boä  xn ,yn ,zn  laø moät boä Pythagore.
b) Chöùng minh 2 xn y n  1 laø moät soá chính phöông.
xn
c) Tính xn ,yn , zn theo n . d) Tính lim .
n  z
n

1
Ñaùp soá: d) .
2
68. Cho daõy soá an xaùc ñònh bôûi:

Văn Phú Quốc-GV. Trường THPT chuyên Nguyễn Bỉnh Khiêm  13


Bài tập dãy số và giới hạn dành cho hs thi Quốc gia

a1  1
 n 1 .
an  1.an1  2 an 2  ...  1 n  1a1 ,n  2

a) Chöùng minh raèng: a2  1,a3  3 a2  0,an 2  3 an1  an  0 n  2 .


b) Tìm soá dö trong pheùp chia an cho 3.
c) Xaùc ñònh an theo n . Haõy chæ ra raèng coù nhieàu voâ haïn soá haïng
thuoäc daõy Fibonacci xuaát hieän trong daõy an .
2n
k
d) Chöùng minh raèng: C
k 1
a
2 n k 1
 F2n1 .

e) Chöùng minh raèng: an 1an1  1  an2 ,n  4 .

Văn
 14Phú Quốc-GV. Trường THPT chuyên Nguyễn Bỉnh Khiêm
Bài tập Hình học không gian dành cho HSG thi Quốc gia

BÀI TẬP HÌNH HỌC KHÔNG GIAN, BỒI DƯỠNG HSG THI QUỐC GIA
Văn Phú Quốc-GV. Trường THPT chuyên Nguyễn Bỉnh Khiêm
1. Cho ABC vuoâng taïi A .Treân ñöôøng thaúng d vuoâng goùc vôùi maët phaúng
ABC taïi B ta laáy ñieåm S sao cho: SB  BA  AC  1 .  P laø maët phaúng
song song vôùi caùc caïnh SB vaø AC caét caùc caïnh SA,SC ,BC ,BA laàn löôït taïi
D,E,F ,H .Xaùc ñònh vò trí cuûa maët phaúng  P sao cho dieän tích cuûa töù
giaùc DEFH lôùn nhaát.
2. Cho töù dieän ABCD chæ coù caïnh AD lôùn hôn 1, ñaët BC  x . Tìm x ñeå theå
tích cuûa töù dieän ABCD lôùn nhaát.
3. Cho hình choùp tam giaùc ñeàu S.ABC coù caïnh ñaùy baèng a .
a) Ta xem hình choùp ñaõ cho laø töù dieän SABC coù troïng taâm O ; goïi  laø
goùc giöõa maët phaúng SAB vaø  ABC . Haõy tính cos ñeå O caùch ñeàu taát
caû caùc maët cuûa SABC
  300 . Xeùt maët phaúng P thay ñoåi ñi qua A , sao cho P caét
b) Bieát ASB  
caùc ñoaïn thaúng SB,SC theo thöù töï taïi B,C .Tìm giaù trò nhoû nhaát cuûa
chu vi tam giaùc ABC theo a .
4. Cho maët phaúng  P trong ñoù coù moät ñöôøng thaûng d coá ñònh vaø moät ñieåm
A coá ñònh khoâng thuoäc d . Treân tia Az vuoâng goùc vôùi  P ta laáy moät
ñieåm D coá ñònh. Goùc vuoâng xAy quay quanh A sao cho d caét Ax , Ay laàn
löôït taïi B vaø C . Goïi H laø hình chieáu cuûa A treân BCD , K laø ñieåm ñoái
xöùng cuûa H qua d . Chöùng minh töù giaùc DBKC noäi tieáp trong moät ñöôøng
troøn. Tìm quyõ tích taâm cuûa ñöôøng troøn ñoù.
5. Cho ABCD laø töù dieän ñeàu coù caïnh baèng 1, M vaø N laø hai ñieåm di ñoäng
treân AB, AC sao cho DMN luoân vuoâng goùc vôùi  ABC .Xaùc ñònh vò trí
cuûa M vaø N ñeå töù dieän ADMN coù theå tích lôùn nhaát vaø nhoû nhaát.
6. Cho d vaø  laø hai ñöôøng thaúng cheùo nhau. Goïi A,B laø hai ñieåm coá ñònh
treân d vaø CD  1 (khoâng ñoåi) di ñoäng treân  . Haõy tìm vò trí cuûa CD ñeå
dieän tích toaøn phaàn cuûa töù dieän ABCD laø nhoû nhaát.
7. Xeùt taát caû caùc ABC trong khoâng gian.
a) Vôùi ñieàu kieän naøo cuûa caùc goùc A,B,C trong ABC thì seõ toàn taïi ñieåm
 ,BPC
P trong khoâng gian maø caùc goùc APB  ,CPA
 laø caùc goùc vuoâng.

b) Giaû söû toàn taïi ñieåm P thoaû maõn tính chaát ôû caâu a); goïi d laø ñoä daøi
lôùn nhaát trong ba ñoaïn thaúng PA,PB,PC vaø h laø ñoä daøi ñöôøng cao lôùn
6
nhaát trong ABC . Chöùng minh raèng: hd h .
3

Văn Phú Quốc-GV.Trường THPT chuyên Nguyễn Bỉnh Khiêm


Bài tập Hình học không gian dành cho HSG thi Quốc gia

8. Cho töù dieän SABC coù SA,SB,SC vuoâng goùc vôùi nhau töøng ñoâi moät. Goïi
H ,O laàn löôït laø tröïc taâm vaø taâm ñöôøng troøn ngoaïi tieáp cuûa  ABC
OH 2 1
Chöùng minh raèng: 2
 2 .
SH 4cos AcosBcosC
9. Maët caàu taâm O ngoaïi tieáp töù dieän ABCD . Giaû söû O naèm trong töù dieän
vaø dieän tích cuûa caùc maët töù dieän ABCD ñoái dieän vôùi caùc ñænh A,B,C ,D laàn
löôït laø S1 ,S2 ,S3 ,S4 .Baùn kính caùc ñöôøng troøn ngoaïi tieáp tam giaùc
BCD,CDA,DAB,ABC laàn löôït laø R1 ,R2 , R3 ,R4 .Khoaûng caùch töø taâm caùc ñöôøng
troøn caùc ñöôøng troøn ngoaïi tieáp caùc tam giaùc BCD,CDA,DAB,ABC theo thöù
töï ñeán caùc ñænh A,B,C ,D laàn löôït laø d1 ,d2 ,d3 ,d4 .Ñoä daøi caùc ñöôøng cao cuûa töù
dieän xuaát phaùt töø A,B,C ,D laàn löôït laø h1 , h2 , h3 , h4 .Chöùng minh:
d12  R12 d22  R22 d32  R32 d42  R42 1 1 4 2 2 2
a)    2 ; b) V  [S (d  Ri )]
h12 h22 h32 h42 3 2 i 1 i i

10. Cho töù dieän ABCD coù maët caàu noäi tieáp vaø baùn kính r khoâng ñoåi. Goïi
RA ,RB ,RC ,RD laàn löôït laø baùn kính maët caàu baøng tieáp vôùi ñænh A,B,C ,D cuûa
töù dieän ABCD . Haõy xaùc ñònh töù dieän aáy ñeå RA  RB  RC  RD coù giaù trò
nhoû nhaát.
11. Cho hình laäp phöông ABCD.ABCD caïnh a . Ñieåm M thuoäc ñoaïn BC ,
ñieåm N thuoäc ñoaïn AB . MN taïo vôùi  ABCD moät goùc  . Chöùng minh
a
raèng: MN  .
2cos  sin
12. Trong khoâng gian cho boán tia Ox ,Oy ,Oz,Ot sao cho caùc goùc taïo bôûi hai tia
baát kì baèng nhau. Treân caùc tia Ox ,Oy ,Oz,Ot laàn löôït laáy caùc ñieåm A,B,C ,D .
Chöùng minh raèng vôùi moïi ñieåm M trong khoâng gian ta coù:
MA  MB  MC  MD  OA  OB  OC  OD .
13. Cho hình choùp tam giaùc ñeàu S.ABC . Goïi a laø goùc hôïp bôûi SA vôùi  ABC ;
b laø goùc hôïp bôûi SBC vaø  ABC ; c laø goùc hôïp bôûi SAB vaø SBC vôùi
 
a ,b ,c   0 ;  .
 2
Giaû söû a,b,c thay ñoåi sao cho a  c  2b ñoàng thôøi döïng thieát dieän qua BC
vaø vuoâng goùc vôùi SA taïi D . Chöùng minh raèng:
VSBCD
ñaït giaù trò lôùn nhaát  a,b,c laø moät caáp soá coäng.
VABCD

14. (Mathematics and Youth Magazine 10/275).

Văn Phú Quốc-GV.Trường THPT chuyên Nguyễn Bỉnh Khiêm


Bài tập Hình học không gian dành cho HSG thi Quốc gia

Treân maët phaúng  P cho ñöôøng troøn ñöôøng kính AB . Laáy ñieåm C treân tia
AB sao cho AC  2 AB . Moät ñöôøng thaúng qua C caét ñöôøng troøn taïi M vaø
N . Döïng ñieåm D sao cho DB  AB vaø DB vuoâng goùc vôùi maët phaúng  P .
  sin 2 BDN
Chöùng minh raèng: sin 2 BDM  1
.
2
15. (Mathematics and Youth Magazine 8/271). Goïi S laø dieän tích toaøn phaàn
cuûa töù dieän ABCD . Chöùng minh baát ñaúng thöùc sau:
2 1 2 2 2
AB  AC  AD  2S 3  BC  CD  CD  DB  DB  BC   .
2  
16. (Mathematics and Youth Magazine 10/273).
Cho hình truï T1 . Ta goïi hình truï T2 laø noäi tieáp ngang T1 neáu moãi ñaùy cuûa
T1 chöùa ñuùng moät ñöôøng sinh cuûa T2 vaø maët xung quanh cuûa T1 chöùa
boán ñieåm cuûa ñöôøng troøn ñaùy cuûa T2 . Hình truï T1 phaûi thoûa maõn ñieàu
kieän gì ñeå coù voâ haïn hình truï T1 ,T2 ,...,Tn ,... maø moãi hình truï ñöùng sau noäi
tieáp ngang hình truï tröôùc.
17. (Mathematics and Youth Magazine 8/272).
Xeùt töù dieän A1A2 A3 A4 cuøng ngoaïi tieáp moät maët caàu cho tröôùc. Moãi tieáp
dieän cuûa maët caàu song song vôùi moät maët cuûa töù dieän naøy, caét ra khoûi töù
dieän ñoù moät töù dieän nhoû. Goïi vi i  1,2,3,4 laø theå tích cuûa töù dieän nhoû coù
ñænh Ai vaø V laø theå tích cuûa töù dieän A1A2 A3 A4 . Tìm giaù trò nhoû nhaát cuûa
v1  v2  v3  v4
vaø xaùc ñònh daïng cuûa nhöõng töù dieän A1A2 A3 A4 nhö theá.
V
18. (Mathematics and Youth Magazine 8/263).
Tìm ñieàu kieän caàn vaø ñuû ñoái vôùi töù dieän ABCD sao cho toång khoaûng caùch
töø moät ñieåm M baát kì naèm trong töù dieän ñeán caùc maët cuûa noù laø khoâng
ñoåi.
19. (Mathematics and Youth Magazine 10/285).
Cho naêm ñieåm phaân bieät A1 ,A2 , A3 , A4 ,A5 khoâng ñoàng phaúng nhöng cuøng
naèm treân moät maët caàu. Chöùng minh raèng caùc maët phaúng, moãi maët phaúng ñi
qua troïng taâm cuûa tam giaùc coù caùc ñænh laø ba trong naêm ñænh noùi treân vaø
vuoâng goùc vôùi ñöôøng thaúng noái hai ñieåm coøn laïi thì ñoàng quy.
20. (Mathematics and Youth Magazine 10/280).
Cho töù dieän ABCD coù boán ñöôøng cao caét nhau taïi moät ñieåm H . Chöùng
minh raèng coù moät vaø chæ moät ñieåm M trong khoâng gian thoûa maõn
HG1  HG2  HG3  HG4 trong ñoù G1 ,G2 ,G3 ,G4 laàn löôït laø troïng taâm cuûa caùc
töù dieän MBCD, MCDA, MDAB, MABC .

Văn Phú Quốc-GV.Trường THPT chuyên Nguyễn Bỉnh Khiêm


Bài tập Hình học không gian dành cho HSG thi Quốc gia

21.(Mathematics and Youth Magazine 10/287).


Giaû söû M laø moät ñieåm naèm beân trong töù dieän ABCD . Caùc ñöôøng thaúng
AM ,BM ,CM ,DM theo thöù töï caét caùc maët phaúng BCD,CDA,DAB, ABC
taïi A,B,C,D . Maët phaúng   qua M , song song vôùi maët phaúng BCD laàn
löôït caét AB, AC,AD taïi X ,Y ,Z . Chöùng minh raèng: M laø troïng taâm cuûa
XYZ .
22. (Mathematics and Youth Magazine 10/284).
Cho hình choùp S.ABC . Chöùng minh raèng neáu caùc trung tuyeán cuûa caùc tam
giaùc SAB,SBC ,SCA keû töø ñænh S taïo vôùi caùc caïnh ñaùy AB,BC ,CA nhöõng
goùc khoâng tuø baèng nhau thì dieän tích cuûa moãi maët beân nhoû hôn toång
dieän tích caùc maët beân coøn laïi.
23. (Mathematics and Youth Magazine 10/286).
Cho töù dieän ABCD coù caùc caëp caïnh ñoái dieän baèng nhau. Goïi E laø tieáp
ñieåm cuûa caùc maët BCD vaø maët caàu taâm O noäi tieáp töù dieän. Goïi K laø
tieáp ñieåm cuûa maët BCD vaø maët caàu baøng tieáp töù dieän öùng vôùi ñænh A .
Chöùng minh raèng:
a) K laø tröïc taâm cuûa BCD
b) FA  2EF vôùi F laø giao ñieåm cuûa AE vaø KO .
24. (Mathematics and Youth Magazine 8/261).
Giaû söû moät töù dieän ñeàu ñöôïc phaân chia thaønh moät soá töù dieän nhoû sao
cho toång theå tích caùc hình caàu ngoaïi tieáp caùc töù dieän nhoû baèng theå tích
hình caàu ngoaïi tieáp töù dieän ban ñaàu. Chöùng minh raèng caùc töù dieän nhoû laø
töù dieän ñeàu.
25. (Mathematics and Youth Magazine 10/257).
Cho töù dieän ABCD coù troïng taâm G .Laáy M laø moät ñieåm baát kì trong
 
khoâng gian, N laø ñieåm thoûa maõn ñieàu kieän: MN  4 MG . Chöùng minh
raèng: NA  NB  NC  ND  2MN  MA  MB  MC  MD .
Daáu “=” xaûy ra khi naøo?
26. (Mathematics and Youth Magazine 10/264).
Treân caïnh CD cuûa hình töù dieän ABCD laáy ñieåm N N  C ,D . Kyù hieäu
pXYZ laø chu vi XYZ . Chöùng minh raèng:
a) NC.pDAB  ND.pCAB  CD.p NAB
NC CA2  CB2
b)  khi NA  NB .
ND DA2  DB2
27. (Mathematics and Youth Magazine 10/244).

Văn Phú Quốc-GV.Trường THPT chuyên Nguyễn Bỉnh Khiêm


Bài tập Hình học không gian dành cho HSG thi Quốc gia

Goïi l, R laàn löôït laø toång ñoä daøi caùc caïnh vaø baùn kính maët caàu ngoaïi tieáp
l
töù dieän. Hoûi trong soá caùc töù dieän, töù dieän naøo coù tæ soá ñaït giaù trò lôùn
R
nhaát? Tìm giaù trò lôùn nhaát ñoù.
28. (Mathematics and Youth Magazine 10/252).
Cho töù dieän ABCD vaø ñieåm M thoûa maõn ñieàu kieän:
    
 MA   MB   MC   MD  0 .
Ñöôøng thaúng  baát kì qua M , caét caùc maët phaúng BCD,CDA,DAB, ABC
   
theo thöù töï taïi A1 ,B1 ,C1 ,D1 . Chöùng minh raèng:     0.
MA1 MB1 MC1 MD1
29. (Mathematics and Youth Magazine 10/253).
Cho töù dieän ABCD , troïng taâm G , taâm maët caàu ngoaïi tieáp O . Goïi I laø
ñieåm ñoái xöùng cuûa O qua G . Bieát O naèm trong töù dieän. Chöùng minh
raèng I cuõng naèm trong töù dieän.
30. Cho maët caàu S coù baùn kính R khoâng ñoåi. Moät hình hoäp ABCD.ABCD
ngoaïi tieáp maët caàu S . Giaû söû goùc giöõa hai maët phaúng  AADD vaø
8 R2
CCDD baèng x . Chöùng minh raèng: V 
sin x
.

Văn Phú Quốc-GV.Trường THPT chuyên Nguyễn Bỉnh Khiêm


Bài tập Hình học không gian dành cho HSG thi Quốc gia

Văn Phú Quốc-GV.Trường THPT chuyên Nguyễn Bỉnh Khiêm


Problems in Geometry
Prithwijit De
ICFAI Business School, Kolkata
Republic of India
email: de.prithwijit@gmail.com

Problem 1 [BMOTC]

Prove that the medians from the vertices A and B of triangle ABC are
mutually perpendicular if and only if |BC|2 + |AC|2 = 5|AB|2 .

Problem 2 [BMOTC]

Suppose that ∠A is the smallest of the three angles of triangle ABC. Let D
be a point on the arc BC of the circumcircle of ABC which does not contain
A. Let the perpendicular bisectors of AB, AC intersect AD at M and N
respectively. Let BM and CN meet at T . Prove that BT + CT ≤ 2R where
R is the circumradius of triangle ABC.

Problem 3 [BMOTC]

Let triangle ABC have side lengths a, b and c as usual. Points P and Q
lie inside this triangle and have the properties that ∠BP C = ∠CP A =
∠AP B = 120◦ and ∠BQC = 60◦ + ∠A, ∠CQA = 60◦ + ∠B, ∠AQB =
60◦ + ∠C. Prove that
(|AP | + |BP | + |CP |)3 .|AQ|.|BQ|.|CQ| = (abc)2 .

Problem 4 [BMOTC]

The points M and N are the points of tangency of the incircle of the isosceles
triangle ABC which are on the sides AC and BC. The sides of equal length
are AC and BC. A tangent line t is drawn to the minor arc M N . Suppose
that t intersects AC and BC at Q and P respectively. Suppose that the lines
AP and BQ meet at T .
(a) Prove that T lies on the line segment M N .
(b) Prove that the sum of the areas of triangles AT Q and BT P is
minimized when t is parallel to AB.

Problem 5 [BMOTC]

In a hexagon with equal angles, the lengths of four consecutive edges are 5,
3, 6 and 7 (in that order). Find the lengths of the remaining two edges.

1
Problem 6 [BMOTC]

The incircle γ of triangle ABC touches the side AB at T . Let D be the point
on γ diametrically opposite to T , and let S be the intersection of the line
through C and D with the side AB. Show that |AT | = |SB|.

Problem 7 [BMOTC]

Let S and r be the area and the inradius of the triangle ABC. Let rA denote
the radius of the circle touching the incircle, AB and AC. Define rB and
rC similarly. The common tangent of the circles with radii r and rA cuts a
little triangle from ABC with area SA . Quantities SB and SC are defined in
a similar fashion. Prove that
SA SB SC S
rA
+ rB
+ rC
= r

Problem 8 [BMOTC]

Triangle ABC in the plane Π is said to be good if it has the following property:
for any point D in space, out of the plane Π, it is possible to construct a
triangle with sides of lengths |AD|, |BD| and |CD|. Find all good triangles.

Problem 9 [BMO]

Circle γ lies inside circle θ and touches it at A. From a point P (distinct


from A) on θ, chords P Q and P R of θ are drawn touching γ at X and Y
respectively. Show that ∠QAR = 2∠XAY .

Problem 10 [BMO]

AP , AQ, AR, AS are chords of a given circle with the property that

∠P AQ = ∠QAR = ∠RAS.

Prove that

AR(AP + AR) = AQ(AQ + AS).

Problem 11 [BMO]

The points Q, R lie on the circle γ, and P is a point such that P Q, P R are
0
tangents to γ. A is a point on the extension of P Q and γ is the circumcircle
0
of triangle P AR. The circle γ cuts γ again at B and AR cuts γ at the point
C. Prove that ∠P AR = ∠ABC.

2
Problem 12 [BMO]

In the acute-angled triangle ABC, CF is an altitude, with F on AB and BM


is a median with M on CA. Given that BM = CF and ∠M BC = ∠F CA,
prove that the triangle ABC is equilateral.

Problem 13 [BMO]

A triangle ABC has ∠BAC > ∠BCA. A line AP is drawn so that ∠P AC =


∠BCA where P is inside the triangle. A point Q outside the triangle is
constructed so that P Q is parallel to AB, and BQ is parallel to AC. R is the
point on BC (separated from Q by the line AP ) such that ∠P RQ = ∠BCA.
Prove that the circumcircle of ABC touches the circumcircle of P QR.

Problem 14 [BMO]

ABP is an isosceles triangle with AB=AP and ∠P AB acute. P C is the


line through P perpendicular to BP and C is a point on this line on the
same side of BP as A. (You may assume that C is not on the line AB). D
completes the parallelogram ABCD. P C meets DA at M . Prove that M is
the midpoint of DA.

Problem 15 [BMO]

In triangle ABC, D is the midpoint of AB and E is the point of trisection


of BC nearer to C. Given that ∠ADC = ∠BAE find ∠BAC.

Problem 16 [BMO]

ABCD is a rectangle, P is the midpoint of AB and Q is the point on P D


such that CQ is perpendicular to P D. Prove that BQC is isosceles.

Problem 17 [BMO]

Let ABC be an equilateral triangle and D an internal point of the side BC.
A circle, tangent to BC at D, cuts AB internally at M and N and AC
internally at P and Q. Show that BD + AM + AN = CD + AP + AQ.

Problem 18 [BMO]

Let ABC be an acute-angled triangle, and let D, E be the feet of the per-
pendiculars from A, B to BC and CA respectively. Let P be the point where
the line AD meets the semicircle constructed outwardly on BC and Q be the
point where the line BE meets the semicircle constructed outwardly on AC.
Prove that CP = CQ.

3
Problem 19 [BMO]

Two intersecting circles C1 and C2 have a common tangent which touches


C1 at P and C2 at Q. The two circles intersect at M and N , where N is
closer to P Q than M is. Prove that the triangles M N P and M N Q have
equal areas.

Problem 20 [BMO]

Two intersecting circles C1 and C2 have a common tangent which touches C1


at P and C2 at Q. The two circles intersect at M and N , where N is closer
to P Q than M is. The line P N meets the circle C2 again at R. Prove that
M Q bisects ∠P M R.

Problem 21 [BMO]

Triangle ABC has a right angle at A. Among all points P on the perimeter
of the triangle, find the position of P such that AP + BP + CP is minimized.

Problem 22 [BMO]

Let ABCDEF be a hexagon (which may not be regular), which circumscribes


a circle S. (That is, S is tangent to each of the six sides of the hexagon.)
The circle S touches AB, CD, EF at their midpoints P , Q, R respectively.
Let X, Y , Z be the points of contact of S with BC, DE, F A respectively.
Prove that P Y , QZ, RX are concurrent.

Problem 23 [BMO]

The quadrilateral ABCD is inscribed in a circle. The diagonals AC, BD


meet at Q. The sides DA, extended beyond A, and CB, extended beyond
B, meet at P . Given that CD = CP = DQ, prove that ∠CAD = 60◦ .

Problem 24 [BMO]

The sides a, b, c and u, v, w of two triangles ABC and U V W are related by


the equations
u(v + w − u) = a2
v(w + u − v) = b2
w(u + v − w) = c2
Prove that triangle ABC is acute-angled and express the angles U , V , W in
terms of A, B, C.

4
Problem 25 [BMO]

Two circles S1 and S2 touch each other externally at K; they also touch a
circle S internally at A1 and A2 respectively. Let P be one point of intersec-
tion of S with the common tangent to S1 and S2 at K. The line P A1 meets
S1 again at B1 and P A2 meets S2 again at B2 . Prove that B1 B2 is a common
tangent to S1 and S2 .

Problem 26 [BMO]

Let ABC be an acute-angled triangle and let O be its circumcentre. The


circle through A, O and B is called S. The lines CA and CB meet the
circle S again at P and Q respectively. Prove that the lines CO and P Q are
perpendicular.

Problem 27 [BMO]

Two circles touch internally at M . A straight line touches the inner circle at
P and cuts the outer circle at Q and R. Prove that ∠QM P = ∠RM P .

Problem 28 [BMO]

ABC is a triangle, right-angled at C. The internal bisectors of ∠BAC and


∠ABC meet BC and CA at P and Q, respectively. M and N are the feet
of the perpendiculars from P and Q to AB. Find the measure of ∠M CN .

Problem 29 [BMO]

The triangle ABC, where AB < AC, has circumcircle S. The perpendicular
from A to BC meets S again at P . The point X lies on the segment AC
and BX meets S again at Q. Show that BX = CX if and only if P Q is a
diameter of S.

Problem 30 [BMO]

Let ABC be a triangle and let D be a point on AB such that 4AD = AB.
The half-line l is drawn on the same side of AB as C, starting from D and
making an angle of θ with DA where θ = ∠ACB. If the circumcircle of ABC
meets the half-line l at P , show that P B = 2P D.

5
Problem 31 [BMO]
Let BE and CF be the altitudes of an acute triangle ABC, with E on AC
and F on AB. Let O be the point of intersection of BE and CF . Take any
line KL through O with K on AB and L on AC. Suppose M and N are
located on BE and CF respectively, such that KM is perpendicular to BE
and LN is perpendicular to CF . Prove that F M is parallel to EN .

Problem 32 [BMO]

In a triangle ABC, D is a point on BC such that AD is the internal bisector


of ∠A. Suppose ∠B = 2∠C and CD = AB. Prove that ∠A = 72◦ .

Problem 33 [Putnam]

Let T be an acute triangle. Inscribe a rectangle R in T with one side along


a side of T . Then inscribe a rectangle S in the triangle formed by the side
of R opposite the side on the boundary of T , and the other two sides of T ,
with one side along the side of R. For any polygon X, let A(X) denote the
area of X. Find the maximum value, or show that no maximum exists, of
A(R)+A(S)
A(T )
where T ranges over all triangles and R, S over all rectangles as
above.

Problem 34 [Putnam]

A rectangle, HOM F , has sides HO=11 and OM =5. A triangle ABC has
H as the orthocentre, O as the circumcentre, M the midpoint of BC and F
the foot of the altitude from A. What is the length of BC?

Problem 35 [Putnam]

A right circular cone has base of radius 1 and height 3. A cube is inscribed
in the cone so that one face of the cube is contained in the base of the cone.
What is the side-length of the cube?

Problem 36 [Putnam]

Let A, B and C denote distinct points with integer coordinates in R2 . Prove


that if (|AB| + |BC|)2 < 8[ABC] + 1 then A, B, C are three vertices of a
square. Here |XY | is the length of segment XY and [ABC] is the area of
triangle ABC.

6
Problem 37 [Putnam]
Right triangle ABC has right angle at C and ∠BAC = θ; the point D is
chosen on AB so that |AC| = |AD| = 1; the point E is chosen on BC so
that ∠CDE = θ. The perpendicular to BC at E meets AB at F . Evaluate
limθ→0 |EF |.

Problem 38 [BMO]

Let ABC be a triangle and D, E, F be the midpoints of BC, CA, AB


respectively. Prove that ∠DAC = ∠ABE if, and only if, ∠AF C = ∠ADB.

Problem 39 [BMO]

The altitude from one of the vertex of an acute-angled triangle ABC meets
the opposite side at D. From D perpendiculars DE and DF are drawn to the
other two sides. Prove that the length of EF is the same whichever vertex
is chosen.

Problem 40

Two cyclists ride round two intersecting circles, each moving with a constant
speed. Having started simultaneously from a point at which the circles in-
tersect, the cyclists meet once again at this point after one circuit. Prove
that there is a fixed point such that the distances from it to the cyclists are
equal all the time if they ride: (a) in the same direction (clockwise); (b) in
opposite direction.

Problem 41

Prove that four circles circumscribed about four triangles formed by four
intersecting straight lines in the plane have a common point. (Michell’s
Point).

Problem 42

Given an equilateral triangle ABC. Find the locus of points M inside the
triangle such that ∠M AB + ∠M BC + ∠M CA = π2 .

Problem 43

In a triangle ABC, on the sides AC and BC, points M and N are taken,
respectively and a point L on the line segment M N . Let the areas of the
triangles ABC, AM L and BN L be equal to S, P and Q, respectively. Prove
that

7
1 1 1
S 3 ≥ P 3 + Q3 .

Problem 44
bc+a2
For an arbitrary triangle, prove the inequality bcb+c
cos A
+a < p < a
, where
a, b and c are the sides of the triangle and p its semiperimeter.

Problem 45

Given in a triangle are two sides: a and b (a > b). Find the third side if it is
known that a + ha ≤ b + hb , where ha and hb are the altitudes dropped on
these sides (ha the altitude drawn to the side a).

Problem 46

One of the sides in a triangle ABC is twice the length of the other and
∠B = 2∠C. Find the angles of the triangle.

Problem 47

In a parallelogram whose area is S, the bisectors of its interior angles are


drawn to intersect one another. The area of the quadrilateral thus obtained
is equal to Q. Find the ratio of the sides of the parallelogram.

Problem 48

Prove that if one angle of a triangle is equal to 120◦ , then the triangle formed
by the feet of its angle bisectors is right-angled.

Problem 49

Given a rectangle ABCD where |AB| = 2a, |BC| = a 2. With AB is
diameter a semicircle is constructed externally. Let M be an arbitrary point
on the semicircle, the line M D intersect AB at N , and the line M C at L.
Find |AL|2 + |BN |2 .

Problem 50

Let A, B and C be three points lying on the same line. Constructed on AB,
BC and AC as diameters are three semicircles located on the same side of
the line. The centre of a circle touching the three semicircles is found at a
distance d from the line AC. Find the radius of this circle.

8
Problem 51

In an isosceles triangle ABC, |AC| = |BC|, BD is an angle bisector, BDEF


is a rectangle. Find ∠BAF if ∠BAE = 120◦ .

Problem 52

Let M1 be a point on the incircle of triangle ABC. The perpendiculars to


the sides through M1 meet the incircle again at M2 , M3 , M4 . Prove that the
geometric
√ mean of the six lengths Mi Mj , 1 ≤ i ≤ j ≤ 4, is less than or equal
3
to r 4, where r denotes the inradius. When does the equality hold?

Problem 53 [AMM]

Let ABC be a triangle and let I be the incircle of ABC and let r be the
radius of I. Let K1 , K2 and K3 be the three circles outside I and tangent
to I and to two of the three sides of ABC. Let ri be the radius of Ki for
1 ≤ i ≤ 3. Show that
√ √ √
r = r1 r 2 + r2 r 3 + r3 r 1

Problem 54 [Prithwijit’s Inequality]

In triangle ABC suppose the lengths of the medians are ma , mb and mc


respectively. Prove that
ama +bmb +cmc 1
(a+b+c)(ma +mb +mc )
≤ 3

Problem 55 [Loney]

The base a of a triangle and the ratio r(< 1) of the sides are given. Show
ar
that the altitude h of the triangle cannot exceed 1−r 2 and that when h has

this value the vertical angle of the triangle is 2 − 2 tan−1 r.


π

Problem 56 [Loney]

The internal bisectors of the angles of a triangle ABC meet the sides in D,
2∆abc
E and F . Show that the area of the triangle DEF is equal to (a+b)(b+c)(c+a) .

Problem 57 [Loney]

If a, b, c are the sides of a triangle, λa, λb, λc the sides of a similar triangle
inscribed in the former and θ the angle between the sides a and λa, prove
that 2λ cos θ = 1.

9
Problem 58

Let a, b and c denote the sides of a triangle and a + b + c = 2p. Let G be the
median point of the triangle and O, I and Ia the centres of the circumscribed,
inscribed and escribed circles, respectively (the escribed circle touches the
side BC and the extensions of the sides AB and AC), R, r and ra being
their radii, respectively. Prove that the following relationships are valid:

(a) a2 + b2 + c2 = 2p2 − 2r2 − 8Rr


2 2 2
(b) |OG|2 = R2 − a +b9 +c
2 2
(c) |IG|2 = p +5r 9−16Rr
(d) |OI|2 = R2 − 2Rr
(e) |OIa |2 = R2 + 2Rra
(f) |IIa |2 = 4R(ra − r)

Problem 59

M N is a diameter of a circle, |M N | = 1, A and B are points on the circles


situated on one side of M N , C is a point on the other semicircle. Given: A
is the midpoint of semicircle, M B = 35 , the length of the line segment formed
by the intersection of the diameter M N with the chords AC and BC is equal
to a. What is the greatest value of a?

Problem 60

Given a parallelogram ABCD. A straight line passing through the vertex C


intersects the lines AB and AD at points K and L, respectively. The areas
of the triangles KBC and CDL are equal to p and q, respectively. Find the
area of the parallelogram ABCD.

Problem 61 [Loney]

Three circles, whose radii are a, b and c, touch one another externally and the
tangents at their points of contact meet in a point; prove
q that the distance
abc
of this point from either of their points of contact is a+b+c .

Problem 62 [Loney]

If a circle be drawn touching the inscribed and circumscribed circles of a


triangle and the side BC externally, prove that its radius is ∆a tan2 A2 .

10
Problem 63

Characterize all triangles ABC such that

AIa : BIb : CIc = BC : CA : AB

where Ia ; Ib , Ic are the vertices of the excentres corresponding to A, B, C


respectively.

Problem 64

On the sides AB and BC of triangle ABC, points K and M are chosen such
that the quadrilaterals AKM C and KBM N are cyclic, where
N = AM ∩ CK. If these quadrilaterals have the same circumradii then find
∠ABC.

Problem 65 [AMM]

Let B 0 and C 0 be points on the sides AB and AC, respectively, of a given


triangle ABC, and let P be a point on the segment B 0 C 0 . Determine the
maximum value of
min([BP B 0 ],[CP C 0 ])
[ABC]

where [F ] denotes the area of F .

Problem 66 [AMM]

For each point O on diameter AB of a circle, perform the following construc-


tion. Let the perpendicular to AB at O meet the circle at point P . Inscribe
circles in the figures bounded by the circle and the lines AB and OP . Let
R and S be the points at which the two incircles to the curvilinear trian-
gles AOP and BOP are tangent to the diameter AB. Show that ∠RP S is
independent of the position of O.

Problem 67

Let E be a point inside the triangle ABC such that ∠ABE = ∠ACE. Let F
and G be the feet of the perpendiculars from E to the internal and external
bisectors, respectively, of angle BAC. Prove that the line F G passes through
the mid-point of BC.

11
Problem 68
Let A, B, C and D be points on a circle with centre O and let P be the
point of intersection of AC and BD. Let U and V be the circumcentres
of triangles AP B and CP D, respectively. Determine conditions on A, B,
C and D that make O, U , P and V collinear and prove that, otherwise,
quadrilateral OU P V is a parallelogram.
Problem 69 [AMM]
Let R and r be the circumradius and inradius, respectively of triangle ABC.
(a) Show that ABC has a median whose length is at most 2R − r.
(b) Show that ABC has an altitude whose length is at least 2R − r.
Problem 70 [AMM]
Let ABCD be a convex quadrilateral. Prove that if there is point P in the
interior of ABCD such that
∠P AB = ∠P BC = ∠P CD = ∠P DA = 45◦
then ABCD is a square.
Problem 71 [AMM]
Let M be any point in the interior of triangle ABC and let D, E and F
be points on the perimeter such that AD, BE and CF are concurrent at
M . Show that if triangles BM D, CM E and AM F all have equal areas and
equal perimeters then triangle ABC is equilateral.
Problem 72
The perpendiculars AD, BE, CF are produced to meet the circumscribed
circle in X, Y , Z prove that
AX BY CZ
AD
+ BE
+ CF
=4
Problem 73 [AMM]
Given an odd positive integer n, let A1 , A2 ,...,An be a regular polygon with
circumcircle Γ. A circle Oi with radius r is drawn externally tangent to Γ at
Ai for i = 1, 2, · · · , n. Let P be any point on Γ between An and A1 . A circle
C (with any radius) is drawn externally tangent to Γ at P . Let ti be the
length
Pof the common external tangent between the circles C and Oi . Prove
n
that i=1 (−1)i ti = 0.

12
Problem 74 [INMO]

The circumference of a circle is divided into eight arcs by a convex quadrilat-


eral ABCD, with four arcs lying inside the quadrilateral and the remaining
four lying outside it. The lengths of the arcs lying inside the quadrilateral
are denoted by p, q, r, s in counter-clockwise direction starting from some
arc. Suppose p + r = q + s. Prove that ABCD is a cyclic quadrilateral.

Problem 75 [INMO]

In an acute-angled triangle ABC, points D, E, F are located on the sides


BC, CA, AB respectively such that
CD CA AE AB BF BC
CE
= ,
CB AF
= ,
AC BD
= BA
.

Prove that AD, BE, CF are the altitudes of ABC.

Problem 76

In trapezoid ABCD, AB is parallel to CD and let E be the mid-point of


BC. Suppose we can inscribe a circle in ABED and also in AECD. Then
if we denote |AB| = a, |BC| = b, |CD| = c, |DA| = d prove that:
b 1 1
a+c= 3
+d , a
+ c
= 3b .

Problem 77 [BMO]

Let ABC be a triangle with AC > AB. The point X lies on the side BA
extended through A and the point Y lies on the side CA in such a way that
BX = CA and CY = BA. The line XY meets the perpendicular bisector
of side BC at P . Show that

∠BP C + ∠BAC = 180◦

Problem 78 [Loney]

If D, E, F are the points of contact of the inscribed circle with the sides BC,
CA, AB of a triangle, show that if the squares of AD, BE, CF are in arith-
metic progression, then the sides of the triangle are in harmonic progression.

13
Problem 79 [Loney]

Through the angular points of a triangle straight lines making the same angle
α with the opposite sides are drawn. Prove that the area of the triangle
formed by them is to the area of the original triangle as 4 cos2 α : 1.

Problem 80 [Loney]

If D, E, F be the feet of the perpendiculars from ABC on the opposite sides


and ρ, ρ1 , ρ2 , ρ3 be the radii of the circles inscribed in the triangles DEF ,
AEF , BF D, CDE, prove that r3 ρ = 2Rρ1 ρ2 ρ3 .

Problem 81 [Loney]

A point O is situated on a circle of radius R and with centre O another


circle of radius 3R
2
is described. Inside the crescent-shaped area intercepted
between these circles a circle of radius R8 is placed. Show that if the small
circle moves in contact with the original circle of radius R, the length of arc
described by its centre in moving from one extreme position to the other is
7
12
πR.

Problem 82 [Crux]

A Gergonne cevian is the line segment from a vertex of a triangle to the point
of contact, on the opposite side, of the incircle. The Gergonne point is the
point of concurrency of the Gergonne cevians.
In an integer triangle ABC, prove that the Gergonne point Γ bisects the
Gergonne cevian AD if and only if b, c, |3a−b−c|
2
form a triangle where the
measure of the angle between b and c is π3 .

Problem 83

Prove that the line which divides the perimeter and the area of a triangle in
the same ratio passes through the centre of the incircle.

Problem 84

Let ma , mb , mc and wa , wb , wc denote, respectively, the lengths of the medi-


ans and angle bisectors of a triangle. Prove that
√ √ √ √ √ √
ma + mb + mc ≥ wa + wb + wc .

14
Problem 85

A quadrilateral has one vertex on each side of a square of side-length 1.


Show that the lengths a, b, c and d of the sides of the quadrilateral satisfy
the inequalities
2 ≤ a2 + b2 + c2 + d2 ≤ 4.

Problem 86 [Purdue Problem of the Week]

Given a triangle ABC, find a triangle A1 B1 C1 so that

(1) A1 ∈ BC, B1 ∈ CA, C1 ∈ AB


(2) the centroids of triangles ABC and A1 B1 C1 coincide
and subject to (1) and (2) triangle A1 B1 C1 has minimal area.

Problem 87

Prove that if the perpendiculars dropped from the points A1 , B1 and C1 on


the sides BC, CA and AB of the triangle ABC, respectively, intersect at the
same point, then the perpendiculars dropped from the points A, B and C on
the lines B1 C1 , C1 A1 and A1 B1 also intersect at one point.

Problem 88

Drawn through the intersection point M of medians of a triangle ABC is a


straight line intersecting the sides AB and AC at points K and L, respec-
tively, and the extension of the side BC at a point P (C lying between P
and B). Prove that
1 1 1
|M K|
= |M L|
+ |M P |

Problem 89

Prove that the area of the octagon formed by the lines joining the vertices of
a parallelogram to the midpoints of the opposite sides is 1/6 of the area of
the parallelogram.

Problem 90

Prove that if the altitude of a triangle is 2 times the radius of the circum-
scribed circle, then the straight line joining the feet of the perpendiculars
dropped from the foot of this altitude on the sides enclosing it passes through
the centre of the circumscribed circle.

15
Problem 91
Prove that the projections of the foot of the altitude of a triangle on the sides
enclosing this altitude and on the two other altitudes lie on one straight line.
Problem 92
Let a, b, c and d be the sides of an inscribed quadrilateral (a is opposite to
c), ha , hb , hc and hd the distances from the centre of the circumscribed circle
to the corresponding sides. Prove that if the centre of the circle is inside the
quadrilateral, then
ahc + cha = bhd + dhb
Problem 93
Prove that three lines passing through the vertices of a triangle and bisecting
its perimeter intersect at one point (called Nagell’s point). Let M denote
the centre of mass of the triangle, I the centre of the inscribed circle, S the
centre of the circle inscribed in the triangle with vertices at the midpoints of
the sides of the given triangle. Prove that the points N , M , I and S lie on
a straight line and |M N | = 2|IM |, |IS| = |SN |.
Problem 94 [Loney]
If ∆0 be the area of the triangle formed by joining the points of contact of
the inscribed circle with the sides of the given triangle whose area is ∆ and
∆1 , ∆2 and ∆3 the corresponding areas for the escribed circles prove that
∆1 + ∆2 + ∆3 − ∆0 = 2∆
Problem 95
Prove that the radius of the circle circumscribed about the triangle formed
by the medians of an acute-angled triangle is greater than 5/6 of the radius
of the circle circumscribed about the original triangle.
Problem 96
Let K denote the intersection point of the diagonals of a convex quadrilateral
ABCD, L a point on the side AD, N a point on the side BC, M a point on
the diagonal AC, KL and M N being parallel to AB, LM parallel to DC.
Prove that KLM N is a parallelogram and its area is less than 8/27 of the
area of the quadrilateral ABCD (Hattori’s Theorem).

16
Problem 97

Two triangles have a common side. Prove that the distance between the
centres of the circles inscribed in them is less than the distance between
their non-coincident vertices (Zalgaller’s problem).

Problem 98

Prove that the sum of the distances from a point inside a triangle to its
vertices is not less than 6r, where r is the radius of the inscribed circle.

Problem 99

Given a triangle. The triangle formed by the feet of its angle bisectors is
isosceles. Is the given triangle isosceles?

Problem 100

Prove that the perpendicular bisectors of the line segments joining the inter-
section points of the altitudes to the centres of the circumscribed circles of
the four triangles formed by four arbitrary straight lines in the plane intersect
at one point (Herwey’s point).

Problem 101 [Crux]

Given triangle ABC with AB < AC. Let I be the incentre and M be the
mid-point of BC. The line M I meets AB and AC at P and Q respectively.
A tangent to the incircle meets sides AB and AC at D and E respectively.
Prove that
AP AQ PQ
BD
+ CE
= 2M I

Problem 102 [Crux]

Let ABC be a triangle with ∠BAC = 60◦ . Let AP bisect ∠BAC and let
BQ bisect ∠ABC, with P on BC and Q on AC. If AB + BP = AQ + QB,
what are the angles of the triangle?

Problem 103

Prove that the sum of the squares of the distances from an arbitrary point in
the plane to the sides of a triangle takes on the least value for such a point
inside the triangle whose distances to the corresponding sides are propor-
tional to these sides. Prove also that this point is the intersection point of
the symmedians of the given triangle (Lemoine’s Point).

17
Problem 104

Given a triangle ABC. AA1 , BB1 and CC1 are its altitudes. Prove that
Euler’s lines of the triangles AB1 C1 , A1 BC1 and A1 B1 C intersect at a point
P of the nine-point circles such that one of the line segments P A1 , P B1 , P C1
is equal to sum of the other two (Thebault’s problem).

Problem 105

Let M be an arbitrary point in the plane and G, the centroid of triangle


ABC. Prove that
3|M G|2 = |M A|2 + |M B|2 + |M C|2 − 31 (|AB|2 + |BC|2 + |CA|2 )
(Leibnitz’s Theorem)

Problem 106

Let ABC be a regular triangle with side a and M some point in the plane
found at a distance d from the centre of the triangle ABC. Prove that the
area of the triangle whose sides are equal to the line segments M A, M B and
M C can be expressed by the formula


3 2
S= 12
|a − 3d2 |

Problem 107 [Todhunter]

If Q be any point in the plane of a triangle and R1 , R2 , R3 the radii of the


circles about QBC, QCA, QAB prove that
a2 b2 c2
( Ra1 + b
R2
+ c
R3
)(− Ra1 + b
R2
+ c
R3
)( Ra1 − b
R2
+ c
R3
)( Ra1 + b
R2
− c
R3
) = R12 R22 R32

Problem 108 [Mathematical Gazette]

P QRS is a quadrilateral inscribed in a circle with centre O. E is the inter-


section of the diagonals P R and QS. Let F be theintersection of P Q and
RS and G the intersection of P S and QR. The circle on F G as diameter
meets OE at X. The perpendicular bisectors of SX and P X meet at A and
B, C, D are defined similarly by cyclic change of letters.
(i) Prove that the tangents at P and Q and the line OB are concurrent.
(ii) Prove that P Q, AC, SR, F G are concurrent at F .
(iii)Prove that AD, BC, F G are concurrent.

18
Problem 109 [AMM]

Let X, Y and Z be three distinct points in the interior of an equilateral


triangle ABC. Let α, β and γ be positive numbers adding up to π3 with the
property that ∠XBA = ∠Y AB=α, ∠Y CB = ∠ZBC = β and ∠ZAC =
∠XCA = γ. Find the angles of triangle XY Z in terms of α, β and γ.

Problem 110 [Todhunter]

If O be the centre of the circle inscribed in a triangle ABC and ra , rb , rc the


radii of the circles inscribed in the triangles OBC, OCA, OAB, show that
a b c
ra
+ rb
+ rc
= 2(cot( A4 ) + cot( B4 ) + cot( C4 ))

Problem 111 [BMO]

Let P be an internal point of triangle ABC and let α, β, γ be defined by

α = ∠BP C − ∠BAC
β = ∠CP A − ∠CBA
γ = ∠AP B − ∠ACB

Prove that

P A sin(∠BAC)
sin(α)
= P B sin(∠CBA)
sin(β)
= P C sin(∠ACB)
sin(γ)

Problem 112

Let ABC be a triangle with incentre I and inradius r. Let D, E, F be the


feet of the perpendiculars from I to the sides BC, CA and AB respectively.
If r1 , r2 and r3 are the radii of circles inscribed in the quadrilaterals AF IE,
BDIF and CEID respectively, prove that
r1 r2 r3 r1 r2 r3
r−r1
+ r−r2
+ r−r3
= (r−r1 )(r−r2 )(r−r3 )

Problem 113 [Loney]

Given the product p of the sines of the angles of a triangle and the product
q of the cosines, show that the tangents of the angles are the roots of the
equation

qx3 − px2 + (1 + q)x − p = 0

19
Problem 114

The altitude of a right triangle drawn to the hypotenuse is equal to h. Prove


that the vertices of the acute angles of the triangle and the projections of
the foot of the altitude on the legs all lie on the same circle. Determine the
length of the chord cut by this circle on the line containing the altitude and
the segments of the chord into which it is divided by the hypotenuse.

Problem 115

Four villages are situated at the vertices of a square of side 2 Km. The
villages are connected by roads so that each village is joined to any other. Is
it possible for the total length of the roads to be less than 5.5 Km?

Problem 116

Prove that if the lengths of the internal


√ angle bisectors of a triangle are less
3
than 1, then its area is less than 3 .

Problem 117

Given a convex quadrilateral ABCD circumscribed about a circle of diameter


1. Inside ABCD, there is a point M such that
|M A|2 + |M B|2 + |M C|2 + |M D|2 = 2.
Find the area of ABCD.

Problem 118

The circle inscribed in a triangle ABC divides the median BM into three
equal parts. Find the ratio |BC| : |CA| : |AB|.

Problem 119

Prove that if the centres of the squares constructed externally on the sides
of a given triangle serve as the vertices of the triangle whose area is twice
the area of the given triangle, then the centres of the squares constructed
internally on the sides of the triangle lie on a straight line.

Problem 120

Prove that the median drawn to the largest side of a triangle forms with
the sides enclosing this median angles each of which is not less than half the
smallest angle of the triangle.

20
Problem 121

Three squares BCDE, ACF G and BAHK are constructed externally on


the sides BC, CA and AB of a triangle ABC. Let F CDQ and EBKP
be parallelograms. Prove that the triangle AP Q is a right-angled isosceles
triangle.

Problem 122

Three points are given in a plane. Through these points three lines are drawn
forming a regular triangle. Find the locus of centres of those triangles.

Problem 123

Drawn in an inscribed polygon are non-intersecting diagonals separating the


polygon into triangles. Prove that the sum of the radii of the circles inscribed
in those triangles is independent of the way the diagonals are drawn.

Problem 124

A polygon is circumscribed about a circle. Let l be an arbitrary line touching


the circle and coinciding with no side of the polygon. Prove that the ratio of
the product of the distances from the verices of the polygon to the line l to
the product of the distances from the points of tangency of the sides of the
polygon with the circle to l is independent of the position of the line l.

Problem 125 [Loney]

If 2φ1 , 2φ2 , 2φ3 are the angles subtended by the circle escribed to the side a
of a triangle at the centres of the inscribed circle and the other two escribed
circles, prove that
r12
sin(φ1 ) sin(φ2 ) sin(φ3 ) = 16R2

Problem 126

If from any point in the plane of a regular polygon perpendiculars are drawn
on the sides, show that the sum of the squares of these perpendiculars is equal
to the sum of the squares on the lines joining the feet of the perpendiculars
with the centre of the polygon.

21
Problem 127 [Loney]

The three medians of a triangle ABC make angles α, β, γ with each other.
Prove that

cot α + cot β + cot γ + cot A + cot B + cot C = 0

Problem 128 [Loney]

A railway curve, in the shape of a quadrant of a circle, has n telegraph posts


at its ends and at equal distances along the curve. A man stationed at a
point on one of the extreme radii produced sees the pth and qth posts from
the end nearest him in a straight line. Show that the radius of the curve is
a cos(p+q)φ π
2 sin(pφ) sin(qφ)
, where φ = 4(n−1) , and a is the distance from the man to the
nearest end of the curve.

Problem 129

Let D be an arbitrary point on the side BC of a triangle ABC. Let E and F


be points on the sides AC and AB such that DE is parallel to AB and DF is
parallel to AC. A circle passing through D, E and F intersects for the second
time BC, CA and AB at points D1 , E1 and F1 , respectively. Let M and N
denote the intersection points of DE and F1 D1 , DF and D1 E1 , respectively.
Prove that M and N lie on the symedian emanating from the vertex A. If
D coincides with the foot of the symedian, then the circle passing through
D, E and F touches the side BC.(This circle is called Tucker’s Circle.)

Problem 130

Let ABCD be a cyclic quadrilateral. The diagonal AC is equal to a and


forms angles α and β with the sides AB and AD, respectively. Prove that
2
the magnitude of the area of the quadrilateral lies between a sin(α+β)
2 sin α
sin β
and
a2 sin(α+β) sin α
2 sin β

Problem 131

A triangle has sides of lengths a, b, c and respective altitudes of lengths ha ,hb ,


hc . If a ≥ b ≥ c show that a + ha ≥ b + hb ≥ c + hc .

22
Problem 132 [Crux]

Given a right-angled triangle ABC with ∠BAC = 90◦ . Let I be the incen-
tre and let D and E be the intersections of BI and CI with AC and AB
respectively. Prove that
|BI|2 +||ID|2 |AB|2
|IC|2 +|IE|2
= |AC|2

Problem 133 [Hobson]

Straight lines whose lengths are successively proportional to 1, 2, 3, · · · , n


form a rectilineal figure whose exterior angles are each equal to 2π n
; if a
polygon be formed by joining the extremities of the first and last lines, show
that its area is
n(n+1)(2n+1)
24
cot( πn ) + n
16
cot( πn ) csc2 ( πn )

Problem 134

An arc AB of a circle is divided into three equal parts by the points C and
D (C is nearest to A). When rotated about the point A through an angle of
π
3
, the points B, C and D go into points B1 , C1 and D1 . F is the point of
intersection of the straight lines AB1 and DC1 ; E is a point on the bisector
of the angle B1 BA such that |BD| = |DE|. Prove that the triangle CEF is
regular (Finlay’s theorem).

Problem 135

In a triangle ABC, a point D is taken on the side AC. Let O1 be the centre
of the circle touching the line segments AD, BD and the circle circumscribed
about the triangle ABC and let O2 be the centre of the circle touching the line
segments CD, BD and the circumscribed circle. Prove that the line O1 O2
passes through the centre O of the circle inscribed in the triangle ABC and
|O1 O| : |OO2 | = tan2 (φ/2), where φ = ∠BDA (Thebault’s theorem).

Problem 136

Prove the following statement. If there is an n-gon inscribed in a circle α and


circumscribed about another circle β, then there are infinitely many n-gons
inscribed in the circle α and circumscribed about the circle β and any point
of the circle can be taken as one of the vertices of such an n-gon (Poncelet’s
theorem).

23
Problem 137 [Loney]

A point is taken in the plane of a regular polygon of n sides at a distance c


from the centre and on the line joining the centre to a vertex, and the radius
of the inscribed circle is r. Show that the product of the distances of the
point from the sides of the polygon is
cn
2n−2
cos2 ( n2 cos−1 rc ) if c > r and
cn
2n−2
cosh2 ( n2 cosh−1 rc ) if c < r

Problem 138 [Loney]

An infinite straight line is divided by an infinite number of points into por-


tions each of length a. Prove that the sum of the fourth powers of the
reciprocals of the distances of a point O on the line from all the points of
division is
π4
3a4
(3 csc4 πb
a
− 2 csc2 πb
a
)

Problem 139 [Loney]

If ρ1 , ρ2 , · · · , ρn be the distances of the vertices of a regular polygon of n sides


from any point P in its plane, prove that
1 1 1 n r 2n −a2n
ρ21
+ ρ22
+ ··· + ρ2n
= r 2 −a2 r2n −2an rn cos(nθ)+a2n

where a is the radius of the circumcircle of the polygon, r is the distance of


P from its centre O and θ is the angle that OP makes with the radius to any
angular point of the polygon.

Problem 140

Given an angle with vertex A and a circle inscribed in it. An arbitrary


straight line touching the given circle intersects the sides of the angle at
points B and C. Prove that the circle circumscribed about the triangle
ABC touches the circle inscribed in the given angle.

Problem 141

Let ABCDEF be an inscribed hexagon in which |AB| = |CD| = |EF | = R,


where R is the radius of the circumscribed circle, O its centre. Prove that
the points of pairwise intersections of the circles circumscribed about the
triangles BOC, DOE, F OA, distinct from O, serve as the vertices of an
equilateral triangle with side R.

24
Problem 142

The diagonals of an inscribed quadrilateral are mutually perpendicular. Prove


that the midpoints of its sides and the feet of the perpendiculars dropped
from the point of intersection of the diagonals on the sides lie on a circle.
Find the radius of that circle if the radius of the given circle is R and the
distance from its centre to the point of intersection of the diagonals of the
quadrilateral is d.

Problem 143

Prove that if a quadrialateral is both inscribed in a circle and circumscribed


about a circle of radius r, the distance between the centres of those circles
being d, then the relationship
1 1 1
(R+d)2
+ (R−d)2
= r2

is true.

Problem 144

Let ABCD be a convex quadrilateral. Consider four circles each of which


touches three sides of this quadrilateral.

(a) Prove that the centres of these circles lie on one circle.
(b) Let r1 , r2 ,r3 and r4 denote the radii of these circles (r1 does not touch
the side DC, r2 the side DA, r3 the side AB and r4 the side BC). Prove
that
|AB| |CD| |BC| |AD|
r1
+ r3
= r2
+ r4

Problem 145

The sides of a square is equal to a and the products of the distances from
the opposite vertices to a line l are equal to each other. Find the distance
from the centre of the square to the line l if it is known that neither of the
sides of the square is parallel to l.

Problem 146

Find the angles of a triangle if the distance between the centre of the cir-
cumcircle and the intersection point of the altitudes is one-half the length of
the largest side and equals the smallest side.

25
Problem 147

Prove that for the perpendiculars dropped from the points A1 , B1 and C1 on
the sides BC, CA and AB of a triangle ABC to intersect at the same point,
it is necessary and sufficient that
|A1 B|2 − |BC1 |2 + |C1 A|2 − |AB1 |2 + |B1 C|2 − |CA1 |2 = 0.

Problem 148

Each of the sides of a convex quadrilateral is divided into (2n + 1) equal


parts. The division points on the opposite sides are joined correspondingly.
Prove that the area of the central quadrilateral amounts to 1/(2n + 1)2 of
the area of the entire quadrilateral.

Problem 149

A straight line intersects the sides AB, BC and the extension of the side
AC of a triangle ABC at points D, E and F , respectively. Prove that
the midpoints of the line segments DC, AE and BF lie on a straight line
(Gaussian line).

Problem 150

Given two intersecting circles. Find the locus of centres of rectangles with
vertices lying on these circles.

Problem 151

An equilateral triangle is inscribed in a circle. Find the locus of intersection


points of the altitudes of all possible triangles inscribed in the circle if two
sides of the triangles are parallel to those of the given one.

Problem 152

Given two circles touching each other internally at a point A. A tangent to


the smaller circle intersects the larger one at points B and C. Find the locus
of centres of circles inscribed in triangles ABC.

Problem 153 [Loney]

Two circles, the sum of whose radii is a, are placed in the same plane with
their centres at a distance 2a and an endless string is fully stretched so as
partly to surround the circles√and to cross between them. Show that the
length of the string is ( 4π
3
+ 2 3)a.

26
Problem 154 [Loney]

If p, q, r are the perpendiculars from the vertices of a triangle upon any


straight line meeting the sides externally in D, E, F , prove that
a2 (p − q)(p − r) + b2 (q − r)(q − p) + c2 (r − p)(r − q) = 4∆2 .

Problem 155 [Loney]

A regular polygon is inscribed in a circle; show that the arithmetic mean of


the squares of the distances of its corners from any point (not necessarily in
its plane) is equal to the arithmetic mean of the sum of the squares of the
longest and shortest distances of the point from the circle.

Problem 156

In the cyclic quadrilateral ABCD, the diagonal AC bisects the angle DAB.
The side AD is extended beyond D to a point E. Show that CE = CA if
and only if DE = AB.

Problem 157 [BMO]

Let G be a convex quadrilateral. Show that there is a point X in the plane


of G with the property that every straight line through X divides G into two
regions of equal area if and only if G is a parallelogram.

Problem 158

Given a triangle ABC and a point M . A straight line passing through the
point M intersects the lines AB, BC and CA at points C1 , A1 and B1 ,
respectively. The lines AM , BM and CM intersect the circle circumscribed
about the triangle ABC at points A2 , B2 and C2 , respectively. Prove that
the lines A1 A2 , B1 B2 and C1 C2 intersect at a point situated on the circle
circumscribed about the triangle ABC.

Problem 159 [AMM]

Let P be a point in the interior of triangle ABC and let r1 , r2 , r3 denote


the distances from P to the sides of the triangle with lengths a1 , a2 , a3 ,
respectively. Let R be the circumradius of ABC and let 0 < a < 1 be a real
number. Let b = 2a/(1 − a). Prove that
1
r1a + r2a + r3a ≤ (2R)a
(ab1 + ab2 + ab3 )1−a

27
Problem 160 [AMM]

Let K be the circumcentre and G the centroid of a triangle with side lengths
a, b, c and area ∆.

(a) Show that the distance d from K to G satisfies

12∆d = a2 b2 c2 − (b2 + c2 − a2 )(c2 + a2 − b2 )(a2 + b2 − c2 )


abc abc abc
(b) Show that d(< 12∆ , = 12∆ ,> 12∆
) when the triangle is respectively
(acute, right-angled,obtuse).

Problem 161 [AMM]

Let ABC be an acute-triangle and let P be a point in its interior. Denote by


a, b, c the lengths of the triangle’s sides, by da , db , dc the distances from P to
the triangle’s sides, and by Ra , Rb , Rc the distances from P to the vertices
A, B, C respectively. Show that

d2a + d2b + d2c ≥ Ra2 sin2 (A/2) + Rb2 sin2 (B/2) + Rc2 sin2 (C/2) ≥ (da + db + dc )2 /3

Problem 162 [Loney]

A1 A2 · · · An is a regular polygon of n sides which is inscribed in a circle, whose


radius is a and whose centre is O; prove that the product of the distances of
its angular points from a straight line at right angles to OA and at a distance
b(> a) from the centre is

bn [cosn ( 21 sin−1 ab ) − sinn ( 12 sin−1 ab )]2

Problem 163 [Loney]

The radii of an infinite series of concentric circles are a, a2 , a3 · · · . From a point


at a distance c(> a) from their common centre a tangent is drawn to each
circle. Prove that
pc
sin(θ1 ) sin(θ2 ) sin(θ3 ) · · · = πa sin πa
c

where θ1 , θ2 , θ3 , · · · are the angles that the tangents subtend at the common
centre.

28
Problem 164 [Crux]

Construct equilateral triangles A0 BC, B 0 CA, C 0 AB exterior to triangle ABC


and take points P , Q, R on AA0 , BB 0 , CC 0 , respectively, such that
AP BQ CR
AA0
+ BB 0
+ CC 0
= 1.

Prove that ∆P QR is equilateral.

Problem 165 [Crux]

Given a triangle ABC, we take variable points P on segment AB and Q on


segment AC. CP meets BQ in T . Where should P and Q be located so that
area of ∆P QT is maximized?

Problem 166 [Crux]

Let ABC be a triangle and A1 , B1 , C1 the common points of the inscribed


circle with the sides BC, CA, AB, respectively. We denote the length of the
arc B1 C1 (not containing A1 ) of the incircle by Sa , and similarly define Sb
and Sc . Prove that

a b c 9 3
Sa
+ Sb
+ Sc
≥ π

Problem 167 [AMM]

A cevian of a triangle is a line segment that joins a vertex to the line con-
taining the opposite side. An equicevian point of a triangle ABC is a point
P (not necessarily inside the triangle) such that the cevians on the lines AP ,
BP and CP have equal length. Let SBC be an equilateral triangle and let
A be chosen in the interior of SBC on the altitude dropped from S.

(a) Show that ABC has two equicevian points.


(b) Show that the common length of the cevians through either of the
equicevian points is constant, independent of the choice of A.
(c) Show that the equicevian points divide the cevian through A in a
constant ratio, independent of the choice of A.
(d) Find the locus of the equicevian points as A varies.
(e) Let S 0 be the reflection of S in the line BC. Show that (a), (b) and (c)
hold if A moves on any ellipse with S and S 0 as its foci. Find the locus of
the equicevian points as A varies on the ellipse.

29
Problem 168 [Crux]

Let ABCD be a trapezoid with AD parallel to BC. M , N , P , Q, O are the


midpoints of AB, CD, AC, BD, M N , respectively. Circles m, n, p, q all
pass through O and are tangent to AB at M , to CD at N , to AC at P , and
to BD at Q, respectively. Prove that the centres of m, n, p, q are collinear.

Problem 169 [AMM]

Let ABC be an equilateral triangle inscribed in a circle with radius 1 unit.


32
Suppose P is a point inside the triangle. Prove that |P A||P B||P C| ≤ 27 .
Generalize the result to a regular polygon of n sides. (Erdos)

Problem 170

Given two circles. Find the locus of points M such that the ratio of the
lengths of the tangents drawn from M to the given circles is a constant k.

Problem 171

In a quadrilateral ABCD, P is the intersection point of BC and AD, Q


that of CA and BD and R that of AB and CD. Prove that the intersection
points of BC and QR, CA and RP , AB and P Q are collinear.

Problem 172

Given two squares whose sides are respectively parallel. Determine the locus
of points M such that for any point P of the first square there is a point Q of
the second one such that the triangle M P Q is equilateral. Let the side of the
first square be a and that of the second square be b. For what relationship
between a and b is the desired locus non-empty?

Problem 173 [AMM]

Let C1 C2 . . . Cn be a regular n-gon and let Cn+1 = C1 . Let O be the inscribed


circle. For 1 ≤ k ≤ n, let Tk be the point at which O is tangent to Ck Ck+1 .
Let X be a point on the open arc (Tn−1 Tn ) and let Y be a point other than
X on O. For 1 ≤ i ≤ n, let Bi be the second point at which the line XCi
meets O and let pi = |XBi ||XCi |. Let Mi be the mid-point of chord Ti Ti+1
and let Ni be the second point, other than Y , at which Y Mi meets O. Let
Pn n
P
qi = |Y Mi ||Y Ni |. Prove that qi = ( p i ) − p n .
i=1 i=1

30
Problem 174 [AMM]

Let ABC be an acute triangle, with semi-perimeter p and with inscribed and
circumscribed circles of radius r and R, respectively.

(a) Show that ABC has a median of length at most p/ 3.
(b) Show that ABC has a median of length at most R + r.
(c) Show that ABC has an altitude of length at least R + r.

Problem 175 [RMO, India]

Let ABC be an acute-angled triangle and CD be the altitude through C. If


AB = 8 and CD = 6 find the distance between the mid-points of AD and
BC.

Problem 176 [RMO, India]

Let ABCD be a rectangle with AB = a and BC = b. Suppose r1 is the radius


of the circle passing through A and B and touching CD; and similarly r2 is
the radius of the circle passing through B and C and touching AD. Show
that
r1 + r2 ≥ 85 (a + b)

Problem 177 [INMO]

Two circles C1 and C2 intersect at two distinct points P and Q in a plane. Let
a line passing through P meet the circles C1 and C2 in A and B respectively.
Let Y be the mid-point of AB and QY meet the circles C1 and C2 in X and
Z respectively. Show that Y is also the mid-point of XZ.

Problem 178 [INMO]

In a triangle ABC angle A is twice angle B. Show that a2 = b(b + c).

Problem 179 [INMO]

The diagonals AC and BD of a cyclic quadrilateral ABCD intersect at P .


Let O be the circumcentre of triangle AP B and H be the orthocentre of
triangle CP D. Show that the points H, P , O are collinear.

31
Problem 180 [INMO]

Let ABC be a triangle in a plane Σ. Find the set of all points P (distinct
from A, B, C) in the plane Σ such that the circumcircles of triangles ABP ,
BCP and CAP have the same radii.

Problem 181 [INMO]

Let ABC be a triangle right-angled at A and S be its circumcircle. Let S1 be


the circle touching the lines AB and AC and the circle S internally. Further
let S2 be the circle touching the lines AB and AC and the circle S externally.
If r1 and r2 be the radii of the circles S1 and S2 respectively, show that
r1 .r2 = 4Area(ABC)

Problem 182 [INMO]

Show that there exists a convex hexagon in the plane such that
(a) all its interior angles are equal.
(b) all its sides are 1,2,3,4,5,6 in some order.

Problem 183 [INMO]

Let G be the centroid of a triangle ABC in which the angle C is obtuse and
AD and CF be the medians from A and C respectively onto the sides BC
and AB. If the four points B, D, G and F are concyclic, show that
AC

BC
> 2
If further P is a point on the line BG extended such that AGCP is a paral-
lelogram, show that the triangle ABC and GAP are similar.

Problem 184 [INMO]

A circle passes through a vertex C of a rectangle ABCD and touches its


sides AB and AD at M and N respectively. If the distance from C to the
line segment M N is equal to 5 units, find the area of the rectangle ABCD.

32
Problem 185 [RMO, India]

In a quadrilateral ABCD, it is given that AB is parallel to CD and the


diagonals AC and BD are perpendicular to each other. Show that
(a) AD.BC ≥ AB.CD
(b) AD + BC ≥ AB + CD

Problem 186 [RMO, India]

In the triangle ABC, the incircle touches the sides BC, CA and AB respec-
tively at D, E and F . If the radius of the incircle is 4 units and if BD, CE
and AF are consecutive integers , find the sides of the triangle ABC.

Problem 187 [RMO, India]

Let ABCD be a square and M , N points on sides AB, BC, respectively,


such that ∠M DN = 45◦ . If R is the midpoint of M N show that RP = RQ
where P , Q are the points of intersection of AC with the lines M D and N D.
Problem 188 [RMO, India]
Let AC and BD be two chords of a circle with centre O such that they
intersect at right angles inside the circle at the point M . Suppose K and L
are the mid-points of the chord AB and CD respectively. Prove that OKM L
is a parallelogram.

Problem 189 [AMM]

Let P be a convex n-gon inscribed in a circle O and let ∆ be a triangulation


of P without new vertices. Compute the sum of the squares of distances from
the centre O to the incentres of the triangles of ∆ and show that this sum is
independent of ∆.

Problem 190 [AMM]

Let T1 and T2 be triangles such that for i ∈ 1, 2, triangle Ti has circumradius


Ri , inradius ri and side lengths ai , bi and ci . Show that
8R1 R2 + 4r1 r2 ≥ a1 a2 + b1 b2 + c1 c2 ≥ 36r1 r2
and determine when equality holds.

33
Problem 191 [AMM]

Let ABC be an acute triangle. T the mid-point of arc BC of the circle


circumscribing ABC. Let G and K be the projections of A and T respectively
on BC, let H and L be the projections of B and C on AT and let E be the
mid-point of AB. Prove that:

(a) KH||AC, GL||BT , GH||T C, LK||AB.


(b)G, H, K and L are concyclic.
(c) The centre of the circle through G, H and K lies on the Euler circle of
ABC.

Problem 192 [AMM]

A trapezoid ABRS with AB||RS is inscribed in a non-circular ellipse E with


axes of symmetry a and b. The points A and B are reflected through a to
points P and Q on E.

(a) Show that P , Q, R and S are concyclic.


(b) Show that if the line P Q intersects the line RS at T , then the angle
bisector of ∠P T R is parallel to a.

Problem 193 [RMO, India]

ABCD is a cyclic quadrilateral with AC ⊥ BD; AC meets BD at E. Prove


that EA2 + EB 2 + EC 2 + ED2 = 4R2 .

Problem 194 [RMO, India]

ABCD is a cyclic quadrilateral; x, y, z are the distances of A from the lines


BD, BC, CD respectively. Prove that
BD BC CD
x
= y
+ z

Problem 195 [RMO, India]

ABCD is a quadrilateral and P , Q are mid-points of CD, AB respectively.


Let AP , DQ meet at X and BP , CQ meet at Y . Prove that

area(ADX)+area(BCY ) = area(P XQY ).

34
Problem 196 [RMO, India]

The cyclic octagon ABCDEF GH has sides a, a, a, a, b, b, b, b respectively.


Find the radius of the circle that circumscribes ABCDEF GH in terms of a
and b.

Problem 197 [AMM]

Prove that in an acute triangle with angles A, B and C


(1−cos A)(1−cos B)(1−cos C) 8(tan A+tan B+tan C)3
cos A cos B cos C
≥ 27(tan A+tan B)(tan C+tan A)(tan B+tan C)

Problem 198 [Mathscope, Vietnam]

In a triangle ABC, denote by la , lb , lc the internal angle bisectors, ma , mb ,


mc the medians and ha , hb , hc the altitudes to the sides a, b, c of the triangle.
Prove that
ma mb mc 3
lb +hb
+ hc +lc
+ la +ha
≥ 2

Problem 199 [Mathscope, Vietnam]

Let AM , BN , CP be the medians of triangle ABC. Prove that if the radius


of the incircles of triangles BCN , CAP and ABM are equal in length, then
ABC is an equilateral triangle.

Problem 200 [Mathscope, Vietnam]

Given a triangle with incentre I, let l be a variable line passing through I.


Let l intersect the ray CB, sides AC, AB at M , N , P respectively. Prove
that the value of
AB AC BC
P A.P B
+ N A.N C
− M B.M C

is independent of the choice of l.

Problem 201 [Mathscope, Vietnam]

Let I be the incentre of triangle ABC and let ma , mb , mc be the lengths of


the medians from vertices A, B and C, respectively. Prove that
IA2 IB 2 IC 2 3
m2a
+ m2b
+ m2c
≤ 4

35
Problem 202 [Mathscope, Vietnam]
Let R and r be the circumradius and inradius of triangle ABC; the incircle
touches the sides of the triangle at three points which form a triangle of
perimeter p. Suppose that q is the perimeter of triangle ABC. Prove that
r p 1
R
≤ q
≤ 2

Problem 203 [AMM]


Let a, b and c be the lengths of the sides of a triangle and let R and r be the
circumradius and inradius of that triangle, respectively. Show that
2 (b−c)2 (c−a)2
R
2r
≥ exp( (a−b)
2c2
+ 2a2
+ 2b2
)
Problem 204 [AMM]
Consider an acute triangle with sides of lengths a, b and c and with an
inradius of r and circumradius of R. Show that
√ 2
r 2(2a −(b−c)2 )(2b2 −(c−a)2 )(2c2 −(a−b)2 )
R
≤ (a+b)(b+c)(c+a)
.
Problem 205 [AMM]
Let a, b and c be the lengths of the sides of a triangle, and let R and r denote
the circumradius and inradius of the triangle. Show that
R 4a 2 4b 2 4c 2
2
2r
≥ ( 4a2 −(b−c) 2 4b2 −(c−a)2 4c2 −(a−b)2 )

Problem 206 [AMM]


Let ABC be a triangle with sides a, b and c all different, and corresponding
angles α, β and γ. Show that
(a) (a + b) cot(β + γ2 ) + (b + c) cot(γ + α2 ) + (c + a) cot(α + β2 ) = 0.
(b) (a − b) tan(β + γ2 ) + (b − c) tan(γ + α2 ) + (c − a) tan(α + β2 ) = 4(R + r).
Problem 207 [AMM]
Let r, R and s be the radii of the incircle, circumcircle and semi-perimeter
of a triangle. Prove that

3 2
q
2
r s ≤ r +4Rr3
≤ 3s
Problem 208 [AMM]
Let a, b and c be the lengths of the sides of a nondegenerate triangle, let
p = (1/2)(a + b + c), and let r and R be the inradius and circumradius of the
triangle, respectively. Show that
a 4r−R
p
2
( R
) ≤ (p − b)(p − c) ≤ a2 .

36
PHƯƠNG TRÌNH HÀM - BỒI DƯỠNG HỌC SINH GIỎI THI QUỐC GIA, QUỐC TẾ

BÀI TẬP PHƯƠNG TRÌNH HÀM THI QUỐC GIA, QUỐC TẾ


VĂN PHÚ QUỐC
I. PHƯƠNG PHÁP THẾ BIẾN
1. Tồn tại hay không hàm f :    sao cho với mọi x, y   ta có:
f  xy   max  f  x  , y  min  f  y  , x ?
2
2. (Australia 1992). Tìm tất cả các hàm số f :  \     thỏa mãn:
3
 2x  2
2 f  x  f    996 x , x   \   .
 3x  2  3
3. (VMO 2000 , bảng B). Tìm tất cả các hàm số f :    thỏa mãn điều kiện:
x 2 f  x   f 1  x   2 x  x 4 , x  
4. Tìm tất cả các hàm số f :    thỏa mãn các điều kiện:
  
 f  0   2012 ; f    2013
 2 .
 f  x  y   f  x  y   2 f  x  cos y x, y  

5. (Belarus 1995). Tìm tất cả các hàm số f :    thỏa mãn
f  f  x  y    f  x  y   f  x  f  y   xy , x, y   .
6. (VMO 2005). Xác định tất cả các hàm f :    thỏa mãn điều kiện:
f  f  x  y    f  x  f  y   f  x   f  y   xy , x, y   .
7. (Australia 1995). Tìm tất cả các hàm f :     thỏa mãn các điều kiện sau:
1 3 3
f 1  ; f  xy   f  x  f    f  y  f   , x, y    .
2  y x
8. (CAMO 2000). Tìm tất cả các hàm f xác định trên  thỏa mãn:
f  x  y    x
2 2
 2 yf  x   f 2  y  , x, y   .
9. (USAMO). Tìm tất cả các hàm f :    thỏa mãn điều kiện:
f  x 2  y 2   xf  x   yf  y  , x, y   .
10. (Shortlist IMO 1979). Cho hàm số f :    thỏa mãn điều kiện:
f  xy  x  y   f  xy   f  x   f  y  , x, y   .
Chứng minh: f  x  y   f  x   f  y  , x, y   .
11. Tìm tất cả các hàm f :    sao cho với mọi m, n, k   ta đều có:

f  km   f  kn   f  k  f  mn   1 .

12. Có hay không một hàm số f :    thỏa mãn:


 
f x  y  sin x  sin y  2 , x , y  

13. (VMO 2002 B). Tìm tất cả các hàm f x   xác định trên  và thỏa mãn điều kiện:

VĂN PHÚ QUỐC GV. Trường THPT chuyên Nguyễn Bỉnh Khiêm 0982 333 443 1
PHƯƠNG TRÌNH HÀM - BỒI DƯỠNG HỌC SINH GIỎI THI QUỐC GIA, QUỐC TẾ


f yf x    f x 2002
  
 y  2001yf x , x, y   .

14. (Mathematical and Youth 9/361). Tìm tất cả các hàm số f :    thỏa mãn điều kiện

       
f x 3  y  2y 3 f 2 x  y 2  f y  f x , x, y   .

15. (AMM). Tìm tất cả các hàm số f :    thỏa mãn điều kiện: f 2  2 và 
f
x y  f x  f y  
, x  y .


x y  f x  f y   
16. (Korea 2003). Tìm tất cả các hàm f :    thỏa mãn

f xf y     f x   xf y   f  f y   , x, y  

II. PHƯƠNG PHÁP QUY NẠP


1. Tìm tất cả các hàm số f :    thỏa: f  x  1  f  x   1 ; f  x 2   f 2  x  , x    .

2. Tìm tất cả các hàm f :    thỏa mãn điều kiện:

f  x  y   f  x  y   2 f  x   2 f  y  x, y   .

3. Tìm tất cả các hàm f :    thỏa mãn: f  x  y   f  x   f  y   2 xy x, y   .

4. (BMO 1979). Tìm tất cả các hàm f :    thỏa: f  x  f  y    f  x  f  y  , x, y   .

Chứng minh f là hàm hằng.

5. (TST 2005). Tìm tất cả các hàm f :    thỏa: f  x3  y 3  z 3   f 3  x   f 3  y   f 3  z  .

6. (China 1996). Cho hàm số f :    thỏa mãn điều kiện:

f  x 3  y 3    x  y  f 2  x   f  x  f  y   f 2  y  , x, y   .

Chứng minh rằng f 1996 x   1996 f  x  , x   .

III. PHƯƠNG PHÁP SỬ DỤNG TÍNH CHẤT ĐƠN ÁNH-TOÀN ÁNH-SONG ÁNH
CỦA HÀM SỐ.
1. Tìm tất cả các hàm f :    thỏa mãn các điều kiện:

f  f  n    n  2; f  f  n  1  1  n  4; f  0   1 n   .

3. Tồn tại hay không hàm f :    thỏa mãn điều kiện: f  x  f  y    f  x   y x, y   ?

4. Cho f : *  * thỏa mãn các điều kiện: f  m 2 f  n    mnf  m  m, n  * .

VĂN PHÚ QUỐC GV. Trường THPT chuyên Nguyễn Bỉnh Khiêm 0982 333 443 2
PHƯƠNG TRÌNH HÀM - BỒI DƯỠNG HỌC SINH GIỎI THI QUỐC GIA, QUỐC TẾ

Chứng minh rằng nếu f  2003  a 2 thì a là số nguyên tố.

5. ( Việt Nam TST 1988). Xác định hàm số f :    thỏa mãn điều kiện:

f  f  n   f  m    n  m n, m   .

6. Tìm tất cả các hàm f :    thỏa mãn các điều kiện:

(i) f  f  n    f  n 

(ii) f  f  m   f  n    f  m  n 

(iii) f nhận vô số giá trị.

7. Tìm tất cả các hàm f :    thỏa mãn điều kiện: f  x 3  f  y    y  f 3  x  x, y   .

8. Chứng minh rằng tồn tại vô số các hàm số f :    thỏa mãn các điều kiện:
* *

(i) f  f  n    n n  * (ii) f  n   n n  * .

9. (Irish 2002). Tìm tất cả các hàm f :    thỏa mãn điều kiện: f  x  f  y    f  x   y ,

x, y   .

10. Chứng minh rằng không tồn tại song ánh f : *   thỏa mãn điều kiện:

f  mn   f  m   f  n   3 f  m  f  n  m, n  *

11. Tìm tất cả các hàm f :    thỏa mãn điều kiện: f  f  n    2 f  n   3n  8 ,n   .

12. (Balkan 1997). Tìm tất cả các hàm f :    thỏa mãn điều kiện:

f  xf  x   f  y    f 2  x   y, x, y  

13. ( Việt Nam TST 2002). Tìm tất cả các hàm f :    thỏa mãn:

f  f  x   y   2 x  f  f  y   x  , x, y   .

14. (IMO 1992). Tìm tất cả các hàm số f :    thỏa mãn điều kiện:

f  x 2  f  y    f 2  x   y, x, y   .

15. ( Việt Nam TST 2004). Tìm tất cả các giá trị của a sao cho tồn tại duy nhất một hàm

f :    thỏa mãn điều kiện: f  x 2  y  f  y    f 2  x   ay , x, y   .

16. ( Đề nghị IMO 2002). Tìm tất cả các hàm số f :    thỏa mãn:

VĂN PHÚ QUỐC GV. Trường THPT chuyên Nguyễn Bỉnh Khiêm 0982 333 443 3
PHƯƠNG TRÌNH HÀM - BỒI DƯỠNG HỌC SINH GIỎI THI QUỐC GIA, QUỐC TẾ

f  f  x   y   2 x  f  f  y   x  , x, y   .

17. (Indonesia TST 2010). Xác định tất cả các số thực a sao cho có một hàm số
thỏa mãn: x  f  y   a. f  y  f  x   , với mọi x, y   .
18. (MEMO 2009). Tìm tất cả các hàm số f :    thỏa mãn đẳng thức:
f  xf  y    f  f  x   f  y    yf  x   f  x  f  y   , với mọi x, y   .
18. (Journal of Mathematical and youth 5/ 2011). Tìm tất cả các hàm số f xác định trên
tập  , lấy giá trị trong  và thỏa mãn phương trình:
f  x  y  f  y    f  f  x    2 y , với mọi x, y   .
19. (Iran TST 2011). Tìm tất cả các song ánh f :    sao cho:
f  x  f  x   2 f  y    f  2 x   f  2 y  ,với mọi x, y   .
20.(Journal of Mathematical and youth 01/2011).Với mỗi n  * , kí hiệu an là số tất cả
các song ánh f : 1, 2, 3,..., n  1, 2, 3,..., n thỏa mãn điều kiện với mọi k  1, 2, 3,..., n thì
f  f  k    k . Chứng minh:
a) an là số chẵn với mọi n  2 ; b) Với mọi n  10 và n 3 thì an  an 9  3 .
21. Xét tất cả các hàm đơn ánh f :    thỏa mãn điều kiện: f  x  f  x    2 x
,
với mọi x   . Chứng minh rằng hàm số f  x   x là một song ánh.(19)
22. Xét tất cả các hàm f , g , h :    sao cho f là đơn ánh và h là song ánh thỏa mãn điều
kiện f  g  x    h  x  , với mọi x   .Chứng minh rằng g  x  là một hàm song ánh.
23. Xét tất cả các hàm f :    0   thỏa mãn đồng thời hai điều kiện sau:
(i) f  x  y   f  x   f  y  , với mọi x, y     0
(ii) Số phần tử của tập hợp  x f  x   0, x     0 là hữu hạn.
Chứng minh rằng f là một hàm đơn ánh.
IV. PHƯƠNG PHÁP SỬ DỤNG TÍNH ĐƠN ĐIỆU CỦA HÀM SỐ
1. Tìm tất cả các hàm đơn điệu f :    thỏa mãn f  x  f  y    f  x   y , x, y   .

2. (Journal of Mathematical and youth T8/295). Tìm tất cả các hàm số f : 1;    1;  

thỏa mãn: f  xf  y    yf  x  , x, y  1;   .

3. (Greece 1997). Giả sử f :  0;     thỏa mãn ba điều kiện:

1  1
(i). f tăng nghiêm ngặt (ii). f  x    , x  0 (iii). f  x  f  f  x     1, x  0 .
x  x

Tính f 1 .

VĂN PHÚ QUỐC GV. Trường THPT chuyên Nguyễn Bỉnh Khiêm 0982 333 443 4
PHƯƠNG TRÌNH HÀM - BỒI DƯỠNG HỌC SINH GIỎI THI QUỐC GIA, QUỐC TẾ

4. Hãy tìm các hàm tăng thực sự f :    thỏa mãn f  xf  y    yf  2 x  , x, y   .

5. (IMO 2002). Tìm tất cả các hàm f :    thỏa:

 f  x   f  z    f  y   f  t    f  xy  zt   f  xt  yz  .
6. (Bulgaria 1996). Tìm tất cả các hàm tăng thực sự f :     thỏa mãn:

 x2 
f  x, x    .
 f  x  
 
7. (Iran 1997). Cho hàm số f :    là hàm giảm thỏa mãn:

 
f  x  y   f  f  x   f  y    f f  x  f  y    f  y  f  x   , x   .

Chứng minh rằng: f  f  x    x, x   .

V. PHƯƠNG PHÁP SỬ DỤNG TÍNH LIÊN TỤC CỦA HÀM SỐ


1. Tìm tất cả các hàm f  x  xác định trên  sao cho:

a) f  2012 x   f  x  , x   ; f  x  liên tục tại 0.

 x 
b) f  x   f  2 
, x   ; f  x  liên tục tại 0.
 1 x 
2. Tìm tất cả các hàm f  C    và thỏa mãn điều kiện:

f  4 x   f  9 x   2 f  6 x  , x  


3. Tìm tất cả các hàm f  x  xác định trên  , liên tục tại 1 và f  x    f x 2012 , x   . 
4. Tìm tất cả các hàm f  C    và thỏa mãn điều kiện f  x   f  sin x  , x   .

5. Tìm f  C    và thỏa điều kiện: f  f  x    e 2012 x , x   .

 1
6. (Bulgaria 1997). Tìm f :    liên tục và thỏa mãn: f  x   f  x 2   , x   .
 4
2x
7. (VMO 2001). Cho hàm số g  x   . Hãy tìm các hàm f  x  xác định, liên tục trên khoảng
1  x2
2
 1;1 và thỏa mãn hệ thức 1  x 2  f  g  x    1  x 2  . f  x  , x, y   1;1 .

8. Cho a   . Tìm tất cả các hàm f  x  xác định và liên tục trên  sao cho:

f  x  y   f  x   f  y   axy , x, y  

VĂN PHÚ QUỐC GV. Trường THPT chuyên Nguyễn Bỉnh Khiêm 0982 333 443 5
PHƯƠNG TRÌNH HÀM - BỒI DƯỠNG HỌC SINH GIỎI THI QUỐC GIA, QUỐC TẾ

9. Tìm f  C    thỏa mãn: f 1  1 ; f  


x 2  y 2  f  x   f  y  , x, y   .

 
10. Tìm f  C   sao cho f  xy   f  x   f  y  , x, y    .

11. Tìm tất cả các hàm f  x  xác định và liên tục trên   thỏa mãn điều kiện

 x
  f  x   f  y  , x, y   .

f
 y

12. Tìm tất cả các hàm f  x  xác định, liên tục trên  1;1 và thỏa mãn điều kiện

 
f  x   f  y   f x 1  y 2  y 1  x 2 , x, y   1;1 .

13. Tìm tất cả các hàm f  x  xác định và liên tục trên  thỏa mãn điều kiện:

 x  y  f  x  f  y
f  , x, y   .
 2  2

14. Tìm tất cả các hàm f  x  xác định và liên tục trên   và thỏa mãn điều kiện:

f  x  f  y
f  
xy 
2
, x, y    .

15. Tìm hàm f  x  xác định và liên tục trên   thỏa mãn điều kiện:

2
f  
xy 
1 1
, x, y    .

f  x f  y

16. Tìm tất cả các hàm f  x  xác định và liên tục trên * thỏa mãn điều kiện:

 
 2  2
f  , x, y  , x  y  0 .
11  1

1
x y  f  x f  y
 
17. (KOMAL- A.286, Hungary 2002). Tìm tất cả các hàm liên tục f :    thỏa mãn:

 x  y  f  x f  y
f  , x, y  ,1  xy  0 .
 1  xy  1  xy

18. (Bulgaria 1998). Cho hàm số f  x  xác định và liên tục trên  0;1 sao cho:

 2x  y 
(i). f  0   f 1  0 (ii). 2 f  x   f  y   3 f   , x, y   0;1 .
 3 

VĂN PHÚ QUỐC GV. Trường THPT chuyên Nguyễn Bỉnh Khiêm 0982 333 443 6
PHƯƠNG TRÌNH HÀM - BỒI DƯỠNG HỌC SINH GIỎI THI QUỐC GIA, QUỐC TẾ

19. (Romania 1997). Tìm tất cả các hàm liên tục f :    0;1 sao cho:

   
f x 2  y 2  f x 2  y 2  f  2 xy  , x, y  

1  x  1  x 
20. Tìm tất cả các hàm liên tục f :  0,1   thỏa f  x   f   f   , x   0,1 .
2   2  2 
21. Tìm tất cả các hàm f :    liên tục thỏa mãn:

f  x  y   f  x   f  y   sin x sin y cos  x  y  , x, y   ..

22. Cho f là hàm xác định và liên tục trên  0;1 sao cho với mỗi x   0;1 tồn tại h với

f  x  h  f  x  h
0  x  h  x  h  1 và f  x   . Tìm hàm số f .
2
23. (VMO 2006, bảng B). Tìm tất cả các hàm f  x  xác định, liên tục và nhận giá trị trong 

thỏa mãn điều kiện: f  x  y  f  y  z  f  z  x   8  0, x, y, z   .

24. Tìm tất cả các hàm f :    liên tục tại x  0 và thỏa mãn nf  nx   f  x   nx

( trong đó n  1 là số tự nhiên cố định nào đó).


VI. PHƯƠNG SỬ DỤNG ĐẠO HÀM
1. (VMS 1999). Xác định hàm số f  x  thỏa mãn điều kiện:

f  x  h   f  x  h   h 2 x  , h  0

2. Tìm f  x   D    thỏa f   x  .sin x  f  x  cos x  sin 2 x x   .

4. (VMS 2000). Tìm f  D    thỏa: f  x  y   f  x   f  y   2 xy, x, y  

f  x  f  y
5. Tìm tất cả các hàm f  x   D    thỏa mãn điều kiện: f  x  y   x, y   .
1 f  x f  y

6. (VMS 2003). Tìm tất cả các hàm f xác định trên đoạn  0;1 , khả vi trên khoảng  0;1 thỏa:

(i). f  0   f 1  1 ; (ii). 2003 f   x   2004 f  x   2004 x   0;1 .

7. Tìm tất cả các hàm f :    có đạo hàm liên tục trên  và thỏa mãn:

f  3 x  2   3 f  x  x   .

8. Tìm tất cả các hàm f xác định trên  thỏa mãn điều kiện:
2 3
 f  x   f  y    x  y , x, y  

VĂN PHÚ QUỐC GV. Trường THPT chuyên Nguyễn Bỉnh Khiêm 0982 333 443 7
PHƯƠNG TRÌNH HÀM - BỒI DƯỠNG HỌC SINH GIỎI THI QUỐC GIA, QUỐC TẾ

9. Tìm f  D    thỏa f  x  f  y    f  y  f  x   , x, y   .

10. Cho f :  0;     khả vi tại 1 và f  xy   f 1  f  x   f  y  x, y  0 .

Chứng minh rằng: f  x  khả vi và tìm f  x  .

11. Tìm tất cả các hàm f :  1;1   khả vi và thỏa mãn:

 x y 
f  x  f  y  f   , x, y   1;1
 1  xy 
12. Tìm tất cả các hàm f  x  xác định và khả vi 3 lần trên  thỏa: f  0   0 , f 1  e  2 ,

f  1  e 1 và f   x   f   x   0 x   .

13. Tìm hàm khả vi f  x  sao cho x  0; 3 x 2 f   x   x3 f   x   1 và f 1  1 , f  2   1 .

14. Giả sử f  x   C 2  ,   với x, h bất kỳ ta có đồng nhất thức:

 h
f  x  h   f  x   hf   x   . Chứng minh rằng: f  x   ax 2  bx  c .
 2
15. Cho a   . Tìm tất cả các hàm f :    thỏa mãn:

f x  f  y
(i). f có đạo hàm trên  ; (ii).  f   ax  1  a  y  x  y .
x y
16. (VMS 1995). Tìm tất cả các hàm f :    thỏa mãn các điều kiện sau:

f  x
(i). f  x  y   f  x   f  y  x, y   (ii). lim  1.
x0 x
17. Tìm tất cả các hàm f , g :  0;     thỏa mãn các điều kiện sau:

g  x f  x
(i). f có đạo hàm trên  (ii). f   x    ; g x   x  0 .
x x
18. Tìm tất cả các hàm f :    thỏa mãn:

(i). f có đạo hàm trên  (ii). f  x  y   f  x  y   y  f   x  y   f   x  y   x, y .

19. Tìm tất cả các hàm f  x  , g  x   C    thỏa mãn điều kiện:


m 2
f  y   f  x   f  x  y  x   M y  x , x, y  

trong đó M , m là hai số dương cho trước.

VĂN PHÚ QUỐC GV. Trường THPT chuyên Nguyễn Bỉnh Khiêm 0982 333 443 8
PHƯƠNG TRÌNH HÀM - BỒI DƯỠNG HỌC SINH GIỎI THI QUỐC GIA, QUỐC TẾ

20. Tìm f  x   C    thỏa mãn điều kiện: thỏa mãn điều kiện:

f  a   f  d   2003 f  b   f  c  với bộ bốn số a, b, c, d   theo thứ tự lập thành một cấp số

cộng.
VII. PHƯƠNG PHÁP SỬ DỤNG ĐIỂM BẤT ĐỘNG CỦA HÀM SỐ
1. (IMO 1983). Tìm hàm số f :    thỏa mãn hai điều kiện sau:
lim f  x   0 và f  xf  y    yf  x  , x, y    .
x 

2. (IMO 1994). Giả sử S là tập hợp các số thực lớn hơn 1 . Tìm tất cả các hàm f : S  S
sao cho các điều kiện sau được thỏa mãn:
(i). f  x  f  y   xf  y    y  f  x   yf  x  x  S
f  x
(ii). là hàm tăng thực sự trên các khoảng  1;0  ,  0;   .
x
3. (IMO 1996). Tìm tất cả các hàm số f :    sao cho
f  m  f  n    f  f  m    f  n  m, n   .
4. (AMM, E984). Tìm tất cả các hàm f :    sao cho f  f  x    x 2  2, x   .
5. Tìm tất cả các hàm f :    thỏa mãn:
(i). f  f  x   y   xf  y   f  f  x   f  y   , x, y  
(ii). f có một điểm bất động.
6. Tìm tất cả các hàm f :    thỏa mãn: f  x  y   f  x  .e f  y  1 , x, y   .

VIII. BẤT ĐẲNG THỨC HÀM


1. Tìm hàm số f :    thỏa mãn điều kiện: f  x  y   f  x  f  y   2012 x  y , x, y   .
2. (VMO 1994). Tìm tất cả các hàm số f :    thỏa mãn:
1 1 1
f  xy   f  xz   f  x  f  yz   , x, y , z   .
2 2 4
3. (VMS 2004). Tìm tất cả các hàm số f  x  xác định trên tập  và thỏa mãn các điều kiện:
(i) f  x   e 2004 x , x   (ii) f  x  y   f  x  f  y  , x, y   .
4. Tìm tất cả các hàm f :    thỏa mãn hai điều kiện:
(i) f  x   x, x   (ii) f  x  y   g  x   g  y  , x, y   .
5. (Russia 2000). Tìm tất cả các hàm f :    thỏa mãn điều kiện:
f  x  y   f  y  z   f  z  x   3 f  x  2 y  3 z  , x, y, z  
6. (Eotvos - Kurschak 1979). Cho hàm số f :    thỏa:
f  x   x và f  x  y   f  x   f  y  , x, y   .
Chứng minh rằng: f  x   x, x   .
7. (Crux 2003 - Canada). Tìm tất cả các hàm f :    thỏa mãn điều kiện:

VĂN PHÚ QUỐC GV. Trường THPT chuyên Nguyễn Bỉnh Khiêm 0982 333 443 9
PHƯƠNG TRÌNH HÀM - BỒI DƯỠNG HỌC SINH GIỎI THI QUỐC GIA, QUỐC TẾ

f  x 3  x   x  f 3  x   f  x  , x   .
8. (Bulgaria 1997). Tìm hàm số f :  0;     0;   thỏa mãn bất đẳng thức hàm:
f 2  x   f  x  y  f  f  x   y  , x, y   0;   .
9. (Japan 2007). Tìm tất cả các hàm f :     thỏa mãn hai điều kiện sau đây:
f  x  y f x f  y f  x  y
(i). f  x   f  y   ; (ii).   , x, y   
2 x y x y
y
10. Cho hàm số f :    thỏa mãn điều kiện: f  x  y   f  x   x, y   .
x
n n  n  1
Chứng minh rằng với mọi số tự nhiên n ta có:  f 2   f 2  
i 1
n i

2
.

IX. PHƯƠNG PHÁP THÊM BIẾN


1. (Idia 2004). Tìm các hàm số f :    thỏa mãn điều kiện:
f  x  y   f  x  f  y   c sin x sin y, x, y   , c là hằng số lớn hơn 1.
2. ( Đề nghị IMO 2005). Tìm tất cả các hàm f :  0;     0;   thỏa mãn điều kiện:
f  x  f  y   2 f  x  yf  x   , x, y  0 .
3. (Đề nghị OLP 30/4/2009). Cho hàm số f liên tục trên  và thỏa mãn điều kiện:
f  x  f  y   f  x  y   sin x sin y, x, y   .
1 1 1
Chứng minh rằng:    2.
1  f  2x 1  f 4x 1  f 6x
4. (OLP 30/4/2011). Tìm tất cả các hàm f : 1;    1;   thỏa mãn điều kiện:
f  xf  y    yf  x  , x, y  1;   .
5. (OLP 30/4/2004). Tìm tất cả các hàm liên tục f :    thỏa:
f  xf  y    yf  x  , x, y  

X. PHƯƠNG PHÁP ĐƯA VỀ PHƯƠNG TRÌNH SAI PHÂN


1. Tìm tất cả các hàm f :    thỏa mãn điều kiện: f  f  n    f  n   2n  3k , n  
( k là số tự nhiên cho trước)
2. Tìm tất cả các hàm f :    sao cho với mọi n   :
 
f f  f  n    6 f  n   3 f  f  n    4n  2013 .
3. (Balkan 2002). Tìm tất cả các hàm f :    thỏa mãn với mọi n   sao cho:
f  f  n    f  n   2n  2011 hoặc f  f  n    f  n   2n  2012 .
4. Cho f :    thỏa mãn điều kiện: f  0   1; f  f  x    x  f  x  , x   .
Tìm mọi số nguyên n  1 sao cho f n  0  chia hết cho 20 . Ở đây f n  x   f  f n1  x   .
5. Tìm tất cả các hàm f :      thỏa:

VĂN PHÚ QUỐC GV. Trường THPT chuyên Nguyễn Bỉnh Khiêm 0982 333 443 10
PHƯƠNG TRÌNH HÀM - BỒI DƯỠNG HỌC SINH GIỎI THI QUỐC GIA, QUỐC TẾ

 
f  f  x    20112012 f  x   20112012  20122013 .20122013 x , x    .
6. Tìm tất cả các hàm f :    thỏa mãn điều kiện:
f  n  f  m   f  m  n   f  n  m  , m, n  , n  m .
7. (IMO Shortlist 1992). Cho a, b    . Tìm tất các hàm f :      thỏa mãn điều kiện:
f  f  x    af  x   b  a  b  x, x    .
8. Tìm tất cả các hàm f :  0;1   0;1 thỏa mãn điều kiện: f  2 x  f  x    x, x   0;1 .
XI. PHƯƠNG PHÁP KHẢO SÁT TẬP HỢP
1. (Romania 1986). Cho f :    là một toàn ánh và g :    là một đơn ánh. Biết rằng
f n   g n  với mọi n . Chứng minh rằng: f  g .
2. Cho song ánh f :    . Chứng minh rằng tồn tại bộ ba số a,b, c  , a  b  c sao cho
f a   f c   2 f b  .
3. (IMO 1999). Tìm tất cả các hàm f :    thỏa mãn điều kiện:
   
f x  f y   f f y   xf y   f x   1 , x , y   .

4. (Korea 1999). Tìm tất cả các hàm f :  *   * biết rằng tồn tại n 0  * sao cho

 
f n 0   1 và f n  f n   f n  với mọi n  * .

XII. PHƯƠNG PHÁP ĐỐI LẬP ( ĐÁNH GIÁ BẤT ĐẲNG THỨC)
1. (Baltic MO). Tìm tất cả các hàm f : *  * thỏa mãn:
(i). f  0   0 , f 1  1 ; (ii). f  0   f 1  f  2   ... ;
 
(iii). f x 2  y 2  f 2  x   f 2  y  , x, y  *
2.(Austrian 2002). Tìm tất cả các hàm f : *  * thỏa mãn:
(i). f  x  22   f  x   
(ii). f x 2 y  f 2  x  f  y  , x, y  * .
3. (Cono Sur Olympiad 1995). Tìm tất cả các hàm f : *  * thỏa mãn:
(i). Nếu x  y thì f  x   f  y  (ii). f  yf  x    x 2 f  xy  , x, y  * .
4. (Korea 1996). Tìm tất cả các hàm f :    thỏa mãn:
 
(i). 2 f m 2  n 2  f 2  m   f 2  n  m, n    
(ii). f m 2  f 2  n  m  n .

 
5. Tìm tất cả các hàm f :  *  * thỏa mãn: f19 n  97 f n   98n  232

  
với fm  n  f f ...f n .   
 
m

6. Tìm tất cả các hàm f :  *  * thỏa mãn:



(i). f 2  2 ;

(ii). f  mn   f m  f n  , m, n   *
 
thỏa m; n  1 ;

VĂN PHÚ QUỐC GV. Trường THPT chuyên Nguyễn Bỉnh Khiêm 0982 333 443 11
PHƯƠNG TRÌNH HÀM - BỒI DƯỠNG HỌC SINH GIỎI THI QUỐC GIA, QUỐC TẾ

   
(iii). f m  f n , m  n .
7. Cho hàm số f :  *  * thỏa mãn:
   
(i). f xy  f x f y  
(ii). f x  x  
(iii). f f 1995    95 .
Tìm giá trị nhỏ nhất của f 135  .
8. Tìm tất cả các hàm số f :  *  * thỏa mãn với mọi x ; y; z  * :
       
f xy  f xz  f x f yz  1 .
1 5 1
9. Đặt q  và gọi f :    là hàm số thỏa mãn điều kiện f  n   qn  n   .
2 q

Chứng minh rằng f  f  n    f  n   n n   .

XIII. PHƯƠNG PHÁP SỬ DỤNG NGUYÊN LÝ CỰC HẠN


1. (IMO 1997). Tìm tất cả các hàm f : *  * thỏa mãn: f  f  n    f  n  1 n   .
2. (Canada 2002). Tìm tất cả các hàm f :    thỏa mãn:
 
xf  y   yf  x    x  y  f x 2  y 2 , x, y   .

 
3. Tìm tất cả các hàm f : *  * thỏa 2 f 3 m 2  n 2   f m  f  n  f m  f n  .
2 2

4. Tìm tất cả các hàm f :    thỏa mãn điều kiện:



f mf n     f  f m    f n  , m, n   .
XIV. PHƯƠNG PHÁP SỬ DỤNG SỐ HỌC
1. Tìm tất cả các hàm f :    sao cho f 2  x   y chia hết cho x 2  f  y  x, y  * .
2. (Iran TST). Tìm tất cả các hàm f :    thỏa mãn: tồn tại số k  * và số nguyên tố p
sao cho với mọi n  k , f  n  p   f  n  và nếu n chia hết cho m thì f  n   1 chia hết cho
f  m  1 .
3. (IMO Shortlist 2004). Tìm tất cả các hàm f : *  * thỏa mãn: m 2  n 2 chia hết cho
f 2 m   f  n .
4. Cho hàm số f  n  xác định trên tập hợp các số nguyên dương * thỏa mãn các điều kiện:
(i) f  p   1 nếu p nguyên tố.
(ii) f  mn   mf  n   nf  m  m, n  *
Hãy tìm giá trị n sao cho f  n   n .

  
5. Tìm tất cả các hàm f :  *  * thỏa mãn: x 2  f y f 2 x  y với mọi x , y  * .
6. (Iran TST 2005). Tìm tất cả các hàm f :    thỏa mãn tồn tại số k   và một số
   
nguyên tố p sao cho với mọi n  k , f n  p  f n và nếu m n thì f m  1 f n  1 .    
7. (IMO Shortlists 2004). Tìm tất cả các hàm f :  *  * thỏa mãn

VĂN PHÚ QUỐC GV. Trường THPT chuyên Nguyễn Bỉnh Khiêm 0982 333 443 12
PHƯƠNG TRÌNH HÀM - BỒI DƯỠNG HỌC SINH GIỎI THI QUỐC GIA, QUỐC TẾ

2
 
f2 m  f n   m 2
n  với mọi m, n   *
8. (USA TST). Cho p là một số nguyên tố lẻ. Tìm tất cả các hàm số f :    thỏa mãn
   
đồng thời: (i). f m  f n nếu m  n mod p        
(ii). f mn  f m f n với m, n   .

9. Tìm tất cả các số nguyên không âm n nhỏ nhất sao cho tồn tại hàm số f :    0;  
khác hằng số thỏa mãn:
        
(i). f xy  f x f y (ii). 2 f x 2  y 2  f x  f y  0;1;...; n , x , y   
Với số n tìm được, tìm mọi hàm số thỏa mãn.

XV. PHƯƠNG PHÁP SỬ DỤNG HÀM SỐ VÀ CƠ SỐ ĐẾM


1. (IMO 1988). Xác định f trên tập hợp các số nguyên dương như sau:
(i). f 1  1, f  3  3
(ii). f  2n   f  n  , f  4n  1  2 f  2n  1  f  n  , f  4n  3  3 f  2n  1  2 f  n  .
2. (IMO Shortlist 2000). Cho hàm f : *  * thỏa mãn:
(i). f  4n   f  2n   f  n  (ii). f  4n  2   f  4n   1 (iii). f  2n  1  f  2n   1

3. Cho hàm số f : *   thỏa mãn các điều kiện:


 n  1
1  f  , n  2m  1
  2 

f 1  1 và f n    n 
1  f   , n  2m
 2
Hãy tìm các giá trị của n sao cho f n  2004 . 
4. Cho hàm số f :  *  * thỏa mãn các điều kiện:

(i). f 1  1  
(ii). f 2n  f n      
(iii). f 2n  1  f 2n  1 .

 
Tìm giá trị lớn nhất của f n với 1  n  1994 .
5. (IMO 1988)
Cho hàm f xác định trên tập các số nguyên dương * sao cho:
       
f 1  1; f 3  3; f 2n  f n ; 2 f 2n

f  4n  1  2 f  2n  1  f  n  ; f  4n  3   3 f  2n  1 .
Hãy xác định các số nguyên dương n  1988 sao cho f  n   n .

VĂN PHÚ QUỐC GV. Trường THPT chuyên Nguyễn Bỉnh Khiêm 0982 333 443 13

You might also like